Download as pdf or txt
Download as pdf or txt
You are on page 1of 191

UPSC 2024 -

Exam Title : Environment,


Sc...
rupeshkr.
Email :
09ee80@gmail.com
Contact :

Note: If the exam is multi-lingual i.e. English and Hindi. Hindi solutions will be
after the completion of English solutions.
QUESTION 1. MTI3Mjk4K1J1cGVzaCBLdW1hcitydXBlc2hrci4wOWVlODBAZ21haWwuY29tK1FVRVNU
SU9OIDA=
Consider the following pairs:

Neutrino Observatory Location

1. Trident South China Sea

2. IceCube Antarctica

3. INO Theni Hills India

How many of the above pairs correct?

a) Only one
b) Only two
c) All three
d) None

Correct Answer: C
Your Answer: A Marks: -0.66/1.00
Explanation

Solution (c)

Explanation:

· China is building the World’s largest telescope called Trident below the South China Sea. It will
be used to detect ‘ghost particles’ or the neutrinos. (Hence pair 1 is correct)

· The IceCube Neutrino Observatory is a neutrino observatory constructed at the Amundsen–


Scott South Pole Station in Antarctica. The project is a recognized CERN experiment. Its
thousands of sensors are located under the Antarctic ice, distributed over a cubic kilometer. (He
nce pair 2 is correct)

· The INO decided on a site in Bodi West Hills (BWH) region near Pottipuram village in Theni
district of Tamil Nadu. (Hence pair 3 is correct)

Important Points/ Value Additions:

· Neutrinos are tiny subatomic particles. They are called ‘ghost particles’ because they barely
interact with anything else, and are thus very difficult to detect.

· Neutrinos do not carry an electric charge. They are not affected by electromagnetic forces. This
allows them to pass through matter including most atoms, with minimal interaction.

· Neutrinos have a mass much smaller than other fundamental particles like electrons or quarks.
Neutrino’s weak charge have made them difficult to observe.

IASbaba
Score:
Web: http://ilp.iasbaba.com/
44.00 /
Email: ilp@iasbaba.com
Page 1 200
UPSC 2024 -
Exam Title : Environment,
Sc...
rupeshkr.
Email :
09ee80@gmail.com
Contact :

QUESTION 2. MTI3Mjk4K1J1cGVzaCBLdW1hcitydXBlc2hrci4wOWVlODBAZ21haWwuY29tK1FVRVNU
SU9OIDE=
Consider the following statements about ‘Small Modular Reactors’(SMR):

1. SMRs require more frequent refueling compared to conventional plants.

2. SMRs have higher power and operating pressure, reducing safety of SMRs.

3. The modular nature of SMRs makes installation difficult and time-consuming.

How many of the above statements correct?

a) Only one
b) Only two
c) All three
d) None

Correct Answer: D
Your Answer: B Marks: -0.66/1.00
Explanation

Solution (d)

Explanation:

· Power plants based on SMRs may require less frequent refuelling, every 3 to 7 years, in
comparison to between 1 and 2 years for conventional plants. Some SMRs are designed to opera
te for up to 30 years without refuelling. (Hence statement 1 is incorrect)

· SMRs have lower power and operating pressure. They are inherently safer than the
conventional reactors because their safety systems rely on natural circulation, convection, gravity
and self-pressurization. (Hence statement 2 is incorrect)

· Smaller size and modular nature make SMRs easier to install. They can be installed in locations
which are unsuitable for large and conventional reactors. (Hence statement 3 is incorrect)

Important Points/Value Additions:

· Small Modular Reactors (SMRs) represent a category of nuclear reactors that differ in size from
traditional large-scale nuclear power plants. Their design emphasizes flexibility, scalability, and
easy deployment in diverse environments.

· The modular aspect allows independent construction of various SMR modules in a factory,
followed by transportation to the installation site. In contrast, conventional nuclear reactors,
which are substantially heavier, are typically constructed on-site.

· SMRs boast a power capacity of up to 300 MW per unit, constituting approximately one-third of
the capacity of traditional nuclear reactors, which typically range from 700 to 800 MW.

IASbaba
Score:
Web: http://ilp.iasbaba.com/
44.00 /
Email: ilp@iasbaba.com
Page 2 200
UPSC 2024 -
Exam Title : Environment,
Sc...
rupeshkr.
Email :
09ee80@gmail.com
Contact :

QUESTION 3. MTI3Mjk4K1J1cGVzaCBLdW1hcitydXBlc2hrci4wOWVlODBAZ21haWwuY29tK1FVRVNU
SU9OIDI=
With reference to e-SIM (Embedded Subscriber Identification Module), consider the following
statements:

1. Change in mobile phone device will lead to change in phone number.

2. It can easily violate privacy of users, as service provider can track user activity.

3. It is not possible to re-programme e-SIMS.

How many of the above statements correct?

a) Only one
b) Only two
c) All three
d) None

Correct Answer: B
Your Answer: A Marks: -0.66/1.00
Explanation

Solution (b)

Explanation:

· Switching devices becomes more challenging, as a change in the mobile phone device results in a
corresponding alteration of the phone number. (Hence statement 1 is correct)

· Mobile services providers gain the ability to monitor user’s activities with heightened precision,
raising concerns when robust privacy protection laws are lacking. (Hence statement 2 is
correct)

· While users have the option to re-programme eSIMs, individuals with lower digital literacy
may encounter difficulties in doing so. (Hence statement 3 is incorrect)

Important Points/Value Additions:

· An eSIM, short for embedded SIM, is a digital or electronic counterpart of a traditional SIM
card. Unlike a physical, removable card, it is integrated into devices like smartphones, tablets, or
smartwatches.

· Its primary function aligns with that of a conventional SIM card, serving to identify a user to a
mobile network and facilitate connectivity.

· eSIMs provide enhanced security, with a reduced likelihood of being lost or removed, simplifying
the onboarding process for new users without the need to wait for SIM card activation.

· The growing acceptance of eSIMs is anticipated to diminish or eliminate the necessity for
telecom service providers to dispatch SIM cards to remote locations.

IASbaba
Score:
Web: http://ilp.iasbaba.com/
44.00 /
Email: ilp@iasbaba.com
Page 3 200
UPSC 2024 -
Exam Title : Environment,
Sc...
rupeshkr.
Email :
09ee80@gmail.com
Contact :

· Recognized for its environmental friendliness, the re-programmable nature of eSIMs eliminates
the requirement for additional plastic and metal in the production of new SIMs.

QUESTION 4. MTI3Mjk4K1J1cGVzaCBLdW1hcitydXBlc2hrci4wOWVlODBAZ21haWwuY29tK1FVRVNU
SU9OIDM=
In the context of ‘Deep Ocean Mission’ of Ministry of Earth Sciences, consider the following
statements:

1. The years 2021-2030 have been designated by the United Nations as the ‘Decade of Ocean
Science’.

2. Deep Ocean Mission is one of nine missions under the Prime Minister’s Science, Technology, and
Innovation Advisory Council (PMSTIAC).

3. Matsya6000 is a deep-ocean submersible designed to accommodate a crew of three members


equipped with a suite of scientific sensors and tools.

4. Exploring the deep ocean presents a significant challenge due to the progressively decreasing
pressure as we delve deeper from the ocean surface.

How many of the above statements are correct?

a) Only one
b) Only two
c) Only three
d) All four

Correct Answer: C
Your Answer: C Marks: 2.00/1.00
Explanation

Solution (c)

Explanation:

· The years 2021-2030 have been designated by the United Nations as the ‘Decade of Ocean
Science’, and Prime Minister Narendra Modi has, on several occasions, emphasised the need for
India to work towards sustainably harnessing the ocean’s potential for the nation’s growth. (Hen
ce statement 1 is correct)

· DOM is one of nine missions under the Prime Minister’s Science, Technology, and Innovation
Advisory Council (PMSTIAC). It is imperative that DOM supports the blue-economy priority area,
blue trade, and blue manufacturing in India. (Hence statement 2 is correct)

· Matsya6000, a deep-ocean submersible designed to accommodate a crew of three members.


Equipped with a suite of scientific sensors and tools. (Hence statement 3 is correct)

· Exploring the depths of the oceans has proved to be more challenging than exploring outer
space. The fundamental distinction lies with the high pressure in the deep oceans. While outer

IASbaba
Score:
Web: http://ilp.iasbaba.com/
44.00 /
Email: ilp@iasbaba.com
Page 4 200
UPSC 2024 -
Exam Title : Environment,
Sc...
rupeshkr.
Email :
09ee80@gmail.com
Contact :

space is akin to a near perfect vacuum, being one-meter underwater puts as much pressure on an
object of one square meter area as if it were carrying about of 10,000kg of weight, which is
equivalent to a huge adult elephant.

· Operating under such high-pressure conditions requires the use of meticulously designed
equipment crafted from durable metals or materials. Additionally, electronics and instruments find
it simpler to function in a vacuum or in space. Conversely, inside the water, poorly designed
objects collapse or implode. (Hence statement 4 is incorrect)

Important Points/Value Additions:

· It is also known as Samudrayaan Mission, aimed to develop a self-propelled manned submersible


to carry 3 human beings to a water depth of 6000 meters in the ocean with a suite of scientific
sensors and tools for deep ocean exploration. It has an endurance of 12 hours of operational
period and 96 hours in case of emergency.

· The projected timeline is five years for the period 2020-2021 to 2025-2026.

· National Institute of Ocean Technology (NIOT), Chennai, an autonomous institute under MoES,
has developed 6000m depth rated Remotely Operated Vehicle (ROV) and various other
underwater instruments such as Autonomous Coring System (ACS), Autonomous Underwater
Vehicle (AUV) and Deep-Sea Mining System (DSM) for the exploration of deep sea.

QUESTION 5. MTI3Mjk4K1J1cGVzaCBLdW1hcitydXBlc2hrci4wOWVlODBAZ21haWwuY29tK1FVRVNU
SU9OIDQ=
Consider the following statements with respect to ‘Wolbachia method’ used to check growth of
viral diseases:

1. Wolbachia is a common fungi that occurs naturally in insect species.

2. Wolbachia prevents viruses such as dengue, chikungunya, and Zika from proliferating in the
bodies of Aedes aegypti mosquitoes.

3. Wolbachia is safe for humans, animals, and the environment.

How many of the above statements correct?

a) Only one
b) Only two
c) All three
d) None

Correct Answer: B
Your Answer: Unanswered Marks: 0/1.00
Explanation

Solution (b)

Explanation:

IASbaba
Score:
Web: http://ilp.iasbaba.com/
44.00 /
Email: ilp@iasbaba.com
Page 5 200
UPSC 2024 -
Exam Title : Environment,
Sc...
rupeshkr.
Email :
09ee80@gmail.com
Contact :

· Wolbachia is a prevalent bacterium occurring naturally in about 60% of insect species ,


including mosquitoes, fruit flies, moths, dragonflies, and butterflies. (Hence statement 1 is
incorrect)

· Wolbachia effectively hinders the growth of viruses like dengue, chikungunya, and Zika within
the bodies of Aedes aegypti mosquitoes. (Hence statement 2 is correct)

· Notably, Wolbachia is deemed safe for humans, animals, and the environment. (Hence
statement 3 is correct)

Important Points/Value Additions:

· Wolbachia typically absent in the Aedes aegypti mosquito, some studies have indicated its
presence in certain populations. Aedes aegypti is the primary species responsible for transmitting
human viruses such as Zika, dengue, chikungunya, and yellow fever.

· In the Wolbachia method, Aedes aegypti mosquitoes are deliberately infected with the bacterium
in a laboratory setting and subsequently released into the wild.

· The Wolbachia-infected mosquitoes mate with their wild counterparts, leading to an increasing
presence of the bacterium in the mosquito population.

· The heightened prevalence of Wolbachia makes it challenging for viruses to reproduce within the
mosquitoes, as the bacterium effectively blocks their growth.

· As the population of Wolbachia-infested mosquitoes expands, the likelihood of virus transmission


diminishes significantly.

QUESTION 6. MTI3Mjk4K1J1cGVzaCBLdW1hcitydXBlc2hrci4wOWVlODBAZ21haWwuY29tK1FVRVNU
SU9OIDU=
Consider the following statements with reference to the context of ‘Aurora lights’:

1. An aurora is caused when electrically-charged particles from the sun collide with particles from
gases such as oxygen and nitrogen present in the Earth’s atmosphere.

2. It is observed only on Earth in our Solar system, as other planets lack dense atmosphere in
polar regions.

Which of the above statements are correct?

a) 1 Only
b) 2 Only
c) Both
d) Neither

Correct Answer: A
Your Answer: C Marks: -0.66/1.00
Explanation

Solution (a)

IASbaba
Score:
Web: http://ilp.iasbaba.com/
44.00 /
Email: ilp@iasbaba.com
Page 6 200
UPSC 2024 -
Exam Title : Environment,
Sc...
rupeshkr.
Email :
09ee80@gmail.com
Contact :

Explanation:

· An aurora is a natural phenomenon which is characterised by a display of a natural-coloured


(green, red, yellow or white) light in the sky. It is a light show which is caused when electrically-
charged particles from the sun collide with particles from gases such as oxygen and nitrogen
present in the Earth’s atmosphere. (Hence statement 1 is correct)

· Both Jupiter and Saturn have similar auroras that are caused by solar wind but particles also
come from nearby active moons like Jupiter’s Io and Saturn’s Enceladus. (Hence statement 2 is
incorrect)

Important Points/Value Additions:

· Recently, astronomers used the James Webb Space Telescope to find a brown dwarf with infrared
emission from methane. This is unexpected because the brown dwarf, larger than Jupiter and
smaller than a star, is cold and does not have a host star. The researchers propose that the
methane may be there due to processes generating auroras.

· Aurora is sometimes referred to as ‘polar light’. It is predominantly seen in the regions of high
altitudes like the Arctic and Antarctic.

· Auroras are seen in latitudes of around 70 degrees. They generally occur in a band known as
‘auroral zone’. The auroral zone is 3 to 6 degrees wide in latitude. It lies between 10 and 20
degrees from the geomagnetic poles.This is visible quite clearly during the night. Auroras can
sometimes be seen at latitudes below the actual auroral zone.

· This natural light effect is known as ‘aurora borealis’ in northern altitudes, while the effect in the
southern latitudes is known as ‘aurora australis’. Aurora borealis is also known as ‘Northern
lights’. Similarly, aurora australis is also known as ‘Southern lights’.

QUESTION 7. MTI3Mjk4K1J1cGVzaCBLdW1hcitydXBlc2hrci4wOWVlODBAZ21haWwuY29tK1FVRVNU
SU9OIDY=
Q.7) Consider the following statements about ‘Artemis Mission’:

1. It is a Moon exploration program that is led by the United States's NASA.

2. Apollo 17 mission was the last moon landing of humans more than five decades back.

3. The program's stated long-term goal is to establish a permanent base on the Moon to facilitate
human missions to Mars.

4. India joined ‘Artemis Accords’ which has larger objective of peaceful exploration of deep space.

How many of the above statements correct?

a) Only one
b) Only two
c) Only three
d) All four

IASbaba
Score:
Web: http://ilp.iasbaba.com/
44.00 /
Email: ilp@iasbaba.com
Page 7 200
UPSC 2024 -
Exam Title : Environment,
Sc...
rupeshkr.
Email :
09ee80@gmail.com
Contact :

Correct Answer: D
Your Answer: D Marks: 2.00/1.00
Explanation

Solution (d)

Explanation:

· The Artemis program is a Moon exploration program that is led by the United States’ NASA and
formally established in 2017 via Space Policy Directive 1. (Hence statement 1 is correct)

· The Artemis program is intended to reestablish a human presence on the Moon for the first time
since Apollo 17 in 1972. (Hence statement 2 is correct)

· The program's stated long-term goal is to establish a permanent base on the Moon to facilitate
human missions to Mars. (Hence statement 3 is correct)

· Prime Minister announced India's decision to join the Artemis Accords during the visit to the
United States. (Hence statement 4 is correct)

Important Points/Value additions:

· National Aeronautics and Space Administration (NASA) and Indian Space Research Organisation
(ISRO) will collaborate to send Indian astronauts, trained at the Johnson Space Center in
Houston, Texas, to the International Space Station (ISS) in 2024.

· Artemis Accords have wide objective. They are established by the U.S. State Department and
NASA with seven other founding members: Australia, Canada, Italy, Japan, Luxembourg, the
United Arab Emirates, and the United Kingdom in 2020 for setting common principles to govern
civil exploration and use of outer space, the moon, Mars, comets, and asteroids, for peaceful
purposes.

· It builds upon the foundation of the Outer Space Treaty of 1967

QUESTION 8. MTI3Mjk4K1J1cGVzaCBLdW1hcitydXBlc2hrci4wOWVlODBAZ21haWwuY29tK1FVRVNU
SU9OIDc=
Consider the following statements in the context of ‘X-ray Imaging and Spectroscopy Mission’:

1. XRISM is ISRO led mission in collaboration with NASA and the European Space Agency.

2. It is designed to study the hottest regions in the universe, along with its largest structures and
the objects with the strongest gravity.

3. XRISM is carrying only two instruments -Resolve and Xtend each of them focuses of the same
X-ray Mirror assembly.

How many of the above statements correct?

a) Only one
b) Only two

IASbaba
Score:
Web: http://ilp.iasbaba.com/
44.00 /
Email: ilp@iasbaba.com
Page 8 200
UPSC 2024 -
Exam Title : Environment,
Sc...
rupeshkr.
Email :
09ee80@gmail.com
Contact :

c) All three
d) None

Correct Answer: B
Your Answer: Unanswered Marks: 0/1.00
Explanation

Solution (b)

Explanation:

· It is led by the Japan Aerospace Exploration Agency in collaboration with NASA and the
European Space Agency. (Hence statement 1 is incorrect)

· The space observatory is designed to detect X-ray radiation with energy up to 12,000 electron
volts. To put that into context, the energy of visible light is between two to three electron volts.
Therefore, it is designed to study the hottest regions in the universe, along with its largest
structures and the objects with the strongest gravity. (Hence statement 2 is correct)

· XRISM is carrying just two instruments — Resolve and Xtend — and each of them focuses of the
same X-ray Mirror assembly. (Hence statement 3 is correct)

Important Points/Value Additions:

· Resolve is a microcalorimeter spectrometer, and it sits inside a refrigerator-sized container of


liquid helium at a fraction of a degree above absolute zero. X-rays warm the device by an amount
related to its energy. Scientists can use the measure of each individual X-ray’s energy to glean
knowledge about the source.

· Xtend, the mission’s second instrument, is an X-ray imager with a large field of view, able to
observe an area about 60 per cent larger than the average apparent size of the full moon.

QUESTION 9. MTI3Mjk4K1J1cGVzaCBLdW1hcitydXBlc2hrci4wOWVlODBAZ21haWwuY29tK1FVRVNU
SU9OIDg=
Consider the following statements about POEM platform of ISRO:

1. It will help perform in-orbit experiments using the final, and otherwise discarded stage of
Geosynchronous Satellite Launch Vehicle (GSLV).

2. It has a dedicated Navigation Guidance and Control (NGC) system for attitude stabilization.

3. It allows space agencies and private companies to design, develop and validate experimental
orbital payloads.

How many of the above statements correct?

a) Only one
b) Only two
c) All three
d) None

IASbaba
Score:
Web: http://ilp.iasbaba.com/
44.00 /
Email: ilp@iasbaba.com
Page 9 200
UPSC 2024 -
Exam Title : Environment,
Sc...
rupeshkr.
Email :
09ee80@gmail.com
Contact :

Correct Answer: B
Your Answer: Unanswered Marks: 0/1.00
Explanation

Solution (b)

Explanation:

· POEM (PSLV Orbital Experimental Module) is an experimental mission by ISRO which performs
in-orbit scientific experiments during the fourth stage of the Polar Satellite Launch Vehicle (PSLV)
launch vehicle as an orbital platform. (Hence statement 1 is incorrect)

· According to ISRO, POEM has a dedicated Navigation Guidance and Control (NGC) system for
attitude stabilisation, which stands for controlling the orientation of any aerospace vehicle within
permitted limits. The NGC will act as the platform’s brain to stabilize it with specified accuracy. (
Hence statement 2 is correct)

· The space agency’s POEM platform has standard interfaces and packaging for power generation,
telemetry, tele-command stabilisation, orbital-station keeping and orbital maneuvering. This allows
space agencies and private companies to design, develop and validate experimental orbital
payloads. (Hence statement 3 is correct)

Important Points/Value Additions:

· The PSLV is a four-stage rocket where the first three spent stages fall back into the ocean, and
the final stage (PS4) — after launching the satellite into orbit — ends up as space junk. It is the
first time that the (fourth stage) PS4 stage would orbit the earth as a stabilized platform.

· POEM will derive its power from solar panels mounted around the PS4 tank, and a Li-Ion battery.
It will navigate using four sun sensors, a magnetometer, gyros & NavIC.

· It carries dedicated control thrusters using Helium gas storage. It is enabled with a
telecommand feature.

QUESTION 10. MTI3Mjk4K1J1cGVzaCBLdW1hcitydXBlc2hrci4wOWVlODBAZ21haWwuY29tK1FVRVN


USU9OIDk=
With reference to XPoSat, consider the following statement:

1. It is launched by NASA in coordination with ESA.

2. It will study X-ray polarisation and its cosmic sources such as black holes and neutron stars.

3. The XPoSat is the only such mission in the world to study X-ray polarisation.

How many of the above statements correct?

a) Only one
b) Only two
c) All three
d) None

IASbaba
Score:
Web: http://ilp.iasbaba.com/
44.00 /
Email: ilp@iasbaba.com
Page 10 200
UPSC 2024 -
Exam Title : Environment,
Sc...
rupeshkr.
Email :
09ee80@gmail.com
Contact :

Correct Answer: A
Your Answer: C Marks: -0.66/1.00
Explanation

Solution (a)

Explanation:

· The Indian Space Research Organisation (ISRO) began the new year with the successful launch
of its first X-ray Polarimeter Satellite (XPoSat). (Hence statement 1 is incorrect)

· X-ray Polarimeter Satellite (XPoSat), which will study X-ray polarisation and its cosmic sources
such as black holes and neutron stars. (Hence statement 2 is correct)

· The XPoSat is the world’s second such mission after the Imaging X-ray Polarimetry Explorer
(IXPE) launched by NASA in 2021. (Hence statement 3 is incorrect)

Important Points/Value Additions:

· It comprises two payloads, including Indian X-ray Polarimeter (POLIX) and X-ray Spectroscopy
and Timing (XSPECT). These have been built by Raman Research Institute and UR Rao Satellite
Centre, both in Bengaluru.

· This is the third time that the ISRO has used the PSLV Orbital Experimental Module (POEM)
platform to demonstrate technologies in space. One of the technologies on-board fuel cell power
systems designed by ISRO’s Vikram Sarabhai Space Centre.

QUESTION 11. MTI3Mjk4K1J1cGVzaCBLdW1hcitydXBlc2hrci4wOWVlODBAZ21haWwuY29tK1FVRVN


USU9OIDEw
“Kill Switch” term was in news recently. It is related to?

a) Nuclear Catastrophe
b) Mass Extinction
c) Cancer Treatment
d) Antimicrobial Resistance

Correct Answer: C
Your Answer: D Marks: -0.66/1.00
Explanation

Solution (c)

Explanation:

· In a major breakthrough, scientists have reportedly discovered a ‘kill switch’ that triggers the
death of cancer cells. (Hence option c is correct)

· The researchers have named it CAR T-cell therapy, which involves collecting T cells from the
patient’s blood and then genetically modifying them in a lab to produce receptors called chimeric

IASbaba
Score:
Web: http://ilp.iasbaba.com/
44.00 /
Email: ilp@iasbaba.com
Page 11 200
UPSC 2024 -
Exam Title : Environment,
Sc...
rupeshkr.
Email :
09ee80@gmail.com
Contact :

antigen receptors (CARs). After modifying them, these cells are then injected back into the
patient’s body, in the bloodstream.

Important Points/Value Additions:

· As of now, the therapy has shown promising efficacy against liquid cancer, leukaemia and other
blood cancers.

· As per scientists, it has shown “meagre success” against solid tumours such as breast, lung, and
bowel cancer.

QUESTION 12. MTI3Mjk4K1J1cGVzaCBLdW1hcitydXBlc2hrci4wOWVlODBAZ21haWwuY29tK1FVRVN


USU9OIDEx
Consider the following statements with reference to ‘Kilonova Explosions’:

1. A kilonova is a transient astronomical event that occurs in a compact binary system when only
two neutron stars merge.

2. Neutron stars are superdense astrophysical objects formed at the end of a massive star’s life in
a supernova explosion

3. Extreme conditions of these high-energy collisions lead to the formation of heavy elements like
gold.

4. It emits gravitational waves along with electromagnetic waves.

How many of the above statements are correct?

a) Only one
b) Only two
c) Only three
d) All four

Correct Answer: C
Your Answer: Unanswered Marks: 0/1.00
Explanation

Solution (c)

Explanation:

· A kilonova occurs in a compact binary system when two neutron stars or a neutron star and a
black hole merge. Not only two Neutron star but one Neutron star and one Blackhole also.
(Hence statement 1 incorrect)

· Neutron stars are superdense astrophysical objects formed at the end of a massive star’s life in a
supernova explosion. Sometimes, neutron stars orbit each other in binary systems. (Hence
statement 2 correct)

IASbaba
Score:
Web: http://ilp.iasbaba.com/
44.00 /
Email: ilp@iasbaba.com
Page 12 200
UPSC 2024 -
Exam Title : Environment,
Sc...
rupeshkr.
Email :
09ee80@gmail.com
Contact :

· Such mergers let researchers study the physical properties of the most extreme conditions in
the universe. For example, the conditions of these high-energy collisions lead to the formation of
heavy elements like gold. (Hence statement 3 correct)

· These systems constantly lose energy by emitting gravitational waves until they eventually
collide and merge. (Hence statement 4 correct)

Important Information/Value Additions:

· An international team of researchers have developed a method to model the observable signs of a
kilonova explosion from the explosive merger of two orbiting neutron stars.

· This could help them describe what exactly happens during a merger, how nuclear matter
behaves under extreme conditions and why the gold on Earth must have been created by such
cosmic events in the distant past.

· The researchers applied the new method to “multi-messenger” observations of binary neutron
star mergers.

QUESTION 13. MTI3Mjk4K1J1cGVzaCBLdW1hcitydXBlc2hrci4wOWVlODBAZ21haWwuY29tK1FVRVN


USU9OIDEy
Recently one of the space phenomena was named as “Amaterasu”. Choose the correct meaning of
this term from the options given below?

a) High energy cosmic particle


b) Solar flares surge
c) Key ingredient for life
d) Rivers of methane

Correct Answer: A
Your Answer: Unanswered Marks: 0/1.00
Explanation

Solution (a)

Explanation:

· Scientists have detected one of the most powerful cosmic rays ever slamming into Earth but they
have no idea what caused it or where it came from. The extremely high-energy particle has been
named ‘Amaterasu’ after the Japanese Sun goddess, and it seemingly arrived from a void in space
where nothing is known to exist. (Hence option a is correct)

· The Amaterasu particle has an energy exceeding 240 exa-electron volts (EeV). That is millions of
times more powerful than the particles produced by the Large Hadron Collider, which is the most
powerful accelerator ever built. It is second only to the “Oh-My-God” particle, another high-
energy cosmic ray detected in 1991. That came in at 320 EeV.

Important Points/Value Additions:

IASbaba
Score:
Web: http://ilp.iasbaba.com/
44.00 /
Email: ilp@iasbaba.com
Page 13 200
UPSC 2024 -
Exam Title : Environment,
Sc...
rupeshkr.
Email :
09ee80@gmail.com
Contact :

· Solar Flares happen when the powerful magnetic fields in and around the sun reconnect. They're
usually associated with active regions, often seen as sun spots, where the magnetic fields are
strongest. Flares are classified according to their strength. The smallest ones are B-class, followed
by C, M and X, the largest.

· Scientists have found evidence of a key ingredient for life and a powerful source of energy that
could fuel it on Saturn’s icy moon Enceladus. The giant plumes of water vapour and ice grains
thrown out by Enceladus are rich with organic compounds, some of which are important for life.

· Vid Flumina is a river of liquid methane and ethane on Saturn's moon Titan. The river has been
compared to the Nile. The surface of Titan is mostly water ice, so Vid Flumina is a river of methane
and ethane flowing across and cutting canyons into ice as though it were bedrock. NASA
scientists think that it likely has rapids, whirlpools and waterfalls, just like rivers on Earth.

QUESTION 14. MTI3Mjk4K1J1cGVzaCBLdW1hcitydXBlc2hrci4wOWVlODBAZ21haWwuY29tK1FVRVN


USU9OIDEz
Consider the following statements about ‘Chandrayan 3’:

1. It became the first spacecraft to land on the far side of the moon.

2. Chandrayan 3 landed near equator due to failure of Chandrayan 2 to land on lunar south pole.

3. Fewer hills and craters on the south pole make it easier for instruments to conduct operation.

How many of the above statements are correct?

a) Only one
b) Only two
c) All three
d) None

Correct Answer: D
Your Answer: A Marks: -0.66/1.00
Explanation

Solution (d)

Explanation:

· There is very good reason why all the landings on the moon so far have happened in the
equatorial region. Even China’s Chang’e 4, which became the first spacecraft to land on the far
side of the moon (the side that does not face the earth), landed near the 45 degree latitude. (Hen
ce statement 1 is incorrect)

· Chandrayaan-3 became world’s first mission to soft-land near the lunar south pole. All the
previous spacecraft have landed a few degrees latitude north or south of the lunar equator. (Hen
ce statement 2 is incorrect)

IASbaba
Score:
Web: http://ilp.iasbaba.com/
44.00 /
Email: ilp@iasbaba.com
Page 14 200
UPSC 2024 -
Exam Title : Environment,
Sc...
rupeshkr.
Email :
09ee80@gmail.com
Contact :

· It is easier and safer to land near the equator. The terrain and temperature are more hospitable,
and conducive for longer and sustained operation of instruments. The surface here is even and
smooth, very steep slopes are almost absent, and there are fewer hills or craters. Sunlight is
present in abundance, at least on the side facing the earth, thus offering regular supply of energy
to solar-powered instruments. (Hence statement 3 is incorrect)

Important Points/Value Additions:

· The polar regions of the Moon are a very different, and difficult, terrain. Many parts lie in a
completely dark region where sunlight never reaches, and temperatures can go below 230
degrees Celsius.

· Lack of sunlight and extremely low temperatures create difficulty in the operation of
instruments. In addition, there are large craters all over the place, ranging from a few
centimetres in size to those extending to several thousands of kilometres.

· Due to their rugged environment, the polar regions of the Moon have remained unexplored. But
several Orbiter missions have provided evidence that these regions could be very interesting to
explore.

· There are indications of the presence of ice molecules in substantial amounts in the deep craters
in this region — India’s 2008 Chandrayaan-1 mission indicated the presence of water on the lunar
surface with the help of its two instruments onboard.

QUESTION 15. MTI3Mjk4K1J1cGVzaCBLdW1hcitydXBlc2hrci4wOWVlODBAZ21haWwuY29tK1FVRVN


USU9OIDE0
James Webb Space Telescope made a breakthrough in the search for understanding how planets
are formed. In this context consider below statements:

Statement 1: According to pebble accretion theory, icy solids that usually inhibit the outer
reaches of protoplanetary discs enter close to the inner disc.

Statements 2: They release a lot of cold-water vapour which delivers both water and solids to
early planets.

Which one of the following is correct in respect of the above statements?

a) Both Statement-1 and Statement-2 are correct and Statement-2 is the correct explanation
for Statement-1.
b) Both Statement-1 and Statement-2 are correct and Statement-2 is not the correct
explanation for Statement-1.
c) Statement-1 is correct but Statement-2 is incorrect.
d) Statement-1 is incorrect but Statement-2 is correct.

Correct Answer: A
Your Answer: Unanswered Marks: 0/1.00
Explanation

Solution (a)

IASbaba
Score:
Web: http://ilp.iasbaba.com/
44.00 /
Email: ilp@iasbaba.com
Page 15 200
UPSC 2024 -
Exam Title : Environment,
Sc...
rupeshkr.
Email :
09ee80@gmail.com
Contact :

Explanation:

· Scientists used the James Webb Space Telescope to make a breakthrough in the search for
understanding how planets are formed . They observed water vapour in distant protoplanetary
discs, the precursor to a planet, confirming a long-proposed theory of how planets are formed.

· According to this pebble accretion theory, icy solids that usually inhibit the outer reaches
of “protoplanetary discs” enter the warmer region close to the inner disc. There, they
release a lot of cold-water vapou r. This process is thought to deliver both water and solids to
early planets. (Hence option a is correct)

· Based on this theory, scientists make one crucial prediction—when the icy pebbles enter the
warmer region within the discs, they should release large amounts of cold water vapour. And that
is exactly what the Webb telescope observed.

Important Points/Value Additions:

· The researchers used Webb’s MIRI (Mid-Infrared Instrument) to study four protoplanetary discs
around Sun-like stars. These stars are estimated to be between 2 and 3 million years old.

· In cosmic terms, that makes them newborns. Two of the discs were compact while the other two
were extended. The results from the telescope revealed excess cool water in the compact disc.

· The observations were made to find out if compact discs have a higher water abundance in their
inner, rocky planet region. This would be true if pebble drift is more efficient and delivers more
solid mass and water to the inner planets.

QUESTION 16. MTI3Mjk4K1J1cGVzaCBLdW1hcitydXBlc2hrci4wOWVlODBAZ21haWwuY29tK1FVRVN


USU9OIDE1
“Good Manufacturing Practices” in news recently, is related to which of the following sector?

a) Leather Industry
b) Pharmaceutical Industry
c) Plastic Industry
d) Chemical Industry

Correct Answer: B
Your Answer: B Marks: 2.00/1.00
Explanation

Solution (b)

Explanation:

· Following recent incidents of several countries reporting deaths allegedly linked to contaminated
India-manufactured drugs, the government has set a deadline for mandatory implementation of
the Good Manufacturing Practices (GMP) which were revised in 2018, bringing them on par with
World Health Organisation (WHO) standards.

IASbaba
Score:
Web: http://ilp.iasbaba.com/
44.00 /
Email: ilp@iasbaba.com
Page 16 200
UPSC 2024 -
Exam Title : Environment,
Sc...
rupeshkr.
Email :
09ee80@gmail.com
Contact :

· Companies with a turnover of over Rs 250 crore will have to implement the revised GMP within
six months, while medium and small-scale enterprises with turnover of less than Rs 250 crore will
have to implement it within a year.

Important Points/Value Additions:

· ONLY 2,000 of the 10,500 manufacturing units in the country have been found to be compliant
with the global WHO-GMP standards. Now, all will have to implement the revised GMP, ensuring
quality medicines for the domestic market and abroad.

· Currently, while companies exporting medicines to other countries already have to be WHO-GMP
certified, those manufacturing medicines for the domestic market can be granted permission if
they meet the requirements listed in Schedule M of rules under the Drugs and Cosmetics Act.
Among other things, this lists the specifications of the manufacturing units, processes that need
to be followed, and equipment needed.

QUESTION 17. MTI3Mjk4K1J1cGVzaCBLdW1hcitydXBlc2hrci4wOWVlODBAZ21haWwuY29tK1FVRVN


USU9OIDE2
Consider the following pairs:

Medicine Disease

1. Semaglutide (Wegovy) Obesity

2. Anastrozole Breast Cancer

3. Donanemab Tuberculosis

How many of the above pairs correct?

a) Only one
b) Only two
c) All three
d) None

Correct Answer: B
Your Answer: D Marks: -0.66/1.00
Explanation

Solution (b)

Explanation:

· The weight-loss drug Wegovy is being made available in the UK as part of a “controlled and
limited launch”. The drug, also known as semaglutide, will be prescribed via specialist NHS weight
management services alongside a reduced calorie diet and exercise. (Hence pair 1 is correct)

· Anastrozole, a drug that was previously used to treat breast cancer in the UK, has now been
licensed as a medicine to prevent the disease. Reportedly, the drug has been found to lessen the

IASbaba
Score:
Web: http://ilp.iasbaba.com/
44.00 /
Email: ilp@iasbaba.com
Page 17 200
UPSC 2024 -
Exam Title : Environment,
Sc...
rupeshkr.
Email :
09ee80@gmail.com
Contact :

chances of breast cancer in women by 50% in postmenopausal women. (Hence pair 2 is


correct)

· Donanemab found effective in checking cognitive decline in people with early Alzheimer’s. It is
developed by the pharmaceutical giant Eli Lilly, Donanemab was found to slow down cognitive
decline by 35% when compared with a placebo in a phase III trial. (Hence pair 3 is incorrect)

Important Points/Value Additions:

· The Union Health Ministry recently launched the new National List of Essential Medicines
(NLEM), expanding the list to include newer therapies for diabetes, such as the medicine
Teneligliptin and the insulin Glargine, and also incorporating four more anti-cancer therapies.

· Obesity puts huge pressure on the health system, using the latest drugs to support people to lose
weight will be a game-changer by helping to tackle dangerous obesity-related health conditions
such as high blood pressure, diabetes and cancer

· Donanemab is a monoclonal antibody that targets the abnormal plaques of amyloid beta protein
characteristically seen in brain images of those with Alzheimer’s. Its mechanism of action is
similar to Lecanemab, the drug developed by Japanese and American companies Eisai and Biogen
that received a fast-track approval from the FDA earlier this year.

QUESTION 18. MTI3Mjk4K1J1cGVzaCBLdW1hcitydXBlc2hrci4wOWVlODBAZ21haWwuY29tK1FVRVN


USU9OIDE3
What is the proposed origin of the continent-sized blobs called as ‘large low-velocity
provinces’(LLVPs) of unusual material under the African continent and the Pacific Ocean, as
suggested by the recent study?

a) Result of volcanic activity


b) Remnants of an ancient planet that collided with Earth
c) Formed by tectonic plate movements
d) Deposits from asteroid impacts

Correct Answer: D
Your Answer: Unanswered Marks: 0/1.00
Explanation

Solution (d)

Explanation:

· It seemed like there were two continent-sized blobs of unusual material deep near the centre of
the planet. One under the African continent and one beneath the Pacific Ocean. Each of these
“blobs” is twice the size of the Moon and is made of different proportions of elements than the
mantle that surrounds them. The blobs are called large low-velocity provinces (LLVPs).

· A study published in the journal Nature suggests that these blobs are what remains of the
ancient planet that collided with the Earth billions of years ago, creating the Moon.

IASbaba
Score:
Web: http://ilp.iasbaba.com/
44.00 /
Email: ilp@iasbaba.com
Page 18 200
UPSC 2024 -
Exam Title : Environment,
Sc...
rupeshkr.
Email :
09ee80@gmail.com
Contact :

· The study also answers another mystery connected with the “giant impact theory” of how the
Moon was created. The smaller planet that collided with the Earth was dubbed Theia but no trace
of it has been found in the asteroid belt or in meteorites. The researchers believe that most of
Theia was absorbed by early Earth, forming these “LLVPs,” with the residual debris from
the impact becoming the Moon.

Important Points/Value Additions:

· The LLVPs were first discovered when scientists measured seismic waves travelling through the
Earth. These waves travel at different speeds through different materials. In the 1980s, studying
these seismic waves hinted at large-scale three-dimensional variations deep inside Earth.

· There
seemed to be two structures near the Earth’s core that had an unusually high level of iron.

QUESTION 19. MTI3Mjk4K1J1cGVzaCBLdW1hcitydXBlc2hrci4wOWVlODBAZ21haWwuY29tK1FVRVN


USU9OIDE4
With reference to the use of Fast Radio Bursts (FRBs) for measuring "missing" matter between
galaxies, consider the following statements:

1. FRBs can be employed to measure the "missing" matter between galaxies, providing a new
method to determine the mass of the Universe.

2. More than half of the normal matter in the Universe is missing, believed to be located in the
space between galaxies, possibly too hot and diffuse to be observed using conventional
techniques.

3. FRBs can detect ionized material in the almost empty space between galaxies, allowing
scientists to measure the amount of matter present.

How many of the above statements are correct?

a) Only one
b) Only two
c) All three
d) None

Correct Answer: C
Your Answer: C Marks: 2.00/1.00
Explanation

Solution (c)

Explanation:

· FRBs (fast radio bursts) can be used to measure “missing” matter between galaxies and can
provide a new way to weigh the Universe, according to the European Southern Observatory.
(ESO) (Hence statement 1 is correct)

IASbaba
Score:
Web: http://ilp.iasbaba.com/
44.00 /
Email: ilp@iasbaba.com
Page 19 200
UPSC 2024 -
Exam Title : Environment,
Sc...
rupeshkr.
Email :
09ee80@gmail.com
Contact :

· Current methods of estimating the universe’s mass give conflicting answers and challenge the
standard model of cosmology.

· We think that the missing matter is hiding in the space between galaxies, but it may just be so
hot and diffuse that it’s impossible to see using normal techniques. (Hence statement 2 is
correct)

· FRBs sense this ionised material. Even in space that is almost completely empty, these bursts can
see all the electrons. This allows scientists to measure how much stuff there is between galaxies. (
Hence statement 3 is correct)

Important Points/Value Additions:

· Researchers discovered a remote blast of cosmic radio waves that lasted less than a millisecond.
This fast radio burst is the most distant ever detected and its source is in a galaxy so far away that
it took eight billion years to reach us.

· It is also one of the most energetic ones ever observed. In a fraction of a second, it released the
equivalent of 30 years of emissions from our Sun.

· While we still don’t know what causes these massive bursts of energy, the paper confirms that
fast radio bursts are common events in the cosmos and that we will be able to use them to detect
matter between galaxies, and better understand the structure of the Universe

QUESTION 20. MTI3Mjk4K1J1cGVzaCBLdW1hcitydXBlc2hrci4wOWVlODBAZ21haWwuY29tK1FVRVN


USU9OIDE5
Consider the following pairs:

Asteroids Mission

1. Bennu OSIRIS-REx

2. Psyche OSIRIS-APEX

3. Dinkinesh Lucy

How many of the above pairs are correct?

a) Only one
b) Only two
c) All three
d) None

Correct Answer: B
Your Answer: B Marks: 2.00/1.00
Explanation

Solution (b)

Explanation:

IASbaba
Score:
Web: http://ilp.iasbaba.com/
44.00 /
Email: ilp@iasbaba.com
Page 20 200
UPSC 2024 -
Exam Title : Environment,
Sc...
rupeshkr.
Email :
09ee80@gmail.com
Contact :

· OSIRIS-REx is the first U.S. mission to collect a sample from an asteroid. It returned to Earth to
drop off material from asteroid Bennu. (Hence pair 1 correct)

· Mission to asteroid Psyche is named as same Psyche mission. OSIRIS-APEX, to explore asteroid
Apophis. (Hence pair 2 incorrect)

· NASA’s Lucy spacecraft flew by not just its first asteroid, but its first two. The first images
returned by Lucy reveal that the small main belt asteroid Dinkinesh is actually a binary pair. (He
nce pair 3 correct)

Important Points/Value Additions:

· OSIRIS-APEX, a follow-on to OSIRIS-REx, will study the physical changes to asteroid Apophis
after the asteroid’s rare close encounter with Earth in 2029.

· Lucy is a NASA space probe on a twelve-year journey to eight different asteroids. It is slated to
visit two main belt asteroids as well as six Jupiter trojans – asteroids that share Jupiter's orbit
around the Sun, orbiting either ahead of or behind the planet.

QUESTION 21. MTI3Mjk4K1J1cGVzaCBLdW1hcitydXBlc2hrci4wOWVlODBAZ21haWwuY29tK1FVRVN


USU9OIDIw
Consider the following statements in context of India's capabilities in radio astronomy and the
Square Kilometer Array (SKA):

1. The Square Kilometer Array will be a single large telescope consisting of thousands of dish
antennas operating individually.

2. None of the SKA facilities would be located in India.

3. India has highly developed capabilities in radio astronomy, with advanced facilities like the
Giant Meterwave Radio Telescope (GMRT) near Pune.

How many of the above statements are correct?

a) Only one
b) Only two
c) All three
d) None

Correct Answer: B
Your Answer: B Marks: 2.00/1.00
Explanation

Solution (b)

Explanation:

· The Square Kilometer Array will not be a single large telescope, but a collection of thousands of
dish antennas operating as a single unit. (Hence statement 1 is incorrect)

IASbaba
Score:
Web: http://ilp.iasbaba.com/
44.00 /
Email: ilp@iasbaba.com
Page 21 200
UPSC 2024 -
Exam Title : Environment,
Sc...
rupeshkr.
Email :
09ee80@gmail.com
Contact :

· None of the SKA facilities would be located in India. (Hence statement 2 is correct)

· Radio astronomy is something in which India already has highly developed capabilities. The Giant
Meterwave Radio Telescope (GMRT) near Pune is one of the most advanced and sought after
facilities in the world, which has been producing remarkable scientific results. There are other
similar facilities in Ooty, Nainital and Bengaluru. (Hence statement 3 is correct)

Important Points/Value Additions:

· India had decided to formally join the Square Kilometer Array (SKA) project, an international
scientific collaboration working to build the world’s largest radio telescope.

· India had already been contributing to the project for the past several years, but the full member
status, which offers greater scientific opportunities to use the upcoming facility, requires
countries to sign and ratify an international treaty, and also make a financial commitment.

· A full member status would provide India preferential access to the SKA facilities.

· The Indian participation in the project is being led by Pune-based National Centre for Radio
Astrophysics (NCRA).

QUESTION 22. MTI3Mjk4K1J1cGVzaCBLdW1hcitydXBlc2hrci4wOWVlODBAZ21haWwuY29tK1FVRVN


USU9OIDIx
In the context of India's Arctic Research Station and scientific research in the Arctic, Consider
following statements:

1. Himadri, India’s Arctic Research Station is at Ny-Ålesund in the Norwegian archipelago of


Svalbard.

2. Scientific research in the Arctic region is governed by international legal instruments like the
Svalbard Treaty of 1920 and the UN Convention on the Law of the Seas.

3. India’s two other stations in Arctic region are Maitri and Bharti apart from Himadri.

4. National Centre for Polar and Ocean Research, Goa, is the nodal agency for India’s polar
explorations.

How many of the above statements are correct?

a) Only one
b) Only two
c) Only three
d) All four

Correct Answer: C
Your Answer: C Marks: 2.00/1.00
Explanation

Solution (c)

IASbaba
Score:
Web: http://ilp.iasbaba.com/
44.00 /
Email: ilp@iasbaba.com
Page 22 200
UPSC 2024 -
Exam Title : Environment,
Sc...
rupeshkr.
Email :
09ee80@gmail.com
Contact :

Explanation:

· Himadri, India’s Arctic Research Station at Ny-Ålesund in the Norwegian archipelago of


Svalbard in the Arctic Ocean, will now remain operational throughout the year. (Hence
statement 1 is correct)

· Scientific research in the Arctic region is governed by international legal instruments like the
Svalbard Treaty of 1920 and the UN Convention on the Law of the Seas, apart from the individual
jurisdictions of the Arctic countries. (Hence statement 2 is correct)

· India’s has two stations in use, Maitri and Bharti in Antarctica. (Hence statement 3 is
incorrect)

· National Centre for Polar and Ocean Research, Goa, the nodal agency for India’s polar
explorations. (Hence statement 4 is correct)

Important Points/Value Additions:

· India signed the Svalbard Treaty in Paris in 1920. But it took until 2007 for the first Indian
expedition comprising five scientists to visit the International Arctic Research Facilities at Ny-
Ålesund with the purpose of initiating studies in Arctic microbiology, atmospheric sciences, and
geology.

· India’s permanent research station, Himadri, began operations in July 2008.

· Himadri has been equipped for observations during polar nights (that last longer than 24 hours),
and supplied with special winter gear, transport, and logistics support from Norwegian agencies.

QUESTION 23. MTI3Mjk4K1J1cGVzaCBLdW1hcitydXBlc2hrci4wOWVlODBAZ21haWwuY29tK1FVRVN


USU9OIDIy
Consider the following statements in context of discovery of ‘Tantalum’ metal, recently discovered
in Sutlej basin:

Statement 1: Tantalum is one of the most corrosion prone metal.

Statement 2: It forms an oxide layer when exposed to air, extremely difficult to remove, even
when it interacts with strong and hot acid environments.

Which one of the following is correct in respect of the above statements?

a) Both Statement-1 and Statement-2 are correct and Statement-2 is the correct explanation
for Statement-1.
b) Both Statement-1 and Statement-2 are correct and Statement-2 is not the correct
explanation for Statement-1.
c) Statement-1 is correct but Statement-2 is incorrect.
d) Statement-1 is incorrect but Statement-2 is correct.

Correct Answer: D
Your Answer: Unanswered Marks: 0/1.00

IASbaba
Score:
Web: http://ilp.iasbaba.com/
44.00 /
Email: ilp@iasbaba.com
Page 23 200
UPSC 2024 -
Exam Title : Environment,
Sc...
rupeshkr.
Email :
09ee80@gmail.com
Contact :

Explanation

Solution (d)

Explanation:

· It possesses high corrosion resistance because when exposed to air, it forms an oxide layer that is
extremely difficult to remove, even when it interacts with strong and hot acid environments. (He
nce, Statement 1 is incorrect but statement 2 is correct)

· Tantalum is a rare metal with the atomic number 73 — the number of protons found in one atom
of the element. It’s grey, heavy, very hard, and one of the most corrosion-resistant metals in use
today.

· When pure, tantalum is ductile, meaning it can be stretched, pulled, or drawn into a thin wire or
thread without breaking. Moreover, it “is almost completely immune to chemical attack at
temperatures below 150°C, and is attacked only by hydrofluoric acid, acidic solutions containing
the fluoride ion, and free sulphur trioxide,” according to the US Department of Energy.

· Notably, tantalum also has an extremely high melting point, exceeded only by tungsten and
rhenium.

Important Points/Value Additions:

· A team of researchers from the Indian Institute of Technology (IIT), Ropar has found the
presence of tantalum, a rare metal, in the Sutlej River sand in Punjab.

· Tantalum is most prominently used in the electronics sector. The capacitors made from tantalum
are capable of storing more electricity in smaller sizes without much leakage than any other type
of capacitor. This makes them ideal for use in portable electronic devices such as smartphones,
laptops, and digital cameras.

· As tantalum has a high melting point, it is frequently used as a substitute for platinum, which is
more expensive. The rare metal is also used to make components for chemical plants, nuclear
power plants, aeroplanes and missiles.

· Tantalum does not react with bodily fluids and is used to make surgical equipment and implants,
like artificial joints, according to the US Department of Energy.

· A composite consisting of tantalum carbide and graphite is one of the hardest materials known
and is used on the cutting edges of high-speed machine tools.

QUESTION 24. MTI3Mjk4K1J1cGVzaCBLdW1hcitydXBlc2hrci4wOWVlODBAZ21haWwuY29tK1FVRVN


USU9OIDIz
Consider the following statements about ‘Atmospheric Wave Experiment’:

1. AWE is a first of its kind experimental attempt aimed at studying the interactions between
terrestrial and space weather.

2. AWE will be launched and mounted on the exterior of the Earth-orbiting International Space
Station.

IASbaba
Score:
Web: http://ilp.iasbaba.com/
44.00 /
Email: ilp@iasbaba.com
Page 24 200
UPSC 2024 -
Exam Title : Environment,
Sc...
rupeshkr.
Email :
09ee80@gmail.com
Contact :

3. AWE will perform focused mapping of the colourful airglows in the Earth’s atmosphere.

How many of the above statements are correct?

a) Only one
b) Only two
c) All three
d) None

Correct Answer: C
Your Answer: Unanswered Marks: 0/1.00
Explanation

Solution (c)

Explanation:

· AWE is a first-of-its-kind experimental attempt aimed at studying the interactions between


terrestrial and Space weather. Mission will study the links between how waves in the lower layers
of the atmosphere impact the upper atmosphere, and thus, Space weather. (Hence, statement 1
is correct)

· AWE will be launched and mounted on the exterior of the Earth-orbiting International Space
Station (ISS). From the vantage point, it will look down at the Earth and record the colourful light
bands, commonly known as airglow. (Hence, statement 2 is correct)

· AWE will perform focused mapping of the colourful airglows in the Earth’s atmosphere. (Hence,
statement 3 is correct)

Important Points/Value Additions:

· AWE could open a new window of study, wherein scientists are attempting to understand if
Space weather is affected by terrestrial and bottom-up forces.

· AWE will be able to resolve waves at finer horizontal scales than what satellites can usually see at
those altitudes, which is part of what makes the mission unique.

· AWE will measure the airglow at mesopause (about 85 to 87 km above the Earth’s surface),
where the atmospheric temperatures dip to minus 100 degrees Celsius. At this altitude, it is
possible to capture the faint airglow in the infrared bandwidth, which appears the brightest
enabling easy detection.

QUESTION 25. MTI3Mjk4K1J1cGVzaCBLdW1hcitydXBlc2hrci4wOWVlODBAZ21haWwuY29tK1FVRVN


USU9OIDI0
Arrange the following steps in ascending order to follow De-extinction process:

1. Identifying Primordial Germ Cells (PGCs)

2. Developing Interspecies Surrogates

IASbaba
Score:
Web: http://ilp.iasbaba.com/
44.00 /
Email: ilp@iasbaba.com
Page 25 200
UPSC 2024 -
Exam Title : Environment,
Sc...
rupeshkr.
Email :
09ee80@gmail.com
Contact :

3. Performing Genetic comparison with closest relative

4. Complete Genome Sequencing

Choose the correct code from below options.

a) 1, 2, 3, 4
b) 2, 3, 4, 1
c) 3, 4, 1, 2
d) 4, 3, 1, 2

Correct Answer: D
Your Answer: Unanswered Marks: 0/1.00
Explanation

Solution (d)

Explanation:

· To De-extinct a species, the first thing required is accurate and complete genetic information.
This is known as a species genome ; each genome contains all of the information needed to build
that organism and allow it to grow and develop.

· Dodo’s entire genome is being compared to the genome of the Rodrigues solitaire , the
dodo’s closest (also extinct) relative to identify just what makes a dodo, a dodo.

· Nicobar pigeon, the dodo’s closest extant relative, and f ound its primordial germ cells
(PGCs). PGCs are basically embryonic precursors of a species sperm and egg.

· In theory, when the chicken and rooster reproduce (Interspecies surrogates ), they will
give birth to a dodo offspring.

Important Points/Value Additions:

· Collaboration between genetic engineering company Colossal Biosciences and the Mauritian
Wildlife Foundation promises to not just bring the dodo back to life, but also re-introduce it in its
once-native habitat in Mauritius.

· It will then take years of selective breeding to enhance a small pigeon into a large flightless bird.
Remember, nature took millions of years for this to happen with the dodo

· Restoring the dodo gives us the opportunity to create ‘conservation optimism,’ that hopefully
inspires people around the globe.

QUESTION 26. MTI3Mjk4K1J1cGVzaCBLdW1hcitydXBlc2hrci4wOWVlODBAZ21haWwuY29tK1FVRVN


USU9OIDI1
Consider the following statements with reference to ‘Combined Charging standards’ for electric
vehicles:

IASbaba
Score:
Web: http://ilp.iasbaba.com/
44.00 /
Email: ilp@iasbaba.com
Page 26 200
UPSC 2024 -
Exam Title : Environment,
Sc...
rupeshkr.
Email :
09ee80@gmail.com
Contact :

1. Bureau of Indian Standards (BIS) has approved an indigenously developed AC and DC


combined charging connector standards.

2. The indigenously developed charging standard is the first in the world that combines
alternating current (AC) and direct current (DC) for Light Electric Vehicles as well as electric
four-wheelers.

3. A combined charging standard is attractive because of its interoperability, can be used by


different kinds of EV models and charging infrastructure providers.

4. It mandates EV makers to use a uniform standard across range.

How many of the Statements are correct?

a) Only one
b) Only two
c) Only three
d) All four

Correct Answer: B
Your Answer: Unanswered Marks: 0/1.00
Explanation

Solution (b)

Explanation:

· The Bureau of Indian Standards (BIS) has approved an indigenously developed AC and DC
combined charging connector standard for light electric vehicles (LEVs) such as scooters, bikes,
and rickshaws. (Hence statement 1 is correct)

· The indigenously developed charging standard is the first in the world that combines alternating
current (AC) and direct current (DC) for LEVs . Combined AC and DC charging standards
for electric four-wheelers are already in use around the world , like the Combined
Charging System (CCS) standard that is widely used in Europe. (Hence statement 2 is
incorrect)

· A combined charging standard is attractive because of its interoperability which means that it
can be used by different kinds of EV models and charging infrastructure providers. (Hence
statement 3 is correct)

· Although the new approved standard fixes the problem of different standards for AC and DC
charging by creating a combined standard, it does not mandate EV makers to use a uniform
standard that can help address range anxiety and promote faster adoption of EVs. (Hence
statement 4 is incorrect)

Important Points/Value Additions:

· The new standard, ISI7017 (Part 2 / Sec 7): 2023, cleared by BIS has been developed through
collaboration among NITI Aayog, the Department of Science and Technology, electric two-wheeler
maker Ather Energy, and various other government and industry stakeholders.

IASbaba
Score:
Web: http://ilp.iasbaba.com/
44.00 /
Email: ilp@iasbaba.com
Page 27 200
UPSC 2024 -
Exam Title : Environment,
Sc...
rupeshkr.
Email :
09ee80@gmail.com
Contact :

· The US does not have a national standard, but EV makers have been collaborating to push for
some degree of standardisation.

· In Europe, CCS is the predominant charging connector standard, and is also a European Union
(EU) requirement for EV charging networks.

QUESTION 27. MTI3Mjk4K1J1cGVzaCBLdW1hcitydXBlc2hrci4wOWVlODBAZ21haWwuY29tK1FVRVN


USU9OIDI2
Which of the following statements best describes ‘Skywave communication’?

a) It uses low frequency radio signals.


b) It relies on the reflection of radio waves off the ionosphere for long-distance communication.
c) Skywave communication primarily utilizes fiber-optic technology for data transmission.
d) It is based on direct line-of-sight for signal propagation.

Correct Answer: B
Your Answer: B Marks: 2.00/1.00
Explanation

Solution (b)

Explanation:

· Radio waves of certain high frequencies are reflected back to the ground by ionosphere which
facilitates long distance high frequency communication beyond the horizon, known as Skywave
Communications. (Hence option a is incorrect)

· Skywave communication utilizes the ionosphere, a layer within the Earth's atmosphere, to reflect
radio waves. By transmitting radio waves into the ionosphere at a specific angle, they undergo
refraction and are redirected back towards the Earth's surface. (Hence option b is correct)

· Skywave communication uses radio frequency signals for data transmission. (Hence option c is
incorrect)

· This unique phenomenon facilitates extended-distance communication, proving particularly


advantageous for over-the-horizon and international communication. (Hence option d is
incorrect)

Important Points/Value Additions:

· The ionosphere becomes ionized through solar radiation, consisting of charged particles that
interact with radio waves. The reflective property of the ionosphere allows radio waves to bounce
off, enabling communication across much greater distances compared to ground-based, line-of-
sight methods.

QUESTION 28. MTI3Mjk4K1J1cGVzaCBLdW1hcitydXBlc2hrci4wOWVlODBAZ21haWwuY29tK1FVRVN


USU9OIDI3

IASbaba
Score:
Web: http://ilp.iasbaba.com/
44.00 /
Email: ilp@iasbaba.com
Page 28 200
UPSC 2024 -
Exam Title : Environment,
Sc...
rupeshkr.
Email :
09ee80@gmail.com
Contact :

What is the primary mechanism of action for Abaucin, a compound with narrow-spectrum
antibiotic properties effective against Acinetobacter baumannii?

a) Inhibiting DNA replication


b) Disrupting the function of the CCR2 protein
c) Blocking cell wall synthesis
d) Interfering with protein synthesis

Correct Answer: B
Your Answer: D Marks: -0.66/1.00
Explanation

Solution (b)

Explanation:

· Abaucin is a compound that shows useful activity as a narrow-spectrum antibiotic. It has been
found to be effective against Acinetobacter baumannii.

· It disrupts the normal function of the CCR2 protein in bacteria. This disruption hinders the
movement of certain molecules inside the bacteria, preventing them from reaching the outer
membrane. As a result, the growth of Acinetobacter baumannii is inhibited, reducing its ability to
cause infections. (Hence Option b is correct)

Important Points/Value Additions:

· Abaucin was identified through the support of artificial intelligence using a machine-learning
model strategy. Initially, a neural network system was trained on a dataset consisting of 7,500
molecules screened for their ability to inhibit the growth of Acinetobacter baumannii.

· Out of the 240 molecules suggested by the neural network, 9 demonstrated a significant
inhibition of the target bacteria's growth by more than 80%. To refine the selection, scientists
excluded molecules that the bacteria might have been accustomed to, ultimately isolating Abaucin
as the sole remaining compound.

QUESTION 29. MTI3Mjk4K1J1cGVzaCBLdW1hcitydXBlc2hrci4wOWVlODBAZ21haWwuY29tK1FVRVN


USU9OIDI4
Consider the following statements:

Statement 1: Antibiotics are effective across broad spectrum of infections, making them easy to
administer.

Statement 2: Bacteriophage has capacity to attack specific pathogen and hence less chances of
developing resistance.

Which one of the following is correct in respect of the above statements?

a) Both Statement-1 and Statement-2 are correct and Statement-2 is the correct explanation
for Statement-1.

IASbaba
Score:
Web: http://ilp.iasbaba.com/
44.00 /
Email: ilp@iasbaba.com
Page 29 200
UPSC 2024 -
Exam Title : Environment,
Sc...
rupeshkr.
Email :
09ee80@gmail.com
Contact :

b) Both Statement-1 and Statement-2 are correct and Statement-2 is not the correct
explanation for Statement-1.
c) Statement-1 is correct but Statement-2 is incorrect.
d) Statement-1 is incorrect but Statement-2 is correct.

Correct Answer: B
Your Answer: B Marks: 2.00/1.00
Explanation

Solution (b)

Explanation:

· A bacteriophage is a type of virus specialized in infecting bacteria, earning its name "bacteria
eater" due to its capability to destroy host cells.

· Phage Therapy, a treatment approach, involves administering specific bacteriophage viruses to


individuals with identified bacterial diseases. Successful against various diseases, from bubonic
plague to cholera and dysentery, Phage Therapy is known for its specificity.

· In contrast to antibiotics, which have a broad spectrum of action, Phage Therapy is limited in its
application, making it essential to identify the exact pathogen before administering treatment.

Important Points/Value Additions:

· Despite limited research on Phage Therapy, its specificity reduces the likelihood of resistance
development, enhancing effectiveness and minimizing side effects.

· With advancements in genetic sequencing, the identification of disease-causing pathogens and


the harvesting of phages from successfully treated patients hold the potential to further optimize
Phage Therapy.

QUESTION 30. MTI3Mjk4K1J1cGVzaCBLdW1hcitydXBlc2hrci4wOWVlODBAZ21haWwuY29tK1FVRVN


USU9OIDI5
Consider the following statements in the context of ‘Nanoparticles’:

1. The deviant behaviour of small nanoparticles arises because of the emergence of quantum
effects.

2. Nanoparticles are much smaller compared with atoms depending on the size, atom can pack in
thousands to millions of nanoparticles.

Which of the above statements correct?

a) Only one
b) Only two
c) Both
d) Neither

Correct Answer: A

IASbaba
Score:
Web: http://ilp.iasbaba.com/
44.00 /
Email: ilp@iasbaba.com
Page 30 200
UPSC 2024 -
Exam Title : Environment,
Sc...
rupeshkr.
Email :
09ee80@gmail.com
Contact :

Your Answer: A Marks: 2.00/1.00


Explanation

Solution (a)

Explanation:

· Every element exhibits specific properties, usually determined by the number of electrons in its
atoms and the distribution of these electrons around the nucleus. Also, every piece of a pure
element exhibits exactly the same properties, regardless of its size. This is one of the fundamental
facts of chemistry.

· The deviant behaviour of small nanoparticles arises because of the emergence of quantum
effects. The motion and behaviour of very small particles, like electrons, are radically different,
and strange, when compared with any familiar object in normal human experience. Such strange
behaviour at the sub-atomic level is described by the hugely successful Quantum Theory,
developed by physicists 100 years ago. (Hence statement 1 is correct)

· But nanoparticles are much larger compared with atoms. Depending on the size of the
atom, a nanoparticle can pack in thousands to millions of atoms . (Hence statement 2 is
incorrect)

· Nano-sized particles that behaved slightly differently than larger particles of the same element.
These nanoparticles with special properties were called quantum dots

Important Points/Value Additions:

· Research in Nanotechnology awarded the 2023 Nobel Prize in Chemistry. Nanoparticles with
desired deviant behaviour have become an integral part of a variety of modern appliances,
including television, computer screens, and LED lamps. There are a wide range of applications in
biochemistry and medicine as well.

· One of the most noticeable special properties of the nanoparticles becomes evident when they
interact with light. The colour of any material depends on the wavelengths of the light spectrum
absorbed or reflected by the material. Light wavelength absorbed differed

In different sizes of same element.

QUESTION 31. MTI3Mjk4K1J1cGVzaCBLdW1hcitydXBlc2hrci4wOWVlODBAZ21haWwuY29tK1FVRVN


USU9OIDMw
‘Specific Absorption Rate’ terminology was in discussion recently. Which of the following option
describes it correctly?

a) Radiation energy absorbed by human body.


b) Solar insolation absorbed by earth.
c) CO2 absorption rate of plant at different temperatures.
d) Heat absorbed by oceans in a year.

Correct Answer: A
Your Answer: B Marks: -0.66/1.00

IASbaba
Score:
Web: http://ilp.iasbaba.com/
44.00 /
Email: ilp@iasbaba.com
Page 31 200
UPSC 2024 -
Exam Title : Environment,
Sc...
rupeshkr.
Email :
09ee80@gmail.com
Contact :

Explanation

Solution (a)

Explanation:

· France's Agence Nationale des Fréquences (ANFR) asked Apple to halt iPhone12 sales in France
after tests that it said showed the phone's Specific Absorption Rate (SAR)- a gauge of the rate
of radiofrequency energy absorbed by the body from a piece of equipment - was higher than
legally allowed. (Hence option a is correct)

· The limits - based on the risk of burns or heatstroke from the phone's radiation - are already set
ten times below the level where scientists found evidence of harm.

· The European standard is a specific absorption rate of 4.0 watts per kilogram.

Important Points/Value Additions:

· The WHO and other international health bodies say there is no definitive evidence that radiation
from mobile phones causes other adverse health effects. However, it has called for more research.

· In 2011, the International Agency for Research on Cancer (IARC) classed the radiation from
mobile phones as “possibly carcinogenic”, or class 2B.

QUESTION 32. MTI3Mjk4K1J1cGVzaCBLdW1hcitydXBlc2hrci4wOWVlODBAZ21haWwuY29tK1FVRVN


USU9OIDMx
Consider the following statements in context of nuclear energy:

Statement 1: In some atoms are radioactive they tend to break apart into two lighter elements is
the basis of nuclear energy.

Statement 2: In some atom’s composition of the number of protons and neutrons is such that it
does not allow nucleus to hold itself.

Which one of the following is correct in respect of the above statements?

a) Both Statement-1 and Statement-2 are correct and Statement-2 is the correct explanation
for Statement-1.
b) Both Statement-1 and Statement-2 are correct and Statement-2 is not the correct
explanation for Statement-1.
c) Statement-1 is correct but Statement-2 is incorrect.
d) Statement-1 is incorrect but Statement-2 is correct.

Correct Answer: A
Your Answer: C Marks: -0.66/1.00
Explanation

Solution (a)

Explanation:

IASbaba
Score:
Web: http://ilp.iasbaba.com/
44.00 /
Email: ilp@iasbaba.com
Page 32 200
UPSC 2024 -
Exam Title : Environment,
Sc...
rupeshkr.
Email :
09ee80@gmail.com
Contact :

· Most atoms on Earth are stable due to an equilibrated composition of neutrons and protons in
their nucleus. However, in some unstable atoms, the composition of the number of protons and
neutrons is such that it does not allow the nucleus to hold itself together.

· Such atoms are known to be radioactive, and they tend to break apart or fission into two lighter
elements. This is the basis of most nuclear weapons and atomic energy. (Hence option a is
correct)

Important Points/Value Additions:

· Uranium-235, an extremely rare isotope of the heavy metal uranium, is the most commonly used
nuclear fuel, as it is one of the few elements that can undergo induced fission. This means that the
element can be broken down very quickly by a process put into motion by humans.

· This is done by subjecting a U-235 nucleus to neutrons. The nucleus immediately absorbs an
extra neutron and consequently becomes unstable — and immediately breaks apart into two
lighter atoms, and a few extra neutrons. This process releases what is known as atomic energy.

QUESTION 33. MTI3Mjk4K1J1cGVzaCBLdW1hcitydXBlc2hrci4wOWVlODBAZ21haWwuY29tK1FVRVN


USU9OIDMy
Consider the following statements with reference to ‘Ambergris’:

1. Ambergris is a waxy substance found in the digestive system of sperm whales

2. Ambergris
is primarily used in the production of perfumes due to its rare and expensive nature.

3. Sperm whales are a protected species under Schedule 1 of the Wildlife Protection Act

4. Possession or trade of Ambergris is illegal in India.

How many of the statements are correct?

a) Only one
b) Only two
c) Only three
d) All four

Correct Answer: C
Your Answer: Unanswered Marks: 0/1.00
Explanation

Solution (c)

Explanation:

· Ambergris is a waxy substance that originates from the digestive system of protected sperm
whales. One of the theories about its formation suggests that it is produced in the gastrointestinal
tract of some sperm whales. (Hence statement 1 is correct)

IASbaba
Score:
Web: http://ilp.iasbaba.com/
44.00 /
Email: ilp@iasbaba.com
Page 33 200
UPSC 2024 -
Exam Title : Environment,
Sc...
rupeshkr.
Email :
09ee80@gmail.com
Contact :

· Ambergris is a rare substance, which contributes to its high demand and high price in the
international market. Traditionally, it is used to produce perfumes which have notes of musk.
While there are records of it being used to flavour food, alcoholic beverages and tobacco in some
cultures in the past, it is rarely used for these purposes presently. (Hence statement 2 is
correct)

· Sperm whales are a protected species under Schedule 2 of the Wildlife Protection Act. (Hence
statement 3 is incorrect)

· Possession or trade of any of its by-products, including Ambergris and its byproducts, is illegal
under provisions of the Wildlife Protection Act, 1972. (Hence statement 4 is correct)

Important Points/Value Additions:

· Days after the carcass of a sperm whale washed up on a shore of the Canary Island of La Palma,
found ambergris, also known as ‘floating gold’.

· It has been observed that the gangs smuggling the ambergris procure it from coastal areas and
ship it to destination countries via some other countries with whom India has comparatively less
stringent sea trade.

QUESTION 34. MTI3Mjk4K1J1cGVzaCBLdW1hcitydXBlc2hrci4wOWVlODBAZ21haWwuY29tK1FVRVN


USU9OIDMz
Consider the following statements with reference to Aditya-L1 solar mission:

1. Lagrange point ensures that Aditya-L1 can maintain a constant, uninterrupted view of the Sun.

2. L1 Lagrange point location allows the satellite to observe solar radiation and magnetic storms
in the presence of Earth's magnetic field and atmosphere.

3. L1 Lagrange point location provides gravitational stability minimizes the need for frequent
orbital maintenance efforts.

How many of the above statements correct?

a) Only one
b) Only two
c) All three
d) None

Correct Answer: B
Your Answer: C Marks: -0.66/1.00
Explanation

Solution (b)

Explanation:

IASbaba
Score:
Web: http://ilp.iasbaba.com/
44.00 /
Email: ilp@iasbaba.com
Page 34 200
UPSC 2024 -
Exam Title : Environment,
Sc...
rupeshkr.
Email :
09ee80@gmail.com
Contact :

· The strategic placement at the L1 Lagrange point ensures that Aditya-L1 can maintain a
constant, uninterrupted view of the Sun. (Hence statement 1 is correct)

· This location also allows the satellite to access solar radiation and magnetic storms before
they are influenced by Earth's magnetic field and atmosphere . (Hence statement 2 is
incorrect)

· Additionally, the L1 point’s gravitational stability minimizes the need for frequent orbital
maintenance efforts, optimizing the satellite's operational efficiency. (Hence statement 3 is
correct)

Important Points/Value Additions:

· Aditya L1 shall be the first space based Indian mission to study the Sun. The spacecraft shall be
placed in a halo orbit around the Lagrange point 1 (L1) of the Sun-Earth system, which is about
1.5 million km from the Earth.

· A satellite placed in the halo orbit around the L1 point has the major advantage of continuously
viewing the Sun without any occultation/eclipses. This will provide a greater advantage of
observing the solar activities and its effect on space weather in real time.

· The spacecraft carries seven payloads to observe the photosphere, chromosphere and the
outermost layers of the Sun (the corona) using electromagnetic and particle and magnetic field
detectors.

· Using the special vantage point L1, four payloads directly view the Sun and the remaining three
payloads carry out in-situ studies of particles and fields at the Lagrange point L1, thus providing
important scientific studies of the propagatory effect of solar dynamics in the interplanetary
medium

· The suits of Aditya L1 payloads are expected to provide most crucial information to understand
the problem of coronal heating, coronal mass ejection, pre-flare and flare activities and their
characteristics, dynamics of space weather, propagation of particle and fields etc.

QUESTION 35. MTI3Mjk4K1J1cGVzaCBLdW1hcitydXBlc2hrci4wOWVlODBAZ21haWwuY29tK1FVRVN


USU9OIDM0
Consider the following statements in context of discovery of organic molecule in space:

1. The CH3+ also known as methyl cation, has been identified in space for the first time in Orion
Nebula.

2. CH3+ is a simple organic molecule which reacts with other molecules to form more complex
compounds.

3. The James Webb Space Telescope, despite utilizing spectroscopy failed to detect the fingerprint
of CH3+.

How many of the above statements are correct?

a) Only one

IASbaba
Score:
Web: http://ilp.iasbaba.com/
44.00 /
Email: ilp@iasbaba.com
Page 35 200
UPSC 2024 -
Exam Title : Environment,
Sc...
rupeshkr.
Email :
09ee80@gmail.com
Contact :

b) Only two
c) All three
d) None

Correct Answer: B
Your Answer: B Marks: 2.00/1.00
Explanation

Solution (b)

Explanation:

· The CH3+ also known as methyl cation, has been identified in space for the first time. (Hence
statement 1 is correct)

· Organic molecules, including the simple yet reactive CH3+, typically consist of carbon atoms
bonded to hydrogen and other elements like oxygen, nitrogen, or phosphorus. CH3+ serves as a
catalyst for intricate reactions, hinting at the presence of fundamental components necessary for
life beyond Earth. (Hence statement 2 is correct)

· The breakthrough discovery of CH3+ fingerprints was accomplished through the precise
technique of spectroscopy, showcasing the unique light emissions from molecules in the Orion
Nebula by James Webb Space Telescope. (Hence statement 3 is incorrect)

Important Points/Value Additions:

· The James Webb Space Telescope (JWST) played a pivotal role in identifying the CH3+ molecule
in space, with preliminary results published in the journal Nature.

· The recent detection of the CH3+ molecule marks a milestone in our quest to understand the
origins of life from non-living matter, emphasizing the carbon-based nature of life.

· The molecule's fingerprints were discerned in light emanating from a swirling disk of dust and
gas around a young star in the Orion Nebula, located 1,350 light years away.

QUESTION 36. MTI3Mjk4K1J1cGVzaCBLdW1hcitydXBlc2hrci4wOWVlODBAZ21haWwuY29tK1FVRVN


USU9OIDM1
Which of the following celestial phenomenon is used as ‘Cosmic Clock’ by scientists:

a) Blackholes
b) Pulsars
c) Comets
d) White Dwarfs

Correct Answer: B
Your Answer: B Marks: 2.00/1.00
Explanation

Solution (b)

IASbaba
Score:
Web: http://ilp.iasbaba.com/
44.00 /
Email: ilp@iasbaba.com
Page 36 200
UPSC 2024 -
Exam Title : Environment,
Sc...
rupeshkr.
Email :
09ee80@gmail.com
Contact :

Explanation:

· Pulsars are distant rapidly rotating neutron stars that emit pulses of radiation, observed from
the Earth as bright flashes of light. These bursts take place at extremely precise intervals, and
therefore scientists use pulsars as ‘cosmic clocks’.

· The rotation periods of pulsars are remarkably stable, similar to the ticking of a clock. Some
pulsars have rotation periods that range from milliseconds to a few seconds.

Important Points/Value Additions:

· The stability of their rotation makes them excellent candidates for precision timekeeping in
astronomical studies.

· Scientists use pulsars as natural celestial clocks to study various phenomena, including the
dynamics of binary star systems, the effects of gravitational waves, and the properties of
interstellar and intergalactic mediums.

QUESTION 37. MTI3Mjk4K1J1cGVzaCBLdW1hcitydXBlc2hrci4wOWVlODBAZ21haWwuY29tK1FVRVN


USU9OIDM2
“The space telescope will create a great map of the large-scale structure of the Universe across
space and time by observing billions of galaxies out to 10 billion light-years, across more than a
third of the sky, revealing more about the role of gravity and the nature of dark energy and dark
matter.” The initiative in the question refers to:

a) Juice
b) Fermi Gamma-ray Space Telescope
c) James Webb Space Telescope
d) Euclid

Correct Answer: D
Your Answer: D Marks: 2.00/1.00
Explanation

Solution (d)

Explanation:

· ESA's Euclid mission is designed to explore the composition and evolution of the dark Universe.
The space telescope will create a great map of the large-scale structure of the Universe across
space and time by observing billions of galaxies out to 10 billion light-years, across more than a
third of the sky.

· Euclid will explore how the Universe has expanded and how structure has formed over cosmic
history, revealing more about the role of gravity and the nature of dark energy and dark matter. (
Hence option d is correct)

IASbaba
Score:
Web: http://ilp.iasbaba.com/
44.00 /
Email: ilp@iasbaba.com
Page 37 200
UPSC 2024 -
Exam Title : Environment,
Sc...
rupeshkr.
Email :
09ee80@gmail.com
Contact :

· ESA’s Jupiter Icy Moons Explorer, Juice, will make detailed observations of the giant gas planet
and its three large ocean-bearing moons – Ganymede, Callisto and Europa – with a suite of remote
sensing, geophysical and in situ instruments.

Important Points/Value Additions:

· James Webb Telescope can observe extremely far back in time and zoom into the details.

· Euclid can go fast and wide. In a single observation Euclid can record the data from an area of
the sky more than one hundred times bigger than that imaged by Webb’s camera, NIRCam. This
means that Euclid can map a third of the sky to the required sensitivity in six years in space a feat
that would be impossible with Webb.

· The Fermi Gamma-ray Space Telescope, a superior tool to study how black holes, notorious for
pulling matter in, can accelerate jets of gas outward at fantastic speeds. Physicists are able to
study subatomic particles at energies far greater than those seen in ground-based particle
accelerators.

QUESTION 38. MTI3Mjk4K1J1cGVzaCBLdW1hcitydXBlc2hrci4wOWVlODBAZ21haWwuY29tK1FVRVN


USU9OIDM3
Which of the following statements accurately describes Foucault's Pendulum?

a) It measures the strength of Earth's magnetic field.


b) It is a device used to demonstrate the rotation of the Earth.
c) It is a medical tool for studying cardiovascular health.
d) It is a type of clock mechanism invented in the 19th century.

Correct Answer: B
Your Answer: Unanswered Marks: 0/1.00
Explanation

Solution (b)

Explanation:

· Foucault's Pendulum is based on the principle of demonstrating the rotation of the Earth. The
pendulum consists of a heavy weight suspended from a fixed point so that it can swing freely in
any direction. As the pendulum swings back and forth, the Earth rotates beneath it.

· The key to the demonstration is that the plane of the pendulum's swing remains constant while
the Earth rotates beneath it. This rotation causes an apparent change in the direction of the
pendulum's swing over time. The time it takes for the pendulum to complete a full rotation
depends on its location and the latitude of the Earth.

· This phenomenon was first demonstrated by the French physicist Léon Foucault in 1851.
Foucault's Pendulum became a popular educational tool, illustrating the Earth's rotation and
providing a visual confirmation of the Earth's movement. It's an elegant way to show that the
Earth rotates on its axis, and it has been displayed in various museums and educational
institutions around the world.

IASbaba
Score:
Web: http://ilp.iasbaba.com/
44.00 /
Email: ilp@iasbaba.com
Page 38 200
UPSC 2024 -
Exam Title : Environment,
Sc...
rupeshkr.
Email :
09ee80@gmail.com
Contact :

Important Points/Value Additions:

· Foucault pendulum is installed in India’s newly constructed Parliament building. The pendulum
hangs from a skylight at the top of the Constitution Hall, and signifies the “integration of the idea
of India with the idea of the cosmos”.

· It is created by the National Council of Science Museum (NCSM) in Kolkata, the pendulum is
being dubbed as the largest such piece in India, 22 metre in height, and weighing a staggering 36
kg.

· First in India, It was installed in 1991 at the Inter-University Centre for Astronomy and
Astrophysics (IUCAA) in Pune.

QUESTION 39. MTI3Mjk4K1J1cGVzaCBLdW1hcitydXBlc2hrci4wOWVlODBAZ21haWwuY29tK1FVRVN


USU9OIDM4
Consider
the following statements in context of difference between ‘Submersible and Submarine’:

1. Submersible does not function as an autonomous craft, instead relying on a support platform to
deploy and return.

2. Submersibles
are not capable to carry humans unlike submarines which are crewed by humans.

3. Submarine is umbrella term and submersible is a type of submarine.

How many of the above statements are correct?

a) Only one
b) Only two
c) All three
d) None

Correct Answer: A
Your Answer: A Marks: 2.00/1.00
Explanation

Solution (a)

Explanation:

· Submersible does not function as an autonomous craft, instead relying on a support platform to
deploy and return. (Hence statement 1 is correct)

· Submersibles can be manned or unmanned depending upon type. They can be human-operated
(manned submersibles) or remotely operated (ROVs) and autonomous (AUVs) without a human
presence on board. Recently manned submersible Titan imploded in ocean. (Hence statement 2
is incorrect)

IASbaba
Score:
Web: http://ilp.iasbaba.com/
44.00 /
Email: ilp@iasbaba.com
Page 39 200
UPSC 2024 -
Exam Title : Environment,
Sc...
rupeshkr.
Email :
09ee80@gmail.com
Contact :

· Submarines specifically refer to crewed vehicles designed for independent operation beneath
the water's surface, and submersible is not a type of submarine. (Hence statement 3 is
incorrect)

Important Points/Value Additions:

· Submersible and Submarines are not completely interchangeable. The difference between them
is that a submarine has enough power to leave a port and come back to a port by itself, according
to the US National Oceanic and Atmospheric Administration.

· A submersible, on the other hand, has limited power reserves so it needs to be “launched” by a
mothership that will also have to recover it. It needs to be dropped to a certain point in the ocean
where they can explore before they come back to a ship that will bring them back to a port.

QUESTION 40. MTI3Mjk4K1J1cGVzaCBLdW1hcitydXBlc2hrci4wOWVlODBAZ21haWwuY29tK1FVRVN


USU9OIDM5
Which of the following factors affect ‘Hardy-Weinberg equilibrium’ used to study population
genetics?

1. Gene migration or Gene flow

2. Genetic drift

3. Mutation

4. Random mating

5. Natural selection

How many of the above correct?

a) Only two
b) Only three
c) Only four
d) All five

Correct Answer: C
Your Answer: Unanswered Marks: 0/1.00
Explanation

Solution (c)

Explanation:

· Random mating is one of the key assumptions of the Hardy-Weinberg equilibrium (HWE) model.
Hardy-Weinberg equilibrium describes the theoretical distribution of alleles in a population that is
not evolving. (Hence statement 4 is incorrect)

· Five factors are known to affect Hardy-Weinberg equilibrium. These are gene migration or gene
flow, genetic drift, mutation, genetic recombination and natural selection. When migration of a

IASbaba
Score:
Web: http://ilp.iasbaba.com/
44.00 /
Email: ilp@iasbaba.com
Page 40 200
UPSC 2024 -
Exam Title : Environment,
Sc...
rupeshkr.
Email :
09ee80@gmail.com
Contact :

section of population to another place and population occurs, gene frequencies change in the
original as well as in the new population. (Hence statement 1,2,3 and 5 are correct)

· New genes/alleles are added to the new population and these are lost from the old population.

· There would be a gene flow if this gene migration, happens multiple times. If the same change
occurs by chance, it is called genetic drift.

· Sometimes the change in allele frequency is so different in the new sample of population that
they become a different species. The original drifted population becomes founders and the effect
is called “founder effect”.

Important Points/Value Additions:

· The Founder Effect is a genetic phenomenon that occurs when a small group of individuals
establishes a new population, and due to the limited number of individuals involved, the new
population has a subset of the genetic diversity of the original population. This can lead to a
population with a gene pool that differs from the source population, and certain genetic traits or
disorders may be more prevalent in the new population due to the influence of the founding
individuals.

· For example, if a small group of individuals migrates to a new area and establishes a population,
the genetic makeup of that new population is determined by the genes carried by the founders.
Any genetic variations present in the founding individuals will be disproportionately represented
in the subsequent generations of the population.

· The Founder Effect is a special case of genetic drift, where chance events play a significant role
in determining the genetic composition of a population, and it can have implications for the
genetic diversity and health of the population over time.

QUESTION 41. MTI3Mjk4K1J1cGVzaCBLdW1hcitydXBlc2hrci4wOWVlODBAZ21haWwuY29tK1FVRVN


USU9OIDQw
Which of the following statement is correct in context of difference between convergent and
divergent evolution?

a) Convergent evolution results in the development of similar traits in unrelated species, while
divergent evolution leads to the emergence of different traits in closely related species.
b) Convergent evolution leads to the development of different traits in closely related species,
while divergent evolution results in the emergence of similar traits in unrelated species.
c) Both convergent and divergent evolution lead to the development of similar traits in closely
related species.
d) Both convergent and divergent evolution result in the emergence of different traits in
unrelated species.

Correct Answer: A
Your Answer: A Marks: 2.00/1.00
Explanation

Solution (a)

IASbaba
Score:
Web: http://ilp.iasbaba.com/
44.00 /
Email: ilp@iasbaba.com
Page 41 200
UPSC 2024 -
Exam Title : Environment,
Sc...
rupeshkr.
Email :
09ee80@gmail.com
Contact :

Explanation:

· Convergent evolution and divergent evolution are two patterns of evolution that describe how
species develop similarities or differences over time.

· Convergent Evolution: This occurs when unrelated species independently evolve similar traits or
characteristics due to adapting to similar environmental challenges. These species may not share
a recent common ancestor, but they face analogous selective pressures, leading to the
development of analogous structures or functions

· Divergent Evolution: In contrast, divergent evolution occurs when closely related species evolve
different traits or characteristics. Over time, populations of a common ancestor may adapt to
different environmental conditions, resulting in variations that distinguish the species. (Hence
option a is correct)

Important Points/Value Additions:

· Example of convergent evolution: The wings of bats and birds serve similar purposes in flight,
but these animals are not closely related.

· Example of divergent evolution: Variety of beak shapes found in Darwin's finches, where
different species adapted to different food sources on the Galápagos Islands.

QUESTION 42. MTI3Mjk4K1J1cGVzaCBLdW1hcitydXBlc2hrci4wOWVlODBAZ21haWwuY29tK1FVRVN


USU9OIDQx
Consider the following statements within the framework of criteria must for a molecule to
function as ‘genetic material’:

1. It should be able to generate its replica (Replication).

2. It should be stable chemically and structurally.

3. It should provide the scope for slow changes (mutation) that are required for evolution.

4. It should be able to express itself in the form of 'Mendelian Characters’.

How many of the above statements are correct?

a) Only one
b) Only two
c) Only three
d) All four

Correct Answer: D
Your Answer: D Marks: 2.00/1.00
Explanation

Solution (d)

Explanation:

IASbaba
Score:
Web: http://ilp.iasbaba.com/
44.00 /
Email: ilp@iasbaba.com
Page 42 200
UPSC 2024 -
Exam Title : Environment,
Sc...
rupeshkr.
Email :
09ee80@gmail.com
Contact :

· Genetic material must be capable of replication so that it can pass on its information to the next
generation during cell division or reproduction. This is a fundamental characteristic of genetic
material.

· The genetic material should be stable enough not to change with different stages of life cycle,
age or with change in physiology of the organism.

· Genetic material should allow for mutations, which are changes in the DNA sequence. These
mutations provide the raw material for the process of evolution, allowing organisms to adapt to
their environments over time.

Important Points/Value Additions:

· Mendelian characters are hereditary traits governed by Gregor Mendel's principles of


inheritance. These traits follow dominance and recessiveness, where dominant traits mask the
expression of recessive ones. Segregation ensures alleles separate during gamete formation, and
independent assortment explains the inheritance of traits on different chromosomes.

· Genes, known as unit characters, exist in alternative forms called alleles. The law of dominance
dictates that in heterozygotes, one allele determines the phenotype.

· Mendelian traits include simple genetic characteristics like blood types or certain inherited
disorders, exemplifying the predictable patterns of inheritance discovered by Mendel in his
pioneering pea plant experiments.

QUESTION 43. MTI3Mjk4K1J1cGVzaCBLdW1hcitydXBlc2hrci4wOWVlODBAZ21haWwuY29tK1FVRVN


USU9OIDQy
Consider the following statements in context of DNA and RNA:

1. RNA mutates at a faster rate consequently viruses having RNA genome and having shorter life
span mutate and evolve faster.

2. Both RNA and DNA are able to mutate and both RNA and DNA can function as genetic
material.

3. RNA is preferred for storage of genetic information and DNA for the transmission of genetic
information.

How many of the above statements are correct?

a) Only one
b) Only two
c) All three
d) None

Correct Answer: B
Your Answer: B Marks: 2.00/1.00
Explanation

IASbaba
Score:
Web: http://ilp.iasbaba.com/
44.00 /
Email: ilp@iasbaba.com
Page 43 200
UPSC 2024 -
Exam Title : Environment,
Sc...
rupeshkr.
Email :
09ee80@gmail.com
Contact :

Solution (b)

Explanation:

· Both DNA and RNA are able to mutate. In fact, RNA being unstable, mutate at a faster rate.
Consequently, viruses having RNA genome and having shorter life span mutate and evolve faster.
(Hence statement 1 is correct)

· RNA can directly code for the synthesis of proteins, hence can easily express the characters.
DNA, however, is dependent on RNA for synthesis of proteins.

· Both RNA and DNA can function as genetic material as explained above both are able to mutate.
(Hence statement 2 is correct)

· DNA being more stable is preferred for storage of genetic information. For the
transmission of genetic information, RNA is better. (Hence statement 3 is incorrect)

Important points/Value Additions:

· Deoxyribonucleic acid (DNA) and ribonucleic acid (RNA) are complex molecular structures that
control all hereditary characteristics of cells and thus of organisms. DNA is the master blueprint
for life and constitutes the genetic material in all free-living organisms. RNA uses DNA to code for
the structure of proteins synthesized in cells, and it is also the genetic material of certain viruses.

· Both are made of combinations of four nucleotides, which are special “building block” molecules
with a nitrogen base. DNA is composed of long strands of the nucleotides adenine, guanine,
cytosine, and thymine. In RNA, the thymine is replaced by uracil. The order and patterns of these
nucleotides form the genetic code.

· Physically, DNA is structured as a double helix, with two strands of DNA winding around each
other, while RNA is structured as a single strand.

QUESTION 44. MTI3Mjk4K1J1cGVzaCBLdW1hcitydXBlc2hrci4wOWVlODBAZ21haWwuY29tK1FVRVN


USU9OIDQz
Consider the following statements in context of Immunity:

1. Virus-infected cells secrete proteins called interferons which protect non-infected cells from
further viral infection is type of acquired Immunity.

2. Antibody mediated- acquired immune response involve proteins produced by B-lymphocytes in


response to pathogens.

3. The T-lymphocytes mediate cell-mediated immunity (CMI) is responsible for the graft rejection.

Which of the above statements are correct?

a) 1 and 2
b) 2 and 3
c) 1 and 3

IASbaba
Score:
Web: http://ilp.iasbaba.com/
44.00 /
Email: ilp@iasbaba.com
Page 44 200
UPSC 2024 -
Exam Title : Environment,
Sc...
rupeshkr.
Email :
09ee80@gmail.com
Contact :

d) All of the above

Correct Answer: B
Your Answer: B Marks: 2.00/1.00
Explanation

Solution (b)

Explanation:

· Cytokine barriers: Virus-infected cells secrete proteins called interferons which protect non-
infected cells from further viral infection. This is nonspecific type of defence and it is barrier
through innate immunity. (Hence statement 1 is incorrect)

· In acquired immunity- The primary and secondary immune responses are carried out with the
help of two special types of lymphocytes present in our blood, i.e., B-lymphocytes and T-
lymphocytes. The B-lymphocytes produce an army of proteins in response to pathogens into our
blood to fight with them. These proteins are called antibodies. The T-cells themselves do not
secrete antibodies but help B cells to produce them. (Hence statement 2 is correct)

· The second type is called cell-mediated immune response or cell-mediated immunity (CMI). The
T-lymphocytes mediate CMI. The body is able to differentiate ‘self’ and ‘nonself’ and the cell-
mediated immune response is responsible for the graft rejection. (Hence statement 3 is
correct)

Important Points/Value Additions:

· Very often, when some human organs like heart, eye, liver, kidney fail to function satisfactorily,
transplantation is the only remedy to enable the patient to live a normal life.

· Grafts from just any source – an animal, another primate, or any human beings cannot be made
since the grafts would be rejected sooner or later. Tissue matching, blood group matching are
essential before undertaking any graft/transplant and even after this the patient has to take
immunosuppresants all his/her life.

QUESTION 45. MTI3Mjk4K1J1cGVzaCBLdW1hcitydXBlc2hrci4wOWVlODBAZ21haWwuY29tK1FVRVN


USU9OIDQ0
Which of the following statements about ‘Quantum Mechanics’ is INCORRECT:

a) Quantum entanglement allows particles to be correlated in such a way that the state of one
particle can influence the state of another when distance between them is finite.
b) Heisenberg's Uncertainty Principle states that it is impossible to simultaneously know the
exact position and momentum of a particle with absolute precision.
c) Quantum superposition implies that particles can exist in multiple states at the same time
until observed or measured.
d) Quantum tunneling is a phenomenon where particles can pass through barriers that classical
physics would predict as impenetrable.

Correct Answer: A
Your Answer: D Marks: -0.66/1.00

IASbaba
Score:
Web: http://ilp.iasbaba.com/
44.00 /
Email: ilp@iasbaba.com
Page 45 200
UPSC 2024 -
Exam Title : Environment,
Sc...
rupeshkr.
Email :
09ee80@gmail.com
Contact :

Explanation

Solution (a)

Explanation:

· Quantum entanglement allows particles to be correlated in such a way that the state of one
particle can influence the state of another, regardless of the distance between them . (Hence
statement 1 is incorrect)

· Heisenberg's Uncertainty Principle is a fundamental concept in quantum mechanics, stating that


the more precisely you know the position of a particle, the less precisely you can know its
momentum (and vice versa). This is not a limitation due to measurement devices but is inherent in
the nature of quantum systems.

· Quantum superposition is a concept where a particle can exist in multiple states simultaneously
until a measurement is made, at which point it 'collapses' into one of the possible states. This is a
fundamental aspect of quantum mechanics.

· Quantum tunneling is a real phenomenon in quantum mechanics where particles can pass
through energy barriers that classical physics would consider impenetrable. This occurs due to
the probabilistic nature of quantum states, allowing particles to 'tunnel' through barriers they
wouldn't be able to overcome classically.

Important Points/Value Additions:

· Quantum mechanics is a fundamental theory in physics that describes the behavior of matter
and energy at the smallest scales, typically on the order of atoms and subatomic particles.

· It is a branch of theoretical physics that has proven to be incredibly successful in explaining the
behavior of particles at the quantum level.

QUESTION 46. MTI3Mjk4K1J1cGVzaCBLdW1hcitydXBlc2hrci4wOWVlODBAZ21haWwuY29tK1FVRVN


USU9OIDQ1
Consider the following statements with reference to qualities of Carbon fiber and Titanium:

1. Carbon fibre is a polymer that is known to be quite strong but very heavy than steel.

2. Carbon can be as much as five times stronger than steel and twice as stiff.

3. Titanium is as strong as steel but lighter than steel.

4. Titanium is twice as strong as aluminium.

How many of the above statements are correct?

a) Only one
b) Only two
c) Only three
d) All four

IASbaba
Score:
Web: http://ilp.iasbaba.com/
44.00 /
Email: ilp@iasbaba.com
Page 46 200
UPSC 2024 -
Exam Title : Environment,
Sc...
rupeshkr.
Email :
09ee80@gmail.com
Contact :

Correct Answer: C
Your Answer: C Marks: 2.00/1.00
Explanation

Solution (c)

Explanation:

· Carbon fibre is a polymer that is known to be quite strong despite being lightweight. (Hence
statement 1 is incorrect)

· It can be as much as five times stronger than steel and twice as stiff. (Hence statement 2 is
correct)

· Titanium is as strong as steel but around 45 per cent lighter. (Hence statement 3 is correct)

· It is twice as strong as aluminium but only 60 per cent heavier, according to the United States
Geological Survey. (Hence statement 4 is correct)

Important Points/Value Additions:

· According to OceanGate manufacturer of Titan submersible makes “innovative use of modern


materials” to be “lighter in weight and more cost-efficient to mobilise than any other deep diving
submersible.” The materials the company is referring to are carbon fibre and titanium.

· The vessel was made using a mixture of proprietary technologies and some off-the-shelf
components, which the company claims make it simple to operate and replace parts in the field.

QUESTION 47. MTI3Mjk4K1J1cGVzaCBLdW1hcitydXBlc2hrci4wOWVlODBAZ21haWwuY29tK1FVRVN


USU9OIDQ2
Consider the following statements:

Statement 1: Hubble's work played a crucial role in the development of the Big Bang Theory.

Statement 2: Big Bang Theory suggested that the universe is always expanding but maintaining a
constant average density of matter.

Which one of the following is correct in respect of the above statements?

a) Both Statement-1 and Statement-2 are correct and Statement-2 is the correct explanation
for Statement-1.
b) Both Statement-1 and Statement-2 are correct and Statement-2 is not the correct
explanation for Statement-1.
c) Statement-1 is correct but Statement-2 is incorrect.
d) Statement-1 is incorrect but Statement-2 is correct.

Correct Answer: C
Your Answer: C Marks: 2.00/1.00
Explanation

IASbaba
Score:
Web: http://ilp.iasbaba.com/
44.00 /
Email: ilp@iasbaba.com
Page 47 200
UPSC 2024 -
Exam Title : Environment,
Sc...
rupeshkr.
Email :
09ee80@gmail.com
Contact :

Solution (c)

Explanation:

· Edwin Hubble is famous for his observational work that led to the formulation of Hubble's Law,
which states that the velocity of galaxies is proportional to their distance from us.

· This observation, coupled with the redshift of light from distant galaxies, provided strong
evidence for an expanding universe.

· Hubble's work played a crucial role in the development of the Big Bang Theory, which posits that
the universe began from an extremely hot and dense state and has been expanding ever since. (
Hence statement 1 is correct)

· Steady-state theory in cosmology, a view that the universe is always expanding but
maintaining a constant average density , with matter being continuously created to form new
stars and galaxies at the same rate that old ones become unobservable as a consequence of their
increasing distance and velocity of recession. (Hence statement 2 is incorrect)

· A steady-state universe has no beginning or end in time, and from any point within it the view on
the grand scale i.e., the average density and arrangement of galaxies—is the same. Galaxies of all
possible ages are intermingled.

Important Points/Value Additions:

· Indian Scientists have proposed a new method to determine the value of Hubble Constant.

· Hubble's Law states that the galaxies are moving away from Earth at speeds proportional to
their distance i.e., the farther the galaxy away from the Earth, the faster it moves away.

· Hubble's Law forms the basis that the Universe is expanding.

· The Hubble constant is a unit that describes how fast the universe is expanding at different
distances from a particular point in space. It is measured in (km/s)/Megaparsec. Megaparsec is
unit for large distances, equal to 3.26 million light years.

QUESTION 48. MTI3Mjk4K1J1cGVzaCBLdW1hcitydXBlc2hrci4wOWVlODBAZ21haWwuY29tK1FVRVN


USU9OIDQ3
Which of the following radioactive elements is commonly associated with nuclear fallout and poses
health risks due to its presence in nuclear weapons testing and accidents?

a) Tritium
b) Caesium-137
c) Strontium-90
d) All of the above

Correct Answer: D
Your Answer: D Marks: 2.00/1.00
Explanation

IASbaba
Score:
Web: http://ilp.iasbaba.com/
44.00 /
Email: ilp@iasbaba.com
Page 48 200
UPSC 2024 -
Exam Title : Environment,
Sc...
rupeshkr.
Email :
09ee80@gmail.com
Contact :

Solution (d)

Explanation:

· Twelve years after the Fukushima nuclear meltdown, Japan is releasing the power plant’s cooling
water into the ocean. Japanese officials say it’s safe, but experts are divided.

· Tritium: Tritium is a radioactive isotope of hydrogen. It is commonly used in nuclear weapons


and can also be released into the environment during nuclear power production. Tritium emits
low-energy beta radiation and can be incorporated into water molecules, potentially leading to
health concerns if it enters the food chain.

· Caesium-137 (Option B): Caesium-137 is a radioactive isotope produced during nuclear fission. It
has a relatively long half-life, making it persistent in the environment. Caesium-137 can
contaminate soil and vegetation, posing a risk to human health if consumed. It has been a
significant contributor to radioactive fallout from nuclear testing and accidents.

· Strontium-90 (Option C): Strontium-90 is a radioactive isotope of strontium. It is produced


during nuclear fission and has a long half-life. Strontium-90 behaves chemically similarly to
calcium and can accumulate in bones, posing health risks, especially for developing bones in
children. It is also a component of nuclear fallout.

Important Points/Value Additions:

· The environmental group Greenpeace has accused the Japanese government of distracting
attention from the radiation levels in the water by focusing on tritium — other radionuclides will
remain in the water even after it’s been filtered.

· According to Greenpeace, The contaminated water contains many radionuclides, which we know
impact the environment and human health — including strontium-90

QUESTION 49. MTI3Mjk4K1J1cGVzaCBLdW1hcitydXBlc2hrci4wOWVlODBAZ21haWwuY29tK1FVRVN


USU9OIDQ4
Consider the following statements about ‘Superconductors’:

1. Complete disappearance of electrical resistance in various solids when they are cooled below a
certain temperature called the transition temperature.

2. Expulsion of a magnetic field from the interior of material below transition temperature.

3. No known materials are superconductors in everyday conditions.

How many of the above statements correct?

a) Only one
b) Only two
c) All three
d) None

Correct Answer: C

IASbaba
Score:
Web: http://ilp.iasbaba.com/
44.00 /
Email: ilp@iasbaba.com
Page 49 200
UPSC 2024 -
Exam Title : Environment,
Sc...
rupeshkr.
Email :
09ee80@gmail.com
Contact :

Your Answer: C Marks: 2.00/1.00


Explanation

Solution (c)

Explanation:

· Superconductivity, complete disappearance of electrical resistance in various solids when they


are cooled below a characteristic temperature. This temperature, called the transition
temperature, varies for different materials but generally is below 20 K (−253 °C). (Hence
statement 1 is correct)

· Meissner effect, the expulsion of a magnetic field from the interior of a material that is in the
process of becoming a superconductor, that is, losing its resistance to the flow of electrical
currents when cooled below a certain temperature, called the transition temperature, usually
close to absolute zero. (Hence statement 2 is correct)

· First discovered in 1911, superconductors can seem almost magical. However, no known
materials are superconductors in everyday conditions. (Hence statement 3 is correct)

Important Points/Value Additions:

· Most require ultracold temperatures, and recent advances toward superconductors that
function at higher temperatures require crushing pressures.

· A superconductor that works at everyday temperatures and pressures could find use in M.R.I.
scanners, novel electronic devices and levitating trains.

QUESTION 50. MTI3Mjk4K1J1cGVzaCBLdW1hcitydXBlc2hrci4wOWVlODBAZ21haWwuY29tK1FVRVN


USU9OIDQ5
Consider the following statements in context of Motion Capture technology (Mo-cap) and
Computer-generated Imagery (CGI):

1. Mo-cap tracks specific motions such as facial expressions and body movements, so that the
features can be provided to the computer-generated 3D character on screen.

2. CGI refers to process of creating and manipulating images that do not exist in the physical
environment.

Which of the above statements are correct?

a) 1 Only
b) 2 Only
c) Both
d) Neither

Correct Answer: C
Your Answer: Unanswered Marks: 0/1.00
Explanation

IASbaba
Score:
Web: http://ilp.iasbaba.com/
44.00 /
Email: ilp@iasbaba.com
Page 50 200
UPSC 2024 -
Exam Title : Environment,
Sc...
rupeshkr.
Email :
09ee80@gmail.com
Contact :

Solution (c)

Explanation:

· Motion capture, or mo-cap, is the name for the technology that records the movement of people
or objects. The data thus captured will be transferred to a computer programme and can in turn
be translated to a CGI character.

· Mo-cap, in detail, tracks specific motions such as facial expressions and body movements, so that
the features can be provided to the computer-generated 3D character on screen. (Hence
statement 1 is correct)

· In basic terms, Computer-generated Imagery, or CGI, refers to the utilisation of computers to


create pictures or characters in film and television.

· It can also be defined as the process of creating and manipulating images that do not exist in the
physical environment for being captured on film or video. These images can be either static or
dynamic and are utilised in both 2D and 3D movies. (Hence statement 2 is correct)

Important Points/Value Additions:

· CGI is utilised in a wide range of films, including period/epic or sci-fi genres, to create specific
elements, objects, or even entire background environments.

· Even makers of film working to embrace CGI technology to enhance the overall choreography of
scenes or the film as a whole by incorporating intricate details.

QUESTION 51. MTI3Mjk4K1J1cGVzaCBLdW1hcitydXBlc2hrci4wOWVlODBAZ21haWwuY29tK1FVRVN


USU9OIDUw
“Takakia is a genus that only has two species. And both of them are found together only in one
place-the Tibetan Plateau. An international group of scientists have discovered exactly how it
developed the ability to survive frost, life-threatening UV radiation and other hazardous
conditions.”

It belongs to which of the following group?

a) Algae
b) Moss
c) Liverwort
d) Hornwort

Correct Answer: B
Your Answer: B Marks: 2.00/1.00
Explanation

Solution (b)

Explanation:

IASbaba
Score:
Web: http://ilp.iasbaba.com/
44.00 /
Email: ilp@iasbaba.com
Page 51 200
UPSC 2024 -
Exam Title : Environment,
Sc...
rupeshkr.
Email :
09ee80@gmail.com
Contact :

· Takakia is a moss, not a liverwort, algae, or hornwort. It belongs to the division Bryophyta, which
includes mosses. While Takakia is considered a primitive moss and shares some characteristics
with liverworts, it is taxonomically classified within the moss group. (Hence
option b is correct)

· Mosses, liverworts, and hornworts are all different groups of non-vascular plants, each with
unique characteristics and evolutionary histories.

Important Points/Value Additions:

· The plants took millions of years to adapt to decreasing temperatures and increasing radiation
but now, its habitat is now changing in just decades because of climate change.

· These hardy species don’t seem to cope as well temperature rise compared to others. Their
populations become significantly smaller over the study period while other plants benefited from
the warming. The researchers believe this trend will continue.

· A species that saw the dinosaurs come and go might not survive the coming of humans.

QUESTION 52. MTI3Mjk4K1J1cGVzaCBLdW1hcitydXBlc2hrci4wOWVlODBAZ21haWwuY29tK1FVRVN


USU9OIDUx
Which of the following best describes “Greenwashing”?

a) Implementing eco-friendly practices in manufacturing


b) Deceptive portray of environment sensitiveness of product
c) Establishing green initiatives to combat climate change
d) Advocating for sustainable policies in the corporate sector

Correct Answer: B
Your Answer: B Marks: 2.00/1.00
Explanation

Solution (b)

Explanation:

· Greenwashing refers to the deceptive practice of presenting a company, product, or service as


environmentally friendly or sustainable when, in reality, it may not be.

· This misleading marketing strategy is designed to give the impression that the company is
making significant efforts to be environmentally responsible, often by highlighting specific eco-
friendly features or initiatives.

· However, in many cases, these claims may be exaggerated or lack substance, leading consumers
to believe they are supporting environmentally conscious businesses when they might not be.

· Greenwashing can undermine genuine efforts towards sustainability and make it challenging for
consumers to make informed and responsible choices.

Important Points/Value Additions:

IASbaba
Score:
Web: http://ilp.iasbaba.com/
44.00 /
Email: ilp@iasbaba.com
Page 52 200
UPSC 2024 -
Exam Title : Environment,
Sc...
rupeshkr.
Email :
09ee80@gmail.com
Contact :

· Greenwashing as a practice is prevalent enough to create concerns over climate goals getting
completely undermined, and was considered serious enough for the UN Secretary General to set
up an expert group solely to look into this.

· Recently, United Kingdom’s ad regulator banned advertisements from Air France, Lufthansa,
and Etihad for allegedly misleading consumers regarding the environmental impact of air travel.

QUESTION 53. MTI3Mjk4K1J1cGVzaCBLdW1hcitydXBlc2hrci4wOWVlODBAZ21haWwuY29tK1FVRVN


USU9OIDUy
Consider the following statements in context of ‘Green Credit System’:

1. Market-based mechanism designed to incentivize voluntary environmental actions by Private


sector companies only.

2. In its initial state it is focused on two key activities water conservation and afforestation.

3. Green credit certificate issued by administrator will be tradeable on stock exchanges.

How many of the above statements are correct?

a) Only one
b) Only two
c) All three
d) None

Correct Answer: A
Your Answer: A Marks: 2.00/1.00
Explanation

Solution (a)

Explanation:

· Green Credit Program (GCP) is an innovative market-based mechanism designed to incentivize


voluntary environmental actions across diverse sectors, by various stakeholders like
individuals, communities, private sector industries, and companies . ( Hence statement
1 is incorrect)

· In its initial phase, the GCP focuses on two key activities: water conservation and afforestation. (
Hence statement 2 is correct)

· Once verification is complete, the Administrator will grant a Green Credit certificate which will
be tradable on the green credit platform and not on stock exchanges for now. ( Hence statement
3 is incorrect)

Important Points/Value Additions:

IASbaba
Score:
Web: http://ilp.iasbaba.com/
44.00 /
Email: ilp@iasbaba.com
Page 53 200
UPSC 2024 -
Exam Title : Environment,
Sc...
rupeshkr.
Email :
09ee80@gmail.com
Contact :

· The GCP's governance framework is supported by an inter-ministerial Steering Committee and


The Indian Council of Forestry Research and Education (ICFRE) serves as the GCP Administrator,
responsible for program implementation, management, monitoring, and operation.

· Draft methodologies for awarding Green Credits have been developed and will be notified for
stakeholder consultation. These methodologies set benchmarks for each activity/process, to
ensure environmental impact and fungibility across sectors.

· A user-friendly digital platform will streamline the processes for registration of projects, its
verification, and issuance of Green Credits. The Green Credit Registry and trading platform,
being developed by ICFRE along with experts, would facilitate the registration and thereafter, the
buying and selling of Green Credits.

· To obtain Green Credits, individuals and entities must register their activities through the
central government's dedicated app/website. The Administrator will verify the activity through a
designated agency, with self-verification for small projects.

QUESTION 54. MTI3Mjk4K1J1cGVzaCBLdW1hcitydXBlc2hrci4wOWVlODBAZ21haWwuY29tK1FVRVN


USU9OIDUz
Consider the following statements about ‘Ecomark Scheme’:

1. It provides accreditation and labelling for household and consumer products.

2. It will motivate manufacturers to shift towards environmentally friendly production.

3. The Central Pollution Control Board administers the scheme in partnership with Bureau of
Indian Standards (BIS).

How many of the above statements are correct?

a) Only one
b) Only two
c) All three
d) None

Correct Answer: C
Your Answer: C Marks: 2.00/1.00
Explanation

Solution (c)

Explanation:

· The Ecomark Scheme provides accreditation and labelling for household and consumer products
that meet specific environmental criteria while maintaining quality standards as per Indian
norms. (Hence statement 1 is correct)

IASbaba
Score:
Web: http://ilp.iasbaba.com/
44.00 /
Email: ilp@iasbaba.com
Page 54 200
UPSC 2024 -
Exam Title : Environment,
Sc...
rupeshkr.
Email :
09ee80@gmail.com
Contact :

· Products accredited under the Ecomark Scheme will adhere to specific environmental criteria,
ensuring minimal environmental impact. It will build consumer awareness of environmental issues
and encourage eco-conscious choices.

· It will also motivate manufacturers to shift towards environmentally friendly production. The
scheme seeks to ensure accurate labelling and prevent misleading information about products. (
Hence statement 2 is correct)

· The Central Pollution Control Board administers the Ecomark Scheme in partnership with
Bureau of Indian Standards (BIS), which is the national body for standards and certification. (He
nce statement 3 is correct)

Important Points/Value Additions:

· It marks significant step in promoting sustainable living, environmental conservation, and,


through individual and collective choice, embody eco-friendly practices in India.

· This aligns with global sustainability goals and reflect the government's commitment to
conservation and protection of the environment.

QUESTION 55. MTI3Mjk4K1J1cGVzaCBLdW1hcitydXBlc2hrci4wOWVlODBAZ21haWwuY29tK1FVRVN


USU9OIDU0
Recently indigenous breed locally known as the ‘Bhimthadi’ horse, has now been officially
recognized as an independent breed of India. Consider the following statements in this context:

1. It is the first indigenous horse breed to get recognized as independent breed of India.

2. It is predominant in Deccan region along the bank of Bhima River.

3. National Research Centre of Equines in Bikaner is responsible for identification and


characterization of homogenous populations qualifying for a breed.

How many of the above statements are correct?

a) Only one
b) Only two
c) All three
d) None

Correct Answer: A
Your Answer: Unanswered Marks: 0/1.00
Explanation

Solution (a)

Explanation:

· National Bureau of Animal Genetic Resources has already recognized seven independent Indian
breeds of horses which are Bhutia, Kathiawari, Manipuri, Marwari, Spiti, Zanskari, Kachchhi-
Sindh. And Bhimthadi will be eighth. (Hence statement 1 is incorrect)

IASbaba
Score:
Web: http://ilp.iasbaba.com/
44.00 /
Email: ilp@iasbaba.com
Page 55 200
UPSC 2024 -
Exam Title : Environment,
Sc...
rupeshkr.
Email :
09ee80@gmail.com
Contact :

· The Deccani horse is named after the Bhima river, ‘Bhimthadi’ combines ‘Bhima’ with ‘thadi,’
meaning riverbank. This breed is primarily located on the Deccan plateau, spanning areas in Pune,
Satara, Solapur, Ahmednagar, Sangli, and Kolhapur districts. (Hence statement 2 is correct)

· National Bureau of Animal Genetic Resources (NBAGR) has recognized more than 100 new
breeds and homogenous population in country. However, there are still a number of unique
populations of native AnGR which needs to be identified. Zero non-descript AnGR mission
envisage recognition of about 100 breeds in country. (Hence statement 3 is incorrect)

Important Points/Value Additions:

· Among the total of 3,40,000 indigenous horses and around 10,000 foreign horses, the foreign
ones receive more attention through participation in races or NCC. Unfortunately, no significant
measures are being taken to protect our indigenous breeds, which are on the brink of extinction.

· Bhimthadi horses had historic significance as Maratha army of 17 th and 18 th century


constituted of these horses.

QUESTION 56. MTI3Mjk4K1J1cGVzaCBLdW1hcitydXBlc2hrci4wOWVlODBAZ21haWwuY29tK1FVRVN


USU9OIDU1
Recently discovered species of genus batillipes is named after Dr APJ Abdul Kalam as “Batillipes
kalami”. This species is associated with which of the following animal?

a) Springtail
b) Nymph
c) Gerridae
d) Tardigrade

Correct Answer: D
Your Answer: Unanswered Marks: 0/1.00
Explanation

Solution (d)

Explanation:

· Researchers at the Cochin University of Science and Technology (Cusat) have identified a new
species of marine tardigrade which they have named after the late former President and scientist
A.P.J. Abdul Kalam. (Hence option d is correct)

· Tardigrades are extremely tiny animals whose size is measured in micrometres. Their
microscopic size, however, belies their toughness. They are known for their extraordinary
resilience and survival instincts. Studied using microscopes, these water-dwelling animals also go
by the rather unlikely moniker ‘water bears.’

Important Points/Value Additions:

· It has been found in diverse regions of Earth's biosphere-mountaintops, the deep sea, tropical
rainforests, and the Antarctic.

IASbaba
Score:
Web: http://ilp.iasbaba.com/
44.00 /
Email: ilp@iasbaba.com
Page 56 200
UPSC 2024 -
Exam Title : Environment,
Sc...
rupeshkr.
Email :
09ee80@gmail.com
Contact :

· Tardigrades are among the most resilient animals known, with individual species able to survive
extreme conditions such as exposure to extreme temperatures, extreme pressures (both high and
low), air deprivation, radiation, dehydration, and starvation – that would quickly kill most other
known forms of life.

· Tardigrades have survived exposure to outer space.

QUESTION 57. MTI3Mjk4K1J1cGVzaCBLdW1hcitydXBlc2hrci4wOWVlODBAZ21haWwuY29tK1FVRVN


USU9OIDU2
Consider the following statements in context of “Assessment Report on Invasive Alien Species and
their Control’’:

1. It is released by IPCC-Intergovernmental Panel on Climate Change.

2. Not all alien species establish and spread with negative impacts on biodiversity.

3. Highest percentage of alien invertebrates are known to be invasive as per report amongst alien
plants, alien microbes, alien vertebrates.

4. Many invasive alien species have been intentionally introduced for their perceived benefits.

How many of the above statements are correct?

a) Only one
b) Only two
c) Only three
d) All four

Correct Answer: B
Your Answer: D Marks: -0.66/1.00
Explanation

Solution (b)

Explanation:

· In the most extensive study on invasive species carried out till date, the Intergovernmental
Platform on Biodiversity and Ecosystem Services (IPBES) in its new publication – the “Assessment
Report on Invasive Alien Species and their Control’’ – has found that there are 37,000 alien
species, including plants and animals, that have been introduced by many human activities to
regions and biomes around the world, including more than 3,500 invasive alien species and that
invasive alien species have played a key role in 60% of global plant and animal extinctions
recorded. (Hence statement 1 is incorrect)

· Not all alien species establish and spread with negative impacts on biodiversity, local ecosystems
and species, but a significant proportion do – then becoming known as invasive alien species. (He
nce statement 2 is correct)

IASbaba
Score:
Web: http://ilp.iasbaba.com/
44.00 /
Email: ilp@iasbaba.com
Page 57 200
UPSC 2024 -
Exam Title : Environment,
Sc...
rupeshkr.
Email :
09ee80@gmail.com
Contact :

· About 6% of alien plants; 22% of alien invertebrates; 14% of alien vertebrates; and 11% of alien
microbes are known to be invasive, posing major risks to nature and to people. (Hence
statement 3 is incorrect)

· Many invasive alien species have been intentionally introduced for their perceived benefits,
without consideration or knowledge of their negative impacts in forestry, agriculture,
horticulture, aquaculture, or as pets. (Hence statement 4 is correct)

Important Points/Value Additions:

· Invasive alien species are a key driver of biodiversity loss and under Target 6 of the ambitious
Kunming-Montreal Global Biodiversity Framework (KMGBF), the world has to prevent and reduce
the rate of introduction and establishment of invasive alien species by at least 50 per cent by
2030.

· The IPBES released its report, representatives of the 143 member States have approved the
report.

· IPBES is an independent intergovernmental body established to strengthen the science-policy


interface for biodiversity and ecosystem services, working in a similar way to the IPCC, which is
the UN’s climate science body.

· The study, which has taken place over a period of four years, has been by 86 leading experts from
49 countries, drawing on more than 13,000 references.

QUESTION 58. MTI3Mjk4K1J1cGVzaCBLdW1hcitydXBlc2hrci4wOWVlODBAZ21haWwuY29tK1FVRVN


USU9OIDU3
Consider following pairs:

Species Name IUCN status

1. Himalayan Wolf Least Concerned

2. Polar bear Endangered

3. Lesser Florican Endangered

How many of the above are correct?

a) Only one
b) Only two
c) All three
d) None

Correct Answer: D
Your Answer: A Marks: -0.66/1.00
Explanation

Solution (d)

IASbaba
Score:
Web: http://ilp.iasbaba.com/
44.00 /
Email: ilp@iasbaba.com
Page 58 200
UPSC 2024 -
Exam Title : Environment,
Sc...
rupeshkr.
Email :
09ee80@gmail.com
Contact :

Explanation:

· The Himalayan Wolf (Canis lupus chanco), a prominent lupine predator found across the
Himalayas the taxonomic status of which was a puzzle till late, has been assessed for the first
time in the International Union for Conservation of Nature (IUCN)’s Red List. The animal
has been categorised as ‘Vulnerable’ on the IUCN Red List. (Hence pair 1 is incorrect)

· Polar bears endemic to the region are listed as “vulnerable” in the International Union for
Conservation of Nature (IUCN) Red List of Endangered Species for its loss of habitat — sea ice. (
Hence pair 2 is incorrect)

· Lesser floricans are one of the four species of birds of the bustard family that occur in India.
They have been categorised as critically endangered s pecies on the Red List of Threatened
Species drawn by the International Union for Conservation of Nature (IUCN). (Hence pair 3 is
incorrect)

Important Points/Value Additions:

· IUCN report added that all individuals of Himalayan wolf were in one subpopulation stretching
across the Himalayan range of Nepal and India and across the Tibetan Plateau.

· Recently, Alaska has recorded the first case of a polar bear dying due to bird flu, alarming
experts about the already stressed extant species.

· The polar bear died from highly pathogenic avian influenza (HPAI), contracted by scavenging on
bird carcasses infected with the deadly strain H5N1. If the infection spreads, it could be
dangerous for the Arctic population which is already facing issues due to climate change.

· Lesser Floricans, the smallest birds of the bustard family known for their graceful mating display
and elusive females, are also powerful fliers that cross the Gulf of Khambhat in one go during their
annual migration from their breeding grounds in Gujarat to their wintering grounds in southern
India, a study by the Gujarat Forest Department has concluded

QUESTION 59. MTI3Mjk4K1J1cGVzaCBLdW1hcitydXBlc2hrci4wOWVlODBAZ21haWwuY29tK1FVRVN


USU9OIDU4
Consider the following statements about illegal wildlife trade of ‘Sturgeon fish’:

1. Sturgeon a fish native to Bulgaria, Romania, Serbia and Ukraine, produces caviar.

2. Volga is the last river body with functional populations of beluga, Russian, stellate and sterlet
sturgeons.

3. The Convention on International Trade in Endangered Species of Wild Fauna and Flora (CITES)
in listed the species as endangered.

4. IUCN imposed an international labelling system for all caviar products to curb illegal trade.

How many of the above statements are correct?

a) Only one

IASbaba
Score:
Web: http://ilp.iasbaba.com/
44.00 /
Email: ilp@iasbaba.com
Page 59 200
UPSC 2024 -
Exam Title : Environment,
Sc...
rupeshkr.
Email :
09ee80@gmail.com
Contact :

b) Only two
c) Only three
d) All four

Correct Answer: B
Your Answer: Unanswered Marks: 0/1.00
Explanation

Solution (b)

Explanation:

· Sturgeon, a fish native to countries such as Bulgaria, Romania, Serbia and Ukraine that
produces caviar, has been fished illegally, the researchers found. (Hence statement 1 is
correct)

· Danube is the last river body with functional populations of beluga (Huso huso), Russian
(Acipenser gueldenstaedtii), stellate (Acipenser stellatus) and sterlet (Acipenser ruthenus)
sturgeons, according to the authors of the study. (Hence statement 2 is incorrect)

· The Convention on International Trade in Endangered Species of Wild Fauna and Flora (CITES)
in 1998 listed the species as endangered and put restrictions on fishing of sturgeons from the
Danube and Black Sea. (Hence statement 3 is correct)

· In 2000, CITES even imposed an international labelling system for all caviar products to curb
illegal trade. (Hence statement 4 is incorrect)

Important Points/Value Additions:

· Intensive exploitation of the fish species along with habitat change, however, has pushed them to
the brink of extinction.

· Legal internationally traceable caviar and meat can only be sourced from farmed sturgeons

· Caviar and sturgeon trade urgently needs improvement to ensure that sturgeon populations will
have a future.

QUESTION 60. MTI3Mjk4K1J1cGVzaCBLdW1hcitydXBlc2hrci4wOWVlODBAZ21haWwuY29tK1FVRVN


USU9OIDU5
From which of the following country India received cheetah for reintroduction in India under
‘Project Cheetah’?

1. South Africa

2. Namibia

3. Iran

4. Botswana

How many of the above correct?

IASbaba
Score:
Web: http://ilp.iasbaba.com/
44.00 /
Email: ilp@iasbaba.com
Page 60 200
UPSC 2024 -
Exam Title : Environment,
Sc...
rupeshkr.
Email :
09ee80@gmail.com
Contact :

a) Only one
b) Only two
c) Only three
d) All four

Correct Answer: B
Your Answer: B Marks: 2.00/1.00
Explanation

Solution (b)

Explanation:

· The project flagged off on September, 2022 with 20 cheetahs arriving from South Africa and
Namibia.

· India’s Kuno National Park, Madhya Pradesh houses Cheetah reintroduction project which
witnessed a backlash over the deaths of cheetahs due to poor health and criticism of the
management team.

· Iran refused permission to send a male and female cheetah to India for even the research
purposes. Iran is the only country which has population of Asiatic Cheetah.

Important Points/Value Additions:

· South Africa may be the only country that would send cheetahs to India in the future for its
cheetah reintroduction project.

· India is planning to bring 12 to 14 cheetahs from South Africa after careful selection, taking due
care of health parameters and adaptability in the Indian habitat.

· Asiatic Cheetah is critically endangered while African is Vulnerable in IUCN red list.

QUESTION 61. MTI3Mjk4K1J1cGVzaCBLdW1hcitydXBlc2hrci4wOWVlODBAZ21haWwuY29tK1FVRVN


USU9OIDYw
Consider the following pairs:

New Species Unique feature

1. Omorgus Khandesh Role in forensic science, helps to detect the time of death

2. Badis limaakumi Ability to change colour and blend with surrounding

3. Micrixalus kottigeharensis Most threatened amphibian genus of India

How many of the above pairs are correct?

a) Only one
b) Only two
c) All three

IASbaba
Score:
Web: http://ilp.iasbaba.com/
44.00 /
Email: ilp@iasbaba.com
Page 61 200
UPSC 2024 -
Exam Title : Environment,
Sc...
rupeshkr.
Email :
09ee80@gmail.com
Contact :

d) None

Correct Answer: C
Your Answer: Unanswered Marks: 0/1.00
Explanation

Solution (c)

Explanation:

· A new beetle species (Omorgus Khandesh) has been discovered in India which is important for
forensic science as it helps detect the time of death of an animal or human. (Hence pair 1 is
correct)

· Scientists have recently discovered a new fish species from the Milak river, Nagaland. The newly
discovered species Badis limaakumi. It belongs to the Badis assamensis SG, which is characterised
by a distinct, dark opercular blotch.

· Fish from the Badis family are also known as chameleon fish for their ability to change colour.
This helps them blend with the surroundings when under stress. (Hence pair 2 is correct)

· The dancing frogs (Micrixalus kottigeharensis) that are endemic to the Western Ghats are the
most threatened amphibian genus of India, according to the Wildlife Trust of India based on the
second edition of the Global Amphibian Assessment released recently. (Hence
pair 3 is correct)

Important Points/Value Additions:

· Omorgus Khandesh is necrophagous and is, therefore, also called a keratin beetle. During the
decomposition of a body, blowflies are amongst the first ones to arrive in the early stages.
Meanwhile, the final successional stage is with the arrival of the keratin feeders, thus their
importance in forensic science.

· Recent discovery of chameleon fish species indicate rivers in Nagaland are poorly explored
compared to other Northeastern states. Discoveries highlight the need to increase efforts to
identify more unknown fish species.

· The dancing frogs that are found near the streams do a unique display to mate. The males
stretch up their hind legs one at a time and wave their webbed toes in the air in a rapid motion
akin to a dance. This is to attract mates as well as ward off competition, probably preferred
because their mating calls are drowned out by the gurgling of the streams. This act is called “foot
flagging” and gives the species their name.

QUESTION 62. MTI3Mjk4K1J1cGVzaCBLdW1hcitydXBlc2hrci4wOWVlODBAZ21haWwuY29tK1FVRVN


USU9OIDYx
Consider the following statements with respect to the recently released report titled ‘Elephant
Corridors of India, 2023’:

1. It mentioned an increase of 40 per cent of elephant corridors since 2010.

IASbaba
Score:
Web: http://ilp.iasbaba.com/
44.00 /
Email: ilp@iasbaba.com
Page 62 200
UPSC 2024 -
Exam Title : Environment,
Sc...
rupeshkr.
Email :
09ee80@gmail.com
Contact :

2. Karnataka has the highest number of identified elephant corridors in India, accounting for over
17% of all the reported elephant corridors in the country.

3. The Southern region harbors the largest elephant population in India.

How many of the above statements are correct?

a) Only one
b) Only two
c) All three
d) None

Correct Answer: B
Your Answer: C Marks: -0.66/1.00
Explanation

Solution (b)

Explanation:

· A total of 150 elephant corridors were reported from 15 elephant range states across the four
elephant-bearing regions of India. Elephant Task Force of the Government of India listed 88
corridors in to the Gajah report published during 2010. (Hence statement 1 is correct)

· West Bengal has the highest number of identified elephant corridors in India, accounting for over
17% of all the reported elephant corridors in the country. (Hence statement 2 is incorrect)

· Among the four elephant-bearing regions, nearly 35% of the elephant corridors were in the East-
central region, followed by 32% in the North-east region.

· The Southern region, which harbors the largest elephant population in India accounts for 21% of
the elephant corridors in India. (Hence statement 3 is correct)

· The Northern region that harbors the smallest of the four regional elephant populations, has the
least number of elephant corridors, accounting for 12% of all the reported elephant corridors in
the country.

Important Points/Value Additions:

· The expansion of range is also causing elephant deaths due to electrocution, falling in wells and
entering human-dominated agricultural landscapes.

· A corridor is supposed to be a small patch of land that provides connectivity for elephant
movement across habitats, largely within a landscape of the elephant reserve.

· Landscapes are partly fragmented due to agricultural contingencies corridors then serve as
crucial links between habitats spread across these landscapes.

QUESTION 63. MTI3Mjk4K1J1cGVzaCBLdW1hcitydXBlc2hrci4wOWVlODBAZ21haWwuY29tK1FVRVN


USU9OIDYy

IASbaba
Score:
Web: http://ilp.iasbaba.com/
44.00 /
Email: ilp@iasbaba.com
Page 63 200
UPSC 2024 -
Exam Title : Environment,
Sc...
rupeshkr.
Email :
09ee80@gmail.com
Contact :

Consider following pairs:

Protected Area State

1. Kaimur Wildlife Sanctury Bihar

2. Yeslur Forest Range Jharkhand

3. Asola Bhatti Wildlife Sanctury Punjab

How many of the above pairs are correct?

a) Only one
b) Only two
c) All three
d) None

Correct Answer: A
Your Answer: B Marks: -0.66/1.00
Explanation

Solution (a)

Explanation:

· The much-awaited second tiger reserve of Bihar is all set to come up either by the end of 2023 or
early 2024, according to the officials of the Bihar Forest, Environment and Climate Change
Department.

· The officials have been working towards obtaining the National Tiger Reserve
Conservation Authority’s (NTCA) approval for declaring Kaimur Wildlife Sanctuary as
the state’s second tiger reserve after the Valmiki Tiger Reserve (VTR). (Hence option 1 is
correct)

· The state of Karnataka’s beloved elephant Arjuna was killed in a freak encounter with a wild
tusker during an elephant capture operation in the Yeslur forest range in Hassan district .
The operation was part of a mass radio collaring of elephants as per an order by the forest
department. (Hence option 2 is incorrect)

· Restraining the Delhi government’s Forest department from holding a proposed event ‘Walk with
Wildlife’ inside the city’s Asola Bhatti Wildlife Sanctuary, the Delhi High Court said that the
department gave permission for the event in a mechanical manner. It added that the department
did not analyse the threat perception as the event has trappings of a misadventure.

· The high court in its order also remarked that while wildlife sanctuaries are made for the
preservation and protection of wildlife in their natural habitats, “lately, man has been encroaching
on the habitat of wildlife”. (Hence option 3 is incorrect)

Important Points/Value Additions:

· The surge in the tiger population, attributed to restrictions on poaching at the same time habitat
fragmentation and prey scarcity, has led to a notable rise in human-animal conflicts. Maharashtra
has witnessed a significant number of fatalities resulting from tiger attacks.

IASbaba
Score:
Web: http://ilp.iasbaba.com/
44.00 /
Email: ilp@iasbaba.com
Page 64 200
UPSC 2024 -
Exam Title : Environment,
Sc...
rupeshkr.
Email :
09ee80@gmail.com
Contact :

· Establishing additional tiger reserves, such as Kaimur, is a potential solution to address the
escalating conflicts. Additionally, designating the Yeslur forest range as a Wildlife Sanctuary is
imperative to manage and mitigate the frequent encounters between humans and animals,
particularly in the context of the fragmented landscape.

· The updated ‘Guidelines on Sustainable Eco-Tourism in Forest and Wildlife Areas’ mandate
promotion of eco-tourism based on scientific planning, including “demarcation of eco-tourism
zones upon assessment of management requirements of target wildlife, the habitat or geographic
entity and their behavioural and ecological entities”. It also mandates monitoring “stress on
wildlife vis-a-vis the number and pattern of tourist visitation”.

QUESTION 64. MTI3Mjk4K1J1cGVzaCBLdW1hcitydXBlc2hrci4wOWVlODBAZ21haWwuY29tK1FVRVN


USU9OIDYz
Ruminants are responsible for significant methane emission from human activity. Which of the
following animals is NOT a ruminant species?

a) Cow
b) Sheep
c) Horse
d) Goat

Correct Answer: C
Your Answer: B Marks: -0.66/1.00
Explanation

Solution (c)

Explanation:

· Ruminant animals, such as cows, sheep, goats, buffaloes, and others, are characterized by their
hooves and herbivorous grazing habits, engaging in the unique behavior of cud-chewing.

· This specialized adaptation allows these hoofed mammals to effectively break down plant
cellulose, extracting essential nutrients from the cell walls.

· A key aspect of their digestive process involves methane production. Ruminants possess
stomachs with four compartments, with the rumen being crucial for storing partially digested
food and facilitating the fermentation process.

· This intricate digestive system plays a vital role in the conversion of plant material into nutrients,
highlighting the remarkable efficiency of ruminant species in utilizing cellulose for their
nutritional needs.

· Horses are non-ruminant, simple-stomached herbivores. They are hindgut fermenters, meaning
the large intestine is the main site of fermentation of fibrous feedstuffs. This differs from ruminant
animals like cattle, goats, deer, and sheep, which are foregut fermenters with a rumen and
multicompartment stomach. (Hence option c is correct answer)

Important Points/Value Additions:

IASbaba
Score:
Web: http://ilp.iasbaba.com/
44.00 /
Email: ilp@iasbaba.com
Page 65 200
UPSC 2024 -
Exam Title : Environment,
Sc...
rupeshkr.
Email :
09ee80@gmail.com
Contact :

· Methane,
a potent greenhouse gas that is released mainly through burping of ruminant livestock.

· It is estimated that the ruminant digestive system is responsible for 27% of all methane
emissions from human activity.

QUESTION 65. MTI3Mjk4K1J1cGVzaCBLdW1hcitydXBlc2hrci4wOWVlODBAZ21haWwuY29tK1FVRVN


USU9OIDY0
What is the primary objective of the "MISHTI" program recently launched by the Government of
India?

a) To enhance urban infrastructure in coastal districts through sustainable practices.


b) To promote sustainable agriculture in mangrove-rich regions of India.
c) To establish a research alliance for climate change in coastal areas.
d) To undertake mangrove reforestation and afforestation along coastal districts.

Correct Answer: D
Your Answer: D Marks: 2.00/1.00
Explanation

Solution (d)

Explanation:

· Mangrove Initiative for Shoreline Habitats and Tangible Incomes (MISHTI) scheme was recently
launched by Government of India with an objective to take up mangrove reforestation and
afforestation along the coastal districts of India by adopting best practices that already exist in
India as well as from other countries including Indonesia. (Hence option d is correct)

· This program is also envisaged to develop mangrove associated ecotourism initiatives and
livelihood generation in coastal states.

· MISHTI will contribute to the efforts of ‘Mangrove Alliance for Climate (MAC)’, an
intergovernmental alliance for promotion of Mangrove, of which India became its active member
during the (COP27)

Important Points/Value Addition:

· Currently, there is approximately 5000 sq kms of area under mangroves and through the
MISHTI program an additional area of 540 sq km is proposed to be covered across 9 States and 4
Union Territories.

· The scheme is planned to be implemented for a period of five years. MISHTI is to be implemented
by converging CAMPA Fund, MGNREGS, and other sources.

QUESTION 66. MTI3Mjk4K1J1cGVzaCBLdW1hcitydXBlc2hrci4wOWVlODBAZ21haWwuY29tK1FVRVN


USU9OIDY1

IASbaba
Score:
Web: http://ilp.iasbaba.com/
44.00 /
Email: ilp@iasbaba.com
Page 66 200
UPSC 2024 -
Exam Title : Environment,
Sc...
rupeshkr.
Email :
09ee80@gmail.com
Contact :

Consider following with reference to concept of "planetary boundaries," as proposed by


environmental scientists, involves defining limits to human activities to prevent catastrophic
environmental changes:

1. Ocean Acidification

2. Land Use Change

3. Stratospheric Ozone Depletion

4. Global Freshwater Use

How many of the above are ‘planetary boundaries’?

a) Only one
b) Only two
c) Only three
d) All four

Correct Answer: D
Your Answer: D Marks: 2.00/1.00
Explanation

Solution (d)

Explanation:

· The planetary boundaries framework was first proposed by Johan Rockström and a group of 28
internationally renowned scientists in 2009 to define the environmental limits within which
humanity can safely operate to maintain Earth's stability and biodiversity.

· These nine Planetary Boundaries are Climate change, Change in biosphere integrity (biodiversity
loss and species extinction), Stratospheric ozone depletion, Ocean acidification, Biogeoche
mical flows (phosphorus and nitrogen cycles), Land-system change (for example deforestation),
Freshwater use (alterations across the entire water cycle over land), Atmospheric aerosol
loading (microscopic particles in the atmosphere that affect climate and living organisms),
Introduction of novel entities(consisting of microplastics, endocrine disruptors, and organic
pollutants). (Hence option d is correct answer)

Important Points/Value Additions:

· According to a new study, published in the journal Science Advances. The world has breached six
out of nine planetary boundaries necessary to maintain Earth's stability and resilience.

· Breaching of these boundaries doesn't indicate an immediate catastrophe but raises the risk of
irreversible environmental changes.

· This situation could lead to conditions on Earth that no longer support our current way of life.

QUESTION 67. MTI3Mjk4K1J1cGVzaCBLdW1hcitydXBlc2hrci4wOWVlODBAZ21haWwuY29tK1FVRVN


USU9OIDY2

IASbaba
Score:
Web: http://ilp.iasbaba.com/
44.00 /
Email: ilp@iasbaba.com
Page 67 200
UPSC 2024 -
Exam Title : Environment,
Sc...
rupeshkr.
Email :
09ee80@gmail.com
Contact :

What advanced technology is commonly employed in the electrolysis process for producing green
hydrogen, ensuring a higher efficiency and reduced environmental impact?

a) Proton Exchange Membrane (PEM) Electrolysis


b) Steam Methane Reforming
c) Coal gasification through carbon capture technology.
d) Haber-Bosch Process

Correct Answer: A
Your Answer: A Marks: 2.00/1.00
Explanation

Solution (a)

Explanation:

· Proton exchange membrane electrolysis has gained considerable attention as an energy


conversion system for hydrogen production. It is considered the preferred choice for green
hydrogen production owing to its energy efficiency, low capital cost, flexibility, safety, and
durability. (Hence option a is correct)

· Grey Hydrogen: It is produced via coal or lignite gasification (black or brown), or via a process
called steam methane reformation (SMR) of natural gas or methane. (Hence option b is
incorrect)

· Blue Hydrogen: It is produced via natural gas or coal gasification combined with carbon capture
storage (CCS) or carbon capture use (CCU) technologies to reduce carbon emissions. (Hence
option c is incorrect)

· One way of making green ammonia is by using hydrogen from water electrolysis and nitrogen
separated from the air. These are then fed into the Haber process (also known as Haber-Bosch), all
powered by sustainable electricity. (Hence option d is incorrect)

Important Points/Value Additions:

· India launched National Green Hydrogen mission to make India the Global Hub for production,
usage and export of Green Hydrogen and its derivatives. This will contribute to India’s aim to
become Aatmanirbhar through clean energy and serve as an inspiration for the global Clean
Energy Transition.

· The Mission will lead to significant decarbonisation of the economy, reduced dependence on fossil
fuel imports, and enable India to assume technology and market leadership in Green Hydrogen.

· In the Haber process, hydrogen and nitrogen are reacted together at high temperatures and
pressures to produce ammonia, NH3.

QUESTION 68. MTI3Mjk4K1J1cGVzaCBLdW1hcitydXBlc2hrci4wOWVlODBAZ21haWwuY29tK1FVRVN


USU9OIDY3
Consider the following criteria in context of 'unabated fossil fuels’:

IASbaba
Score:
Web: http://ilp.iasbaba.com/
44.00 /
Email: ilp@iasbaba.com
Page 68 200
UPSC 2024 -
Exam Title : Environment,
Sc...
rupeshkr.
Email :
09ee80@gmail.com
Contact :

1. It refers to fossil fuels produced and used without interventions that substantially reduce the
amount of GHG emitted throughout the life cycle.

2. Fossil fuel use which does not capture 100% CO2 from power plants.

3. Fossil fuel use which does not capture 50–80% of fugitive methane emissions from energy
production and supply.

Which of the above statements are correct?

a) 1 and 2
b) 2 and 3
c) 1 and 3
d) All of the above

Correct Answer: C
Your Answer: B Marks: -0.66/1.00
Explanation

Solution (c)

Explanation:

· AR6 benchmark report of the UN Intergovernmental Panel on Climate Change (IPCC) says
unabated fossil fuels are those “without interventions that substantially reduce” greenhouse gas
emissions. (Hence 1 st statement is correct )

· Capturing 90% or more CO2 from power plants. (Hence 2nd statement is incorrect )

· Or 50–80% of fugitive methane emissions from energy supply. (Hence 3 rd statement is


correct)

Important Points/Value Additions:

· In its report, Carbon Capture, Utilisation and Storage, the International Energy Agency (IEA)
said power and industrial plants that are equipped with modern CCS technologies capture around
90% of the CO2.

· However, a 2022 study by the Institute for Energy Economics and Financial Analysis (IEEFA) — a
global think tank that examines issues related to energy markets, trends, and policies — found
that most of the 13 flagship CCS projects worldwide that it analysed have either underperformed
or failed entirely.

· Another analysis by Climate Analytics, a Germany-based climate science and policy institute,
revealed that reliance on CCS could release an extra 86 billion tonnes of greenhouse gases into
the atmosphere between 2020 and 2050.

QUESTION 69. MTI3Mjk4K1J1cGVzaCBLdW1hcitydXBlc2hrci4wOWVlODBAZ21haWwuY29tK1FVRVN


USU9OIDY4

IASbaba
Score:
Web: http://ilp.iasbaba.com/
44.00 /
Email: ilp@iasbaba.com
Page 69 200
UPSC 2024 -
Exam Title : Environment,
Sc...
rupeshkr.
Email :
09ee80@gmail.com
Contact :

Consider the following statements in context of ‘GM Mustard’:

1. Hybridisation involves crossing two genetically dissimilar plant varieties that must be from
different species only.

2. The first-generation (F1) offspring from such crosses tend to have higher yields than what
either parent can individually give.

3. GM crops are derived from plants whose genes are artificially modified, usually by inserting
genetic material from another organism.

Which of the following statements are correct?

a) 1 and 2
b) 2 and 3
c) 1 and 3
d) All of the above

Correct Answer: B
Your Answer: B Marks: 2.00/1.00
Explanation

Solution (b)

Explanation:

· Hybridisation involves crossing two genetically dissimilar plant varieties that can even be
from the same species, not necessarily need different species. (Hence statement 1 is
incorrect)

· The first-generation (F1) offspring from such crosses tend to have higher yields than what either
parent can individually give. (Hence statement 2 is correct)

· GM crops are derived from plants whose genes are artificially modified, usually by inserting
genetic material from another organism, in order to give it new properties, such as increased
yield, tolerance to a herbicide, resistance to disease or drought, or improved nutritional value. (H
ence statement 3 is correct)

Important Points/Value Additions:

· Hybridisation isn’t easy in mustard, as its flowers have both female (pistil) and male (stamen)
reproductive organs, making the plants largely self-pollinating.

· Since the eggs of one plant cannot be fertilised by the pollen grains from another, it limits the
scope for developing hybrids unlike in cotton, maize or tomato, where this can be done through
simple emasculation or physical removal of anthers.

· By genetic modification (GM), Scientists at Delhi University’s Centre for Genetic Manipulation of
Crop Plants (CGMCP) have developed the hybrid mustard DMH-11 containing two alien
genes(Barnase and Barstar) isolated from a soil bacterium called Bacillus amyloliquefaciens.

· DMH-11 is an indigenously developed transgenic mustard. It is a genetically modified variant of


Herbicide Tolerant (HT) mustard.

IASbaba
Score:
Web: http://ilp.iasbaba.com/
44.00 /
Email: ilp@iasbaba.com
Page 70 200
UPSC 2024 -
Exam Title : Environment,
Sc...
rupeshkr.
Email :
09ee80@gmail.com
Contact :

· DMH-11 is a result of a cross between Indian mustard variety ‘Varuna’ and East European ‘Early
Heera-2’ mustard.

QUESTION 70. MTI3Mjk4K1J1cGVzaCBLdW1hcitydXBlc2hrci4wOWVlODBAZ21haWwuY29tK1FVRVN


USU9OIDY5
At the COP26 summit in Scotland's Glasgow, what significant aspect marked the ‘Glasgow pact’?

a) Introduction of new climate targets


b) Inclusion of nuclear energy in the pact
c) Phase down of coal and phase out of fossil fuels
d) Establishment of a global carbon tax system

Correct Answer: C
Your Answer: C Marks: 2.00/1.00
Explanation

Solution (c)

Explanation:

· Reached at the COP26 summit in Scotland’s Glasgow, the pact called for phase down of coal and
phase out of fossil fuel.

· This was the first time that a UN climate agreement explicitly mentioned coal. The pact also
marked the resolution of the deadlock over carbon markets. ( Hence option c is correct)

Important Points/Value Additions:

· COP is an international climate meeting organised annually by the United Nations (UN). COP is
short for Conference of the Parties. ‘Parties’ is a reference to (now) 198 countries that have joined
the international treaty called the UN Framework Convention on Climate Change (UNFCCC).

· ‘Parties’ to the treaty have pledged to take voluntary actions to prevent “dangerous
anthropogenic [human-caused] interference with the climate system.”

· Carbon markets are essentially trading systems in which carbon credits are sold and bought.
They allow countries, or industries, to earn carbon credits for the greenhouse gas emission
reductions they make in excess of their targets.

· These carbon credits can be traded to the highest bidder in exchange for money. The buyers of
carbon credits can show the emission reductions as their own and use them to meet their
reduction targets.

· One tradable carbon credit is equal to one tonne of carbon dioxide or the equivalent amount of a
different greenhouse gas reduced, sequestered or avoided. Once a credit is used to reduce,
sequester, or avoid emissions, it becomes an offset and can no longer be tradable.

IASbaba
Score:
Web: http://ilp.iasbaba.com/
44.00 /
Email: ilp@iasbaba.com
Page 71 200
UPSC 2024 -
Exam Title : Environment,
Sc...
rupeshkr.
Email :
09ee80@gmail.com
Contact :

QUESTION 71. MTI3Mjk4K1J1cGVzaCBLdW1hcitydXBlc2hrci4wOWVlODBAZ21haWwuY29tK1FVRVN


USU9OIDcw
Consider the following statements in context of ‘Global climate-change trends detected in
indicators of ocean ecology’:

1. It says one of the most affected areas is the tropical ocean near the equator, where the water is
turning from blue to green.

2. It suggests colour shift is more in regions where the oceans are getting more stratified.

3. Ocean appears blue and navy blue because blue and violet have much shorter wavelength
which get reflected back.

4. Ocean appear green due to existence of phytoplankton.

Which of the above statements are correct?

a) 1, 2 and 3
b) 2, 3 and 4
c) 3 and 4
d) All of the above

Correct Answer: D
Your Answer: Unanswered Marks: 0/1.00
Explanation

Solution (d)

Explanation:

· The study, ‘ Global climate-change trends detected in indicators of ocean ecology ’, was published
earlier in July in the journal Nature.

· The study says one of the most affected areas is the Tropical Ocean regions, near the equator,
where the water is turning from blue to green. (Hence statement 1 is correct)

· The findings suggest that a shift in colour is happening in those regions where the oceans are
getting more stratified. (Hence statement 2 is correct)

· In most regions across the world, the oceans appear blue or navy blue for a reason. This happens
due to “the absorption and scattering of light,” according to a report by NASA. (Hence
statement 3 is correct)

· In other parts of the world, the oceans appear green, which happens due to the existence of
phytoplankton on the upper surface of the water. (Hence statement 4 is correct)

Important Points/Value Additions:

· The colour of the Earth’s oceans has significantly altered over the past two decades, most likely
due to human-induced climate change, according to a new study. Over 56 per cent of the oceans,
more than the total land area on the planet, has experienced the shift in colour, it added.

IASbaba
Score:
Web: http://ilp.iasbaba.com/
44.00 /
Email: ilp@iasbaba.com
Page 72 200
UPSC 2024 -
Exam Title : Environment,
Sc...
rupeshkr.
Email :
09ee80@gmail.com
Contact :

· When the sunlight falls on deep and clear water, colours with longer wavelengths, such as red,
yellow and green, are absorbed by the water molecules but blue and violet, which have a much
shorter wavelength, are reflected back.

· where major rivers merge into the Atlantic Ocean, the ocean exudes a brown tint because of
dead leaves and sediments spewing from the rivers.

· In other parts of the world, the oceans appear green, which happens due to the existence of
phytoplankton on the upper surface of the water.

QUESTION 72. MTI3Mjk4K1J1cGVzaCBLdW1hcitydXBlc2hrci4wOWVlODBAZ21haWwuY29tK1FVRVN


USU9OIDcx
Consider the following statements about ‘Plastic Pollution’:

1. New marine microbial habitat has been named as the "plastisphere".

2. Only 30% of annual plastic waste get recycled.

3. Cigarette butts are the most common type of plastic waste found in the environment.

4. PET (Polyethylene terephthalate) is the world’s most common plastic and also the most
recyclable plastic.

Which of the above statements are correct?

a) 1 and 2
b) 1, 3 and 4
c) 2, 3 and 4
d) All of the above

Correct Answer: B
Your Answer: A Marks: -0.66/1.00
Explanation

Solution (b)

Explanation:

· Plastics including microplastics are now ubiquitous in our natural environment. They are
becoming part of the Earth's fossil record and a marker of the Anthropocene, our current
geological era. They have even given their name to a new marine microbial habitat called the
"plastisphere". (Hence statement 1 is correct)

· Only 9% of annual plastic waste recycled. (Hence statement 2 is incorrect)

· Cigarette butts — whose filters contain tiny plastic fibres — are the most common type of plastic
waste found in the environment. (Hence statement 3 is correct)

IASbaba
Score:
Web: http://ilp.iasbaba.com/
44.00 /
Email: ilp@iasbaba.com
Page 73 200
UPSC 2024 -
Exam Title : Environment,
Sc...
rupeshkr.
Email :
09ee80@gmail.com
Contact :

· Polyethylene terephthalate, the world’s most common plastic and also PET is the most recyclable
plastic and there is a strong market for its byproduct used to make drink bottles, food containers
or fibres for clothes. (Hence statement 4 is correct)

Important Points/Value Additions:

· Food wrappers, plastic bottles, plastic bottle caps, plastic grocery bags, plastic straws, and
stirrers are the next most common items after cigarettes butts. Many of us use these products
every day.

· But the harder plastics numbered 3-7 have a very small market since the value of the raw
material is lower than the cost of recycling.

QUESTION 73. MTI3Mjk4K1J1cGVzaCBLdW1hcitydXBlc2hrci4wOWVlODBAZ21haWwuY29tK1FVRVN


USU9OIDcy
“The National Park encompasses the catchments of Kiar, Nath and Kibar Nalas, all of which drain
south-west into Marwah River (also known as Marusudar River). The terrain is generally rugged
and steep with narrow valleys bounded by high ridges opening out in their upper glacial parts. It
encompasses breathtaking alpine meadow.”

The above description reflects which one of the following National Park? 127298

a) Kishtwar National Park


b) Dachigam National Park
c) Hemis National Park
d) Salim Ali National Park

Correct Answer: A
Your Answer: B Marks: -0.66/1.00
Explanation

Solution (a)

Explanation:

· Recently, the research team of the Department of Wildlife Protection has c onfirmed the
presence of snow leopard in Kishtwar High Altitude National Park through camera trap
photographs. (Hence option a is correct)

· Camera traps had captured two snow leopards in Nant Nullah area. In Marwah and Dachhan
areas, camera traps have captured images of three snow leopards.

Important Points/Value Additions:

· Snow leopard listed as vulnerable on the International Union for Conservation of Nature red list
in view of their depleting numbers.

· There are estimated to be fewer than 10,000 mature snow leopards across the globe.

IASbaba
Score:
Web: http://ilp.iasbaba.com/
44.00 /
Email: ilp@iasbaba.com
Page 74 200
UPSC 2024 -
Exam Title : Environment,
Sc...
rupeshkr.
Email :
09ee80@gmail.com
Contact :

· These animals usually live above the tree line on alpine meadows and in rocky regions at
elevations of 2,700 metres to 6,000 metres during summer. In winter, they come down to
elevations around 1,200 m to 2,000 m.

QUESTION 74. MTI3Mjk4K1J1cGVzaCBLdW1hcitydXBlc2hrci4wOWVlODBAZ21haWwuY29tK1FVRVN


USU9OIDcz
Consider following statements:

1. Balaram Ambaji and Jessore (Banaskantha district), Jambughoda (Panchmahal district),


Ratanmahal (Dahod district) are wildlife sanctuaries located in a state.

2. They offer safe homes to flagship species of sloth bears along with leopards, striped hyenas,
jackals, wolves, jungle cats, blue bulls, wild boars, Indian foxes etc.

3. CAG report highlighted widespread violations, poor management and encroachments.

Which of the following state is associated with above mentioned description?

a) Gujarat
b) Madhya Pradesh
c) Uttar Pradesh
d) Rajasthan

Correct Answer: A
Your Answer: D Marks: -0.66/1.00
Explanation

Solution (a)

Explanation:

· A series of violations and other discrepancies have been threatening wildlife in Gujarat,
especially in six sanctuaries managed by the state forest department, flagged the Comptroller
Auditor General of India (CAG).

· Six sanctuaries in Gujarat — Balaram Ambaji and Jessore (Banaskantha district); Jambughoda
(Panchmahal district), Ratanmahal (Dahod district), Shoolpaneshwar (Narmada district) and
Purna (Tapi and Dangs districts) — offer safe homes to sloth bears, leopards, striped hyenas,
jackals, wolves, jungle cats, blue bulls, wild boars, Indian foxes, Indian hares, common langurs,
reptiles, birds, barking deers, antelopes and chitals, among others.

· The Ratanmahal sanctuary hosts sloth bears, a flagship species in the area.

· Poor spending of funds has led to mismanagement, leaving the forests open for encroachment,
mining and deforestation, added CAG in its report tabled in the state assembly on September 16,
2023.

IASbaba
Score:
Web: http://ilp.iasbaba.com/
44.00 /
Email: ilp@iasbaba.com
Page 75 200
UPSC 2024 -
Exam Title : Environment,
Sc...
rupeshkr.
Email :
09ee80@gmail.com
Contact :

· Less than 1 per cent of the allotted fund was spent by the forest department in 2016-17. Though
the state government allocated Rs 85,557.78 crore, just Rs 1,000 crore or 0.78 per cent was
spent, CAG pointed out.

Important Points/Value Additions:

· Gujarat Bear Conservation and Welfare Action Plan did not develop any provisions for monitoring
the bear population. The department formed no ecotourism plans and lacked transparency in
proving how the pilgrimage areas such as Kedarnatha were regulated to keep the bears
undisturbed.

· State hasn’t declared any Critical Wildlife Habitats areas in national parks and sanctuaries
dedicated to wildlife conservation in the past 14 years since the implementation of the Scheduled
Tribes and Other Traditional Forest Dwellers (Recognition of Forest Rights) Act, 2006.

QUESTION 75. MTI3Mjk4K1J1cGVzaCBLdW1hcitydXBlc2hrci4wOWVlODBAZ21haWwuY29tK1FVRVN


USU9OIDc0
“It is one of the few and one of the smallest national parks located within a city. One of the few
remaining remnants of Southern Tropical Dry Evergreen Forests forms a pattern of mosaic with
grasslands and scrubs. Well-developed Banyan trees typically dominate the woodlands.
Blackbucks, Spotted Deer, Jackals, Palm Civets are well-adapted to open grasslands of diverse
sizes.”

Which of the following National Park is associated with above description?

a) Sanjay Gandhi National Park


b) Guindy National Park
c) Mount Harriet National Park
d) Bannerghatta National Park

Correct Answer: B
Your Answer: D Marks: -0.66/1.00
Explanation

Solution (b)

Explanation:

· Sanjay Gandhi National Park located in the northern part of Mumbai, this national park is known
for its biodiversity and the ancient Kanheri Caves. The most dominating habitat-type here is the
Forest habitat. Of flowering plants alone, an estimated 1300 species exist. Much of the forest here
conforms to the southern mixed-deciduous forest, dominated by the Tectona, Albizzia, Terminalia,
Holarrhena, Firmiana, Dalbergia etc. In fauna Spotted Deer, Sambhar, Barking Deer, Black-naped
Hare and Leopard dominate. (Hence option a is incorrect)

· Guindy National Park is situated in the city of Chennai. It is one of the smallest national parks in
India. It is home to various species of flora and fauna. It is one of the few national parks located
within a city. One of the few remaining remnants of Southern Tropical Dry Evergreen Forests
forms a pattern of mosaic with grasslands and scrubs in the National Park, but well-developed

IASbaba
Score:
Web: http://ilp.iasbaba.com/
44.00 /
Email: ilp@iasbaba.com
Page 76 200
UPSC 2024 -
Exam Title : Environment,
Sc...
rupeshkr.
Email :
09ee80@gmail.com
Contact :

Banyan trees typically dominate the woodlands. Blackbucks and Spotted Deer are well-adapted to
open grasslands of diverse sizes. (Hence option b is correct)

· Mount Harriet National Park is situated on the northern part of South Andaman Island,
approximately 15 kilometers by road from Port Blair, the capital city of the Andaman and Nicobar
Islands. The national park is known for its diverse flora and fauna. The forested area is home to a
variety of plant species, including orchids, ferns, and other tropical vegetation. The park is also
inhabited by various bird species, making it a destination for birdwatching. (Hence option c is
incorrect)

· Bannerghatta National Park is a national park in India, located near Bangalore, Karnataka. The
park has evergreen primary forests, and at Chiriyatapu the forest type is mixed deciduous, a
combination of primary and secondary forests. The three types of forests are categorized as
tropical evergreen, hilltop tropical evergreen and littoral. Overall 134 plant and tree species are
reported, including 74 native and 51 introduced species. (Hence option d is incorrect)

Important Points/Value Additions:

· Six oil-drenched spot-billed pelicans rescued from Ennore are under rehabilitation at Guindy
National Park and will be tagged before being released back into the wild.

· Following an oil spill in Kosasthalaiyar in early December, several birds were adversely affected.
A wildlife team comprising experts from Wildlife Trust of India, Besant Memorial Animal
Dispensary, and the Forest Department have been assessing the effect of the spill and found about
two dozen pelicans and painted storks to be heavily impacted.

QUESTION 76. MTI3Mjk4K1J1cGVzaCBLdW1hcitydXBlc2hrci4wOWVlODBAZ21haWwuY29tK1FVRVN


USU9OIDc1
Which of the following plant species are invasive?

1. Eucalyptus

2. Prosopis juliflora

3. Cassia Fistula

4. Lantana camara

5. Conocarpus

Choose correct code from below options.

a) 2, 3 and 4
b) 2, 3, 4 and 5
c) 1, 2, 4 and 5
d) All of the above

Correct Answer: C
Your Answer: C Marks: 2.00/1.00
Explanation

IASbaba
Score:
Web: http://ilp.iasbaba.com/
44.00 /
Email: ilp@iasbaba.com
Page 77 200
UPSC 2024 -
Exam Title : Environment,
Sc...
rupeshkr.
Email :
09ee80@gmail.com
Contact :

Solution (c)

Explanation:

· In the Western Ghats, where vast plantations of eucalyptus and wattle were raised in the past by
converting grasslands and shola forests, the original habitat of the Nilgiri Tahr has been
devastated. Nowadays, the Indian Bison makes frequent visits to the Kodaikanal town in Tamil
Nadu because of the non-availability of food plants due to extensive plantations of alien species. (
Hence 1 st is invasive)

· Recently, concerns over the management of the invasive Conocarpus species of trees have
recently led Gujarat and, last year, Telangana, to ban their use. (Hence 5 th is invasive)

· Several other states could follow suit which is likely to discourage horticulturists and nurseries
from multiplying the species and using lakhs of its saplings in afforestation and landscaping
projects across the country over the next year. Along with Conocarpus, Leucaena and Mangium
also invasive plant species.

· Prosopis juliflora and Lantana camara also dominate public lands and commons. (Hence 2 nd a
nd 4 th is invasive)

· Cassia fistula is a flowering plant in the family Fabaceae. The species is native to the Indian
subcontinent a nd adjacent regions of Southeast Asia. (Hence 3 rd is not invasive)

Important Points/Value Additions:

· Many alien species of flora, introduced mostly by the British, have multiplied to a great extent
mainly in the Western Ghats.

· Vast plantations of eucalyptus, wattle (Acacia) and pinus can be seen across the upper slopes of
the Nilgiris and Pulney hills interspersed with Lantana camara.

· Prosopis juliflora, Parthenium hysterophorus and Eupatorium odoratum can be seen on the lower
slopes. Most species such as Eucalyptus, Wattle (Acacia), introduced from Australia, have become
highly invasive.

· Cassia fistula is the official state flower of Kerala state in India. It is also a popular ornamental
plant and is also used in herbal medicine. Delhi’s heritage area has plantation of Cassia fistula.
Need to use indigenous species of ornamental purpose in common and public area.

QUESTION 77. MTI3Mjk4K1J1cGVzaCBLdW1hcitydXBlc2hrci4wOWVlODBAZ21haWwuY29tK1FVRVN


USU9OIDc2
Consider the following statements:

1. Ecocline is a zone of gradual but continuous change from one ecosystem to another.

2. Ecocide is ecological alternative to the chemical pesticides and herbicides.

3. Ecotone is a zone of junction or a transition area between two biomes.

IASbaba
Score:
Web: http://ilp.iasbaba.com/
44.00 /
Email: ilp@iasbaba.com
Page 78 200
UPSC 2024 -
Exam Title : Environment,
Sc...
rupeshkr.
Email :
09ee80@gmail.com
Contact :

How many of the above statements are correct?

a) Only one
b) Only two
c) All three
d) None

Correct Answer: B
Your Answer: C Marks: -0.66/1.00
Explanation

Solution (b)

Explanation:

· Ecocline is a zone of gradual but continuous change from one ecosystem to another when there
is no sharp boundary between the two in terms of species composition. (Hence statement 1 is
correct)

· Ecocide or ‘killing one’s home’ or ‘environment’ refers to acts like port expansion projects,
deforestation, illegal sand mining, polluting rivers and releasing untreated sewage, etc., that
destroy fragile natural ecosystems and local livelihoods. ( Hence statement 2 is incorrect)

· An ecotone is a zone of junction or a transition area between two biomes (diverse ecosystems. (
Hence statement 3 is correct)

· Ecotone is the zone where two communities meet and integrate. For e.g. the mangrove forests
represent an ecotone between marine and terrestrial ecosystem. Other examples are grassland
(between forest and desert), estuary (between fresh water and salt water) and riverbank or
marshland (between dry and wet).

Important Points/Value Additions:

· Ecocline occurs across the environmental gradient (gradual change in abiotic factors such as
altitude, temperature (thermocline), salinity (halocline), depth, etc.

· Ecocide means unlawful or wanton acts committed with knowledge that there is a substantial
likelihood of severe and either widespread or long-term damage to the environment being caused
by those acts.

· Ecotone has conditions intermediate to the adjacent ecosystems. Hence it is a zone of tension.
Usually, the number and the population density of the species of an outgoing community
decreases as we move away from the community or ecosystem. A well-developed ecotone contains
some organisms which are entirely different from that of the adjoining communities.

QUESTION 78. MTI3Mjk4K1J1cGVzaCBLdW1hcitydXBlc2hrci4wOWVlODBAZ21haWwuY29tK1FVRVN


USU9OIDc3
Arrange following National Park in North to South order:

1. Jim Corbett National Park

IASbaba
Score:
Web: http://ilp.iasbaba.com/
44.00 /
Email: ilp@iasbaba.com
Page 79 200
UPSC 2024 -
Exam Title : Environment,
Sc...
rupeshkr.
Email :
09ee80@gmail.com
Contact :

2. Dudhwa National Park

3. Govind Pashu Vihar National Park

4. Rajaji National Park

Choose correct code from below.

a) 1, 2, 3, 4
b) 2, 3. 4. 1
c) 3, 4, 1, 2
d) 4, 1, 2, 3

Correct Answer: C
Your Answer: A Marks: -0.66/1.00
Explanation

Solution (c)

Explanation:

· Three national parks are of Uttarakhand and only one is from Uttar Pradesh. Knowing that
national park of UP gives correct answer.

· As Dudhwa National Park is located in Uttar Pradesh and other three are north of it, option c is
correct.

Important Points/Value Additions:

· Govind Pashu Vihar National Park is located in Uttarkashi district, Uttarakhand. Nestled in the
western Himalayas is known for its diverse flora and fauna. It encompasses alpine meadows,
glaciers, and high-altitude landscapes, providing habitat to species like snow leopards, Himalayan
tahr, and numerous bird species.

· Rajaji National Park spread across the Shivalik ranges in Uttarakhand and named after C.
Rajagopalachari, the last Governor-General of India. It is a national park and tiger reserve, known
for its rich biodiversity, including elephants, tigers, leopards, and several species of deer. The park
is characterized by diverse landscapes, including river valleys and dense forests.

· Jim Corbett National Park located in the Nainital district of Uttarakhand. It is India's oldest
national park, established in 1936, renowned for its diverse wildlife, including Bengal tigers,
elephants, and various bird species. It is spread across the Himalayan foothills, encompassing
riverine belts, grasslands, and dense forests.

· Dudhwa National Park situated on the India-Nepal border in the Lakhimpur Kheri district of
Uttar Pradesh, India. It is a protected area known for its swamp deer population. Dudhwa
National Park features grasslands, dense forests, and the enchanting Terai ecosystem. The park is
home to tigers, leopards, and a variety of bird species.

IASbaba
Score:
Web: http://ilp.iasbaba.com/
44.00 /
Email: ilp@iasbaba.com
Page 80 200
UPSC 2024 -
Exam Title : Environment,
Sc...
rupeshkr.
Email :
09ee80@gmail.com
Contact :

QUESTION 79. MTI3Mjk4K1J1cGVzaCBLdW1hcitydXBlc2hrci4wOWVlODBAZ21haWwuY29tK1FVRVN


USU9OIDc4
Consider the following statements about IPCC’s AR6 Synthesis Report:

1. Over 3 billion people in highly vulnerable areas being 15 times more likely to die from climate-
related events compared to least vulnerable.

2. Existing adaptation gaps that will continue to grow due to limited resources, low climate
literacy, and a lack of political commitment.

3. It stresses need for climate-resilient development, reducing fossil fuel use, climate investments
and low-carbon lifestyles.

Which of the statements above are accurate?

a) 1 and 2
b) 2 and 3
c) 1 and 3
d) All of the above

Correct Answer: D
Your Answer: Unanswered Marks: 0/1.00
Explanation

Solution (d)

Explanation:

· Vulnerable communities, historically less responsible for climate change, are disproportionately
affected. It states that over three billion people in highly vulnerable areas are 15 times more likely
to die from climate-related events. (Hence statement 1 is correct.)

· Despite some progress in curtailing greenhouse gas emissions, there are existing adaptation
gaps. These gaps are attributed to factors such as limited resources, low climate literacy, and a
lack of political commitment. (Hence statement 2 is correct.)

· It stresses need climate-resilient development, reducing fossil fuel use, increasing finance for
climate investments, and facilitating low-carbon lifestyles. (Hence statement 3 is correct.)

Important Points/Value Additions:

· Intergovernmental Panel on Climate Change (IPCC), an UN-backed body of world’s leading


climate scientists, warned the world is on track to breach the 1.5-degree Celsius global warming
limit by the 2030s.

· Crossing the 1.5-degree Celsius limit, as agreed in the Paris Agreement, could lead to various
severe consequences, such as an unpredictable global water cycle, devastating floods, extreme
sea level events, and more intense tropical cyclones.

IASbaba
Score:
Web: http://ilp.iasbaba.com/
44.00 /
Email: ilp@iasbaba.com
Page 81 200
UPSC 2024 -
Exam Title : Environment,
Sc...
rupeshkr.
Email :
09ee80@gmail.com
Contact :

QUESTION 80. MTI3Mjk4K1J1cGVzaCBLdW1hcitydXBlc2hrci4wOWVlODBAZ21haWwuY29tK1FVRVN


USU9OIDc5
Eco bridges and Eco ducts were in news recently, consider following statements:

1. It aims to enhance wildlife connectivity and create contiguous look to landscape.

2. Uttarakhand has Eco bridge for reptiles and small animals.

3. Tiger, leopard used Eco bridge in Pench tiger reserve.

Which of the above statements are correct?

a) 1 and 2
b) 2 and 3
c) 1 and 3
d) All of the above

Correct Answer: D
Your Answer: A Marks: -0.66/1.00
Explanation

Solution (d)

Explanation:

· Eco-ducts or eco-bridges aim to enhance wildlife connectivity that can be disrupted because of
highways or logging. These include canopy bridges (usually for monkeys, squirrels and other
arboreal species); concrete underpasses or overpass tunnels or viaducts (usually for larger
animals); and amphibian tunnels or culverts. (Hence statement 1 is correct)

· Usually these bridges are overlaid with planting from the area to give it a contiguous look with
the landscape.

· Ramnagar Forest Division in Nainital district, Uttarakhand, recently built its first eco-bridge for
reptiles and smaller mammals. (Hence statement 2 is correct)

· A 2020 study by the Wildlife Institute of India (WII) noted that nearly 50,000 km of road projects
have been identified for construction in India over the next five to six years, while many highways
are being upgraded to four lanes.

· The National Tiger Conservation Authority, New Delhi, had identified three major sites that were
cutting across animal corridors, including National Highway 37 through the Kaziranga-Karbi
Anglong landscape in Assam, and State Highway 33 through the Nagarhole Tiger Reserve in
Karnataka.

· Cameras captured nearly 18 species that used these underpasses, including tiger, leopard, and
golden jackal. (Hence statement 3 is correct)

Important Points/Value Additions:

· Pench has 750m long bridge possibly the world’s largest underpass. Most species use this. In
this 750m bridge, the sloth bear took two years to cross, the wolf and the pangolin took less than a
year, while the spotted deer and jungle cat barely took a month.

IASbaba
Score:
Web: http://ilp.iasbaba.com/
44.00 /
Email: ilp@iasbaba.com
Page 82 200
UPSC 2024 -
Exam Title : Environment,
Sc...
rupeshkr.
Email :
09ee80@gmail.com
Contact :

· One of the largest underpasses – 1.4km – for animal conservation is India is being built along the
Madhya Pradesh-Maharashtra border.

QUESTION 81. MTI3Mjk4K1J1cGVzaCBLdW1hcitydXBlc2hrci4wOWVlODBAZ21haWwuY29tK1FVRVN


USU9OIDgw
Consider the following statements about ‘Hope Spot Network’:

1. It is a joint initiative of Conservation International and IUCN.

2. A hope spot is an area of high species richness at least 1500 species of vascular plants and high
degree of species endemism in terrestrial region.

3. Hope spots can only be Marine Protected Areas (MPAs).

How many of the above statements are correct?

a) One only
b) Two only
c) All three
d) None

Correct Answer: D
Your Answer: Unanswered Marks: 0/1.00
Explanation

Solution (d)

Explanation:

· Hope Spot Network is a joint initiative of Mission Blue & IUCN. (Hence statement 1 is
incorrect)

· A hope spot is an area of an ocean that needs special protection because of its wildlife &
significant underwater habitats. They are chosen for their contributions to biodiversity, the
carbon sink, and important habitat. (Hence statement 2 is incorrect)

· They can be Marine Protected Area's (MPA - defined by IUCN) that need attention or any new
sites. (Hence statement 3 is incorrect)

Important Points/Value Additions:

· Hope Spots allow us to plan for the future and look beyond current marine protected areas
(MPAs), which are like national parks on land where exploitative uses like fishing and deep-sea
mining are restricted.

· Hope Spots are often areas that need new protection, but they can also be existing MPAs where
more action is needed. They can be large, they can be small, but they all provide hope due to: A
special abundance or diversity of species, unusual or representative species, habitats or
ecosystems, Particular populations of rare, threatened or endemic species, A site with potential to
reverse damage from negative human impacts, The presence of natural processes such as major

IASbaba
Score:
Web: http://ilp.iasbaba.com/
44.00 /
Email: ilp@iasbaba.com
Page 83 200
UPSC 2024 -
Exam Title : Environment,
Sc...
rupeshkr.
Email :
09ee80@gmail.com
Contact :

migration corridors or spawning grounds, Significant historical, cultural or spiritual values,


Particular economic importance to the community.

· The idea is that anyone can nominate a site special to him or her—a site that gives HOPE.
Collectively all of these Hope Spots will create a global wave of community support for ocean
conservation that leaders and policymakers can’t ignore. Creating more Hope Spots and
supporting our existing Hope Spots allow us to move closer to achieving our goal of protecting
30% of the oceans by 2030.

QUESTION 82. MTI3Mjk4K1J1cGVzaCBLdW1hcitydXBlc2hrci4wOWVlODBAZ21haWwuY29tK1FVRVN


USU9OIDgx
‘Florianopolis Declaration’ is related to which of the following?

a) Promoting commercial whaling as a necessary economic activity


b) Allowing the indefinite protection of the world's whale population
c) Encouraging pro-whaling states to increase their activities
d) Providing binding regulations on commercial whaling activities

Correct Answer: B
Your Answer: D Marks: -0.66/1.00
Explanation

Solution (b)

Explanation:

· Florianopolis Declaration of 2018 reaffirms that the moratorium on commercial whaling, which
has been in effect since 1986, has contributed to the recovery of some whale populations, and
aware of cumulative effects of multiple, existing and emerging threats to whale populations such
as entanglement, bycatch, underwater noise, ship strikes, marine debris and climate change. (He
nce option b is correct)

· It Agrees that the role of the International Whaling Commission in the 21st Century includes
inter alia its responsibility to ensure the recovery of whale populations to their pre-industrial
levels, and in this context reaffirms the importance in maintaining the moratorium on commercial
whaling.

Important Points/Value Additions:

· Acknowledges the existence of an abundance of contemporary non-lethal cetacean research


methods and therefore agrees that the use of lethal research methods is unnecessary; Seeks to
ensure that aboriginal subsistence whaling for the benefit of indigenous communities should meet
the Commission's management and conservation objectives, taking into account the safety of
hunters and the welfare of cetaceans.

· Japan left IWC after adoption of this declaration and resumed commercial whaling.

IASbaba
Score:
Web: http://ilp.iasbaba.com/
44.00 /
Email: ilp@iasbaba.com
Page 84 200
UPSC 2024 -
Exam Title : Environment,
Sc...
rupeshkr.
Email :
09ee80@gmail.com
Contact :

QUESTION 83. MTI3Mjk4K1J1cGVzaCBLdW1hcitydXBlc2hrci4wOWVlODBAZ21haWwuY29tK1FVRVN


USU9OIDgy
Consider the following statements in context of Indian Forest Act 1927:

1. All activities are prohibited unless permitted in Protected Forests

2. All activities are permitted unless it is prohibited in Reserve Forests

3. State government can assign to any village community the rights of government over any land
constituted as reserved forest.

Which of the above statements are INCORRECT?

a) 1 and 2
b) 2 and 3
c) 1 and 3
d) All of the above.

Correct Answer: A
Your Answer: B Marks: -0.66/1.00
Explanation

Solution (a)

Explanation:

· State government may constitute any forest land or waste land as reserved forest which is the
property of Government & may sell the produce from these forest.

· Before issuing official notification state government needs to appoint a Forest Settlement Officer
(has power of Civil Courts) to inquire & to provide settlements to persons belonging to that area.

· All activities are prohibited unless permitted in reserve forests. (Hence statement 1 is
incorrect)

· Any land not included in Reserve Forest is considered as Protected Forest. Protected forests are
of 2 kinds: Demarcated protected forests and Undemarcated protected forests

· In protected forests, all activities are permitted unless it is prohibited. (Hence statement 2 is
incorrect)

· State government can assign to any village community the rights of government (to or over) any
land which has been constituted as reserved forest. (Hence statement 3 is correct)

· State government may make rules for regulating the management of village forest, prescribing
conditions under which the community may be provided with timber or other forest produce or
pasture.

Important Points/Value Additions:

· The Indian Forest Act of 1927, largely influenced by its predecessor from 1878, serves as a legal
framework for forest protection and management, outlining duties related to timber.

IASbaba
Score:
Web: http://ilp.iasbaba.com/
44.00 /
Email: ilp@iasbaba.com
Page 85 200
UPSC 2024 -
Exam Title : Environment,
Sc...
rupeshkr.
Email :
09ee80@gmail.com
Contact :

· It delineates procedures for designating areas as Reserve, Protected, or Village Forests, each
with varying degrees of protection.

· The 2017 amendment removed bamboo from the tree definition, allowing its economic use
without permission in non-forest areas.

QUESTION 84. MTI3Mjk4K1J1cGVzaCBLdW1hcitydXBlc2hrci4wOWVlODBAZ21haWwuY29tK1FVRVN


USU9OIDgz
Consider the following statements about ‘EDGE Species’:

1. The EDGE species have few or no close relatives on the tree of life.

2. EDGE species are usually extremely distinct in the way they look, live, and behave, as well as in
their genetic make-up.

Which of the above statements are correct?

a) 1 Only
b) 2 Only
c) Both
d) Neither

Correct Answer: C
Your Answer: B Marks: -0.66/1.00
Explanation

Solution (c)

Explanation:

· The EDGE have few or no close relatives on the tree of life. (Hence statement 1 is correct)

· EDGE species are usually extremely distinct in the way they look, live, and behave as well as in
their genetic make-up. (Hence statement 2 is correct)

Important Point/Value Addition

· The EDGE of Existence programme is a global conservation initiative to focus specifically on


threatened species that represent a significant amount of unique evolutionary history.

· If they disappear, there will be nothing like them left on the planet.

QUESTION 85. MTI3Mjk4K1J1cGVzaCBLdW1hcitydXBlc2hrci4wOWVlODBAZ21haWwuY29tK1FVRVN


USU9OIDg0
Consider the following statements in context of ‘Dark Sky Park’:

1. Pench Tiger Reserve becomes India’s first Dark Sky Park.

IASbaba
Score:
Web: http://ilp.iasbaba.com/
44.00 /
Email: ilp@iasbaba.com
Page 86 200
UPSC 2024 -
Exam Title : Environment,
Sc...
rupeshkr.
Email :
09ee80@gmail.com
Contact :

2. It is primarily driven by the need to shield the night sky from light pollution.

3. It is funded by the District Planning Community (DPC), designates the area for immersive
stargazing.

Which of the above statements are correct?

a) 1 and 2
b) 2 and 3
c) 1 and 3
d) All of the above

Correct Answer: D
Your Answer: Unanswered Marks: 0/1.00
Explanation

Solution (d)

Explanation:

· The Pench Tiger Reserve (PTR) in Maharashtra has attained the distinction of becoming India’s
inaugural Dark Sky Park and the fifth in Asia. (Hence statement 1 is correct)

· The decision to designate Pench as the first Dark Sky Park is driven by the need to shield the
night sky from light pollution. (Hence statement 2 is correct)

· As India’s first and Asia’s eighth Dark Sky Park, the Pench Tiger Reserve underscores its
commitment to preserving an unspoiled night sky.

· The recently inaugurated night sky observatory, funded by the District Planning Community
(DPC), designates the area near Bagholi for immersive stargazing. (Hence statement 3 is
correct)

· A dark-sky preserve is an area, typically surrounding a park or observatory, that limits artificial
light pollution.

Important Points/Value Additions:

· The primary objective of the dark-sky movement is generally to endorse astronomy.

· As part of the initiatives, over 100 street and community lights in Wagholi, Sillari, Pipariya, and
Khapa villages of the Paoni UC Range buffer area have been replaced with lights facing the
ground to alleviate light pollution.

· India’s first dark-sky preserve is the Indian Astronomical Observatory (IAO), a high-altitude
astronomy station situated in Hanle and operated by the Indian Institute of Astrophysics.

QUESTION 86. MTI3Mjk4K1J1cGVzaCBLdW1hcitydXBlc2hrci4wOWVlODBAZ21haWwuY29tK1FVRVN


USU9OIDg1
Consider the following statements about the 'Basel Ban Amendment' to the Basel Convention:

IASbaba
Score:
Web: http://ilp.iasbaba.com/
44.00 /
Email: ilp@iasbaba.com
Page 87 200
UPSC 2024 -
Exam Title : Environment,
Sc...
rupeshkr.
Email :
09ee80@gmail.com
Contact :

1. It required ratification by 3/4 of the parties to the Convention to become a law and after
Croatia’s ratification came into force.

2. The Amendment prohibits the export of hazardous waste, including electronic wastes, from a
list of developed countries to developing countries except for recycling.

3. India has ratified Basel Ban Amendment.

How many of the above statements are correct?

a) Only one
b) Only two
c) All three
d) None

Correct Answer: A
Your Answer: Unanswered Marks: 0/1.00
Explanation

Solution (a)

Explanation:

· Basel Ban Amendment required ratification by 3/4 of the parties to the Convention to become a
law. It finally became an international law in September 2019 after Croatia ratified it. (Hence
statement 1 is correct)

· The Amendment prohibits the export of hazardous waste (including electronic wastes) from a list
of developed (mostly OECD) countries to developing countries. The Basel Ban applies to export for
any reason, including recycling. (Hence statement 2 is incorrect)

· It will become a new Article in the Convention and will enter into force in the 97 countries after
90 days. However, countries like the US, Canada, Japan, Australia, - New Zealand, South Korea,
Russia, India, Brazil, and Mexico are yet to ratify the ban. (Hence statement 3 is incorrect)

Important Points/Value Additions:

· Many believed a full ban on shipment of hazardous waste was needed including exports for
recycling. This led to the adoption of an amendment to the Basel Convention in 1995 termed the
Basel Ban Amendment.

· The Basel Ban Amendment is hailed as a landmark agreement for global environmental justice.

· The US produces the most waste per-capita but has actively opposed the Ban Amendment.

· In 2021 Budget speech, Union finance minister spoke about doubling the ship recycling capacity
by 2024 and attracting more ships to India from Europe and Japan.

QUESTION 87. MTI3Mjk4K1J1cGVzaCBLdW1hcitydXBlc2hrci4wOWVlODBAZ21haWwuY29tK1FVRVN


USU9OIDg2

IASbaba
Score:
Web: http://ilp.iasbaba.com/
44.00 /
Email: ilp@iasbaba.com
Page 88 200
UPSC 2024 -
Exam Title : Environment,
Sc...
rupeshkr.
Email :
09ee80@gmail.com
Contact :

Which of the following committees are related to the protection of ‘Eco Sensitive Zones (ESZ)’ of
Western Ghat?

1. Madhav Gadgil Committee

2. K. Kasturirangan Committee

3. Shailesh Nayak Committee

Choose correct code.

a) 1 and 2
b) 2 and 3
c) 1 and 3
d) All of the above

Correct Answer: A
Your Answer: Unanswered Marks: 0/1.00
Explanation

Solution (a)

Explanation:

· In order to protect and conserve Western Ghats, MoEFCC had constituted two Committees viz.,
the Western Ghats Ecology Expert Panel (WGEEP) headed by Prof. Madhav Gadgil. (Hence 1 st i
s correct)

· The High Level Working Group (HLWG) headed by Dr. K. Kasturirangan, to review the
recommendations of the WGEEP. (Hence 2nd is correct)

· On the basis of the report of the HLWG, MoEFCC has brought out a draft notification, declaring
biologically and culturally diverse areas as Eco-sensitive Area, requiring protection and
conservation.

· The draft of Western Ghats Eco-sensitive Area was re-notified for the fifth time on 6th July, 2022
and a Committee has also been specifically constituted to assist in consultation with the State
Governments for finalizing the notification. The validity of the draft notification is till 30th June
2024.

· Shailesh Nayak committee was not related to western ghat and appointed to review Coastal
Regulation Zones. (Hence 3 rd is incorrect)

Important Points/Value Additions:

· The Ministry of Environment, Forest and Climate Change (MoEFCC) is implementing a number
of schemes for conservation of forest, wildlife and environment in different states and Union
Territories, including Western Ghats States.

· Funds under Compensatory Afforestation Management & Planning Authority are also utilized for
conservation of forest and wildlife in the Western Ghats.

IASbaba
Score:
Web: http://ilp.iasbaba.com/
44.00 /
Email: ilp@iasbaba.com
Page 89 200
UPSC 2024 -
Exam Title : Environment,
Sc...
rupeshkr.
Email :
09ee80@gmail.com
Contact :

QUESTION 88. MTI3Mjk4K1J1cGVzaCBLdW1hcitydXBlc2hrci4wOWVlODBAZ21haWwuY29tK1FVRVN


USU9OIDg3
What is the meaning of the term 'Dark Fermentation'?

a) It is a process aimed at decreasing atmospheric CO2 levels.


b) It is a technique for generating Hydrogen fuel from wastewater.
c) It is a method for the disposal of nuclear wastes.
d) It is a process for producing methane from organic waste.

Correct Answer: B
Your Answer: Unanswered Marks: 0/1.00
Explanation

Solution (b)

Explanation:

· Dark fermentation is the fermentative conversion of organic substrate to bio-hydrogen.

· It is a complex process manifested by diverse groups of bacteria, involving a series of


biochemical reactions using three steps similar to anaerobic conversion.

· Wastewater is used as a potential substrate for bio-hydrogen production in the dark


fermentation process.

Important Points/Value Additions:

· Dark fermentation is carried out by obligate anaerobes and facultative anaerobes in the absence
of light and oxygen.

· In dark fermentation, bacteria act on the substrate and generate hydrogen. The substrate for
the dark fermentation is lignocellulosic biomass, carbohydrate materials like wastewater from
industry, sugar-containing crop residues, and municipal solid waste.

· In the first step, the pretreatment of the biomass greatly affects the efficiency of the dark
fermentation. The other efficiency affecting parameters are the microorganism involved and the
sugar content of the substrate.

QUESTION 89. MTI3Mjk4K1J1cGVzaCBLdW1hcitydXBlc2hrci4wOWVlODBAZ21haWwuY29tK1FVRVN


USU9OIDg4
Consider the following statements about Corals:

1. Corals can reproduce either asexually or sexually.

2. Black corals are carnivores.

3. Reef building corals can be found in both shallow and deep waters across the oceans.

IASbaba
Score:
Web: http://ilp.iasbaba.com/
44.00 /
Email: ilp@iasbaba.com
Page 90 200
UPSC 2024 -
Exam Title : Environment,
Sc...
rupeshkr.
Email :
09ee80@gmail.com
Contact :

How many of the above statements are correct?

a) Only one
b) Only two
c) All three
d) None

Correct Answer: B
Your Answer: D Marks: -0.66/1.00
Explanation

Solution (b)

Explanation:

· Corals have multiple reproductive strategies – they can be male or female or both, and can
reproduce either asexually or sexually.

· Asexual reproduction is important for increasing the size of the colony, and sexual reproduction
increases genetic diversity and starts new colonies that can be far from the parents. (Hence
statement 1 is correct)

· Black corals are carnivores. Black corals can be found growing both in shallow waters and down
to depths of over 26,000 feet (8,000 meters), and some individual corals can live for over 4,000
years. (Hence statement 2 is correct)

· Black corals are filter feeders and eat tiny zooplankton that are abundant in deep waters. While
colourful shallow-water corals rely on the sun and photosynthesis for energy.

· Corals are found across the world’s ocean, in both shallow and deep water, but reef-building
corals are only found in shallow tropical and subtropical waters. (Hence statement 2 is
incorrect)

· This is because the algae found in their tissues need light for photosynthesis and they prefer
water temperatures between 70-85°F (22-29°C).

Important Points/Value Additions:

· Recently Scientists discovered five new species of black corals living as deep as 2,500 feet (760
meters) below the surface in the Great Barrier Reef and Coral Sea off the coast of Australia by
using a remote-controlled submarine.

· Black corals or antipatharians are colonial animals which are related to sea anemones and stony
corals. They are named for the colour of their stiff, black or brownish skeleton. More than 150
species of black corals have been described. At least 14 species of black corals are currently
known from Hawaii.

· Black corals are found in all oceans, but are most common in deep water habitats of tropical and
subtropical seas.

IASbaba
Score:
Web: http://ilp.iasbaba.com/
44.00 /
Email: ilp@iasbaba.com
Page 91 200
UPSC 2024 -
Exam Title : Environment,
Sc...
rupeshkr.
Email :
09ee80@gmail.com
Contact :

QUESTION 90. MTI3Mjk4K1J1cGVzaCBLdW1hcitydXBlc2hrci4wOWVlODBAZ21haWwuY29tK1FVRVN


USU9OIDg5
Consider the following statements about ‘Crystalline Silicon technology’:

1. It primarily utilizes organic materials for energy conversion.

2. It is limited to large-scale applications due to its efficiency.

3. It is exclusively employed in wind energy systems.

How many of the above statements are correct?

a) Only one
b) Only two
c) All three
d) None

Correct Answer: D
Your Answer: Unanswered Marks: 0/1.00
Explanation

Solution (d)

Explanation:

· Crystalline Silicon technology primarily uses crystalline semiconductor materials, not organic
polymers. (Hence statement 1 is incorrect)

· It is not limited to small-scale applications; it is widely used in both small-scale and large-scale
solar energy systems. (Hence statement 2 is incorrect)

· Crystalline Silicon solar cells are made from semiconductor materials with a crystalline
structure. There are two main types: monocrystalline silicon and polycrystalline silicon.

· It is used in both photovoltaic (PV) systems and concentrated solar power systems. (Hence
statement 3 is incorrect)

Important Points/Value Additions:

· International Solar Alliance’s- World Solar Technology Report 2023 focuses on solar
photovoltaics (solar PV), highlighting the remarkable advancements in crystalline silicon
technology.

· Solar PV has experienced exceptional growth, expected to capture 56.4% of total renewable
energy share by 2050.

· Crystalline silicon technology dominates the market with a 98% share; in particular,
monocrystalline and emerging technologies like organic PV and perovskite PV hold promise for the
future.

IASbaba
Score:
Web: http://ilp.iasbaba.com/
44.00 /
Email: ilp@iasbaba.com
Page 92 200
UPSC 2024 -
Exam Title : Environment,
Sc...
rupeshkr.
Email :
09ee80@gmail.com
Contact :

QUESTION 91. MTI3Mjk4K1J1cGVzaCBLdW1hcitydXBlc2hrci4wOWVlODBAZ21haWwuY29tK1FVRVN


USU9OIDkw
Consider the following statements about ‘Seaweeds’:

1. Seaweeds are specialised angiosperms (marine flowering plants).

2. Seaweeds are the only group of higher plants adapted to life in the salt water

3. Seaweeds are used as a drug for goiter treatment, intestinal and stomach disorders.

4. Products like agar-agar and alginates, iodine which are of commercial value, are extracted from
seaweeds.

How many of the above statements are correct?

a) Only one
b) Only two
c) Only three
d) All four

Correct Answer: B
Your Answer: D Marks: -0.66/1.00
Explanation

Solution (b)

Explanation:

· Sea grasses are specialised angiosperms (marine flowering plants) that resemble grass in
appearance. They produce flowers, have strap-like or oval leaves and a root system, whereas
Seaweeds are (thalloid plants) macroscopic algae, which mean they have no differentiation of true
tissues such as roots, Stems and leaves. (Hence statement 1 is incorrect)

· Sea grasses grow in shallow coastal waters with sandy or muddy bottoms & require
comparatively calm areas.

· Sea grasses are the only group of higher plants adapted to life in the salt water. (Hence
statement 2 is incorrect)

· Seaweeds are important as food for humans, feed for animals, and fertilizer for plants. Seaweeds
are used as a drug for goiter treatment, intestinal and stomach disorders. (Hence statement 3
is correct) Products like agar-agar and alginates, iodine which are of commercial value, are
extracted from seaweeds. (Hence statement 4 is correct)

Important Points/Value Additions:

· Major Sea grass meadows in India occur along the south east coast of Tamil Nadu and in the
lagoons of a few Lakshadweep Islands. There are few grass beds around Andaman and Nicobar
islands also. The rich growth of seagrasses along the Tamil Nadu coast and Lakshadweep islands is
mainly due to high salinity clarity of the water and sandy substratum.

· Seaweeds, the larger and visible marine plants are found attached to rocks, corals and other
submerged strata in the intertidal and shallow sub tidal zones of the sea.

IASbaba
Score:
Web: http://ilp.iasbaba.com/
44.00 /
Email: ilp@iasbaba.com
Page 93 200
UPSC 2024 -
Exam Title : Environment,
Sc...
rupeshkr.
Email :
09ee80@gmail.com
Contact :

QUESTION 92. MTI3Mjk4K1J1cGVzaCBLdW1hcitydXBlc2hrci4wOWVlODBAZ21haWwuY29tK1FVRVN


USU9OIDkx
Which of the following are ‘Phytoplanktons’?

1. Cyanobacteria

2. Silica-encased diatoms

3. Dinoflagellates

4. Green algae

5. Chalk-coated coccolithophores.

Choose correct code.

a) 1, 2, 3 and 4
b) 1, 3, 4 and 5
c) 1, 3 and 4
d) 1, 2, 3, 4 and 5

Correct Answer: D
Your Answer: C Marks: -0.66/1.00
Explanation

Solution (d)

Explanation:

· The term 'plankton' refers to the group of organisms which float in the surface waters of the
rivers, lakes and oceans.

· Derived from the Greek words phyto (plant) and plankton (made to wander or drift,
phytoplankton are microscopic plant organisms that live in aquatic environments, both salty and
fresh.

· Some phytoplankton are bacteria, some are protists, and most are single-celled plants. Among
the common kinds are cyanobacteria, silica-encased diatoms, dinoflagellates, green algae, and
chalk-coated coccolithophores. (Hence option d is correct)

Important Points/Value Additions:

· Like land plants, phytoplankton have chlorophyll to capture sunlight, and they use
photosynthesis to turn it into chemical energy. They consume carbon dioxide, and release oxygen.

· All phytoplankton photosynthesize, but some get additional energy by consuming other
organisms. These micro-algae are present throughout the lighted regions of all the seas and
oceans including the Polar Regions.

· Their total biomass is many times greater than that of the total plants on land and they serve as
the "pasture grounds" in the aquatic environment.

IASbaba
Score:
Web: http://ilp.iasbaba.com/
44.00 /
Email: ilp@iasbaba.com
Page 94 200
UPSC 2024 -
Exam Title : Environment,
Sc...
rupeshkr.
Email :
09ee80@gmail.com
Contact :

· Based on their size, phytoplankton can be classified as 1. macroplankton (more than 1 mm), 2.
microplankton (less than 1 mm, retained by nets of mesh size 0.06 mm), 3. nanoplankton (between
5 and 60 micrometers) and 4. ultraplankton (less than 5 micrometers).

QUESTION 93. MTI3Mjk4K1J1cGVzaCBLdW1hcitydXBlc2hrci4wOWVlODBAZ21haWwuY29tK1FVRVN


USU9OIDky
Consider the following statements about ‘Composite Index of Agricultural Sustainability (CIAS)’:

1. NITI Aayog has prepared a Composite Index of Agricultural Sustainability (CIAS).

2. It shows that Indian agriculture practices are moderately sustainable.

3. At present, most sustainable agriculture is practiced in Mizoram.

Which of the above statements are correct?

a) 1 and 2
b) 2 and 3
c) 1 and 3
d) All of the above

Correct Answer: B
Your Answer: Unanswered Marks: 0/1.00
Explanation

Solution (b)

Explanation:

· Indian Council for Agricultural Research have prepared a Composite Index of Agricultural
Sustainability (CIAS). (Hence statement 1 is incorrect)

· Index shows that Indian agriculture practices are moderately sustainable. (Hence statement 2
is correct)

· At present, the most sustainable agriculture is practised in Mizoram, Kerala, Andhra Pradesh,
Madhya Pradesh and West Bengal while agriculture in arid Rajasthan is the least sustainable. (H
ence statement 3 is correct)

Important Points/Value Additions:

· According to the index, the better-performing states have experienced sizable crop
diversification, improvement in agriculture infrastructure, farm credit and sustainable input use.

· The CIAS has 51 indicators to measure various patterns of agricultural sustainability. They are
categorised under sections of soil health, water resources, biodiversity and ecology and socio-
economic.

IASbaba
Score:
Web: http://ilp.iasbaba.com/
44.00 /
Email: ilp@iasbaba.com
Page 95 200
UPSC 2024 -
Exam Title : Environment,
Sc...
rupeshkr.
Email :
09ee80@gmail.com
Contact :

· States in the Indo-Gangetic Plains—Uttar Pradesh, Punjab, Bihar, and Haryana—as also in the
rice-dominant Jharkhand and Assam are at greater risk. These states produce much of India’s
staple, thereby putting food security at risk if sustainability is threatened.

QUESTION 94. MTI3Mjk4K1J1cGVzaCBLdW1hcitydXBlc2hrci4wOWVlODBAZ21haWwuY29tK1FVRVN


USU9OIDkz
Consider the following statements draughts in India:

1. India recognises and monitors only four types of droughts meteorological, hydrological,
agricultural and ecological.

2. Ecological drought is episodic deficit in water availability that drives ecosystems beyond
thresholds of vulnerability, impacts ecosystem services.

Which of the above statements are correct?

a) 1 Only
b) 2 Only
c) Both
d) Neither

Correct Answer: B
Your Answer: Unanswered Marks: 0/1.00
Explanation

Solution (b)

Explanation:

· India monitors three types of droughts – meteorological, hydrological and agricultural. (Hence
statement 1 is incorrect)

· Ecological drought defines it as “an episodic deficit in water availability that drives ecosystems
beyond thresholds of vulnerability, impacts ecosystem services and triggers feedbacks in natural
and/or human systems.” (Hence statement 2 is correct)

· What sets ecological drought apart is its emphasis on the interconnectedness between humans
and ecosystems within the context of drought.

Important Points/Value Additions:

· India has reported a drought at least once every three years in the last five decades. It has lost
between two and five percent of its GDP due to the impacts of drought, according to a 2022
report from the United Nations Convention on Combating Desertification (UNCCD), which also
included a global drought index that compared India’s drought vulnerability to that of sub-
Saharan Africa.

· A meteorological drought is when there is a shortfall in precipitation compared to the long term
average for a duration of time. The India Meteorological Department specifies that a

IASbaba
Score:
Web: http://ilp.iasbaba.com/
44.00 /
Email: ilp@iasbaba.com
Page 96 200
UPSC 2024 -
Exam Title : Environment,
Sc...
rupeshkr.
Email :
09ee80@gmail.com
Contact :

meteorological drought is a situation when the seasonal rainfall received over the area is less than
75% of its long term average value.

· It is further classified as “moderate drought” if the rainfall deficit is between 26-50% and
“severe drought” when the deficit exceeds 50% of the normal value.

· Rainfall is considered the biggest trigger of drought, but it isn’t the only factor that counts when
it comes to declaring a drought.

· Hydrological droughts are defined by low surface and sub-surface water supplies, such as in
streams, reservoirs and groundwater sources.

· They are a crucial indicator of drought conditions and can “arise even in times of average (or
above average) precipitation when increased usage of water diminishes the reserves,” says the
Ministry of Jal Shakti, India’s water authority.

· Agricultural drought is when a meteorological drought persists for four weeks, leading to
reductions in soil moisture and vegetation.

· Signs of distress, such as the sale of cattle, unavailability of fodder, out migration, limited
drinking water supply, higher demand for labour employment and price of commodities are
indications that the effects of drought have spilled over and affected livelihoods.

· For states to declare a drought, they must qualify for some of the criteria laid down by the
Drought Manual of 2016

QUESTION 95. MTI3Mjk4K1J1cGVzaCBLdW1hcitydXBlc2hrci4wOWVlODBAZ21haWwuY29tK1FVRVN


USU9OIDk0
Consider the following statements about Bears in India:

1. Sloth bear species is the smallest bear species found across the globe.

2. Himalayan brown Bear is least concerned based on its global stable population.

3. Recently a Tibetan brown bear has been spotted in India for the first time.

4. Sun Bear occurs in the North-eastern region is adapted to ‘myrmecophagy’.

How many of the above statements are correct?

a) Only one
b) Only two
c) Only three
d) All four

Correct Answer: B
Your Answer: Unanswered Marks: 0/1.00
Explanation

Solution (b)

IASbaba
Score:
Web: http://ilp.iasbaba.com/
44.00 /
Email: ilp@iasbaba.com
Page 97 200
UPSC 2024 -
Exam Title : Environment,
Sc...
rupeshkr.
Email :
09ee80@gmail.com
Contact :

Explanation:

· The Sun Bear (Helarctos malayanus) is mostly distributed in South-East Asia. In India it
occurs in the North-eastern region, though it is not common. It is the smallest of the eight
bear species found across the globe. T he Sun Bear is threatened by trade both for pets and
to provide various products. IUCN lists the Sun Bear as Vulnerable. (Hence statement 1 is
incorrect)

· The Brown Bear (Ursus arctos) is distinguished from the Himalayan Black Bear by its heavier
built and brown coat. IUCN lists this species as one of Least Concern based on its globally
stable population trend. However in much of its range in Asia (and India) it occurs patchily with
little information about its population or connectivity of these scattered pockets. It is speculated
that it is already extinct in Bhutan. (Hence statement 2 is correct)

· India has recorded its first-ever confirmed record of a rare Tibetan Brown Bear. The bear was
spotted in high altitudes of Northern Sikkim in the camera traps installed by the Sikkim Forest
Department and WWF-India. (Hence statement 3 is correct)

· The Sloth Bear (Melursus ursinus) is endemic to the Indian subcontinent and occurs in
India, Nepal, Bhutan and Sri Lanka.

· This particular species is the inspiration for the much loved character Baloo in Rudyard Kipling’s
Junglebook.

· The Sloth Bear is unique among the bear species in being adapted to ‘myrmecophagy’, i.e to
eating ants and termites, which form a large proportion of its diet. IUCN lists the Sloth Bear as
Vulnerable. (Hence statement 4 is incorrect)

Important Points/Value Additions:

· Tibetan Brown bear subspecies added to the India’s existing four species of bears Asiatic
Black(along with Himalayan black), Sloth, Sun and Himalayan Brown Bear.

· All Indian Bear species are listed under Appendix I in CITES and Schedule I of the Wildlife
(Protection) Act, 1972. This provides complete protection to the species from hunting and trade.

QUESTION 96. MTI3Mjk4K1J1cGVzaCBLdW1hcitydXBlc2hrci4wOWVlODBAZ21haWwuY29tK1FVRVN


USU9OIDk1
Consider the following statements in context of ‘Global Green Growth Institute (GGGI)’:

1. The Global Green Growth Institute (GGGI) is a treaty-based international, inter-governmental


organization.

2. It is headquartered in Seoul, South Korea.

3. It is dedicated to promoting green growth with its Green Growth Index.

4. Global Green Growth Institute and Energy Efficiency Services Ltd (EESL) will collaborate to
implement the Asia Low Carbon Buildings Transition (ALCBT) project.

How many of the above statements are correct?

IASbaba
Score:
Web: http://ilp.iasbaba.com/
44.00 /
Email: ilp@iasbaba.com
Page 98 200
UPSC 2024 -
Exam Title : Environment,
Sc...
rupeshkr.
Email :
09ee80@gmail.com
Contact :

a) Only one
b) Only two
c) Only three
d) All four

Correct Answer: D
Your Answer: B Marks: -0.66/1.00
Explanation

Solution (d)

Explanation:

· The Global Green Growth Institute (GGGI) is a treaty-based international, inter-governmental


organization dedicated to supporting and promoting strong, inclusive and sustainable economic
growth in developing countries and emerging economies. (Hence statement 1 is correct)

· It is based in Seoul, South Korea. (Hence statement 2 is correct)

· GGGI aims for global transition toward a model of green growth While this will be differentiated
at the country level, at its core will be strategies that simultaneously achieve poverty reduction,
social inclusion, environmental sustainability, and economic growth.

· Ensuring the world’s progress towards a green transformation requires comprehensive


measurement and tracking systems. With this report, GGGI is presenting a new platform for
measuring and tracking the green growth performance of countries worldwide through the Green
Growth Index. (Hence statement 3 is correct)

· Energy Efficiency Services Limited (EESL) in collaboration with the Global Green Growth
Institute (GGGI), an international intergovernmental organization, announced the launch of the
Asia Low Carbon Buildings Transition (ALCBT) Project in India. (Hence
statement 4 is correct)

Important Points/Value Additions:

· ALCBT- Under the guidance of the Ministry of Housing and Urban Affairs (MoHUA) as the nodal
ministry, this strategic initiative aims to combat the unprecedented surge in peak electricity
demand in the country, primarily propelled by the escalating need for cooling solutions.

· The Asia Low Carbon Buildings Transition (ALCBT) Project in collaboration with the Global Green
Growth Institute (GGGI) geared to address the challenges of surging electricity demand and
contribute significantly to carbon emission reduction.

· A key focus includes retrofitting air conditioning systems in 60 Indian buildings, demonstrating
our commitment to energy efficiency.

QUESTION 97. MTI3Mjk4K1J1cGVzaCBLdW1hcitydXBlc2hrci4wOWVlODBAZ21haWwuY29tK1FVRVN


USU9OIDk2
Consider the following statements related to the United Nations Convention on the Law of the
Seas (UNCLOS):

IASbaba
Score:
Web: http://ilp.iasbaba.com/
44.00 /
Email: ilp@iasbaba.com
Page 99 200
UPSC 2024 -
Exam Title : Environment,
Sc...
rupeshkr.
Email :
09ee80@gmail.com
Contact :

1. UNCLOS exclusively governs the management of maritime territories and exclusive economic
zones of individual countries.

2. The international ocean floor is not subject to UNCLOS unless a country has signed or ratified
the convention.

3. The "common heritage of mankind" principle applies to the seabed and its mineral resources as
per UNCLOS.

How many of the above statements are correct?

a) Only one
b) Only two
c) All three
d) None

Correct Answer: A
Your Answer: C Marks: -0.66/1.00
Explanation

Solution (a)

Explanation:

· Countries manage their own maritime territory and exclusive economic zones, while the high
seas and the international ocean floor are governed by the United Nations Convention on the Law
of the Seas. (Hence, statement 1 is incorrect)

· It is considered to apply to states regardless of whether or not they have signed or ratified it. (
Hence, statement 2 is incorrect)

· Under the treaty, the seabed and its mineral resources are considered the “common heritage of
mankind” that must be managed in a way that protects the interests of humanity through the
sharing of economic benefits, support for marine scientific research, and protecting marine
environments. (Hence, statement 3 is correct)

Important Points/Value Additions:

· The International Seabed Authority, the United Nations body that regulates the world’s ocean
floor is preparing to resume negotiations that could open the international seabed for mining,
including for materials critical for the green energy transition.

· Years long negotiations are reaching a critical point where the authority will soon need to begin
accepting mining permit applications, adding to worries over the potential impacts on sparsely
researched marine ecosystems and habitats of the deep sea.

· Mining companies interested in deep sea exploitation are partnering with countries to help them
get exploration licenses.

IASbaba
Score:
Web: http://ilp.iasbaba.com/
44.00 /
Email: ilp@iasbaba.com
Page 100 200
UPSC 2024 -
Exam Title : Environment,
Sc...
rupeshkr.
Email :
09ee80@gmail.com
Contact :

QUESTION 98. MTI3Mjk4K1J1cGVzaCBLdW1hcitydXBlc2hrci4wOWVlODBAZ21haWwuY29tK1FVRVN


USU9OIDk3
“Scientists have rediscovered a long-lost species of mammal described as having the spines of a
hedgehog, the snout of an anteater and the feet of a mole, more than 60 years after it was last
recorded. Attenborough’s long-beaked echidna, named after British naturalist David
Attenborough.”

In which of the following country above-described species of Echidna discovered?

a) Australia
b) Papua New Guinea
c) India
d) Indonesia

Correct Answer: D
Your Answer: A Marks: -0.66/1.00
Explanation

Solution (d)

Explanation:

· It is found in the Cyclops Mountains of Indonesian New Guinea.

· The reason it appears so unlike other mammals is because it is a member of the monotremes – an
egg-laying group that separated from the rest of the mammal tree-of-life about 200 million years
ago.

· The species has only been scientifically recorded once before, by a Dutch botanist in 1961. A
different echidna species is found throughout Australia and lowland New Guinea.

Important Points/Value additions:

· Echidnas share their name with a half-woman, half-serpent Greek mythological creature, and
were described by the team as shy, nocturnal burrow-dwellers who are notoriously difficult to
find.

QUESTION 99. MTI3Mjk4K1J1cGVzaCBLdW1hcitydXBlc2hrci4wOWVlODBAZ21haWwuY29tK1FVRVN


USU9OIDk4
Consider the following statements about Sundarban Tigers:

1. It is different subspecies of Panthera tigris tigris.

2. Sundarban tigers exhibit trait of man eater tigers more than other tigers.

3. They have adapted to a semi-aquatic lifestyle and are capable of hunting in water.

Which of the above statements are INCORRECT?

a) 1 and 2

IASbaba
Score:
Web: http://ilp.iasbaba.com/
44.00 /
Email: ilp@iasbaba.com
Page 101 200
UPSC 2024 -
Exam Title : Environment,
Sc...
rupeshkr.
Email :
09ee80@gmail.com
Contact :

b) 2 and 3
c) 1 and 3
d) All of the above

Correct Answer: A
Your Answer: A Marks: 2.00/1.00
Explanation

Solution (a)

Explanation:

· There are 5 subspecies of Tigers (current living population) that have been identified and they
are as follows Indian or Royal Bengal Tiger (Panthera tigris tigris), Indochinese Tiger (Panth
era tigris corbetti), Siberian (Amur) Tiger (Panthera tigris altaica), South China Tiger (Panth
era tigris amoyensis), Sumatran Tiger (Panthera tigris sumatrae). (Hence statement 1 is
incorrect)

· Considering the mutation rates that led to a genetic change, usually an animal that was isolated
for a period of 1 million years was classified as different species and one that was genetically
isolated for between 20,000 and 50,000 years was a different sub-species.

· In the case of Sundarban Tiger, it was a part of a contiguous region with others and was perhaps
separated about 500 to 1000 years ago.

· An estimate provided by the forest department claims that 410 people were attacked by tigers
between 1985 and 2010, leaving just 95 survivors. It is not established that Sunderban tigers are
distinctly more man-eaters. Statistics make it myth. (Hence statement 2 is incorrect)

· The Sundarban Tiger exhibits certain distinctive morphological adaptations that make it
particularly suited to the mangrove habitat of Sundarban islands and capable of hunting in water.
(Hence statement 1 is correct)

Important Points/Value Additions:

· The Tiger (Panthera tigris) is an endangered big cat that has been associated with Indian culture
and history since centuries. It is undeniably one of the most majestic animals alive. There is no
wild animal in the world that evokes feelings of great respect, fear, extreme curiosity or simply
that of Nature’s magic in the same breath.

· The Tiger – whose demographic status is uncertain across its entire distributional range, is
present in 13 Asian countries with 70% of the population being present in the Indian
subcontinent. Because of their large body size and carnivorous diet, tigers naturally occur at low
population densities.

QUESTION 100. MTI3Mjk4K1J1cGVzaCBLdW1hcitydXBlc2hrci4wOWVlODBAZ21haWwuY29tK1FVRV


NUSU9OIDk5
Which of the following best describes “Piscivores raptors”?

a) Predatory birds that feed primarily on small mammals.

IASbaba
Score:
Web: http://ilp.iasbaba.com/
44.00 /
Email: ilp@iasbaba.com
Page 102 200
UPSC 2024 -
Exam Title : Environment,
Sc...
rupeshkr.
Email :
09ee80@gmail.com
Contact :

b) Birds of prey specialized in hunting and consuming fish.


c) Raptors that exclusively target reptiles as their main food source.
d) Avian hunters with a preference for insects and arachnids.

Correct Answer: B
Your Answer: C Marks: -0.66/1.00
Explanation

Solution (b)

Explanation:

· Piscivores are organisms that primarily feed on fish. In the context of raptors, which are birds of
prey, piscivores are those that specialize in hunting and consuming fish. (Hence option b is
correct)

· These raptors often have adaptations such as sharp talons and keen eyesight, enabling them to
catch fish from water bodies. Examples of piscivorous raptors include ospreys and certain eagles.

Important Points/Value Additions:

· Grey-Headed Fish Eagles are experts at scooping their prey from water. Their species name,
ichthyaetus, is derived from the Greek ichthus for ‘fish’ and aetos for ‘eagle’.

· Piscivores usually have the capacity to deal with live prey that are large compared with their
own body size, and so they tend to have large gapes.

· Some swallow their prey whole, and others tear and bite the prey into smaller pieces. However,
the techniques vary widely between species within the demersal fish group.

· Piscivores generally have several rows of conical backwardly pointing teeth both on the jaws and
within the mouth and pharynx. These primarily prevent the prey from escaping and aid
swallowing.

IASbaba
Score:
Web: http://ilp.iasbaba.com/
44.00 /
Email: ilp@iasbaba.com
Page 103 200
UPSC 2024 -
Exam Title : Environment,
Sc...
rupeshkr.
Email :
09ee80@gmail.com
Contact :

Review in Hindi
QUESTION
1.
निम्नलिखित युग्मों पर विचार करें:

न्यूट्रिनो वेधशाला स्थान

1. ट्राइडेंट ( Trident ) दक्षिण चीन सागर

2. आइसक्यूब ( IceCube ) अंटार्क टिका

3. आईएनओ ( INO ) थेनी हिल्स , भारत

उपरोक्त में से कितने जोड़े सही हैं?

a) के वल एक
b) के वल दो
c) सभी तीन
d) कोई नहीं
Correct Answer: C
Your Answer:
Explanation

Q. 1) Solution (c)

स्पष्टीकरण:

· चीन दक्षिण चीन सागर के नीचे ट्राइडेंट नामक विश्व की सबसे बड़ी दूरबीन का निर्माण कर रहा है। इसका उपयोग ' अज्ञात कणों ( ghost
particles ) ' या न्यूट्रिनो का पता लगाने के लिए किया जाएगा । ( अतः जोड़ी 1 सही है )

· आइसक्यूब न्यूट्रिनो वेधशाला अंटार्क टिका के अमुंडसेन-स्कॉट साउथ पोल स्टे श न पर निर्मित एक न्यूट्रिनो वेधशाला है । यह परियोजना एक
मान्यता प्राप्त CERN प्रयोग है। इसके हजारों सेंसर अंटार्क टिक बर्फ के नीचे स्थित हैं, जो एक घन किलोमीटर में फैले हुए हैं। (अतः जोड़ी
2 सही है)

· आईएनओ ने तमिलनाडु के थेनी जिले के पोट्टीपुरम गांव के पास बोडी वेस्ट हिल्स (बीडब्ल्यूएच) क्षेत्र में एक साइट पर निर्णय लिया। (अतः
जोड़ी 3 सही है)

IMPORTANT POINTS/VALUE ADDITIONS:

· न्यूट्रिनो छोटे उपपरमाण्विक कण हैं। उन्हें ' अज्ञात / भूत कण' कहा जाता है क्योंकि वे किसी अन्य वस्तु के साथ मुश्किल से ही संपर्क करते
हैं, और इसलिए उनका पता लगाना बहुत मुश्किल होता है।

· न्यूट्रिनो में विद्युत आवेश नहीं होता है। वे विद्युत चुम्बकीय बलों से प्रभावित नहीं होते हैं। यह उन्हें न्यूनतम अंतःक्रिया के साथ अधिकांश
परमाणुओं सहित पदार्थ से गुजरने की अनुमति दे ता है।

· न्यूट्रिनो का द्रव्यमान इलेक्ट्रॉन या क्वार्क जैसे अन्य मूलभूत कणों की तुलना में बहुत छोटा होता है। न्यूट्रिनो के कमजोर आवेश के कारण
उनका निरीक्षण करना कठिन हो गया है।

QUESTION
2.
'छोटे मॉड्यूलर रिएक्टर' (SMR) के बारे में निम्नलिखित कथनों पर विचार करें:

IASbaba
Score:
Web: http://ilp.iasbaba.com/
44.00 /
Email: ilp@iasbaba.com
Page 104 200
UPSC 2024 -
Exam Title : Environment,
Sc...
rupeshkr.
Email :
09ee80@gmail.com
Contact :

1. पारंपरिक संयंत्र की तुलना में एसएमआर को अधिक बार ईंधन भरने की आवश्यकता होती है।

2. एसएमआर में उच्च शक्ति और परिचालन दाब होता है, जिससे एसएमआर की सुरक्षा कम हो जाती है।

3. एसएमआर की मॉड्यूलर प्रकृ ति स्थापना को कठिन और समय लेने वाली बनाती है।

उपरोक्त में से कितने कथन सही हैं?

a) के वल एक
b) के वल दो
c) सभी तीन
d) कोई नहीं
Correct Answer: D
Your Answer:
Explanation

Q. 2) Solution (d)

स्पष्टीकरण:

· एसएमआर पर आधारित बिजली संयंत्रों को पारंपरिक संयंत्रों के लिए 1 से 2 साल की तुलना में हर 3 से 7 साल में कम बार ईंधन भरने
की आवश्यकता हो सकती है। कु छ एसएमआर को ईंधन भरने के बिना 30 वर्षों तक संचालित करने के लिए डिज़ाइन किया गया है
। (इसलिए कथन 1 गलत है)

· एसएमआर में कम शक्ति और परिचालन दबाव होता है। वे पारंपरिक रिएक्टरों की तुलना में स्वाभाविक रूप से अधिक सुरक्षित हैं
क्योंकि उनकी सुरक्षा प्रणालियाँ प्राकृ तिक परिसंचरण, संवहन, गुरुत्वाकर्षण और आत्म- दाब पर निर्भर करती हैं। (इसलिए कथन 2 गलत
है)

· छोटे आकार और मॉड्यूलर प्रकृ ति एसएमआर को स्थापित करना आसान बनाती है। इन्हें उन स्थानों पर स्थापित किया जा सकता है जो बड़े
और पारंपरिक रिएक्टरों के लिए अनुपयुक्त हैं। (इसलिए कथन 3 गलत है)

IMPORTANT POINTS/VALUE ADDITIONS:

· छोटे मॉड्यूलर रिएक्टर (एसएमआर) परमाणु रिएक्टरों की एक श्रेणी का प्रतिनिधित्व करते हैं जो पारंपरिक बड़े पैमाने के परमाणु ऊर्जा
संयंत्रों से आकार में भिन्न होते हैं। उनका डिज़ाइन लचीलेपन, मापनीयता और विविध वातावरणों में आसान तैनाती पर जोर दे ता है।

· मॉड्यूलर पहलू एक कारखाने में विभिन्न एसएमआर मॉड्यूल के स्वतंत्र निर्माण की अनुमति दे ता है, जिसके बाद स्थापना स्थल तक परिवहन
होता है। इसके विपरीत, पारंपरिक परमाणु रिएक्टर, जो काफी भारी होते हैं, आमतौर पर साइट पर ही बनाए जाते हैं।

· एसएमआर प्रति यूनिट 300 मेगावाट तक की बिजली क्षमता का दावा करते हैं, जो पारंपरिक परमाणु रिएक्टरों की क्षमता का लगभग एक
तिहाई है, जो आम तौर पर 700 से 800 मेगावाट तक होती है।

QUESTION
3.
ई-सिम (एम्बेडेड सब्सक्राइबर आइडेंटिफिके शन मॉड्यूल) के संदर्भ में, निम्नलिखित कथनों पर विचार करें:

1. मोबाइल फोन डिवाइस में बदलाव से फोन नंबर में बदलाव आएगा।

2. यह आसानी से उपयोगकर्ताओं की गोपनीयता का उल्लंघन कर सकता है, क्योंकि सेवा प्रदाता उपयोगकर्ता गतिविधि को ट्रैक कर सकता
है।

3. ई - सिम्स को दोबारा प्रोग्राम करना ( re-programme ) संभव नहीं है ।

IASbaba
Score:
Web: http://ilp.iasbaba.com/
44.00 /
Email: ilp@iasbaba.com
Page 105 200
UPSC 2024 -
Exam Title : Environment,
Sc...
rupeshkr.
Email :
09ee80@gmail.com
Contact :

उपरोक्त में से कितने कथन सही हैं?

a) के वल एक
b) के वल दो
c) सभी तीन
d) कोई नहीं
Correct Answer: B
Your Answer:
Explanation

Q. 3) Solution (b)

स्पष्टीकरण:

· डिवाइस स्विच करना अधिक चुनौतीपूर्ण हो जाता है, क्योंकि मोबाइल फोन डिवाइस में बदलाव के परिणामस्वरूप फोन नंबर में भी बदलाव
होता है। (अतः कथन 1 सही है)

· मोबाइल सेवा प्रदाता अत्यधिक सटीकता के साथ उपयोगकर्ता की गतिविधियों पर नजर रखने की क्षमता हासिल कर लेते हैं, जिससे मजबूत
गोपनीयता सुरक्षा कानूनों की कमी होने पर चिंताएं बढ़ जाती हैं। (अतः कथन 2 सही है)

· जबकि यूजर्स के पास eSIM री - प्रोग्राम करने का विकल्प होता है, कम डिजिटल साक्षरता वाले व्यक्तियों को ऐसा करने में
कठिनाइयों का सामना करना पड़ सकता है। (इसलिए कथन 3 गलत है)

IMPORTANT POINTS/VALUE ADDITIONS:

· एक eSIM , जो एम्बेडेड सिम का संक्षिप्त रूप है, पारंपरिक सिम कार्ड का एक डिजिटल या इलेक्ट्रॉनिक समकक्ष है। भौतिक, हटाने
योग्य कार्ड के विपरीत, इसे स्मार्टफोन, टै बलेट या स्मार्टवॉच जैसे उपकरणों में एकीकृ त किया जाता है।

· इसका प्राथमिक कार्य पारंपरिक सिम कार्ड के साथ संरेखित होता है, जो उपयोगकर्ता को मोबाइल नेटवर्क की पहचान करने और
कनेक्टिविटी की सुविधा प्रदान करता है।

· eSIM उन्नत सुरक्षा प्रदान करते हैं, खो जाने या हटाए जाने की संभावना कम हो जाती है, जिससे सिम कार्ड सक्रियण की प्रतीक्षा किए
बिना नए उपयोगकर्ताओं के लिए ऑनबोर्डिंग प्रक्रिया सरल हो जाती है।

· eSIM की बढ़ती स्वीकार्यता से दूरसंचार सेवा प्रदाताओं के लिए दूरस्थ स्थानों पर सिम कार्ड भेजने की आवश्यकता कम या समाप्त हो
जाएगी।

· eSIM की पुन: प्रोग्राम करने योग्य प्रकृ ति नए सिम के उत्पादन में अतिरिक्त प्लास्टिक और धातु की आवश्यकता को समाप्त कर दे ती है।

QUESTION
4.
पृथ्वी विज्ञान मंत्रालय के 'डीप ओशन मिशन' के संदर्भ में, निम्नलिखित कथनों पर विचार करें:

1. संयुक्त राष्ट्र द्वारा वर्ष 2021-2030 को 'समुद्र विज्ञान दशक' के रूप में नामित किया गया है।

2. डीप ओशन मिशन प्रधान मंत्री के विज्ञान, प्रौद्योगिकी और नवाचार सलाहकार परिषद ( PMSTIAC) के तहत नौ मिशनों में से एक है।

3. मत्स्य 6000 एक गहरे समुद्र में चलने वाली पनडु ब्बी है जिसे वैज्ञानिक सेंसर और उपकरणों से सुसज्जित तीन सदस्यों के दल को
समायोजित करने के लिए डिज़ाइन किया गया है।

4. जैसे-जैसे हम समुद्र की सतह से गहराई में उतरते हैं, उत्तरोत्तर घटते दाब के कारण गहरे समुद्र की खोज एक महत्वपूर्ण चुनौती पेश करती
है।

IASbaba
Score:
Web: http://ilp.iasbaba.com/
44.00 /
Email: ilp@iasbaba.com
Page 106 200
UPSC 2024 -
Exam Title : Environment,
Sc...
rupeshkr.
Email :
09ee80@gmail.com
Contact :

उपरोक्त में से कितने कथन सही हैं?

a) के वल एक
b) के वल दो
c) के वल तीन
d) सभी चार
Correct Answer: C
Your Answer:
Explanation

Q. 4) Solution (c)

स्पष्टीकरण:

· वर्ष 2021-2030 को संयुक्त राष्ट्र द्वारा 'समुद्र विज्ञान दशक' के रूप में नामित किया गया है, और प्रधान मंत्री नरेंद्र मोदी ने कई अवसरों
पर,
भारत को राष्ट्र के विकास के लिए महासागर की क्षमता का निरंतर दोहन करने की दिशा में काम करने की आवश्यकता पर जोर दिया है। (
अतः कथन 1 सही है)

· DOM प्रधान मंत्री के विज्ञान, प्रौद्योगिकी और नवाचार सलाहकार परिषद (PMSTIAC) के तहत नौ मिशनों में से एक है। यह जरूरी है
कि DOM भारत में नीली-अर्थव्यवस्था प्राथमिकता क्षेत्र, नीले व्यापार और नीले विनिर्माण का समर्थन करे। (अतः कथन 2 सही है)

· मत्स्य 6000, एक गहरे समुद्र में चलने वाली पनडु ब्बी है जिसे वैज्ञानिक सेंसर और उपकरणों के एक सूट से सुसज्जित तीन सदस्यों के दल
को समायोजित करने के लिए डिज़ाइन किया गया है। ( अतः कथन 3 सही है )

· बाहरी अंतरिक्ष की खोज की तुलना में महासागरों की गहराई की खोज करना अधिक चुनौतीपूर्ण साबित हुआ है। मूलभूत अंतर गहरे
महासागरों में उच्च दाब के साथ है। जबकि बाहरी अंतरिक्ष लगभग पूर्ण निर्वात के समान है, एक मीटर पानी के नीचे रहने से एक वर्ग मीटर
क्षेत्र की वस्तु पर उतना ही दाब पड़ता है जितना कि वह लगभग 10,000 किलोग्राम वजन ले जा रहा हो, जो एक विशाल वयस्क हाथी के
बराबर है।

· ऐसी उच्च दाब वाली परिस्थितियों में संचालन के लिए कठोर धातुओं या सामग्रियों से तैयार किए गए सावधानीपूर्वक डिजाइन किए गए
उपकरणों के उपयोग की आवश्यकता होती है। इसके अतिरिक्त, इलेक्ट्रॉनिक्स और उपकरणों को निर्वात या अंतरिक्ष में कार्य करना आसान
लगता है। इसके विपरीत, पानी के अंदर, खराब डिज़ाइन वाली वस्तुएं ढह जाती हैं या फट जाती हैं। (अतः कथन 4 ग़लत है)

IMPORTANT POINTS/VALUE ADDITIONS:

· इसे समुद्रयान मिशन के रूप में भी जाना जाता है, जिसका उद्दे श्य गहरे समुद्र की खोज के लिए वैज्ञानिक सेंसर और उपकरणों के एक सेट
के साथ समुद्र में 6000 मीटर की गहराई तक 3 मनुष्यों को ले जाने के लिए एक स्व-चालित मानव चालित पनडु ब्बी विकसित करना है।
इसकी परिचालन अवधि 12 घंटे और आपातकाल के मामले में 96 घंटे है।

· अनुमानित समयसीमा 2020-2021 से 2025-2026 की अवधि के लिए पांच वर्ष है।

· नेश नल इंस्टीट्यूट ऑफ ओशन टे क्नोलॉजी (एनआईओटी), चेन्नई, एमओईएस के तहत एक स्वायत्त संस्थान, ने 6000 मीटर गहराई से
संचालित रिमोट संचालित वाहन (आरओवी) और विभिन्न अन्य पानी के नीचे के उपकरण जैसे ऑटोनॉमस कोरिंग सिस्टम (एसीएस),
ऑटोनॉमस अंडरवाटर व्हीकल (एयूवी) और गहरे समुद्र की खोज के लिए समुद्री खनन प्रणाली (डीएसएम) विकसित किया है।

QUESTION
5.
वायरल रोगों की वृद्धि को रोकने के लिए उपयोग की जाने वाली ' वोल्बाचिया विधि ( Wolbachia method ) ' के संबंध में निम्नलिखि
त कथनों पर विचार करें :

1. वोल्बाचिया एक सामान्य कवक है जो प्राकृ तिक रूप से कीट प्रजातियों में पाया जाता है।

IASbaba
Score:
Web: http://ilp.iasbaba.com/
44.00 /
Email: ilp@iasbaba.com
Page 107 200
UPSC 2024 -
Exam Title : Environment,
Sc...
rupeshkr.
Email :
09ee80@gmail.com
Contact :

2. वोल्बाचिया एडीज एजिप्टी मच्छरों के शरीर में डेंगू, चिकनगुनिया और जीका जैसे वायरस को फैलने से रोकता है।

3. वोल्बाचिया मनुष्यों, जानवरों और पर्यावरण के लिए सुरक्षित है।

उपरोक्त में से कितने कथन सही हैं?

a) के वल एक
b) के वल दो
c) सभी तीन
d) कोई नहीं
Correct Answer: B
Your Answer:
Explanation

Q. 5) Solution (b)

स्पष्टीकरण:

· वोल्बाचिया एक प्रचलित जीवाणु है जो मच्छरों, फल मक्खियों, पतंगों, ड्रैगनफलीज़ और तितलियों सहित लगभग 60% कीट प्रजातियों
में प्राकृ तिक रूप से पाया जाता है। (इसलिए कथन 1 ग़लत है)

· वोल्बाचिया एडीज एजिप्टी मच्छरों के शरीर के भीतर डेंगू, चिकनगुनिया और जीका जैसे वायरस के विकास को प्रभावी ढं ग से रोकता है।
(अतः कथन 2 सही है)

· विशेष रूप से, वोल्बाचिया को मनुष्यों, जानवरों और पर्यावरण के लिए सुरक्षित माना जाता है। (अतः कथन 3 सही है)

IMPORTANT POINTS/VALUE ADDITIONS:

· वोल्बाचिया आमतौर पर एडीज एजिप्टी मच्छर में अनुपस्थित होता है, कु छ अध्ययनों ने कु छ आबादी में इसकी उपस्थिति का संकेत दिया
है। एडीज एजिप्टी जीका, डेंगू, चिकनगुनिया और पीला बुखार जैसे मानव वायरस फै लाने के लिए जिम्मेदार प्राथमिक प्रजाति है।

· वोल्बाचिया विधि में, एडीज एजिप्टी मच्छरों को जानबूझकर प्रयोगशाला सेटिंग में जीवाणु से संक्रमित किया जाता है और बाद में जंगल में
छोड़ दिया जाता है।

· वोल्बाचिया-संक्रमित मच्छर अपने जंगली समकक्षों के साथ संभोग करते हैं, जिससे मच्छरों की आबादी में जीवाणु की उपस्थिति बढ़ जाती
है।

· वोल्बाचिया का बढ़ता प्रचलन वायरस के लिए मच्छरों के भीतर प्रजनन को चुनौतीपूर्ण बना दे ता है, क्योंकि जीवाणु प्रभावी रूप से उनके
विकास को रोकता है।

· जैसे-जैसे वोल्बाचिया-संक्रमित मच्छरों की आबादी बढ़ती है, वायरस संचरण की संभावना काफी कम हो जाती है।

QUESTION
6.
'ऑरोरा प्रकाश ' के संदर्भ में निम्नलिखित कथनों पर विचार करें:

1. अरोरा तब उत्पन्न होता है जब सूर्य से विद्युत-आवेशि त कण पृथ्वी के वायुमंडल में मौजूद ऑक्सीजन और नाइट्रोजन जैसी गैसों के कणों से
टकराते हैं।

2. यह हमारे सौर मंडल में के वल पृथ्वी पर ही दे खा जाता है, क्योंकि अन्य ग्रहों के ध्रुवीय क्षेत्रों में घने वातावरण का अभाव है।

उपरोक्त में से कौन सा कथन सही है?

IASbaba
Score:
Web: http://ilp.iasbaba.com/
44.00 /
Email: ilp@iasbaba.com
Page 108 200
UPSC 2024 -
Exam Title : Environment,
Sc...
rupeshkr.
Email :
09ee80@gmail.com
Contact :

a) के वल 1
b) के वल 2
c) दोनों
d) कोई भी नहीं
Correct Answer: A
Your Answer:
Explanation

Q. 6) Solution (a)

स्पष्टीकरण:

· अरोरा एक प्राकृ तिक घटना है जो आकाश में प्राकृ तिक रंग (हरा, लाल, पीला या सफे द) प्रकाश के प्रदर्शन की विशेषता है। यह एक लाइट
शो है जो तब होता है जब सूर्य से विद्युत-आवेशि त कण पृथ्वी के वायुमंडल में मौजूद ऑक्सीजन और नाइट्रोजन जैसे गैसों के कणों से टकराते
हैं। (अतः कथन 1 सही है)

· बृहस्पति और शनि दोनों में समान ध्रुवीय किरणें हैं जो सौर पवन के कारण होती हैं लेकिन कण बृहस्पति के आयो और शनि के एन्सेलाडस
जैसे पास के सक्रिय चंद्रमाओं से भी आते हैं। (इसलिए कथन 2 गलत है)

IMPORTANT POINTS/VALUE ADDITIONS:

· हाल ही में , खगोलविदों ने मीथेन से अवरक्त उत्सर्जन के साथ एक भूरे रंग के बौने ग्रह को खोजने के लिए जेम्स वेब स्पेस टे लीस्कोप का उ
पयोग किया। यह अप्रत्याशित है क्योंकि भूरा बौना ग्रह , बृहस्पति से बड़ा और एक तारे से छोटा , ठं डा है और इसमें कोई मेजबान तारा नहीं
है। शोधकर्ताओं का प्रस्ताव है कि अरोरा उत्पन्न करने वाली प्रक्रियाओं के कारण मीथेन वहां हो सकती है।

· अरोरा को कभी-कभी 'ध्रुवीय प्रकाश' भी कहा जाता है। यह मुख्य रूप से आर्क टिक और अंटार्क टिक जैसे ऊं चाई वाले क्षेत्रों में दे खा जाता
है।

· अरोरा लगभग 70 डिग्री के अक्षांश ों में दिखाई दे ते हैं। वे आम तौर पर 'ऑरोरल ज़ोन' नामक बैंड में होते हैं। ऑरोरल ज़ोन अक्षांश में 3 से
6 डिग्री चौड़ा है। यह भू-चुंबकीय ध्रुवों से 10 से 20 डिग्री के बीच स्थित है । यह रात के दौरान काफी स्पष्ट दिखाई दे ता है। ऑरोरा को
कभी-कभी वास्तविक ऑरोरल ज़ोन के नीचे अक्षांश ों पर दे खा जा सकता है।

· इस प्राकृ तिक प्रकाश प्रभाव को उत्तरी ऊं चाई पर 'ऑरोरा बोरेलिस' के नाम से जाना जाता है, जबकि दक्षिणी अक्षांश ों में प्रभाव को
'ऑरोरा ऑस्ट्रेलिस' के नाम से जाना जाता है। ऑरोरा बोरेलिस को 'नॉर्दर्न लाइट् स' के नाम से भी जाना जाता है । इसी तरह ऑरोरा
ऑस्ट्रेलिस को 'सदर्न लाइट् स' के नाम से भी जाना जाता है ।

QUESTION
7.
' आर्टेमिस मिशन ( Artemis Mission ) ' के बारे में निम्नलिखित कथनों पर विचार करें :

1. यह एक चंद्रमा अन्वेषण कार्यक्रम है जिसका नेतृत्व संयुक्त राज्य अमेरिका के नासा द्वारा किया गया है।

2. अपोलो 17 मिशन पांच दशक से भी अधिक समय पहले मानव की चंद्रमा पर आखिरी लैंडिंग थी।

3. कार्यक्रम का घोषित दीर्घकालिक लक्ष्य मंगल ग्रह पर मानव मिशन को सुविधाजनक बनाने के लिए चंद्रमा पर एक स्थायी आधार स्थापित
करना है।

4. भारत 'आर्टेमिस समझौते' में शामिल हुआ जिसका उद्दे श्य गहरे अंतरिक्ष की शांतिपूर्ण खोज करना है।

उपरोक्त में से कितने कथन सही हैं?

a) के वल एक

IASbaba
Score:
Web: http://ilp.iasbaba.com/
44.00 /
Email: ilp@iasbaba.com
Page 109 200
UPSC 2024 -
Exam Title : Environment,
Sc...
rupeshkr.
Email :
09ee80@gmail.com
Contact :

b) के वल दो
c) के वल तीन
d) सभी चार
Correct Answer: D
Your Answer:
Explanation

Q. 7) Solution (d)

स्पष्टीकरण:

· आर्टेमिस कार्यक्रम एक चंद्रमा अन्वेषण कार्यक्रम है जिसका नेतृत्व संयुक्त राज्य अमेरिका के नासा द्वारा किया जाता है और औपचारिक रूप
से 2017 में अंतरिक्ष नीति निर्देश 1 के माध्यम से स्थापित किया गया है। (इसलिए कथन 1 सही है)

· आर्टेमिस कार्यक्रम का उद्दे श्य 1972 में अपोलो 17 के बाद पहली बार चंद्रमा पर मानव उपस्थिति को फिर से स्थापित करना है। (इसलिए
कथन 2 सही है)

· कार्यक्रम का घोषित दीर्घकालिक लक्ष्य मंगल ग्रह पर मानव मिशन को सुविधाजनक बनाने के लिए चंद्रमा पर एक स्थायी आधार स्थापित
करना है। (अतः कथन 3 सही है)

· प्रधान मंत्री ने संयुक्त राज्य अमेरिका की यात्रा के दौरान आर्टेमिस समझौते में शामिल होने के भारत के फैसले की घोषणा की। (अतः
कथन 4 सही है)

IMPORTANT POINTS/VALUE ADDITIONS:

· नेश नल एरोनॉटिक्स एंड स्पेस एडमिनिस्ट्रेश न (NASA) और भारतीय अंतरिक्ष अनुसंधान संगठन (ISRO) 2024 में ह्यूस्टन, टे क्सास के
जॉनसन स्पेस सेंटर में प्रशिक्षित भारतीय अंतरिक्ष यात्रियों को अंतर्राष्ट्रीय अंतरिक्ष स्टे श न (ISS) में भेजने के लिए सहयोग करेंगे।

· आर्टेमिस समझौते का उद्दे श्य व्यापक है। इन्हें अमेरिकी विदे श विभाग और नासा द्वारा सात अन्य संस्थापक सदस्यों : ऑस्ट्रेलिया , कनाडा ,
इटली , जापान , लक्ज़मबर्ग , संयुक्त अरब अमीरात और यूनाइटे ड किंगडम के साथ 2020 में नागरिक अन्वेषण तथा बाह्य अंतरिक्ष , शांति
पूर्ण उद्दे श्यों के लिए चंद्रमा , मंगल , धूमके तु और क्षुद्रग्रह के उपयोग को नियंत्रित करने के लिए सामान्य सिद्धांत स्थापित करने के लिए स्था
पित किया गया है।

· यह 1967 की बाह्य अंतरिक्ष संधि की नींव पर आधारित है

QUESTION
8.
' एक्स - रे इमेजिंग और स्पेक्ट्रोस्कोपी मिशन ( XRISM ) ' के संदर्भ में निम्नलिखित कथनों पर विचार करें :

1. XRISM , नासा और यूरोपीय अंतरिक्ष एजेंसी के सहयोग से इसरो के नेतृत्व वाला मिशन है।

2. इसे ब्रह्मांड के सबसे गर्म क्षेत्रों, इसकी सबसे बड़ी संरचनाओं और सबसे मजबूत गुरुत्वाकर्षण वाली वस्तुओं का अध्ययन करने के लिए
डिज़ाइन किया गया है।

3. XRISM में के वल दो उपकरण - रिज़ॉल्व और एक्सटें ड ( Resolve and Xtend ) हैं , इनमें से प्रत्येक एक ही एक्स - रे मिरर असें
बली पर कें द्रित है।

उपरोक्त में से कितने कथन सही हैं?

a) के वल एक
b) के वल दो
c) सभी तीन

IASbaba
Score:
Web: http://ilp.iasbaba.com/
44.00 /
Email: ilp@iasbaba.com
Page 110 200
UPSC 2024 -
Exam Title : Environment,
Sc...
rupeshkr.
Email :
09ee80@gmail.com
Contact :

d) कोई नहीं
Correct Answer: B
Your Answer:
Explanation

Q. 8) Solution (b)

स्पष्टीकरण:

· इसका नेतृत्व नासा और यूरोपीय अंतरिक्ष एजेंसी के सहयोग से जापान एयरोस्पेस एक्सप्लोरेश न एजेंसी द्वारा किया गया है। (इसलिए
कथन 1 ग़लत है)

· अंतरिक्ष वेधशाला को 12,000 इलेक्ट्रॉन वोल्ट तक ऊर्जा वाले एक्स-रे विकिरण का पता लगाने के लिए डिज़ाइन किया गया है। इसे
संदर्भ में रखने के लिए, दृश्य प्रकाश की ऊर्जा दो से तीन इलेक्ट्रॉन वोल्ट के बीच होती है। इसलिए, इसे ब्रह्मांड के सबसे गर्म क्षेत्रों, साथ ही
इसकी सबसे बड़ी संरचनाओं और सबसे मजबूत गुरुत्वाकर्षण वाली वस्तुओं का अध्ययन करने के लिए डिज़ाइन किया गया है। (अतः कथन
2 सही है)

· XRISM में के वल दो उपकरण हैं - रिज़ॉल्व और एक्सटें ड - और उनमें से प्रत्येक एक ही एक्स-रे मिरर असेंबली पर कें द्रित है। (अतः
कथन 3 सही है)

IMPORTANT POINTS/VALUE ADDITIONS:

· रिज़ॉल्व एक माइक्रोकै लोरीमीटर स्पेक्ट्रोमीटर है, और यह तरल हीलियम के एक रेफ्रिजरेटर आकार के कं टे नर के अंदर पूर्ण शून्य से एक
डिग्री ऊपर है। एक्स-रे उपकरण को उसकी ऊर्जा से संबंधित मात्रा तक गर्म करती है। स्रोत के बारे में ज्ञान प्राप्त करने के लिए वैज्ञानिक
प्रत्येक व्यक्तिगत एक्स-रे की ऊर्जा के माप का उपयोग कर सकते हैं।

· एक्सटें ड , मिशन का दूसरा उपकरण , एक बड़े दृश्य क्षेत्र वाला एक्स - रे इमेजर है , जो पूर्णिमा के औसत स्पष्ट आकार से लगभग 60 प्र
तिशत बड़े क्षेत्र का निरीक्षण करने में सक्षम है।

QUESTION
9.
इसरो के POEM प्लेटफॉर्म के बारे में निम्नलिखित कथनों पर विचार करें:

1. यह जियोसिंक्रोनस सैटेलाइट लॉन्च व्हीकल ( जीएसएलवी ) के अंतिम और अन्यथा शेष चरण का उपयोग करके कक्षा में ( in-orbit
experiments ) प्रक्षेपण में मदद करेगा।

2. इसमें स्थिरीकरण के लिए एक समर्पित नेविगेश न मार्गदर्शन और नियंत्रण (एनजीसी) प्रणाली है।

3. यह अंतरिक्ष एजेंसियों और निजी कं पनियों को प्रायोगिक कक्षीय पेलोड को डिजाइन, विकसित और मान्य करने की अनुमति दे ता है।

उपरोक्त में से कितने कथन सही हैं?

a) के वल एक
b) के वल दो
c) सभी तीन
d) कोई नहीं
Correct Answer: B
Your Answer:
Explanation

Q. 9) Solution (b)

IASbaba
Score:
Web: http://ilp.iasbaba.com/
44.00 /
Email: ilp@iasbaba.com
Page 111 200
UPSC 2024 -
Exam Title : Environment,
Sc...
rupeshkr.
Email :
09ee80@gmail.com
Contact :

स्पष्टीकरण:

· POEM (पीएसएलवी ऑर्बिटल एक्सपेरिमेंटल मॉड्यूल) इसरो का एक प्रायोगिक मिशन है जो ध्रुवीय उपग्रह प्रक्षेपण यान (पीएसएलवी)
प्रक्षेपण यान के चौथे चरण के दौरान एक कक्षीय मंच के रूप में कक्षा में वैज्ञानिक प्रयोग करता है। (इसलिए कथन 1 गलत है)

· इसरो के अनुसार, पीओईएम में स्थिरीकरण के लिए एक समर्पित नेविगेश न मार्गदर्शन और नियंत्रण (एनजीसी) प्रणाली है , जो अनुमत
सीमा के भीतर किसी भी एयरोस्पेस वाहन के अभिविन्यास को नियंत्रित करने के लिए है। एनजीसी निर्दिष्ट सटीकता के साथ इसे स्थिर करने
के लिए प्लेटफ़ॉर्म के मस्तिष्क के रूप में कार्य करेगा। (अतः कथन 2 सही है)

· अंतरिक्ष एजेंसी के पीओईएम प्लेटफॉर्म में बिजली उत्पादन, टे लीमेट्री, टे ली-कमांड स्थिरीकरण , ऑर्बिटल-स्टे श न कीपिंग और ऑर्बिटल बद
लाव के लिए मानक इंटरफे स और पैकेजिंग है। यह अंतरिक्ष एजेंसियों और निजी कं पनियों को प्रायोगिक कक्षीय पेलोड को डिजाइन,
विकसित और मान्य करने की अनुमति दे ता है। (अतः कथन 3 सही है)

IMPORTANT POINTS/VALUE ADDITIONS:

· पीएसएलवी एक चार चरणों वाला रॉके ट है जहां पहले तीन चरण वापस समुद्र में गिर जाते हैं, और अंतिम चरण (पीएस4) - उपग्रह को
कक्षा में लॉन्च करने के बाद - अंतरिक्ष कबाड़ के रूप में समाप्त हो जाता है। यह पहली बार है कि (चौथा चरण) PS4 चरण एक स्थिर मंच
के रूप में पृथ्वी की परिक्रमा करेगा।

· POEM अपनी शक्ति PS4 टैं क के चारों ओर लगे सौर पैनलों और ली - आयन बैटरी से प्राप्त करेगा। यह चार सौर सेंसर , एक मैग्नेटोमी
टर , जायरोस ( gyros ) और NavIC का उपयोग करके नेविगेट करेगा।

· यह हीलियम गैस भंडारण का उपयोग करके समर्पित नियंत्रण थ्रस्टर्स रखता है। यह एक टे लीकमांड सुविधा के साथ सक्षम है।

QUESTION
10.
Q. 10 ) XPoSat के संदर्भ में , निम्नलिखित कथन पर विचार करें:

1. इसे नासा ने ईएसए ( ESA ) के सहयोग से लॉन्च किया है।

2. यह एक्स-रे ध्रुवीकरण और इसके ब्रह्मांडीय स्रोतों जैसे ब्लैक होल और न्यूट्रॉन तारों का अध्ययन करेगा।

3. XPoSat एक्स- रे ध्रुवीकरण का अध्ययन करने वाला विश्व का एकमात्र ऐसा मिशन है।

उपरोक्त में से कितने कथन सही हैं?

a) के वल एक
b) के वल दो
c) सभी तीन
d) कोई नहीं
Correct Answer: A
Your Answer:
Explanation

Q. 10) Solution (a)

स्पष्टीकरण:

· भारतीय अंतरिक्ष अनुसंधान संगठन (ISRO) ने नए साल की शुरुआत अपने पहले एक्स-रे पोलारिमीटर सैटेलाइट (XPoSat) के सफल
प्रक्षेपण के साथ की। (इसलिए कथन 1 ग़लत है)

IASbaba
Score:
Web: http://ilp.iasbaba.com/
44.00 /
Email: ilp@iasbaba.com
Page 112 200
UPSC 2024 -
Exam Title : Environment,
Sc...
rupeshkr.
Email :
09ee80@gmail.com
Contact :

· एक्स-रे पोलारिमीटर सैटेलाइट ( XPoSat ), जो एक्स-रे ध्रुवीकरण और इसके ब्रह्मांडीय स्रोतों जैसे ब्लैक होल और न्यूट्रॉन तारों का
अध्ययन करेगा। (अतः कथन 2 सही है)

· XPoSat विश्व का दूसरा ऐसा मिशन है। ( इसलिए कथन 3 गलत है)

IMPORTANT POINTS/VALUE ADDITIONS:

· इसमें दो पेलोड शामिल हैं, जिनमें भारतीय एक्स-रे पोलारिमीटर (POLIX) तथा एक्स-रे स्पेक्ट्रोस्कोपी और टाइमिंग (XSPECT) शामिल
हैं। इन्हें बेंगलुरु में रमन रिसर्च इंस्टीट्यूट और यूआर राव सैटेलाइट सेंटर द्वारा बनाया गया है।

· यह तीसरी बार है कि इसरो ने अंतरिक्ष में प्रौद्योगिकियों का प्रदर्शन करने के लिए पीएसएलवी ऑर्बिटल एक्सपेरिमेंटल मॉड्यूल ( पीओईएम
) प्लेटफॉर्म का उपयोग किया है। जो इसरो के विक्रम साराभाई अंतरिक्ष कें द्र द्वारा डिज़ाइन की गई ऑन - बोर्ड ईंधन सेल पावर सिस्टम प्रौद्
योगिकियों में से एक है ।

QUESTION
11.
“ किल स्विच ( Kill Switch ) ” शब्द हाल ही में खबरों में था। यह किससे संबंधित है ?

a) परमाणु आपदा
b) सामूहिक विनाश
c) कैं सर उपचार
d) रोगाणुरोधी प्रतिरोध
Correct Answer: C
Your Answer:
Explanation

Q. 11) Solution (c)

स्पष्टीकरण:

· एक बड़ी सफलता में, वैज्ञानिकों ने कथित तौर पर एक 'किल स्विच' की खोज की है जो कैं सर कोशिकाओं की मृत्यु को ट्रिगर करता है। (
इसलिए विकल्प सी सही है)

· शोधकर्ताओं ने इसे सीएआर टी-सेल थेरेपी नाम दिया है, जिसमें रोगी के रक्त से टी कोशिकाओं को इकट्ठा करना और फिर उन्हें प्रयोगशाला
में आनुवंशि क रूप से संश ोधित करके काइमेरिक एंटीजन रिसेप्टर्स (सीएआर) नामक रिसेप्टर्स का उत्पादन करना शामिल है। उन्हें संश ोधित
करने के बाद, इन कोशिकाओं को रोगी के शरीर में, रक्तप्रवाह में वापस इंजेक्ट कर दिया जाता है।

IMPORTANT POINTS/VALUE ADDITIONS:

· अब तक, थेरेपी ने तरल कैं सर, ल्यूकेमिया और अन्य रक्त कैं सर के खिलाफ आशाजनक प्रभावकारिता दिखाई है।

· स्तन, फे फड़े और आंत्र कैं सर जैसे ठोस ट्यूमर के खिलाफ "अल्प सफलता" दिखाई है ।

QUESTION
12.
Q. 12 ) ' किलोनोवा विस्फोट ( Kilonova Explosions ) ' के संदर्भ में निम्नलिखित कथनों पर विचार करें :

1. किलोनोवा एक क्षणिक खगोलीय घटना है जो एक कॉम्पैक्ट बाइनरी सिस्टम में घटित होती है जब के वल दो न्यूट्रॉन तारे विलीन होते हैं।

2. न्यूट्रॉन तारे अति सघन खगोलीय पिंड हैं जो एक विशाल तारे के जीवन के अंत में सुपरनोवा विस्फोट से बनते हैं।

IASbaba
Score:
Web: http://ilp.iasbaba.com/
44.00 /
Email: ilp@iasbaba.com
Page 113 200
UPSC 2024 -
Exam Title : Environment,
Sc...
rupeshkr.
Email :
09ee80@gmail.com
Contact :

3. इन उच्च - ऊर्जा टकरावों की चरम स्थितियों से सोने ( gold ) जैसे भारी तत्वों का निर्माण होता है।

4. यह विद्युत चुम्बकीय तरंगों के साथ-साथ गुरुत्वाकर्षण तरंगें भी उत्सर्जित करता है।

उपरोक्त में से कितने कथन सही हैं?

a) के वल एक
b) के वल दो
c) के वल तीन
d) सभी चार
Correct Answer: C
Your Answer: Unanswered
Explanation

Q. 12) Solution (c)

स्पष्टीकरण:

· एक किलोनोवा एक कॉम्पैक्ट बाइनरी सिस्टम में तब होता है जब दो न्यूट्रॉन तारे या एक न्यूट्रॉन तारा और एक ब्लैक होल विलीन हो जाते
हैं। न के वल दो न्यूट्र ॉन तारे बल्कि एक न्यूट्रॉन तारा और एक ब्लैक होल भी। (इसलिए कथन 1 गलत है)

· न्यूट्रॉन तारे अत्यधिक सघन खगोलीय पिंड हैं जो एक सुपरनोवा विस्फोट में एक विशाल तारे के जीवन के अंत में बनते हैं। कभी-कभी,
न्यूट्रॉन तारे बाइनरी सिस्टम में एक दूसरे की परिक्रमा करते हैं। (अतः कथन 2 सही है)

· इस तरह के विलय से शोधकर्ताओं को ब्रह्मांड की सबसे चरम स्थितियों के भौतिक गुणों का अध्ययन करने में मदद मिलती है। उदाहरण के
लिए, इन उच्च-ऊर्जा टकरावों की स्थितियों से सोने जैसे भारी तत्वों का निर्माण होता है। (अतः कथन 3 सही है)

· ये प्रणालियाँ गुरुत्वाकर्षण तरंगों का उत्सर्जन करके लगातार ऊर्जा खोती रहती हैं जब तक कि वे अंततः टकराकर विलीन नहीं हो जातीं। (
अतः कथन 4 सही है)

महत्वपूर्ण सूचना/मूल्यवर्धन:

· दो परिक्रमा करने वाले न्यूट्रॉन तारों के विस्फोटक विलय से एक किलोनोवा विस्फोट के अवलोकन योग्य संकेतों को मॉडल करने के लिए
एक विधि विकसित की है।

· इससे उन्हें यह बताने में मदद मिल सकती है कि विलय के दौरान वास्तव में क्या होता है, चरम स्थितियों में परमाणु पदार्थ कै से व्यवहार करता
है और सुदूर अतीत में ऐसी ब्रह्मांडीय घटनाओं से पृथ्वी पर सोना क्यों बना होगा।

· शोधकर्ताओं ने बाइनरी न्यूट्रॉन स्टार विलय के "मल्टी-मैसेंजर" अवलोकनों के लिए नई विधि लागू की।

QUESTION
13.
हाल ही में अंतरिक्ष की एक घटना को " अमेतरासु ( Amaterasu ) " नाम दिया गया। नीचे दिए गए विकल्पों में से इस शब्द का सही अर्थ
चुनें ?

a) उच्च ऊर्जा ब्रह्मांडीय कण


b) बढ़ती सौर ज्वालाएँ
c) जीवन के लिए मुख्य घटक
d) मीथेन नदियाँ
Correct Answer: A
Your Answer:

IASbaba
Score:
Web: http://ilp.iasbaba.com/
44.00 /
Email: ilp@iasbaba.com
Page 114 200
UPSC 2024 -
Exam Title : Environment,
Sc...
rupeshkr.
Email :
09ee80@gmail.com
Contact :

Explanation

Q. 13) Solution (a)

स्पष्टीकरण:

· वैज्ञानिकों ने पृथ्वी पर अब तक गिरने वाली सबसे शक्तिशाली ब्रह्मांडीय किरणों में से एक का पता लगाया है, लेकिन उन्हें पता नहीं है कि
इसका कारण क्या था या यह कहां से आई थी। अत्यधिक उच्च-ऊर्जा वाले कण को जापानी सूर्य दे वी के नाम पर 'अमेतरासु' नाम दिया गया
है, और यह अंतरिक्ष में एक शून्य से आया प्रतीत होता है जहां कु छ भी अस्तित्व में नहीं है। (इसलिए विकल्प ए सही है)

· अमेतरासु कण की ऊर्जा 240 एक्सा-इलेक्ट्रॉन वोल्ट ( ईईवी ) से अधिक है। यह लार्ज हैड्रॉन कोलाइडर द्वारा उत्पन्न कणों से लाखों गुना
अधिक शक्तिशाली है, जो अब तक का सबसे शक्तिशाली त्वरक है। यह "ओह-माय-गॉड" कण के बाद दूसरे स्थान पर है, जो 1991 में
खोजी गई एक और उच्च-ऊर्जा वाली ब्रह्मांडीय किरण है। यह 320 ईईवी पर आई थी।

IMPORTANT POINTS/VALUE ADDITIONS:

· सौर ज्वालाएँ तब घटित होती हैं जब सूर्य के अंदर और उसके चारों ओर शक्तिशाली चुंबकीय क्षेत्र पुनः जुड़ जाते हैं। वे आमतौर पर सक्रिय
क्षेत्रों से जुड़े होते हैं, जिन्हें अक्सर सूर्य धब्बे के रूप में दे खा जाता है, जहां चुंबकीय क्षेत्र सबसे मजबूत होते हैं। फ्लेयर्स को उनकी सामर्थ्य के
अनुसार वर्गीकृ त किया गया है। सबसे छो टी बी-क्लास हैं, उसके बाद सी, एम और एक्स, सबसे बड़ी हैं।

· वैज्ञानिकों को शनि के बर्फीले चंद्रमा एन्सेलेडस पर जीवन के लिए एक प्रमुख घटक और ऊर्जा के एक शक्तिशाली स्रोत का प्रमाण मिला है।
एन्सेलाडस द्वारा फें के गए जलवाष्प के विशाल ढे र और बर्फ के कण कार्बनिक यौगिकों से समृद्ध हैं, जिनमें से कु छ जीवन के लिए महत्वपूर्ण
हैं।

· विड फ्लुमिना शनि के चंद्रमा टाइटन पर तरल मीथेन और ईथेन की एक नदी है। इस नदी की तुलना नील नदी से की गई है। टाइटन की
सतह ज्यादातर पानी की बर्फ है, इसलिए विड फ्लुमिना मीथेन और ईथेन की एक नदी है जो बहती है और घाटियों को बर्फ में काटती है जैसे
कि यह आधारशिला हो। नासा के वैज्ञानिकों का मानना है कि इसमें संभवतः पृथ्वी पर नदियों की तरह रैपिड् स, भँवर और झरने हैं।

QUESTION
14.
'चंद्रयान 3' के बारे में निम्नलिखित कथनों पर विचार करें:

1. यह चंद्रमा के सुदूर हिस्से पर उतरने वाला पहला अंतरिक्ष यान बन गया।

2. चंद्रयान 2 के चंद्रमा के दक्षिणी ध्रुव पर उतरने में विफलता के कारण चंद्रयान 3 भूमध्य रेखा के पास उतरा।

3. दक्षिणी ध्रुव पर कम पहाड़ियाँ और गड्ढे उपकरणों के संचालन को आसान बनाते हैं।

उपरोक्त में से कितने कथन सही हैं?

a) के वल एक
b) के वल दो
c) सभी तीन
d) कोई नहीं
Correct Answer: D
Your Answer:
Explanation

Q. 14) Solution (d)

स्पष्टीकरण:

IASbaba
Score:
Web: http://ilp.iasbaba.com/
44.00 /
Email: ilp@iasbaba.com
Page 115 200
UPSC 2024 -
Exam Title : Environment,
Sc...
rupeshkr.
Email :
09ee80@gmail.com
Contact :

· यह बहुत अच्छा कारण है कि चंद्रमा पर अब तक हुई सभी लैंडिंग भूमध्यरेखीय क्षेत्र में हुई हैं। यहां तक कि चीन का चांग ' ई 4, जो चंद्रमा
के सुदूर हिस्से ( वह हिस्सा जो पृथ्वी का सामना नहीं करता ) पर उतरने वाला पहला अंतरिक्ष यान बन गया , जो 45 डिग्री अक्षांश के पास उ
तरा। ( इसलिए कथन 1 ग़लत है )

· चंद्रयान-3 चंद्रमा के दक्षिणी ध्रुव के पास सॉफ्ट-लैंडिंग करने वाला विश्व का पहला मिशन बन गया। पिछले सभी अंतरिक्ष यान चंद्र भूमध्य
रेखा के उत्तर या दक्षिण में कु छ डिग्री अक्षांश पर उतरे हैं। (इसलिए कथन 2 गलत है)

· भूमध्य रेखा के पास उतरना आसान और सुरक्षित है। इलाके और तापमान अधिक अनुकूल हैं, और उपकरणों के लंबे और निरंतर संचालन
के लिए अनुकूल हैं। यहां की सतह समतल और चिकनी है, बहुत तीव्र ढलान लगभग अनुपस्थित हैं, और कम पहाड़ियाँ या गड्ढे हैं। सूर्य का
प्रकाश प्रचुर मात्रा में मौजूद है, कम से कम पृथ्वी की ओर वाले हिस्से में, इस प्रकार सौर ऊर्जा से चलने वाले उपकरणों को ऊर्जा की नियमित
आपूर्ति होती है। (इसलिए कथन 3 गलत है)

IMPORTANT POINTS/VALUE ADDITIONS:

· चंद्रमा के ध्रुवीय क्षेत्र बहुत अलग और कठिन भूभाग हैं। कई हिस्से पूरी तरह से अंधेरे क्षेत्र में स्थित हैं जहां सूरज की रोशनी कभी नहीं
पहुंचती है, और तापमान 230 डिग्री सेल्सियस से नीचे जा सकता है।

· सूर्य के प्रकाश की कमी और अत्यधिक कम तापमान उपकरणों के संचालन में कठिनाई पैदा करते हैं। इसके अलावा, हर जगह बड़े-बड़े गड्ढे
हैं, जिनका आकार कु छ सेंटीमीटर से लेकर कई हज़ार किलोमीटर तक फै ला हुआ है ।

· अपने बीहड़ वातावरण के कारण, चंद्रमा के ध्रुवीय क्षेत्र अज्ञात बने हुए हैं। लेकिन कई ऑर्बिटर मिशनों ने साक्ष्य दिया है कि इन क्षेत्रों का
पता लगाना बहुत दिलचस्प हो सकता है।

· इस क्षेत्र में गहरे गड्ढों में पर्याप्त मात्रा में बर्फ के अणुओं की उपस्थिति के संकेत हैं - भारत के 2008 चंद्रयान -1 मिशन ने अपने दो
उपकरणों की मदद से चंद्र सतह पर पानी की उपस्थिति का संकेत दिया था।

QUESTION
15.
ग्रह कै से बनते हैं , यह समझने की खोज में जेम्स वेब स्पेस टे लीस्कोप ने एक बड़ी उपलब्धि हासिल की है। इस संदर्भ में निम्नलिखित कथनों प
र विचार करें :

कथन 1: कं कड़ अभिवृद्धि सिद्धांत ( pebble accretion theory ) के अनुसार , बर्फीले ठोस जो आमतौर पर प्रोटोप्लेनेटरी डिस्क (
protoplanetary discs) की बाहरी पहुंच को रोकते हैं , आंतरिक डिस्क के करीब प्रवेश करते हैं।

कथन 2: वे बहुत सारे ठं डे पानी के वाष्प छोड़ते हैं जो शुरुआती ग्रहों पर पानी और ठोस पदार्थ दोनों पहुंचाते हैं।

उपरोक्त कथनों के संबंध में निम्नलिखित में से कौन सा सही है?

a) कथन-1 और कथन-2 दोनों सही हैं और कथन-2, कथन-1 की सही व्याख्या है।
b) कथन-1 और कथन-2 दोनों सही हैं और कथन-2, कथन-1 की सही व्याख्या नहीं है।
c) कथन-1 सही है लेकिन कथन-2 गलत है।
d) कथन-1 ग़लत है लेकिन कथन-2 सही है।
Correct Answer: A
Your Answer:
Explanation

Q. 15) Solution (a)

स्पष्टीकरण:

IASbaba
Score:
Web: http://ilp.iasbaba.com/
44.00 /
Email: ilp@iasbaba.com
Page 116 200
UPSC 2024 -
Exam Title : Environment,
Sc...
rupeshkr.
Email :
09ee80@gmail.com
Contact :

· ग्रहों का निर्माण कै से होता है , यह समझने की खोज में सफलता हासिल करने के लिए वैज्ञानिकों ने जेम्स वेब स्पेस टे लीस्कोप का उपयोग
किया । उन्होंने सुदूर प्रोटोप्लेनेटरी डिस्क में जल वाष्प दे खा , जो एक ग्रह का अग्रदूत था , जिससे ग्रह कै से बनते हैं , इसके लंबे समय से प्रस्
तावित सिद्धांत की पुष्टि हुई।

· इस कं कड़ अभिवृद्धि सिद्धांत के अनुसार, बर्फीले ठोस पदार्थ जो आमतौर पर "प्रोटोप्लेनेट री डिस्क" की बाहरी पहुंच को रोकते हैं,
आंतरिक डिस्क के करीब गर्म क्षेत्र में प्रवेश करते हैं। वहां, वे बहुत अधिक मात्रा में ठं डे पानी की वाष्प छोड़ते हैं । ऐसा माना जाता है
कि यह प्रक्रिया प्रारंभिक ग्रहों तक पानी और ठोस पदार्थ दोनों पहुंचाती है। (इसलिए विकल्प ए सही है)

· इस सिद्धांत के आधार पर, वैज्ञानिक एक महत्वपूर्ण भविष्यवाणी करते हैं - जब बर्फीले कं कड़ डिस्क के गर्म क्षेत्र में प्रवेश करते हैं, तो उन्हें
बड़ी मात्रा में ठं डा पानी और वाष्प छोड़ना चाहिए तथा वेब टे लीस्कोप ने ठीक यही दे खा।

IMPORTANT POINTS/VALUE ADDITIONS:

· शोधकर्ताओं ने सूर्य जैसे तारों के चारों ओर चार प्रोटोप्लेनेटरी डिस्क का अध्ययन करने के लिए वेब के एमआईआरआई (मिड-इन्फ्रारेड
इंस्ट्रूमेंट) का उपयोग किया। अनुमान है कि ये तारे 2 से 3 मिलियन वर्ष पुराने हैं।

· लौकिक दृष्टि से, यह उन्हें नवजात बनाता है। दो डिस्क कॉम्पैक्ट थीं जबकि अन्य दो विस्तारित थीं। दूरबीन के परिणामों से पता चला कि
कॉम्पैक्ट डिस्क में अतिरिक्त ठं डा पानी है।

· यह अवलोकन यह पता लगाने के लिए किया गया था कि क्या कॉम्पैक्ट डिस्क के आंतरिक, चट्टानी ग्रह क्षेत्र में पानी की प्रचुरता अधिक है।
यह सच होगा यदि कं कड़ विस्थापन अधिक कु शल हो और आंतरिक ग्रहों पर अधिक ठोस द्रव्यमान और पानी पहुंचाए।

QUESTION
16.
हाल ही में खबरों में रहा "गुड मैन्युफैक्चरिंग प्रैक्टिसेज (Good Manufacturing Practices)" निम्नलिखित में से किस क्षेत्र से
संबंधित है?

a) चमड़ा उद्योग
b) फार्मास्युटिकल / दवा उद्योग
c) प्लास्टिक उद्योग
d) रसायन उद्योग
Correct Answer: B
Your Answer:
Explanation

Q. 16) Solution (b)

स्पष्टीकरण:

· कई दे श ों में कथित तौर पर दूषित भारत - निर्मित दवाओं से होने वाली मौतों की रिपोर्ट करने की हालिया घटनाओं के बाद , सरकार ने गुड
मैन्युफैक्चरिंग प्रैक्टिस ( जीएमपी ) के अनिवार्य कार्यान्वयन के लिए एक समय सीमा तय की है , जिसे 2018 में संश ोधित किया गया था ,
जिससे उन्हें विश्व स्वास्थ्य संगठन ( डब्ल्यूएचओ ) के समान मानक पर लाया गया।

· 250 करोड़ रुपये से अधिक टर्नओवर वाली कं पनियों को छह महीने के भीतर संश ोधित जीएमपी लागू करना होगा, जबकि 250 करोड़
रुपये से कम टर्नओवर वाले मध्यम और लघु उद्यमों को इसे एक साल के भीतर लागू करना होगा।

IMPORTANT POINTS/VALUE ADDITIONS:

· दे श में 10,500 विनिर्माण इकाइयों में से के वल 2,000 को वैश्विक WHO-GMP मानकों के अनुरूप पाया गया है। अब, सभी को घरेलू
बाजार और विदे श के लिए गुणवत्तापूर्ण दवाएं सुनिश्चित करते हुए संश ोधित जीएमपी लागू करना होगा।

IASbaba
Score:
Web: http://ilp.iasbaba.com/
44.00 /
Email: ilp@iasbaba.com
Page 117 200
UPSC 2024 -
Exam Title : Environment,
Sc...
rupeshkr.
Email :
09ee80@gmail.com
Contact :

· वर्तमान में, जबकि अन्य दे श ों में दवाएँ निर्यात करने वाली कं पनियों को पहले से ही WHO-GMP प्रमाणित होना पड़ता है, घरेलू बाज़ार
के लिए दवाएँ बनाने वाली कं पनियों को अनुमति दी जा सकती है यदि वे ड्रग्स और कॉस्मेटिक्स अधिनियम के तहत नियमों की अनुसूची एम
में सूचीबद्ध आवश्यकताओं को पूरा करते हैं। अन्य बातों के अलावा, इसमें विनिर्माण इकाइयों की विशिष्टताओं, पालन की जाने वाली
प्रक्रियाओं और आवश्यक उपकरणों की सूची दी गई है।

QUESTION
17.
निम्नलिखित युग्मों पर विचार करें:

औष धि रोग

1. सेमाग्लूटाइड - वेगोवी ( Semaglutide - Wegovy ) मोटापा

2. एनास्ट्रोज़ोल ( Anastrozole ) स्तन कैं सर

3. डोनानेमब ( Donanemab ) क्षय रोग

उपरोक्त में से कितने जोड़े सही हैं?

a) के वल एक
b) के वल दो
c) सभी तीन
d) कोई नहीं
Correct Answer: B
Your Answer:
Explanation

Q. 17) Solution (b)

स्पष्टीकरण:

· वजन घटाने वाली दवा वेगोवी ( Wegovy ) को यूके में " नियंत्रित और सीमित लॉन्च " के हिस्से के रूप में उपलब्ध कराया जा रहा है। द
वा , जिसे सेमाग्लूटाइड भी कहा जाता है , कम कै लोरी वाले आहार और व्यायाम के साथ - साथ विशेषज्ञ एनएचएस वजन प्रबंधन सेवाओं के
माध्यम से निर्धारित की जाएगी। ( अतः जोड़ी 1 सही है )

· एनास्ट्रोज़ोल, एक दवा जिसका उपयोग पहले ब्रिटे न में स्तन कैं सर के इलाज के लिए किया जाता था, अब इस बीमारी को रोकने के लिए
एक दवा के रूप में लाइसेंस प्राप्त कर लिया गया है। कथित तौर पर, यह पाया गया है कि यह दवा रजोनिवृत्ति के बाद महिलाओं में स्तन
कैं सर की संभावना को 50% तक कम कर दे ती है। (अतः जोड़ी 2 सही है)

· डोनानेमैब को प्रारंभिक अल्जाइमर वाले लोगों में संज्ञानात्मक गिरावट को रोकने में प्रभावी पाया गया। इसे फार्मास्युटिकल दिग्गज एली
लिली द्वारा विकसित किया गया है, तीसरे चरण के परीक्षण में प्लेसबो की तुलना में डोनानेमैब को संज्ञानात्मक गिरावट को 35% तक धीमा
पाया गया। (अतः जोड़ी 3 गलत है)

IMPORTANT POINTS/VALUE ADDITIONS:

· कें द्रीय स्वास्थ्य मंत्रालय ने हाल ही में आवश्यक दवाओं की नई राष्ट्रीय सूची (एनएलईएम) लॉन्च की है, जिसमें मधुमेह के लिए नए उपचारों
जैसे टे नेलिग्लिप्टिन और इंसुलिन ग्लार्गिन को शामिल करने के लिए सूची का विस्तार किया गया है, तथा चार और कैं सर विरोधी उपचारों को
भी शामिल किया गया है।

· मोटापा स्वास्थ्य प्रणाली पर भारी दबाव डालता है, लोगों को वजन कम करने में मदद करने के लिए नवीनतम दवाओं का उपयोग उच्च
रक्तचाप, मधुमेह और कैं सर जैसी खतरनाक मोटापे से संबंधित स्वास्थ्य स्थितियों से निपटने में मदद करके गेम-चेंजर साबित होगा।

IASbaba
Score:
Web: http://ilp.iasbaba.com/
44.00 /
Email: ilp@iasbaba.com
Page 118 200
UPSC 2024 -
Exam Title : Environment,
Sc...
rupeshkr.
Email :
09ee80@gmail.com
Contact :

QUESTION
18.
जैसा कि हाल के अध्ययन से पता चला है , अफ्रीकी महाद्वीप और प्रशांत महासागर के नीचे असामान्य सामग्री के ' विशाल निम्न - वेग वाले
प्रांत ' (large low-velocity provinces - LLVPs) कहे जाने वाले महाद्वीप के आकार की आकृ ति की प्रस्तावित उत्पत्ति क्या है ?

a) ज्वालामुखीय गतिविधि का परिणाम


b) एक प्राचीन ग्रह के अवशेष जो पृथ्वी से टकरा ए थे
c) टे क्टोनिक प्लेट संच लनों द्वारा निर्मित
d) क्षुद्रग्रह प्रभाव से निक्षेप
Correct Answer: D
Your Answer:
Explanation

Q. 18) Solution (d)

स्पष्टीकरण:

· ग्रह के कें द्र के पास गहराई में असामान्य सामग्री के दो महाद्वीप आकार के पिंड थे । एक अफ़्रीकी महाद्वीप के नीचे और एक प्रशांत महासा
गर के नीचे है । इनमें से प्रत्येक चंद्रमा के आकार से दोगुना है और उन्हें घेरने वाले आवरण की तुलना में तत्वों के विभिन्न अनुपात से बना है। इ
नके विशाल निम्न - वेग प्रांत ( एलएलवीपी ) कहा जाता है।

· नेचर जर्नल में प्रकाशित एक अध्ययन से पता चलता है कि ये पिंड उस प्राचीन ग्रह के अवशेष हैं जो अरबों साल पहले पृथ्वी से टकराया था,
जिससे चंद्रमा का निर्माण हुआ।

· यह अध्ययन " विशाल प्रभाव सिद्धांत " से जुड़े एक अन्य रहस्य का भी उत्तर दे ता है कि चंद्रमा का निर्माण कै से हुआ। पृथ्वी से टकराने वाले
छोटे ग्रह को थिया ( Theia ) नाम दिया गया था लेकिन क्षुद्रग्रह बेल्ट या उल्कापिंडों में इसका कोई निशान नहीं पाया गया है। शोधकर्ताओं
का मानना है कि थिया का अधिकांश भाग प्रारंभिक पृथ्वी द्वारा अवशोषित कर लिया गया था , जिससे इन " एलएलवीपी " का नि
र्माण हुआ , जिसके प्रभाव से बचा हुआ मलबा चंद्रमा बन गया।

IMPORTANT POINTS/VALUE ADDITIONS:

· एलएलवीपी की खोज पहली बार तब हुई जब वैज्ञानिकों ने पृथ्वी के माध्यम से यात्रा करने वाली भूकंपीय तरंगों को मापा। ये तरंगें विभिन्न
सामग्रियों के माध्यम से अलग-अलग गति से यात्रा करती हैं। 1980 के दशक में, इन भूकंपीय तरंगों के अध्ययन से पृथ्वी के अंदर बड़े पैमाने
पर त्रि-आयामी विविधताओं का संकेत मिला।

· ऐसा प्रतीत होता है कि पृथ्वी के कें द्र के पास दो संरचनाएँ थीं जिनमें असामान्य रूप से उच्च स्तर का लोहा था।

QUESTION
19.
आकाशगंगाओं के बीच " लुप्त " पदार्थ को मापने के लिए फास्ट रेडियो बर्स्ट्स (FRBs) के उपयोग के संदर्भ में, निम्नलिखित कथनों पर
विचार करें:

1. एफआरबी का उपयोग आकाशगंगाओं के बीच " लुप्त " पदार्थ को मापने के लिए किया जा सकता है, जो ब्रह्मांड के द्रव्यमान को
निर्धारित करने के लिए एक नई विधि प्रदान करता है।

2. ब्रह्मांड में आधे से अधिक सामान्य पदार्थ लुप्त है, माना जाता है कि यह आकाशगंगाओं के बीच के स्थान में स्थित है, संभवतः इतना गर्म
और फै ला हुआ है कि इसे पारंपरिक तकनीकों का उपयोग करके नहीं दे खा जा सकता है।

3. एफआरबी आकाशगंगाओं के बीच लगभग खाली जगह में आयनित सामग्री का पता लगा सकते हैं, जिससे वैज्ञानिकों को मौजूद पदार्थ की
मात्रा को मापने की अनुमति मिलती है।

उपरोक्त में से कितने कथन सही हैं?

IASbaba
Score:
Web: http://ilp.iasbaba.com/
44.00 /
Email: ilp@iasbaba.com
Page 119 200
UPSC 2024 -
Exam Title : Environment,
Sc...
rupeshkr.
Email :
09ee80@gmail.com
Contact :

a) के वल एक
b) के वल दो
c) सभी तीन
d) कोई नहीं
Correct Answer: C
Your Answer:
Explanation

Q. 19) Solution (c)

स्पष्टीकरण:

· यूरोपीय दक्षिणी वेधशाला के अनुसार, एफआरबी (फास्ट रेडियो बर्स्ट) का उपयोग आकाशगंगाओं के बीच " लुप्त " पदार्थ को मापने के
लिए किया जा सकता है और ब्रह्मांड को जान ने का एक नया तरीका प्रदान कर सकता है। (इसलिए कथन 1 सही है)

· ब्रह्माण्ड के द्रव्यमान का अनुमान लगाने की वर्तमान विधियाँ परस्पर विरोधी उत्तर दे ती हैं और ब्रह्माण्ड विज्ञान के मानक मॉडल को चुनौती
दे ती हैं।

· हमें लगता है कि लुप्त पदार्थ आकाशगंगाओं के बीच की जगह में छिपा हुआ है, लेकिन यह इतना गर्म और फैला हुआ हो सकता है कि
सामान्य तकनीकों का उपयोग करके इसे दे खना असंभव है। (अतः कथन 2 सही है)

· एफआरबी इस आयनित सामग्री को महसूस करते हैं। यहां तक कि अंतरिक्ष में जो लगभग पूरी तरह से खाली है, ये विस्फोट सभी इलेक्ट्रॉनों
को दे ख सकते हैं। इससे वैज्ञानिकों को यह मापने में मदद मिलती है कि आकाशगंगाओं के बीच कितना पदार्थ है। (अतः कथन 3 सही है)

IMPORTANT POINTS/VALUE ADDITIONS:

· शोधकर्ताओं ने ब्रह्मांडीय रेडियो तरंगों के एक दूरस्थ विस्फोट की खोज की जो एक मिलीसेकंड से भी कम समय तक चला। यह तेज़ रेडियो
विस्फोट अब तक पता लगाया गया सबसे दूर का विस्फोट है और इसका स्रोत इतनी दूर एक आकाशगंगा में है कि इसे हम तक पहुँचने में आठ
अरब साल लग गए।

· यह अब तक दे खे गए सबसे ऊर्जावान कणों में से एक है। एक सेकंड के एक अंश में, इसने हमारे सूर्य से 30 वर्षों के बराबर उत्सर्जन जारी
किया।

· हालाँकि हम अभी भी नहीं जानते हैं कि ऊर्जा के इन बड़े विस्फोटों का क्या कारण है, तथ्य पुष्टि करता है कि तेज़ रेडियो विस्फोट ब्रह्मांड में
सामान्य घटनाएँ हैं और हम उनका उपयोग आकाशगंगाओं के बीच पदार्थ का पता लगाने और इसकी संरचना को बेहतर ढं ग से समझ पाएंगे।

QUESTION
20.
निम्नलिखित युग्मों पर विचार करें:

क्षुद्रग्रह मिशन

1. बेन्नु OSIRIS-REx

2. साइकी ( Psyche ) OSIRIS-APEX

3. दिन्किनेश ( Dinkinesh ) लुसी ( Lucy )

उपरोक्त में से कितने जोड़े सही हैं?

a) के वल एक
b) के वल दो

IASbaba
Score:
Web: http://ilp.iasbaba.com/
44.00 /
Email: ilp@iasbaba.com
Page 120 200
UPSC 2024 -
Exam Title : Environment,
Sc...
rupeshkr.
Email :
09ee80@gmail.com
Contact :

c) सभी तीन
d) कोई नहीं
Correct Answer: B
Your Answer:
Explanation

Q. 20) Solution (b)

स्पष्टीकरण:

· OSIRIS-Rex किसी क्षुद्रग्रह से नमूना एकत्र करने वाला पहला अमेरिकी मिशन है। यह क्षुद्रग्रह बेन्नु से सामग्री लाने के लिए पृथ्वी पर लौ
ट आया। ( इसलिए जोड़ी 1 सही है )

· क्षुद्रग्रह साइकी के नाम पर मिशन को साइकी मिशन का नाम दिया गया है। OSIRIS-APEX , क्षुद्रग्रह एपोफिस का पता लगाने के लिए
है । ( इसलिए जोड़ी 2 गलत है )

· नेश नल एरोनॉटिक्स एंड स्पेस एडमिनिस्ट्रेश न ( NASA) का बृहस्पति ट्रोजन क्षुद्रग्रहों का पता लगाने के लिये पहला ' लूसी ' (Lucy) मि
शन लॉन्च किया गया है। लुसी द्वारा भेजी गई पहली छवियों से पता चलता है कि छोटा मुख्य बेल्ट क्षुद्रग्रह डिंकिनेश वास्तव में एक द्विआधारी
जोड़ी ( binary pair ) है। ( इसलिए जोड़ी 3 सही है )

IMPORTANT POINTS/VALUE ADDITIONS:

· OSIRIS-APEX, OSIRIS-REx का अनुवर्ती , 2029 में क्षुद्रग्रह की पृथ्वी के साथ दुर्लभ निकटता के बाद क्षुद्रग्रह एपोफिस में होने
वाले भौतिक परिवर्तनों का अध्ययन करेगा।

· लुसी आठ अलग-अलग क्षुद्रग्रहों की बारह साल की यात्रा पर नासा का एक अंतरिक्ष यान है।

· मिशन को विभिन्न क्षुद्रग्रहों की संरचना को समझने हेतु डिज़ाइन किया गया है जो ट्रोजन क्षुद्रग्रहों का एक हिस्सा है , इसका उपयोग सामग्री
के द्रव्यमान और घनत्व को निर्धारित करने के लिये एवं ट्रोजन क्षुद्रग्रहों की परिक्रमा करने वाले उपग्रहों एवं रिंग्स को दे खने तथा उनका
अध्ययन करने के लिये किया जाएगा।

QUESTION
21.
रेडियो खगोल विज्ञान और स्क्वायर किलोमीटर एरे (SKA) में भारत की क्षमताओं के संदर्भ में निम्नलिखित कथनों पर विचार करें:

1. स्क्वायर किलोमीटर ऐरे एक एकल बड़ी दूरबीन होगी जिसमें व्यक्तिगत रूप से संचालित होने वाले हजारों डिश एंटेना शामिल होंगे।

2. SKA का कोई भी प्रतिष्ठान भारत में स्थित नहीं हो गा ।

3. विशाल मीटरवेव रेडियो टे लीस्कोप ( GMRT) जैसी उन्नत सुविधाओं के साथ, भारत के पास रेडियो खगोल विज्ञान में अत्यधिक
विकसित क्षमताएं हैं।

उपरोक्त में से कितने कथन सही हैं?

a) के वल एक
b) के वल दो
c) सभी तीन
d) कोई नहीं
Correct Answer: B
Your Answer:
Explanation

IASbaba
Score:
Web: http://ilp.iasbaba.com/
44.00 /
Email: ilp@iasbaba.com
Page 121 200
UPSC 2024 -
Exam Title : Environment,
Sc...
rupeshkr.
Email :
09ee80@gmail.com
Contact :

Q. 21) Solution (b)

स्पष्टीकरण:

· स्क्वायर किलोमीटर ऐरे कोई एक बड़ा टे लीस्कोप नहीं होगा, बल्कि एक इकाई के रूप में काम करने वाले हजारों डिश एंटेना का संग्रह
होगा। (इसलिए कथन 1 ग़लत है)

· SKA का कोई भी प्रतिष्ठान भारत में स्थित नहीं होगा। ( अतः कथन 2 सही है )

· रेडियो खगोल विज्ञान एक ऐसी चीज़ है जिसमें भारत के पास पहले से ही अत्यधिक विकसित क्षमताएं हैं। पुणे के पास विशाल मीटरवेव
रेडियो टे लीस्कोप (जीएमआरटी) दुनिया में सबसे उन्नत और मांग वाली सुविधाओं में से एक है, जो उल्लेखनीय वैज्ञानिक परिणाम दे रहा है।
ऊटी, नैनीताल और बेंगलुरु में भी इसी तरह की अन्य सुविधाएं हैं। (अतः कथन 3 सही है)

IMPORTANT POINTS/VALUE ADDITIONS:

· भारत ने औपचारिक रूप से स्क्वायर किलोमीटर ऐरे (एसके ए) परियोजना में शामिल होने का फैसला किया था, जो विश्व के सबसे बड़े
रेडियो टे लीस्कोप के निर्माण के लिए काम करने वाला एक अंतरराष्ट्रीय वैज्ञानिक सहयोग है।

· भारत पिछले कई वर्षों से पहले से ही इस परियोजना में योगदान दे रहा था, लेकिन पूर्ण सदस्य का दर्जा, जो आगामी सुविधा का उपयोग
करने के लिए अधिक वैज्ञानिक अवसर प्रदान करता है, के लिए दे श ों को एक अंतरराष्ट्रीय संधि पर हस्ताक्षर करने और उसका अनुमोदन
करने की आवश्यकता होती है, और एक वित्तीय प्रतिबद्धता भी बनानी पड़ती है।

· पूर्ण सदस्य का दर्जा भारत को SKA सुविधाओं तक अधिमान्य पहुंच प्रदान करेगा।

· परियोजना में भारतीय भागीदारी का नेतृत्व पुणे स्थित नेश नल सेंटर फॉर रेडियो एस्ट्रोफिजिक्स (एनसीआरए) द्वारा किया जा रहा है।

QUESTION
22.
भारत के आर्क टिक अनुसंधान स्टे श न और आर्क टिक में वैज्ञानिक अनुसंधान के संदर्भ में, निम्नलिखित कथनों पर विचार करें:

1. हिमाद्री , भारत का आर्क टिक अनुसंधान स्टे श न स्वालबार्ड के नॉर्वेजियन द्वीपसमूह में नाइल - एलेसुंड ( Ny-Ålesund ) में है।

2. आर्क टिक क्षेत्र में वैज्ञानिक अनुसंधान 1920 की स्वालबार्ड संधि और समुद्र के कानून पर संयुक्त राष्ट्र कन्वेंश न जैसे अंतरराष्ट्रीय कानूनी
उपकरणों द्वारा शासित होता है।

3. आर्क टिक क्षेत्र में भारत के दो अन्य स्टे श न हिमाद्रि के अलावा मैत्री और भारती हैं।

4. राष्ट्रीय ध्रुवीय एवं महासागर अनुसंधान कें द्र, गोवा, भारत की ध्रुवीय खोजों के लिए नोडल एजेंसी है।

उपरोक्त में से कितने कथन सही हैं?

a) के वल एक
b) के वल दो
c) के वल तीन
d) सभी चार
Correct Answer: C
Your Answer:
Explanation

Q. 22) Solution (c)

स्पष्टीकरण:

IASbaba
Score:
Web: http://ilp.iasbaba.com/
44.00 /
Email: ilp@iasbaba.com
Page 122 200
UPSC 2024 -
Exam Title : Environment,
Sc...
rupeshkr.
Email :
09ee80@gmail.com
Contact :

· हिमाद्रि , आर्क टिक महासागर में नॉर्वेजियन द्वीपसमूह स्वालबार्ड के न्य-एलेसुंड में भारत का आर्क टिक अनुसंधान स्टे श न , अब पूरे वर्ष चालू
रहेगा। (अतः कथन 1 सही है)

· आर्क टिक क्षेत्र में वैज्ञानिक अनुसंधान आर्क टिक दे श ों के व्यक्तिगत अधिकार क्षेत्र के अलावा, 1920 की स्वालबार्ड संधि और समुद्र के
कानून पर संयुक्त राष्ट्र कन्वेंश न जैसे अंतरराष्ट्रीय कानूनी उपकरणों द्वारा शासित होता है। (अतः कथन 2 सही है)

· अंटार्क टिका में भारत के दो स्टे श न मैत्री और भारती उपयोग में हैं। (इसलिए कथन 3 गलत है)

· राष्ट्रीय ध्रुवीय एवं महासागर अनुसंधान कें द्र, गोवा, भारत की ध्रुवीय खोजों के लिए नोडल एजेंसी है। (अतः कथन 4 सही है)

IMPORTANT POINTS/VALUE ADDITIONS:

· भारत ने 1920 में पेरिस में स्वालबार्ड संधि पर हस्ताक्षर किए। लेकिन आर्क टिक सूक्ष्म जीव विज्ञान, वायुमंडलीय विज्ञान और भूविज्ञान में
अध्ययन शुरू करने के उद्दे श्य से नाइलुंड में अंतर्राष्ट्रीय आर्क टिक अनुसंधान सुविधाओं का दौरा करने वाले पांच वैज्ञानिकों वाले पहले
भारतीय अभियान को 2007 तक का समय लग गया।

· भारत के स्थायी अनुसंधान स्टे श न, हिमाद्री ने जुलाई 2008 में परिचालन शुरू किया।

· हिमाद्रि को ध्रुवीय रातों ( जो 24 घंटे से अधिक समय तक चलती है ) के दौरान अवलोकन के लिए सुसज्जित किया गया है , और नॉर्वेजि
यन एजेंसियों से विशेष शीतकालीन गियर ( winter gear ) , परिवहन और रसद सहायता प्रदान की गई है।

QUESTION
23.
हाल ही में सतलज बेसिन में खोजी गई ' टैं टलम ( Tantalum ) ' धातु की खोज के संदर्भ में निम्नलिखित कथनों पर विचार करें :

कथन 1: टैं टलम सबसे अधिक संक्षारण प्रवण धातुओं ( corrosion prone metal ) में से एक है।

कथन 2: हवा के संपर्क में आने पर यह एक ऑक्साइड परत बना ती है, जिसे हटाना बेहद मुश्किल होता है, यहां तक कि जब यह मजबूत
और गर्म एसिड वातावरण के साथ संपर्क कर ती है।

उपरोक्त कथनों के संबंध में निम्नलिखित में से कौन सा सही है?

a) कथन-1 और कथन-2 दोनों सही हैं और कथन-2, कथन-1 की सही व्याख्या है।
b) कथन-1 और कथन-2 दोनों सही हैं और कथन-2, कथन-1 की सही व्याख्या नहीं है।
c) कथन-1 सही है लेकिन कथन-2 गलत है।
d) कथन-1 ग़लत है लेकिन कथन-2 सही है।
Correct Answer: D
Your Answer:
Explanation

Q. 23) Solution (d)

स्पष्टीकरण:

· इसमें उच्च संक्षारण प्रतिरोध होता है क्योंकि हवा के संपर्क में आने पर, यह एक ऑक्साइड परत बनाता है जिसे हटाना बेहद मुश्किल होता
है, भले ही यह मजबूत और गर्म एसिड वातावरण के साथ संपर्क करता हो। (इसलिए, कथन 1 गलत है लेकि न कथन 2 सही है)

· टैं टलम एक दुर्लभ धातु है जिसका परमाणु क्रमांक 73 है – जो तत्व के एक परमाणु में पाए जाने वाले प्रोटॉन की संख्या है । यह भूरा , भा
री , बहुत कठोर है और आज उपयोग में आने वाली सबसे अधिक संक्षारण प्रतिरोधी धातुओं में से एक है।

· शुद्ध होने पर, टैं टलम लचीला होता है, जिसका अर्थ है कि इसे बिना टू टे पतले तार या धागे में खींचा, या लंबा किया जा सकता है। इसके
अलावा, अमेरिकी ऊर्जा विभाग के अनुसार, "यह 150 डिग्री सेल्सियस से नीचे के तापमान पर रासायनिक हमले के प्रति लगभग पूरी तरह

IASbaba
Score:
Web: http://ilp.iasbaba.com/
44.00 /
Email: ilp@iasbaba.com
Page 123 200
UPSC 2024 -
Exam Title : Environment,
Sc...
rupeshkr.
Email :
09ee80@gmail.com
Contact :

से प्रतिरक्षित है, और के वल हाइड्रोफ्लोरिक एसिड, फ्लोराइड आयन युक्त अम्लीय समाधान और मुक्त सल्फर ट्राइऑक्साइड द्वारा हमला
किया जाता है।"

· विशेष रूप से, टैं टलम का गलनांक भी अत्यधिक उच्च होता है, जो के वल टं गस्टन और रेनियम से अधिक होता है।

IMPORTANT POINTS/VALUE ADDITIONS:

· भारतीय प्रौद्योगिकी संस्थान (आईआईटी), रोपड़ के शोधकर्ताओं की एक टीम ने पंजाब में सतलुज नदी की रेत में एक दुर्लभ धातु टैं टलम
की उपस्थिति पाई है।

· टैं टलम का सबसे अधिक उपयोग इलेक्ट्रॉनिक्स क्षेत्र में किया जाता है। टैं टलम से बने कै पेसिटर किसी भी अन्य प्रकार के कै पेसिटर की
तुलना में बिना अधिक रिसाव के छोटे आकार में अधिक बिजली संग्रहीत करने में सक्षम हैं। यह उन्हें स्मार्टफोन, लैपटॉप और डिजिटल कै मरे
जैसे पोर्टेबल इलेक्ट्रॉनिक उपकरणों में उपयोग के लिए आदर्श बनाता है।

· चूंकि टैं टलम का गलनांक उच्च होता है, इसलिए इसे अक्सर प्लैटिनम के विकल्प के रूप में उपयोग किया जाता है, जो अधिक महंगा है।
दुर्लभ धातु का उपयोग रासायनिक संयंत्रों, परमाणु ऊर्जा संयंत्रों, हवाई जहाजों और मिसाइलों के लिए घटक बनाने के लिए भी किया जाता
है।

· अमेरिकी ऊर्जा विभाग के अनुसार, टैं टलम शारीरिक तरल पदार्थों के साथ प्रतिक्रिया नहीं करता है और इसका उपयोग कृ त्रिम जोड़ों जैसे
सर्जिकल उपकरण और प्रत्यारोपण बनाने के लिए किया जाता है।

· टैं टलम कार्बाइड और ग्रेफाइट से बना एक मिश्रण ज्ञात सबसे कठोर सामग्रियों में से एक है और इसका उपयोग हाई-स्पीड मशीन टू ल्स के
काटने वाले किनारों पर किया जाता है।

QUESTION
24.
' वायुमंडलीय तरंग प्रयोग ( Atmospheric Wave Experiment ) ' के बारे में निम्नलिखित कथनों पर विचार करें :

1. AWE अपनी तरह का पहला प्रायोगिक प्रयास है जिसका उद्दे श्य स्थलीय और अंतरिक्ष मौसम के बीच परस्पर क्रिया का अध्ययन करना
है।

2. AWE को लॉन्च करके पृथ्वी की परिक्रमा करने वाले अंतर्राष्ट्रीय अंतरिक्ष स्टे श न के बाहरी हिस्से पर स्थापित किया जाएगा।

3. AWE पृथ्वी के वायुमंडल में रंगीन वायु चमक (colourful airglows) की कें द्रित मैपिंग करेगा।

उपरोक्त में से कितने कथन सही हैं?

a) के वल एक
b) के वल दो
c) सभी तीन
d) कोई नहीं
Correct Answer: C
Your Answer:
Explanation

Q. 24) Solution (c)

स्पष्टीकरण:

· AWE अपनी तरह का पहला प्रयोगात्मक प्रयास है जिसका उद्दे श्य स्थलीय और अंतरिक्ष मौसम के बीच परस्पर क्रिया का अध्ययन करना
है। मिशन इस संबंध का अध्ययन करेगा कि वायुमंडल की निचली परतों में तरंगें ऊपरी वायुमंडल को कै से प्रभावित करती हैं। (अतः, कथन
1 सही है)

IASbaba
Score:
Web: http://ilp.iasbaba.com/
44.00 /
Email: ilp@iasbaba.com
Page 124 200
UPSC 2024 -
Exam Title : Environment,
Sc...
rupeshkr.
Email :
09ee80@gmail.com
Contact :

· AWE को लॉन्च करके पृथ्वी की परिक्रमा करने वाले अंतर्राष्ट्रीय अंतरिक्ष स्टे श न (ISS) के बाहरी हिस्से पर स्थापित किया जाएगा।
सुविधाजनक बिंदु से, यह पृथ्वी की ओर दे खेगा और रंगीन प्रकाश बैंडों को रिकॉर्ड करेगा, जिन्हें आमतौर पर एयरग्लो के रूप में जाना जाता
है। (अतः, कथन 2 सही है)

· पृथ्वी के वायुमंडल में रंगीन वायु चमक की कें द्रित मैपिंग करेगा । (अतः, कथन 3 सही है)

IMPORTANT POINTS/VALUE ADDITIONS:

· AWE अध्ययन की एक नई खिड़की खोल सकता है, जिसमें वैज्ञानिक यह समझने का प्रयास कर रहे हैं कि क्या अंतरिक्ष का मौसम स्थलीय
और नीचे से ऊपर के बलों से प्रभावित होता है।

· AWE उन ऊं चाइयों पर उपग्रहों द्वारा आमतौर पर दे खी जा सकने वाली तरंगों की तुलना में अधिक सूक्ष्म क्षैतिज पैमानों पर तरंगों को जान
ने में सक्षम होगा, जो मिशन को अद्वितीय बनाता है।

· AWE मेसोपॉज (पृथ्वी की सतह से लगभग 85 से 87 किमी ऊपर) में वायु की चमक को मापेगा, जहां वायुमंडलीय तापमान शून्य से
100 डिग्री सेल्सियस नीचे तक गिर जाता है। इस ऊं चाई पर, इन्फ्रारेड बैंडविड् थ में हल्की एयरग्लो को जानना संभव है, जो सबसे चमकदार
दिखाई दे ता है जिससे आसानी से पता लगाया जा सकता है।

QUESTION
25.
वि -विलोपन प्रक्रिया (De-extinction process) का पालन करने के लिए निम्नलिखित चरणों को आरोही क्रम (ascending
order) में व्यवस्थित करें:

1. प्राइमर्डियल जर्म कोशिकाओं ( पीजीसी ) की पहचान करना ( Identifying Primordial Germ Cells (PGCs) )

2. अंतर्जातीय सरोगेट्स ( Interspecies Surrogates ) का विकास करना

3. निकटतम संबंधी के साथ आनुवंशि क तुलना करना

4. संपूर्ण जीनोम अनुक्रमण

नीचे दिए गए विकल्पों में से सही कोड चुनें।

a) 1, 2, 3, 4
b) 2, 3, 4, 1
c) 3, 4, 1, 2
d) 4, 3, 1, 2
Correct Answer: D
Your Answer:
Explanation

Q. 25) Solution (d)

स्पष्टीकरण:

· किसी प्रजाति को वि- विलुप्त करने के लिए सबसे पहले आवश्यक चीज़ सटीक और संपूर्ण आनुवंशि क जानकारी है। इसे प्रजाति जीनोम
के रूप में जाना जाता है ; प्रत्येक जीनोम में उस जीव के निर्माण और उसे विकसित होने की अनुमति दे ने के लिए आवश्यक सभी जानकारी
होती है।

· डोडो के पूरे जीनोम की तुलना रोड्रिग्स सॉलिटे यर के जीनोम से की जा रही है , डोडो के सबसे करीबी (विलुप्त भी) जीनोम से यह पता
लगाया जा सकता है कि डोडो कै से निर्मित हुआ है।

IASbaba
Score:
Web: http://ilp.iasbaba.com/
44.00 /
Email: ilp@iasbaba.com
Page 125 200
UPSC 2024 -
Exam Title : Environment,
Sc...
rupeshkr.
Email :
09ee80@gmail.com
Contact :

· निकोबार कबूतर, डोडो का निकटतम संबंधी है, और इसकी प्राइमर्डियल जर्म कोशिकाएं (पीजीसी) मिलीं। पीजीसी मूल रूप से एक
प्रजाति के शुक्राणु और अंडे के भ्रूणीय अग्रदूत हैं।

· सिद्धांत रूप में , जब मुर्गी ( chicken ) और मुर्गा ( rooster ) प्रजनन करते हैं ( अंतर्जातीय सरोगेट्स / Interspecies
Surrogates ), तो वे डोडो संतान को जन्म दें गे।

महत्वपूर्ण बिंदु/मूल्यवर्धन :

· जेनेटिक इंजीनियरिंग कं पनी कोलोसल बायोसाइंसेज और मॉरीशस वाइल्डलाइफ फाउंडेश न के बीच सहयोग न के वल डोडो को वापस
जीवन में लाने का वादा करता है, बल्कि मॉरीशस में इसके मूल निवास स्थान में इसे फिर से पेश करने का भी वादा करता है।

· फिर एक छोटे कबूतर को एक बड़े उड़ानहीन पक्षी में विकसित करने के लिए चयनात्मक प्रजनन में वर्षों लगेंगे। याद रखें, डोडो के साथ
ऐसा होने में प्रकृ ति को लाखों साल लग गए ।

· डोडो को पुनर्स्थापित करने से हमें 'संरक्षण आशावाद' पैदा करने का अवसर मिलता है, जो उम्मीद है कि दुनिया भर के लोगों को प्रेरित
करेगा।

QUESTION
26.
इलेक्ट्रिक वाहनों के लिए ' संयुक्त चार्जिंग मानकों ( Combined Charging standards ) ' के संदर्भ में निम्नलिखित कथनों पर वि
चार करें :

1. भारतीय मानक ब्यूरो (बीआईएस) ने स्वदे श ी रूप से विकसित एसी और डीसी संयुक्त चार्जिंग कनेक्टर मानकों को मंजूरी दे दी है।

2. स्वदे श ी रूप से विकसित चार्जिंग मानक विश्व में पहला है जो हल्के इलेक्ट्रिक वाहनों के साथ-साथ इलेक्ट्रिक चार पहिया वाहनों के लिए
प्रत्यावर्ती धारा (एसी) और प्रत्यक्ष धारा (डीसी) को जोड़ता है।

3. एक संयुक्त चार्जिंग मानक अपनी अंतरसंचालनीयता के कारण आकर्षक है, इसका उपयोग विभिन्न प्रकार के ईवी मॉडल और चार्जिंग
इंफ्रास्ट्रक्चर प्रदाताओं द्वारा किया जा सकता है।

4. यह ईवी निर्माताओं को सभी रेंज में एक समान मानक का उपयोग करने का अधिदे श दे ता है।

इनमें से कितने कथन सही हैं?

a) के वल एक
b) के वल दो
c) के वल तीन
d) सभी चार
Correct Answer: B
Your Answer:
Explanation

Q. 26) Solution (b)

स्पष्टीकरण:

· भारतीय मानक ब्यूरो (बीआईएस) ने स्कू टर, बाइक और रिक्शा जैसे हल्के इलेक्ट्रिक वाहनों (एलईवी) के लिए स्वदे श ी रूप से विकसित
एसी और डीसी संयुक्त चार्जिंग कनेक्टर मानक को मंजूरी दे दी है। (अतः कथन 1 सही है)

· स्वदे श ी रूप से विकसित चार्जिंग मानक विश्व में पहला है जो एलईवी के लिए प्रत्यावर्ती धारा (एसी) और प्रत्यक्ष धारा (डीसी) को जोड़ता है
। इलेक्ट्रि क चार पहिया वाहनों के लिए संयुक्त एसी और डीसी चार्जिंग मानक पहले से ही विश्व भर में उपयोग में हैं , जैसे संयुक्त
चार्जिंग सिस्टम (सीसीएस) मानक जो यूरोप में व्यापक रूप से उपयोग किया जाता है। (इसलिए कथन 2 गलत है)

IASbaba
Score:
Web: http://ilp.iasbaba.com/
44.00 /
Email: ilp@iasbaba.com
Page 126 200
UPSC 2024 -
Exam Title : Environment,
Sc...
rupeshkr.
Email :
09ee80@gmail.com
Contact :

· एक संयुक्त चार्जिंग मानक अपनी अंतरसंचालनीयता के कारण आकर्षक है जिसका अर्थ है कि इसका उपयोग विभिन्न प्रकार के ईवी मॉडल
और चार्जिंग इंफ्रास्ट्रक्चर प्रदाताओं द्वारा किया जा सकता है। (अतः कथन 3 सही है)

· हालाँकि नया स्वीकृ त मानक एक संयुक्त मानक बनाकर एसी और डीसी चार्जिंग के लिए अलग-अलग मानकों की समस्या को ठीक करता
है, लेकिन यह ईवी निर्माताओं को एक समान मानक का उपयोग करने के लिए बाध्य नहीं करता है जो रेंज की चिंता को दूर करने और
ईवी को तेजी से अपनाने को बढ़ावा दे ने में मदद कर सकता है। (अतः कथन 4 गलत है)

IMPORTANT POINTS/VALUE ADDITIONS:

· नया मानक, ISI7017 (भाग 2 / धारा 7): 2023, BIS द्वारा स्वीकृ त, नीति आयोग, विज्ञान और प्रौद्योगिकी विभाग, इलेक्ट्रिक
दोपहिया निर्माता एथर एनर्जी और विभिन्न अन्य सरकारी और उद्योग हितधारकों के बीच सहयोग के माध्यम से विकसित किया गया है।

· अमेरिका के पास कोई राष्ट्रीय मानक नहीं है, लेकिन ईवी निर्माता कु छ हद तक मानकीकरण पर जोर दे ने के लिए सहयोग कर रहे हैं ।

· यूरोप में, सीसीएस प्रमुख चार्जिंग कनेक्टर मानक है, और ईवी चार्जिंग नेटवर्क के लिए यूरोपीय संघ (ईयू) की आवश्यकता भी है।

QUESTION
27.
निम्नलिखित में से कौन सा कथन ' स्काईवेव संचार ( Skywave communication ) ' का सबसे अच्छा वर्णन करता है ?

a) यह निम्न आवृत्ति वाले रेडियो सिग्नल का उपयोग करता है।


b) यह लंबी दूरी के संचार के लिए आयनमंडल से रेडियो तरंगों के परावर्तन पर निर्भर करता है।
c) स्काईवेव संचार मुख्य रूप से डेटा ट्रांसमिशन के लिए फाइबर-ऑप्टिक तकनीक का उपयोग करता है।
d) यह सिग्नल प्रसार के लिए प्रत्यक्ष लाइन - ऑफ़ - विज़न ( line-of-sight ) पर आधारित है।
Correct Answer: B
Your Answer:
Explanation

Q. 27) Solution (b)

स्पष्टीकरण:

· कु छ उच्च आवृत्ति यों की रेडियो तरंगें आयनमंडल द्वारा वापस जमीन पर परावर्तित होती हैं जो क्षितिज से परे लंबी दूरी की उच्च आवृत्ति
संचार की सुविधा प्रदान करती हैं, जिसे स्काईवेव कम्युनिके शंस के रूप में जाना जाता है। (इसलिए विकल्प a ग़लत है)

· स्काईवेव संचार रेडियो तरंगों को प्रतिबिंबित करने के लिए पृथ्वी के वायुमंडल के भीतर एक परत, आयनमंडल का उपयोग करता है।
रेडियो तरंगों को एक विशिष्ट कोण पर आयनमंडल में संचारित करके , वे अपवर्तन से गुजरते हैं और वापस पृथ्वी की सतह की ओर
पुनर्निर्देशित हो जाते हैं। (इसलिए विकल्प बी सही है)

· स्काईवेव संचार डेटा ट्रांसमिशन के लिए रेडियो फ्रीक्वेंसी सिग्नल का उपयोग करता है। (इसलिए विकल्प सी गलत है)

· यह अनूठी घटना विस्तारित-दूरी संचार की सुविधा प्रदान करती है, जो विशेष रूप से ओवर-द-क्षितिज और अंतर्राष्ट्रीय संचार के लिए
फायदे मंद साबित होती है। (इसलिए विकल्प d गलत है)

IMPORTANT POINTS/VALUE ADDITIONS:

· आयनमंडल सौर विकिरण के माध्यम से आयनित हो जाता है, जिसमें आवेशि त कण होते हैं जो रेडियो तरंगों के साथ संपर्क करते हैं।
आयनमंडल की परावर्तक संपत्ति रेडियो तरंगों को उछालने की अनुमति दे ती है, जिससे जमीन-आधारित, लाइन-ऑफ़-विज़न विधियों की
तुलना में बहुत अधिक दूरी पर संचार सक्षम हो जाता है।

QUESTION
28.

IASbaba
Score:
Web: http://ilp.iasbaba.com/
44.00 /
Email: ilp@iasbaba.com
Page 127 200
UPSC 2024 -
Exam Title : Environment,
Sc...
rupeshkr.
Email :
09ee80@gmail.com
Contact :

Q. 28 ) एबाउसीन , जो एसिनेटोबैक्टर बाउमन्नी के विरुद्ध प्रभावी संकीर्ण - स्पेक्ट्रम एंटीबायोटिक गुणों वाला एक यौगिक है , के लिए का
र्रवाई का प्राथमिक तंत्र क्या है ?

a) डीएनए प्रतिकृ ति को रोकना


b) CCR2 प्रोटीन के कार्य को बाधित करना
c) कोशिका भित्ति संश्लेषण को अवरुद्ध करना
d) प्रोटीन संश्लेषण में हस्तक्षेप
Correct Answer: B
Your Answer:
Explanation

Q. 28) Solution (b)

स्पष्टीकरण:

· अबौसिन ( Abaucin ) एक यौगिक है जो एक संकीर्ण - स्पेक्ट्रम एंटीबायोटिक के रूप में उपयोगी गतिविधि दिखाता है। इसे एसिनेटोबै
क्टर बौमन्नी के विरुद्ध प्रभावी पाया गया है।

· यह बैक्टीरिया में CCR2 प्रोटीन के सामान्य कार्य को बाधित करता है। यह व्यवधान बैक्टीरिया के अंदर कु छ अणुओं की गति में बाधा
डालता है, जिससे उन्हें बाहरी झिल्ली तक पहुंचने से रोका जा सकता है। परिणामस्वरूप, एसिनेटोबैक्टर बाउमानी की वृद्धि बाधित हो जाती
है, जिससे संक्रमण पैदा करने की इसकी क्षमता कम हो जाती है। (इसलिए विकल्प बी सही है)

महत्वपूर्ण बिंदु/मूल्यवर्धन :

· मशीन-लर्निंग मॉडल रणनीति का उपयोग करके कृ त्रिम बुद्धिमत्ता के समर्थन के माध्यम से अबाउसिन की पहचान की गई थी। प्रारंभ में,
एक तंत्रिका नेटवर्क प्रणाली को एक डेटासेट पर प्रशिक्षित किया गया था जिसमें एसिनेटोबैक्टर बाउमनी के विकास को रोकने की क्षमता के
लिए 7,500 अणुओं की जांच की गई थी।

· तंत्रिका नेटवर्क द्वारा सुझाए गए 240 अणुओं में से 9 ने लक्ष्य बैक्टीरिया के विकास में 80% से अधिक का महत्वपूर्ण अवरोध प्रदर्शित
किया।

QUESTION
29.
निम्नलिखित कथनों पर विचार करें:

कथन 1: एंटीबायोटिक्स संक्रमण के व्यापक स्पेक्ट्रम में प्रभावी हैं, जिससे उन्हें प्रशासित करना आसान हो जाता है।

कथन 2: बैक्टीरियोफे ज में विशिष्ट रोगज़नक़ पर हमला करने की क्षमता होती है और इसलिए प्रतिरोध विकसित होने की संभावना कम होती
है।

उपरोक्त कथनों के संबंध में निम्नलिखित में से कौन सा सही है?

a) कथन-1 और कथन-2 दोनों सही हैं और कथन-2, कथन-1 की सही व्याख्या है।
b) कथन-1 और कथन-2 दोनों सही हैं और कथन-2, कथन-1 की सही व्याख्या नहीं है।
c) कथन-1 सही है लेकिन कथन-2 गलत है।
d) कथन-1 ग़लत है लेकिन कथन-2 सही है।
Correct Answer: B
Your Answer:
Explanation

Q. 29) Solution (b)

IASbaba
Score:
Web: http://ilp.iasbaba.com/
44.00 /
Email: ilp@iasbaba.com
Page 128 200
UPSC 2024 -
Exam Title : Environment,
Sc...
rupeshkr.
Email :
09ee80@gmail.com
Contact :

स्पष्टीकरण:

· बैक्टीरियोफे ज ( bacteriophage ) एक प्रकार का वायरस है जो बैक्टीरिया को संक्रमित करने में माहिर होता है , मेजबान कोशिकाओं
को नष्ट करने की अपनी क्षमता के कारण इसे " बैक्टीरिया भक्षक (bacteria eater) " नाम मिला है।

· फेज थेरेपी, एक उपचार दृष्टिकोण है, जिसमें चिन्हित किए गए जीवाणु रोगों वाले व्यक्तियों को विशिष्ट बैक्टीरियोफेज वायरस दे ना शामिल
है। बुबोनिक प्लेग से लेकर हैजा और दस्त तक विभिन्न बीमारियों के विरुद्ध सफल, फे ज थेरेपी अपनी विशिष्टता के लिए जानी जाती है।

· एंटीबायोटिक दवाओं के विपरीत, जिसमें कार्रवाई का एक व्यापक स्पेक्ट्रम होता है, फे ज थेरेपी अपने आवेदन में सीमित है, जिससे उपचार
करने से पहले सटीक रोगज़नक़ की पहचान करना आवश्यक हो जाता है।

IMPORTANT POINTS/VALUE ADDITIONS:

· फेज थेरेपी पर सीमित शोध के बावजूद, इसकी विशिष्टता प्रतिरोध विकास की संभावना को कम करती है, प्रभावशीलता को बढ़ाती है और
दुष्प्रभावों को कम करती है।

· आनुवंशि क अनुक्रमण में प्रगति के साथ, रोग पैदा करने वाले रोगजनकों की पहचान और सफलतापूर्वक इलाज किए गए रोगियों से फेज
थेरेपी और अधिक अनुकूलित करने की क्षमता रखती है।

QUESTION
30.
'नैनोकणों' के संदर्भ में निम्नलिखित कथनों पर विचार करें:

1. छोटे नैनोकणों का विचलित व्यवहार क्वांटम प्रभावों के उद्भव के कारण उत्पन्न होता है।

2. आकार के आधार पर नैनोकण परमाणुओं की तुलना में बहुत छोटे होते हैं, जहां परमाणु में हजारों से लाखों नैनोकण समा सकते हैं।

उपरोक्त में से कौन सा कथन सही है?

a) के वल एक
b) के वल दो
c) दोनों
d) कोई भी नहीं
Correct Answer: A
Your Answer:
Explanation

Q. 30) Solution (a)

स्पष्टीकरण:

· प्रत्येक तत्व विशिष्ट गुण प्रदर्शित करता है, जो आमतौर पर उसके परमाणुओं में इलेक्ट्रॉनों की संख्या और नाभिक के चारों ओर इन
इलेक्ट्रॉनों के वितरण से निर्धारित होता है। इसके अलावा, शुद्ध तत्व का प्रत्येक टु कड़ा अपने आकार की परवाह किए बिना बिल्कु ल समान
गुण प्रदर्शित करता है। यह रसायन विज्ञान के मूलभूत तथ्यों में से एक है।

· विचलित व्यवहार क्वांटम प्रभावों के उद्भव के कारण उत्पन्न होता है। सामान्य मानव अनुभव में किसी भी परिचित वस्तु की तुलना में,
इलेक्ट्रॉनों जैसे बहुत छोटे कणों की गति और व्यवहार मौलिक रूप से भिन्न और अजीब होते हैं। उप-परमाणु स्तर पर इस तरह के अजीब
व्यवहार का वर्णन 100 साल पहले भौतिकविदों द्वारा विकसित बेहद सफल क्वांटम सिद्धांत द्वारा किया गया है। (अतः कथन 1 सही है)

· लेकिन नैनोकण परमाणुओं की तुलना में बहुत बड़े होते हैं। परमाणु के आकार के आधार पर, एक नैनोकण में हजारों से लाखों
परमाणु पैक हो सकता है । (इसलिए कथन 2 गलत है)

IASbaba
Score:
Web: http://ilp.iasbaba.com/
44.00 /
Email: ilp@iasbaba.com
Page 129 200
UPSC 2024 -
Exam Title : Environment,
Sc...
rupeshkr.
Email :
09ee80@gmail.com
Contact :

· नैनो-आकार के कण जो एक ही तत्व के बड़े कणों की तुलना में थोड़ा अलग व्यवहार करते हैं। विशेष गुणों वाले इन नैनोकणों को क्वांटम
डॉट् स कहा गया

IMPORTANT POINTS/VALUE ADDITIONS:

· नैनोटे क्नोलॉजी में अनुसंधान को रसायन विज्ञान में 2023 नोबेल पुरस्कार से सम्मानित किया गया। वांछित विचलन व्यवहार वाले नैनोकण
टे लीविजन, कं प्यूटर स्क्रीन और एलईडी लैंप सहित विभिन्न आधुनिक उपकरणों का एक अभिन्न अंग बन गए हैं। जैव रसायन और चिकित्सा
में भी इसके व्यापक अनुप्रयोग हैं।

· नैनोकणों के सबसे उल्लेखनीय विशेष गुणों में से एक तब स्पष्ट हो जाता है जब वे प्रकाश के साथ संपर्क करते हैं। किसी भी सामग्री का रंग
सामग्री द्वारा अवशोषित या परावर्तित प्रकाश स्पेक्ट्रम की तरंग दै र्ध्य पर निर्भर करता है।

QUESTION
31.
'विशिष्ट अवशोषण दर (Specific Absorption Rate)' शब्दावली हाल ही में चर्चा में थी। निम्नलिखित में से कौन सा विकल्प इसका
सही वर्णन करता है?

a) मानव शरीर द्वारा अवशोषित विकिरण ऊर्जा


b) पृथ्वी द्वारा अवशोषित सौर सूर्यातप
c) विभिन्न तापमानों पर पौधे की CO2 अवशोषण दर
d) एक वर्ष में महासागरों द्वारा अवशोषित ऊष्मा
Correct Answer: A
Your Answer:
Explanation

Q. 31) Solution (a)

स्पष्टीकरण:

· फ्रांस की एजेंस नेश नेल डेस फ्रीक्वेंसी (एएनएफआर) ने ऐप्पल से फ्रांस में iPhone12 की बिक्री रोकने के लिए कहा, क्योंकि परीक्षणों में
फोन की विशिष्ट अवशोषण दर (एसएआर) - उपकरण के एक टु कड़े से शरीर द्वारा अवशोषित रेडियोफ्रीक्वेंसी ऊर्जा की दर की एक मा
प – को कानूनी तौर पर अनुमति से अधिक दिखाया गया था। (इसलिए विकल्प ए सही है)

· सीमाएं - फोन के विकिरण से जलने या हीटस्ट्रोक के जोखिम के आधार पर - पहले से ही उस स्तर से दस गुना नीचे निर्धारित की गई हैं जहां
वैज्ञानिकों को नुकसान के साक्ष्य मिले हैं।

· यूरोपीय मानक 4.0 वाट प्रति किलोग्राम की विशिष्ट अवशोषण दर है।

महत्वपूर्ण बिंदु/मूल्यवर्धन :

· डब्ल्यूएचओ और अन्य अंतरराष्ट्रीय स्वास्थ्य निकायों का कहना है कि इस बात का कोई निश्चित प्रमाण नहीं है कि मोबाइल फोन से निकलने
वाला विकिरण स्वास्थ्य पर अन्य प्रतिकू ल प्रभाव डालता है। हालाँकि, इसने और अधिक शोध की मांग की है।

· 2011 में, इंटरनेश नल एजेंसी फॉर रिसर्च ऑन कैं सर (आईएआरसी) ने मोबाइल फोन से निकलने वाले विकिरण को "संभवतः कैं सरकारी"
या वर्ग 2बी के रूप में वर्गीकृ त किया।

QUESTION
32.
परमाणु ऊर्जा के संदर्भ में निम्नलिखित कथनों पर विचार करें:

कथन 1: कु छ परमाणु रेडियोधर्मी होते हैं, वे दो हल्के तत्वों में टू ट जाते हैं, यही परमाणु ऊर्जा का आधार है।

IASbaba
Score:
Web: http://ilp.iasbaba.com/
44.00 /
Email: ilp@iasbaba.com
Page 130 200
UPSC 2024 -
Exam Title : Environment,
Sc...
rupeshkr.
Email :
09ee80@gmail.com
Contact :

कथन 2: कु छ परमाणुओं की संरचना में प्रोटॉन और न्यूट्रॉन की संख्या इतनी होती है कि यह नाभिक को स्वयं को स्थिर नहीं होने दे ती है।

उपरोक्त कथनों के संबंध में निम्नलिखित में से कौन सा सही है?

a) कथन-1 और कथन-2 दोनों सही हैं और कथन-2, कथन-1 की सही व्याख्या है।
b) कथन-1 और कथन-2 दोनों सही हैं और कथन-2, कथन-1 की सही व्याख्या नहीं है।
c) कथन-1 सही है लेकिन कथन-2 गलत है।
d) कथन-1 ग़लत है लेकिन कथन-2 सही है।
Correct Answer: A
Your Answer:
Explanation

Q. 32) Solution (a)

स्पष्टीकरण:

· पृथ्वी पर अधिकांश परमाणु अपने नाभिक में न्यूट्रॉन और प्रोटॉन की संतुलित संरचना के कारण स्थिर हैं। हालाँकि, कु छ अस्थिर परमाणुओं
में प्रोटॉन और न्यूट्रॉन की संख्या की संरचना ऐसी होती है कि यह नाभिक को खुद को एक साथ रखने की अनुमति नहीं दे ती है।

· ऐसे परमाणु रेडियोधर्मी माने जाते हैं और वे टू टकर दो हल्के तत्वों में विखंडित हो जाते हैं। यह अधिकांश परमाणु हथियारों और परमाणु
ऊर्जा का आधार है। (इसलिए विकल्प ए सही है)

IMPORTANT POINTS/VALUE ADDITIONS:

· यूरेनियम-235, भारी धातु यूरेनियम का एक अत्यंत दुर्लभ आइसोटोप, सबसे अधिक इस्तेमाल किया जाने वाला परमाणु ईंधन है, क्योंकि
यह उन कु छ तत्वों में से एक है जो प्रेरित विखंडन से गुजर सकते हैं। इसका मतलब यह है कि मानव द्वारा क्रियान्वित की गई प्रक्रिया द्वारा
तत्व को बहुत तेजी से तोड़ा जा सकता है।

· यह U-235 नाभिक को न्यूट्रॉन के अधीन करके किया जाता है। नाभिक तुरंत एक अतिरिक्त न्यूट्रॉन को अवशोषित कर लेता है और
परिणामस्वरूप अस्थिर हो जाता है - और तुरंत दो हल्के परमाणुओं और कु छ अतिरिक्त न्यूट्रॉन में टू ट जाता है। इस प्रक्रिया से वह चीज़
निकलती है जिसे परमाणु ऊर्जा के रूप में जाना जाता है।

QUESTION
33.
' एम्बरग्रीस ( Ambergris ) ' के संदर्भ में निम्नलिखित कथनों पर विचार करें :

1. एम्बरग्रीस एक मोमी पदार्थ है जो स्पर्म व्हेल के पाचन तंत्र में पाया जाता है

2. एम्बरग्रीस अपनी दुर्लभ और महंगी प्रकृ ति के कारण मुख्य रूप से इत्र के उत्पादन में उपयोग किया जाता है।

3. स्पर्म व्हेल वन्यजीव संरक्षण अधिनियम की अनुसूची 1 के तहत एक संरक्षित प्रजाति है

4. भारत में एम्बरग्रीस का रखना या व्यापार अवैध है।

इनमें से कितने कथन सही हैं?

a) के वल एक
b) के वल दो
c) के वल तीन
d) सभी चार
Correct Answer: C

IASbaba
Score:
Web: http://ilp.iasbaba.com/
44.00 /
Email: ilp@iasbaba.com
Page 131 200
UPSC 2024 -
Exam Title : Environment,
Sc...
rupeshkr.
Email :
09ee80@gmail.com
Contact :

Your Answer:
Explanation

Q. 33) Solution (c)

स्पष्टीकरण:

· एम्बरग्रीस एक मोमी पदार्थ है जो संरक्षित शुक्राणु व्हेल के पाचन तंत्र से उत्पन्न होता है। इसके गठन के बारे में एक सिद्धांत से पता चलता है
कि यह कु छ शुक्राणु व्हेल के जठरांत्र संबंधी मार्ग में उत्पन्न होता है। (अतः कथन 1 सही है)

· एम्बरग्रीस एक दुर्लभ पदार्थ है, जो अंतरराष्ट्रीय बाजार में इसकी उच्च मांग और उच्च कीमत में योगदान दे ता है। परंपरागत रूप से, इसका
उपयोग ऐसे इत्र बनाने के लिए किया जाता है। हालाँकि अतीत में कु छ संस्कृ तियों में इसका उपयोग भोजन, मादक पेय और तम्बाकू को
स्वादिष्ट बनाने के लिए किए जाने के रिकॉर्ड हैं, लेकिन वर्तमान में इन उद्दे श्यों के लिए इसका उपयोग शायद ही कभी किया जाता है। (अतः
कथन 2 सही है)

· स्पर्म व्हेल वन्यजीव संरक्षण अधिनियम की अनुसूची 2 के तहत एक संरक्षित प्रजाति है। (इसलिए कथन 3 गलत है)

· एम्बरग्रीस और इसके उपोत्पादों सहित इसके किसी भी उप-उत्पाद का कब्ज़ा या व्यापार, वन्यजीव संरक्षण अधिनियम, 1972 के
प्रावधानों के तहत अवैध है। (इसलिए कथन 4 सही है)

IMPORTANT POINTS/VALUE ADDITIONS:

· ला पाल्मा के कै नरी द्वीप के तट पर एक स्पर्म व्हेल का शव बहकर आने के कु छ दिनों बाद एम्बरग्रीस मिला, जिसे 'तैरता हुआ सोना' भी
कहा जाता है।

· यह दे खा गया है कि एम्बरग्रीस की तस्करी करने वाले गिरोह इसे तटीय क्षेत्रों से खरीदते हैं और इसे कु छ अन्य दे श ों के माध्यम से गंतव्य दे श ों
में भेजते हैं जिनके साथ भारत का समुद्री व्यापार तुलनात्मक रूप से कम कठोर है।

QUESTION
34.
आदित्य-एल1 सौर मिशन के संदर्भ में निम्नलिखित कथनों पर विचार करें:

1. लैग्रेंज बिंदु ( Lagrange point ) यह सुनिश्चित करता है कि आदित्य - एल 1 सूर्य का निरंतर , निर्बाध दृश्य बनाए रख सकता है।

2. L1 लैग्रेंज बिंदु स्थान उपग्रह को पृथ्वी के चुंबकीय क्षेत्र और वायुमंडल की उपस्थिति में सौर विकिरण और चुंबकीय तूफानों का निरीक्षण
करने की अनुमति दे ता है।

3. L1 लैग्रेंज बिंदु स्थान गुरुत्वाकर्षण स्थिरता प्रदान करता है और बार-बार कक्षीय रखरखाव प्रयासों की आवश्यकता को कम करता है।

उपरोक्त में से कितने कथन सही हैं?

a) के वल एक
b) के वल दो
c) सभी तीन
d) कोई नहीं
Correct Answer: B
Your Answer:
Explanation

Q. 34) Solution (b)

स्पष्टीकरण:

IASbaba
Score:
Web: http://ilp.iasbaba.com/
44.00 /
Email: ilp@iasbaba.com
Page 132 200
UPSC 2024 -
Exam Title : Environment,
Sc...
rupeshkr.
Email :
09ee80@gmail.com
Contact :

· एल1 लैग्रेंज बिंदु पर रणनीतिक प्लेसमेंट यह सुनिश्चित करता है कि आदित्य-एल1 सूर्य का निरंतर, निर्बाध दृश्य बनाए रख सकता है। (अतः
कथन 1 सही है)

· यह स्थान उपग्रह को पृथ्वी के चुंबकीय क्षेत्र और वायुमंडल से प्रभावित होने से पहले सौर विकिरण और चुंबकीय तूफानों तक
पहुंचने की भी अनुमति दे ता है । (इसलिए कथन 2 गलत है)

· इसके अतिरिक्त, L1 बिंदु की गुरुत्वाकर्षण स्थिरता उपग्रह की परिचालन दक्षता को अनुकूलित करते हुए, लगातार कक्षीय रखरखाव
प्रयासों की आवश्यकता को कम करती है। (अतः कथन 3 सही है)

IMPORTANT POINTS/VALUE ADDITIONS:

· आदित्य एल1 सूर्य का अध्ययन करने वाला पहला अंतरिक्ष आधारित भारतीय मिशन होगा। अंतरिक्ष यान को सूर्य-पृथ्वी प्रणाली के लैग्रेंज
बिंदु 1 (एल1) के चारों ओर एक प्रभामंडल कक्षा में रखा जाएगा, जो पृथ्वी से लगभग 1.5 मिलियन किमी दूर है।

· L1 बिंदु के चारों ओर प्रभामंडल कक्षा में रखे गए उपग्रह को बिना किसी ग्रहण/ग्रहण के सूर्य को लगातार दे खने का प्रमुख लाभ होता है।
इससे वास्तविक समय में सौर गतिविधियों और अंतरिक्ष मौसम पर इसके प्रभाव को दे खने का अधिक लाभ मिलेगा।

· अंतरिक्ष यान विद्युत चुम्बकीय और कण और चुंबकीय क्षेत्र डिटे क्टरों का उपयोग करके प्रकाशमंडल, क्रोमोस्फीयर और सूर्य की सबसे
बाहरी परतों (कोरोना) का निरीक्षण करने के लिए सात पेलोड ले जाता है।

· विशेष सुविधाजनक बिंदु L1 का उपयोग करते हुए, चार पेलोड सीधे सूर्य को दे खते हैं और शेष तीन पेलोड लैग्रेंज बिंदु L1 पर कणों और
क्षेत्रों का इन-सीटू अध्ययन करते हैं, इस प्रकार अंतरग्रहीय माध्यम में सौर गतिशीलता के प्रसार प्रभाव का महत्वपूर्ण वैज्ञानिक अध्ययन प्रदान
करते हैं।

· उम्मीद है कि आदित्य एल1 पेलोड के सूट कोरोनल हीटिंग, कोरोनल मास इजेक्शन, प्री-फ्लेयर और फ्लेयर गतिविधियों और उनकी
विशेषताओं, अंतरिक्ष मौसम की गतिशीलता, कण और क्षेत्रों के प्रसार आदि की समस्या को समझने के लिए सबसे महत्वपूर्ण जानकारी प्रदान
करेंगे।

QUESTION
35.
अंतरिक्ष में कार्बनिक अणु की खोज के संदर्भ में निम्नलिखित कथनों पर विचार करें:

1. CH3+ को मिथाइल धनायन ( methyl cation ) के रूप में भी जाना जाता है , इसे पहली बार अंतरिक्ष में ओरियन नेबुला में चिन्हि
त किया गया है।

2. CH3+ एक सरल कार्बनिक अणु है जो अन्य अणुओं के साथ प्रतिक्रिया करके अधिक जटिल यौगिक बनाता है।

3. जेम्स वेब स्पेस टे लीस्कोप, स्पेक्ट्रोस्कोपी का उपयोग करने के बावजूद CH3+ के फिंगरप्रिंट का पता लगाने में विफल रहा।

उपरोक्त में से कितने कथन सही हैं?

a) के वल एक
b) के वल दो
c) सभी तीन
d) कोई नहीं
Correct Answer: B
Your Answer: Unanswered
Explanation

Q. 35) Solution (b)

स्पष्टीकरण:

IASbaba
Score:
Web: http://ilp.iasbaba.com/
44.00 /
Email: ilp@iasbaba.com
Page 133 200
UPSC 2024 -
Exam Title : Environment,
Sc...
rupeshkr.
Email :
09ee80@gmail.com
Contact :

· CH3+ जिसे मिथाइल धनायन के रूप में भी जाना जाता है, पहली बार अंतरिक्ष में चिन्हित किया गया है। (अतः कथन 1 सही है)

· सरल लेकिन प्रतिक्रियाशील CH3+ सहित कार्बनिक अणुओं में आमतौर पर हाइड्रोजन और ऑक्सीजन, नाइट्रोजन या फास्फोरस जैसे
अन्य तत्वों से जुड़े कार्बन परमाणु होते हैं। CH3+ जटिल प्रतिक्रियाओं के लिए उत्प्रेरक के रूप में कार्य करता है, जो पृथ्वी से परे जीवन के
लिए आवश्यक मूलभूत घटकों की उपस्थिति का संकेत दे ता है। (अतः कथन 2 सही है)

· CH3+ फ़िंगरप्रिंट की महत्वपूर्ण खोज स्पेक्ट्रोस्कोपी की सटीक तकनीक के माध्यम से पूरी की गई, जिसमें जेम्स वेब स्पेस टे लीस्कोप द्वारा
ओरियन नेबुला में अणुओं से अद्वितीय प्रकाश उत्सर्जन का प्रदर्शन किया गया था। (इसलिए कथन 3 गलत है)

IMPORTANT POINTS/VALUE ADDITIONS:

· जेम्स वेब स्पेस टे लीस्कोप (JWST) ने अंतरिक्ष में CH3+ अणु की पहचान करने में महत्वपूर्ण भूमिका निभाई, जिसके प्रारंभिक परिणाम
नेचर जर्नल में प्रकाशित हुए।

· CH3+ अणु की हालिया खोज निर्जीव पदार्थ से जीवन की उत्पत्ति को समझने की हमारी खोज में एक मील का पत्थर है, जो जीवन की
कार्बन-आधारित प्रकृ ति पर जोर दे ती है।

· 1,350 प्रकाश वर्ष दूर स्थित ओरियन नेबुला में एक युवा तारे के चारों ओर धूल और गैस की घूमती हुई डिस्क से निकलने वाली रोशनी में
अणु के निशान दे खे गए।

QUESTION
36.
निम्नलिखित में से किस खगोलीय घटना का उपयोग वैज्ञानिकों द्वारा ' कॉस्मिक क्लॉक ( Cosmic Clock ) ' के रूप में किया जाता है :

a) ब्लैक होल
b) पल्सर
c) धूमके तु ( Comets )
d) व्हाइट ड् वॉर्फ (White Dwarfs)
Correct Answer: B
Your Answer:
Explanation

Q. 36) Solution (b)

स्पष्टीकरण:

· पल्सर दूर स्थित तेजी से घूमने वाले न्यूट्रॉन तारे हैं जो विकिरण के स्पंदों का उत्सर्जन करते हैं, जिन्हें पृथ्वी से प्रकाश की चमकदार चमक के
रूप में दे खा जाता है। ये विस्फोट अत्यंत सटीक अंतराल पर होते हैं, और इसलिए वैज्ञानिक पल्सर को 'कॉस्मिक क्लॉक' के रूप में उपयोग
करते हैं ।

· पल्सर की घूर्णन अवधि घड़ी की टिक-टिक के समान उल्लेखनीय रूप से स्थिर होती है। कु छ पल्सर की घूर्णन अवधि मिलीसेकंड से लेकर
कु छ सेकंड तक होती है।

IMPORTANT POINTS/VALUE ADDITIONS:

· उनके घूर्णन की स्थिरता उन्हें खगोलीय अध्ययन में सटीक समय निर्धारण के लिए उत्कृ ष्ट उम्मीदवार बनाती है।

· वैज्ञानिक विभिन्न घटनाओं का अध्ययन करने के लिए पल्सर का उपयोग प्राकृ तिक आकाशीय घड़ियों के रूप में करते हैं, जिसमें बाइनरी
स्टार सिस्टम की गतिशीलता, गुरुत्वाकर्षण तरंगों के प्रभाव और इंटरस्टे लर और इंटरगैलेक्टिक माध्यमों के गुण शामिल हैं।

QUESTION
37.

IASbaba
Score:
Web: http://ilp.iasbaba.com/
44.00 /
Email: ilp@iasbaba.com
Page 134 200
UPSC 2024 -
Exam Title : Environment,
Sc...
rupeshkr.
Email :
09ee80@gmail.com
Contact :

“ अंतरिक्ष दूरबीन आकाश के एक तिहाई से अधिक भाग में 10 अरब प्रकाश वर्ष तक की अरबों आकाशगंगाओं का अवलोकन करके अंत
रिक्ष और समय में ब्रह्मांड की बड़े पैमाने की संरचना का एक बड़ा मानचित्र तैयार करेगा , तथा गुरुत्वाकर्षण की भूमिका एवं डार्क एनर्जी औ
र डार्क मैटर की प्रकृ ति के बारे में अधिक खुलासा करेगा । " प्रश्न में पहल का तात्पर्य है :

a) जूस (Juice)
b) फर्मी गामा-रे स्पेस टे लीस्कोप
c) जेम्स वेब स्पेस टे लीस्कोप
d) यूक्लिड ( Euclid )
Correct Answer: D
Your Answer:
Explanation

Q. 37) Solution (d)

स्पष्टीकरण:

· ईएसए का यूक्लिड मिशन अंधेरे ब्रह्मांड की संरचना और विकास का पता लगाने के लिए डिज़ाइन किया गया है। अंतरिक्ष दूरबीन आकाश
के एक तिहाई से अधिक भाग में 10 अरब प्रकाश वर्ष तक की अरबों आकाशगंगाओं का अवलोकन करके अंतरिक्ष और समय में ब्रह्मांड की
बड़े पैमाने की संरचना का एक बड़ा नक्शा तैयार करेगा।

· यूक्लिड यह पता लगाएगा कि ब्रह्मांड का विस्तार कै से हुआ और ब्रह्मांडीय इतिहास में संरचना कै से बनी, गुरुत्वाकर्षण की भूमिका तथा डा
र्क एनर्जी और डार्क मैटर की प्रकृ ति के बारे में और अधिक खुलासा करेगा । (इसलिए विकल्प d सही है)

· ईएसए का ज्यूपिटर आइसी मून्स एक्सप्लोरर, जूस, रिमोट सेंसिंग, भूभौतिकीय और स्वस्थानी उपकरणों के एक सेट के साथ विशाल गैस
ग्रह और उसके तीन बड़े महासागर-असर वाले चंद्रमाओं - गेनीमेड, कै लिस्टो और यूरोपा का विस्तृत अवलोकन करेगा।

QUESTION
38.
निम्नलिखित में से कौन सा कथन फौकॉल्ट पेंडुलम ( Foucault ’ s Pendulum ) का सटीक वर्णन करता है ?

a) यह पृथ्वी के चुंबकीय क्षेत्र की सामर्थ्य को मापता है।


b) यह एक उपकरण है जिसका उपयोग पृथ्वी के घूर्णन को प्रदर्शित करने के लिए किया जाता है।
c) यह हृदय स्वास्थ्य का अध्ययन करने के लिए एक चिकित्सा उपकरण है।
d) यह 19वीं शताब्दी में आविष्कार किया गया एक प्रकार का घड़ी तंत्र है।
Correct Answer: B
Your Answer:
Explanation

Q. 38) Solution (b)

स्पष्टीकरण:

· फौकॉल्ट पेंडुलम पृथ्वी के घूर्णन को प्रदर्शित करने के सिद्धांत पर आधारित है। पेंडुलम में एक भारी वजन होता है जिसे एक निश्चित बिंदु से
लटकाया जाता है ताकि यह किसी भी दिशा में स्वतंत्र रूप से घूम सके । जैसे पेंडुलम आगे-पीछे घूमता है, पृथ्वी उसके नीचे घूमती है।

· प्रदर्शन की कुं जी यह है कि पेंडुलम के झूले का तल स्थिर रहता है जबकि पृथ्वी उसके नीचे घूमती है। यह घूर्णन समय के साथ पेंडुलम के
घूमने की दिशा में एक स्पष्ट परिवर्तन का कारण बनता है। पेंडुलम को एक संपूर्ण घूर्णन पूरा करने में लगने वाला समय उसके स्थान और पृथ्वी
के अक्षांश पर निर्भर करता है।

· इस घटना को पहली बार 1851 में फ्रांसीसी भौतिक विज्ञानी लियोन फौकॉल्ट द्वारा प्रदर्शित किया गया था। फौकॉल्ट का पेंडुलम एक
लोकप्रिय शैक्षिक उपकरण बन गया, जो पृथ्वी के घूर्णन को दर्शाता है और पृथ्वी की गति की एक दृश्य पुष्टि प्रदान करता है। यह दिखाने का

IASbaba
Score:
Web: http://ilp.iasbaba.com/
44.00 /
Email: ilp@iasbaba.com
Page 135 200
UPSC 2024 -
Exam Title : Environment,
Sc...
rupeshkr.
Email :
09ee80@gmail.com
Contact :

एक सुंदर तरीका है कि पृथ्वी अपनी धुरी पर घूमती है, और इसे दुनिया भर के विभिन्न संग्रहालयों और शैक्षणिक संस्थानों में प्रदर्शित किया
गया है।

IMPORTANT POINTS/VALUE ADDITIONS:

· भारत के नवनिर्मित संसद भवन में फौकॉल्ट पेंडुलम स्थापित किया गया है। पेंडुलम संविधान कक्ष के शीर्ष पर एक रोशनदान से लटका हुआ
है, और "ब्रह्मांड के विचार के साथ भारत के विचार के एकीकरण" का प्रतीक है।

· इसे कोलकाता में नेश नल काउंसिल ऑफ साइंस म्यूजियम (एनसीएसएम) द्वारा बनाया गया है, पेंडुलम को भारत में इस तरह का सबसे बड़ा
टु कड़ा करार दिया जा रहा है, जिसकी ऊं चाई 22 मीटर है और इसका वजन 36 किलोग्राम है।

· भारत में सबसे पहले, इसे 1991 में पुणे में इंटर-यूनिवर्सिटी सेंटर फॉर एस्ट्रोनॉमी एंड एस्ट्रोफिजिक्स (IUCAA) में स्थापित किया गया
था।

QUESTION
39.
' सबमर्सिबल ( Submersible ) और सबमरीन (Submarine) ' के बीच अंतर के संदर्भ में निम्नलिखित कथनों पर विचार करें :

1. सबमर्सिबल एक स्वायत्त क्राफ्ट के रूप में कार्य नहीं करता है, बल्कि तैनाती और वापसी के लिए एक समर्थन मंच पर निर्भर करता है।

2. सबमरीन के विपरीत सबमर्सिबल मनुष्यों को ले जाने में सक्षम नहीं हैं , जिन्हें मनुष्यों द्वारा संचालित किया जाता है।

3. सबमरीन छत्र शब्द है और सबमर्सिबल एक प्रकार की सबमरीन है।

उपरोक्त में से कितने कथन सही हैं?

a) के वल एक
b) के वल दो
c) सभी तीन
d) कोई नहीं
Correct Answer: A
Your Answer:
Explanation

Q. 39) Solution (a)

स्पष्टीकरण:

· सबमर्सिबल एक स्वायत्त क्राफ्ट के रूप में कार्य नहीं करता है , बल्कि तैनाती और वापसी के लिए एक समर्थन मंच पर निर्भर करता है। (
अतः कथन 1 सही है )

· प्रकार के आधार पर सबमर्सिबल को मानवयुक्त या मानवरहित किया जा सकता है। वे मानव संचालित ( मानव चालित सबमर्सिबल ) या दूर
से संचालित ( आरओवी ) और स्वायत्त ( एयूवी ) हो सकते हैं। हाल ही में मानवयुक्त सबमर्सिबल टाइटन समुद्र में फट गया। ( इसलिए कथन
2 गलत है )

· सबमरीन विशेष रूप से पानी की सतह के नीचे स्वतंत्र संचालन के लिए डिज़ाइन किए गए चालक दल के वाहनों को संदर्भित करती हैं, और
सबमर्सिबल एक प्रकार की सबमरीन नहीं है। (इसलिए कथन 3 गलत है)

IMPORTANT POINTS/VALUE ADDITIONS:

· सबमर्सिबल और सबमरीन पूरी तरह से विनिमेय नहीं हैं। यूएस नेश नल ओशनिक एंड एटमॉस्फे रिक एडमिनिस्ट्रेश न के अनुसार, उनके बीच
अंतर यह है कि एक सबमरीन में एक बंदरगाह छोड़ने और अपने आप वापस बंदरगाह पर आने की पर्याप्त शक्ति होती है।

IASbaba
Score:
Web: http://ilp.iasbaba.com/
44.00 /
Email: ilp@iasbaba.com
Page 136 200
UPSC 2024 -
Exam Title : Environment,
Sc...
rupeshkr.
Email :
09ee80@gmail.com
Contact :

· दूसरी ओर, एक सबमर्सिबल में बिजली का भंडार सीमित होता है इसलिए इसे एक स्टे श न द्वारा "लॉन्च" करने की आवश्यकता होती है जिसे
इसे पुनर्प्राप्त भी करना होगा। इसे समुद्र में एक निश्चित बिंदु पर छोड़ने की आवश्यकता है जहां वे जहाज पर वापस आने से पहले अन्वेषण
कर सकें जो उन्हें बंदरगाह पर वापस लाएगा।

QUESTION
40.
निम्नलिखित में से कौन सा कारक जनसंख्या आनुवंशि की का अध्ययन करने के लिए उपयोग किए जाने वाले 'हार्डी-वेनबर्ग संतुलन' को
प्रभावित करता है?

1. जीन प्रवास या जीन प्रवाह

2. आनुवंशि क विस्थापन (Genetic drift)

3. उत्परिवर्तन ( Mutation )

4. यादृच्छिक संभोग ( Random mating )

5. प्राकृ तिक चयन ( Natural selection )

उपरोक्त में से कितने सही हैं?

a) के वल दो
b) के वल तीन
c) के वल चार
d) सभी पांच
Correct Answer: C
Your Answer:
Explanation

Q. 40) Solution (c)

स्पष्टीकरण:

· यादृच्छिक संभोग हार्डी-वेनबर्ग संतुलन (एचडब्ल्यूई) मॉडल की प्रमुख धारणाओं में से एक है। हार्डी-वेनबर्ग संतुलन एक ऐसी आबादी में
एलील्स के सैद्धांतिक वितरण का वर्णन करता है जो विकसित नहीं हो रही है। (अतः कथन 4 ग़लत है)

· हार्डी - वेनबर्ग संतुलन को प्रभावित करने वाले पांच कारक ज्ञात हैं। ये जीन प्रवास या जीन प्रवाह , आनुवंशि क विस्थापन , उत्परिवर्तन ,
आनुवंशि क पुनर्संयोजन और प्राकृ तिक चयन हैं । जब जनसंख्या के एक वर्ग का दूसरे स्थान और जनसंख्या में प्रवास होता है , तो मूल और न
ई जनसंख्या में जीन आवृत्ति याँ बदल जाती हैं। ( अतः कथन 1,2,3 और 5 सही हैं )

· नई जनसंख्या में नए जीन/एलील जुड़ जाते हैं और ये पुरानी जनसंख्या से लुप्त हो जाते हैं।

· यदि यह जीन प्रवास कई बार होता है तो एक जीन प्रवाह होगा। यदि वही परिवर्तन संयोगवश होता है तो इसे आनुवंशि क विस्थापन कहा
जाता है।

· कभी-कभी जनसंख्या के नए नमूने में एलील आवृत्ति में परिवर्तन इतना भिन्न होता है कि वे एक अलग प्रजाति बन जाते हैं। मूल विस्थापित
जनसंख्या संस्थापक बन जाती है और इस प्रभाव को "संस्थापक प्रभाव" कहा जाता है।

IMPORTANT POINTS/VALUE ADDITIONS:

· संस्थापक प्रभाव ( Founder Effect ) एक आनुवांशि क घटना है जो तब घटित होती है जब व्यक्तियों का एक छोटा समूह एक नई आ
बादी स्थापित करता है , और इसमें शामिल व्यक्तियों की सीमित संख्या के कारण , नई आबादी में मूल आबादी की आनुवंशि क विविधता का

IASbaba
Score:
Web: http://ilp.iasbaba.com/
44.00 /
Email: ilp@iasbaba.com
Page 137 200
UPSC 2024 -
Exam Title : Environment,
Sc...
rupeshkr.
Email :
09ee80@gmail.com
Contact :

एक उपसमूह होता है। इससे जीन पूल वाली आबादी स्रोत आबादी से भिन्न हो सकती है , और संस्थापक व्यक्तियों के प्रभाव के कारण नई आ
बादी में कु छ आनुवंशि क लक्षण या विकार अधिक प्रचलित हो सकते हैं।

· उदाहरण के लिए, यदि व्यक्तियों का एक छोटा समूह किसी नए क्षेत्र में प्रवास करता है और आबादी स्थापित करता है, तो उस नई आबादी
का आनुवंशि क ढांचा संस्थापकों द्वारा लाए गए जीन द्वारा निर्धारित किया जाता है। संस्थापक व्यक्तियों में मौजूद किसी भी आनुवांशि क
विविधता को आबादी की बाद की पीढ़ियों में असमान रूप से दर्शाया जाएगा।

· संस्थापक प्रभाव आनुवंशि क विस्थापन का एक विशेष मामला है, जहां आकस्मिक घटनाएं किसी जनसंख्या की आनुवंशि क संरचना को
निर्धारित करने में महत्वपूर्ण भूमिका निभाती हैं, और समय के साथ जनसंख्या की आनुवंशि क विविधता और स्वास्थ्य पर इसका प्रभाव पड़
सकता है।

QUESTION
41.
अभिसरण और अपसारी उद्भव ( convergent and divergent evolution ) के बीच अंतर के संदर्भ में निम्नलिखित में से कौन सा
कथन सही है ?

a) अभिसारी उद्भव के परिणामस्वरूप असंबंधित प्रजातियों में समान लक्षण विकसित होते हैं, जबकि अपसारी उद्भव के परिणामस्वरूप
निकट से संबंधित प्रजातियों में विभिन्न लक्षण उभरते हैं।
b) अभिसरण उद्भव से निकट संबंधी प्रजातियों में विभिन्न लक्षणों का विकास होता है, जबकि अपसारी उद्भव के परिणामस्वरूप
असंबंधित प्रजातियों में समान लक्षण उभरते हैं।
c) अभिसारी और अपसारी दोनों उद्भव निकट संबंधी प्रजातियों में समान लक्षणों के विकास की ओर ले जाते हैं।
d) अभिसारी और अपसारी दोनों उद्भव के परिणामस्वरूप असंबंधित प्रजातियों में विभिन्न लक्षण उभरते हैं।
Correct Answer: A
Your Answer:
Explanation

Q. 41) Solution (a)

स्पष्टीकरण:

· अभिसरण उद्भव ( Convergent evolution ) और अपसारी उद्भव ( divergent evolution ), उद्भव / विकास के दो पैटर्न हैं
जो बताते हैं कि समय के साथ प्रजातियों में समानताएं या अंतर कै से विकसित होते हैं।

· अभिसरण उद्भव : यह तब होता है जब असंबद्ध प्रजातियाँ समान पर्यावरणीय चुनौतियों के अनुकूल होने के कारण स्वतंत्र रूप से समान
लक्षण या विशेषताएँ विकसित करती हैं। हो सकता है कि ये प्रजातियाँ हाल के सामान्य पूर्वज साझा न करें, लेकिन उन्हें अनुरूप चयनात्मक
दबावों का सामना करना पड़ता है, जिससे अनुरूप संरचनाओं या कार्यों का विकास होता है।

· अपसारी उद्भव : इसके विपरीत, अपसारी उद्भव तब होता है जब निकट संबंधी प्रजातियाँ अलग-अलग लक्षण या विशेषताएँ विकसित
करती हैं। समय के साथ, एक सामान्य पूर्वज की आबादी अलग-अलग पर्यावरणीय परिस्थितियों के अनुकूल हो सकती है, जिसके
परिणामस्वरूप विविधताएं पैदा होती हैं जो प्रजातियों को अलग करती हैं। (इसलिए विकल्प ए सही है)

IMPORTANT POINTS/VALUE ADDITIONS:

· अभिसरण उद्भव का उदाहरण: चमगादड़ और पक्षियों के पंख उड़ान में समान उद्दे श्यों को पूरा करते हैं, लेकिन ये जानवर निकट से संबंधित
नहीं हैं।

· अपसारी उद्भव का उदाहरण: डार्विन के फिंच में विभिन्न प्रकार की चोंच के आकार पाए गए, जहां विभिन्न प्रजातियां गैलापागोस द्वीप समूह
पर विभिन्न खाद्य स्रोतों के लिए अनुकूलित हुईं।

QUESTION
42.
एक अणु को 'आनुवंशि क सामग्री' के रूप में कार्य करने के लिए आवश्यक मानदं डों के ढांचे के भीतर निम्नलिखित कथनों पर विचार करें:

IASbaba
Score:
Web: http://ilp.iasbaba.com/
44.00 /
Email: ilp@iasbaba.com
Page 138 200
UPSC 2024 -
Exam Title : Environment,
Sc...
rupeshkr.
Email :
09ee80@gmail.com
Contact :

1. इसे अपनी प्रतिकृ ति (replica) उत्पन्न करने में सक्षम होना चाहिए।

2. यह रासायनिक और संरचनात्मक रूप से स्थिर होना चाहिए।

3. इसे विकास के लिए आवश्यक धीमे परिवर्तनों (उत्परिवर्तन) की गुंजाइश प्रदान करनी चाहिए।

4. इसे स्वयं को 'मेंडेलियन कै रेक्टर' के रूप में अभिव्यक्त करने में सक्षम होना चाहिए।

उपरोक्त में से कितने कथन सही हैं?

a) के वल एक
b) के वल दो
c) के वल तीन
d) सभी चार
Correct Answer: D
Your Answer:
Explanation

Q. 42) Solution (d)

स्पष्टीकरण:

· आनुवंशि क सामग्री को प्रतिकृ ति बनाने में सक्षम होना चाहिए ताकि वह कोशिका विभाजन या प्रजनन के दौरान अपनी जानकारी अगली
पीढ़ी तक पहुंचा सके । यह आनुवंशि क सामग्री की एक मूलभूत विशेषता है।

· आनुवंशि क सामग्री इतनी स्थिर होनी चाहिए कि जीवन चक्र के विभिन्न चरणों, आयु या
जीव के शरीर विज्ञान में परिवर्तन के साथ न बदले।

· आनुवंशि क सामग्री को उत्परिवर्तन की अनुमति दे नी चाहिए, जो डीएनए अनुक्रम में परिवर्तन हैं। ये उत्परिवर्तन विकास की प्रक्रिया के लिए
कच्चा माल प्रदान करते हैं, जिससे जीवों को समय के साथ अपने वातावरण के अनुकूल होने की अनुमति मिलती है।

IMPORTANT POINTS/VALUE ADDITIONS:

· मेंडेलियन लक्षण वंश ानुगत लक्षण हैं जो ग्रेगर मेंडल के वंश ानुक्रम के सिद्धांतों द्वारा शासित होते हैं। ये लक्षण प्रभुत्व और अप्रभावीता का
अनुसरण करते हैं, जहां प्रमुख लक्षण अप्रभावी लक्षणों की अभिव्यक्ति को छिपा दे ते हैं। पृथक्करण युग्मक निर्माण के दौरान एलील्स को
अलग करना सुनिश्चित करता है, और स्वतंत्र वर्गीकरण विभिन्न गुणसूत्रों पर लक्षणों की विरासत की व्याख्या करता है।

· जीन, जिन्हें इकाई वर्ण के रूप में जाना जाता है, वैकल्पिक रूपों में मौजूद होते हैं जिन्हें एलील कहा जाता है। प्रभुत्व का नियम बताता है कि
हेटेरोजाइट् स में, एक एलील फे नोटाइप निर्धारित करता है।

· मेंडेलियन लक्षणों में रक्त प्रकार या कु छ वंश ानुगत विकार जैसी सरल आनुवंशि क विशेषताएं शामिल हैं, जो मेंडल द्वारा अपने अग्रणी मटर
पौधे प्रयोगों में खोजे गए वंश ानुक्रम के पूर्वानुमानित पैटर्न का उदाहरण हैं।

QUESTION
43.
डीएनए और आरएनए के संदर्भ में निम्नलिखित कथनों पर विचार करें:

1. आरएनए तीव्र गति से उत्परिवर्तित होता है, परिणामस्वरूप आरएनए जीनोम वाले और कम जीवन काल वाले वायरस तेजी से उत्परिवर्तित
और विकसित होते हैं।

2. आरएनए और डीएनए दोनों उत्परिवर्तन करने में सक्षम हैं तथा आरएनए और डीएनए दोनों आनुवंशि क सामग्री के रूप में कार्य कर सकते
हैं।

IASbaba
Score:
Web: http://ilp.iasbaba.com/
44.00 /
Email: ilp@iasbaba.com
Page 139 200
UPSC 2024 -
Exam Title : Environment,
Sc...
rupeshkr.
Email :
09ee80@gmail.com
Contact :

3. आनुवंशि क जानकारी के भंडारण के लिए आरएनए को प्राथमिकता दी जाती है तथा आनुवंशि क जानकारी के प्रसारण के लिए डीएनए
को प्राथमिकता दी जाती है।

उपरोक्त में से कितने कथन सही हैं?

a) के वल एक
b) के वल दो
c) सभी तीन
d) कोई नहीं
Correct Answer: B
Your Answer:
Explanation

Q. 43) Solution (b)

स्पष्टीकरण:

· डीएनए और आरएनए दोनों उत्परिवर्तन करने में सक्षम हैं। दरअसल, आरएनए अस्थिर होने के कारण तेज गति से उत्परिवर्तन करता है।
नतीजतन, आरएनए जीनोम वाले और कम जीवन काल वाले वायरस तेजी से उत्परिवर्तित और विकसित होते हैं। (अतः कथन 1 सही है)

· आरएनए प्रोटीन के संश्लेषण के लिए सीधे कोड कर सकता है, इसलिए लक्षणों को आसानी से व्यक्त कर सकता है। हालाँकि, डीएनए
प्रोटीन के संश्लेषण के लिए आरएनए पर निर्भर है।

· आरएनए और डीएनए दोनों आनुवंशि क सामग्री के रूप में कार्य कर सकते हैं जैसा कि ऊपर बताया गया है कि दोनों उत्परिवर्तन करने में
सक्षम हैं। (अतः कथन 2 सही है)

· अधिक स्थिर होने के कारण डीएनए को आनुवंशि क जानकारी के भंडारण के लिए प्राथमिकता दी जाती है। आनुवंशि क
जानकारी के प्रसारण के लिए आरएनए बेहतर है। (इसलिए कथन 3 गलत है)

IMPORTANT POINTS/VALUE ADDITIONS:

· डीऑक्सीराइबोन्यूक्लिक एसिड (डीएनए) और राइबोन्यूक्लिक एसिड (आरएनए) जटिल आणविक संरचनाएं हैं जो कोशिकाओं और इस
प्रकार जीवों की सभी वंश ानुगत विशेषताओं को नियंत्रित करती हैं। डीएनए जीवन का मुख्य खाका है और सभी स्वतंत्र जीवों में आनुवंशि क
सामग्री का निर्माण करता है। आरएनए कोशिकाओं में संश्लेषित प्रोटीन की संरचना को कोड करने के लिए डीएनए का उपयोग करता है,
और यह कु छ वायरस की आनुवंशि क सामग्री भी है।

· दोनों चार न्यूक्लियोटाइड के संयोजन से बने हैं, जो नाइट्रोजन क्षार के साथ विशेष "बिल्डिंग ब्लॉक" अणु हैं। डीएनए न्यूक्लियोटाइड् स
एडेनिन, गुआनिन, साइटोसिन और थाइमिन की लंबी श्रृंखला से बना है । आरएनए में, थाइमिन को यूरैसिल द्वारा प्रतिस्थापित किया जाता है।
इन न्यूक्लियोटाइड् स का क्रम और पैटर्न आनुवंशि क कोड बनाते हैं।

· भौतिक रूप से, डीएनए एक डबल हेलिक्स के रूप में संरचित होता है, जिसमें डीएनए के दो स्ट्रैंड एक-दूसरे के चारों ओर घूमते हैं, जबकि
आरएनए एक एकल स्ट्रैंड के रूप में संरचित होता है।

QUESTION
44.
प्रतिरक्षा ( Immunity ) के संदर्भ में निम्नलिखित कथनों पर विचार करें :

1. वायरस से संक्रमित कोशिकाएं इंटरफे रॉन नामक प्रोटीन का स्राव करती हैं जो गैर-संक्रमित कोशिकाओं को आगे के वायरल संक्रमण से
एक प्रकार की अर्जित प्रतिरक्षा से बचाती हैं।

2. एंटीबॉडी मध्यस्थता- अर्जित प्रतिरक्षा प्रतिक्रिया में रोगजनकों के जवाब में बी-लिम्फोसाइटों द्वारा उत्पादित प्रोटीन शामिल होते हैं।

IASbaba
Score:
Web: http://ilp.iasbaba.com/
44.00 /
Email: ilp@iasbaba.com
Page 140 200
UPSC 2024 -
Exam Title : Environment,
Sc...
rupeshkr.
Email :
09ee80@gmail.com
Contact :

3. टी - लिम्फोसाइट् स मध्यस्थ , कोशिका - मध्यस्थ प्रतिरक्षा (cell-mediated immunity - CMI) ग्राफ्ट/ प्रत्यारोपण अस्वीकृ ति
(graft rejection) के लिए उत्तरदायी है।

उपरोक्त में से कौन सा कथन सही है?

a) 1 और 2
b) 2 और 3
c) 1 और 3
d) उपरोक्त सभी
Correct Answer: B
Your Answer:
Explanation

Q. 44) Solution (b)

स्पष्टीकरण:

· साइटोकिन बाधाएं ( Cytokine barriers ) : वायरस से संक्रमित कोशिकाएं इंटरफे रॉन नामक प्रोटीन का स्राव करती हैं जो गैर - सं
क्रमित कोशिकाओं को आगे के वायरल संक्रमण से बचाती हैं। यह गैर - विशिष्ट प्रकार की सुरक्षा है और यह जन्मजात प्रतिरक्षा के माध्यम से
बाधा है। ( इसलिए कथन 1 गलत है )

· अर्जित प्रतिरक्षा ( acquired immunity ) में - प्राथमिक और माध्यमिक प्रतिरक्षा प्रतिक्रियाएं हमारे रक्त में मौजूद दो विशेष प्रकार के
लिम्फोसाइटों , यानी बी - लिम्फोसाइट् स और टी - लिम्फोसाइट् स की मदद से की जाती हैं। बी - लिम्फोसाइट् स रोगजनकों के जवाब में उनसे
लड़ने के लिए हमारे रक्त में प्रोटीन की एक सेना का उत्पादन करते हैं। इन प्रोटीनों को एंटीबॉडी कहा जाता है। टी - कोशिकाएं स्वयं एंटीबॉ
डी का स्राव नहीं करती हैं बल्कि बी कोशिकाओं को उनके उत्पादन में मदद करती हैं। ( अतः कथन 2 सही है )

· दूसरे प्रकार को कोशिका - मध्यस्थ प्रतिरक्षा प्रतिक्रिया या कोशिका - मध्यस्थ प्रतिरक्षा ( सीएमआई ) कहा जाता है। टी - लिम्फोसाइट् स सी
एमआई में मध्यस्थता करते हैं। शरीर ' स्वयं ' और ' गैर - स्वयं ' में अंतर करने में सक्षम है और कोशिका - मध्यस्थ प्रतिरक्षा प्रतिक्रिया ग्राफ्ट
अस्वीकृ ति के लिए उत्तरदायी है। ( अतः कथन 3 सही है )

IMPORTANT POINTS/VALUE ADDITIONS:

· अक्सर, जब कु छ मानव अंग जैसे हृदय, आंख, यकृ त, गुर्दे संतोषजनक ढं ग से काम करने में विफल हो जाते हैं, तो रोगी को सामान्य जीवन
जीने में सक्षम बनाने के लिए प्रत्यारोपण ही एकमात्र उपाय है।

· किसी भी स्रोत - एक जानवर, अन्य प्राइमेट, या किसी भी इंसान से ग्राफ्ट नहीं बनाया जा सकता क्योंकि ग्राफ्ट को जल्द ही अस्वीकार कर
दिया जाएगा। किसी भी ग्राफ्ट/प्रत्यारोपण से पहले ऊतक मिलान, रक्त समूह मिलान आवश्यक है और इसके बाद भी रोगी को जीवन भर
इम्यूनोसप्रेसेन्ट लेना पड़ता है।

QUESTION
45.
' क्वांटम मैकेनिक्स ( Quantum Mechanics ) ' के बारे में निम्नलिखित में से कौन सा कथन गलत है :

a) क्वांटम उलझाव ( Quantum entanglement ) कणों को इस तरह से सहसंबद्ध करने की अनुमति दे ता है कि एक कण की स्थिति दूसरे क
को प्रभावित कर सकती है जब उनके बीच की दूरी सीमित हो।
b) हाइजेनबर्ग के अनिश्चितता सिद्धांत में कहा गया है कि किसी कण की सटीक स्थिति और गति को पूर्ण सटीकता के साथ जानना असंभव है।
c) क्वांटम सुपरपोज़िशन का तात्पर्य है कि कण एक ही समय में कई अवस्थाओं में मौजूद रह सकते हैं जब तक कि उन्हें दे खा या मापा न जाए।
d) क्वांटम टनलिंग ( Quantum tunneling ) एक ऐसी घटना है जहां कण उन बाधाओं से गुजर सकते हैं जिनके बारे में शास्त्रीय भौतिकी अभ
वाणी करती है।
Correct Answer: A
Your Answer:

IASbaba
Score:
Web: http://ilp.iasbaba.com/
44.00 /
Email: ilp@iasbaba.com
Page 141 200
UPSC 2024 -
Exam Title : Environment,
Sc...
rupeshkr.
Email :
09ee80@gmail.com
Contact :

Explanation

Q. 45) Solution (a)

स्पष्टीकरण:

· क्वांटम उलझाव कणों को इस तरह से सहसंबद्ध करने की अनुमति दे ता है कि एक कण की स्थिति दूसरे की स्थिति को प्रभावित कर सकती
है, भले ही उनके बीच की दूरी कु छ भी हो । (इसलिए कथन 1 गलत है)

· हाइजेनबर्ग का अनिश्चितता सिद्धांत क्वांटम यांत्रिकी में एक मौलिक अवधारणा है, जिसमें कहा गया है कि जितना अधिक सटीक रूप से
आप एक कण की स्थिति को जानते हैं, उतना ही कम आप इसकी गति को जान सकते हैं। यह माप उपकरणों के कारण कोई सीमा नहीं है
बल्कि क्वांटम प्रणालियों की प्रकृ ति में अंतर्निहित है।

· क्वांटम सुपरपोजिशन एक ऐसी अवधारणा है जहां एक कण माप किए जाने तक एक साथ कई अवस्थाओं में मौजूद रह सकता है, जिस
बिंदु पर यह संभावित अवस्थाओं में से एक में 'संक्षिप्त' हो जाता है। यह क्वांटम यांत्रिकी का एक मूलभूत पहलू है।

· क्वांटम टनलिंग क्वांटम यांत्रिकी में एक वास्तविक घटना है जहां कण ऊर्जा बाधाओं से गुजर सकते हैं जिन्हें शास्त्रीय भौतिकी अभेद्य
मानती है। यह क्वांटम अवस्थाओं की संभाव्य प्रकृ ति के कारण होता है, जिससे कणों को उन बाधाओं के माध्यम से 'सुरंग' बनाने की
अनुमति मिलती है।

IMPORTANT POINTS/VALUE ADDITIONS:

· क्वांटम यांत्रिकी भौतिकी में एक मौलिक सिद्धांत है जो सबसे छोटे पैमाने पर पदार्थ और ऊर्जा के व्यवहार का वर्णन करता है , जो
आमतौर पर परमाणुओं और उप - परमाणु कणों के क्रम पर होता है ।

· यह सैद्धांतिक भौतिकी की एक शाखा है जो क्वांटम स्तर पर कणों के व्यवहार को समझाने में अविश्वसनीय रूप से सफल साबित हुई है।

QUESTION
46.
कार्बन फाइबर और टाइटे नियम के गुणों के संदर्भ में निम्नलिखित कथनों पर विचार करें:

1. कार्बन फाइबर एक पॉलिमर है जो काफी मजबूत लेकिन स्टील से बहुत भारी माना जाता है।

2. कार्बन स्टील से पांच गुना अधिक मजबूत और दोगुना कठोर हो सकता है।

3. टाइटे नियम स्टील जितना मजबूत है लेकिन स्टील से हल्का है।

4. टाइटे नियम एल्युमीनियम से दोगुना मजबूत है।

उपरोक्त में से कितने कथन सही हैं?

a) के वल एक
b) के वल दो
c) के वल तीन
d) सभी चार
Correct Answer: C
Your Answer:
Explanation

Q. 46) Solution (c)

स्पष्टीकरण:

IASbaba
Score:
Web: http://ilp.iasbaba.com/
44.00 /
Email: ilp@iasbaba.com
Page 142 200
UPSC 2024 -
Exam Title : Environment,
Sc...
rupeshkr.
Email :
09ee80@gmail.com
Contact :

· कार्बन फाइबर एक ऐसा पॉलिमर है जो हल्का होने के बावजूद काफी मजबूत माना जाता है। (इसलिए कथन 1 गलत है)

· यह स्टील से पांच गुना अधिक मजबूत और दोगुना कठोर हो सकता है। (अतः कथन 2 सही है)

· टाइटे नियम स्टील जितना मजबूत है लेकिन लगभग 45 प्रतिशत हल्का है। (अतः कथन 3 सही है)

· संयुक्त राज्य भूवैज्ञानिक सर्वेक्षण के अनुसार , यह एल्युमीनियम से दोगुना मजबूत है लेकिन के वल 60 प्रतिशत भारी है। (अतः कथन 4
सही है)

IMPORTANT POINTS/VALUE ADDITIONS:

· ओशनगेट के अनुसार टाइटन सबमर्सिबल के निर्माता ने "आधुनिक सामग्रियों का अभिनव उपयोग" किया है, जो "वजन में हल्का है और
किसी भी अन्य गहरी डाइविंग सबमर्सिबल की तुलना में अधिक लागत प्रभावी है। " कं पनी जिन सामग्रियों का जिक्र कर रही है वे कार्बन
फाइबर और टाइटे नियम हैं।

· जहाज को मालिकाना प्रौद्योगिकियों और कु छ ऑफ-द-शेल्फ घटकों के मिश्रण का उपयोग करके बनाया गया था, जिसके बारे में कं पनी
का दावा है कि इसे संचालित करना और क्षेत्र में भागों को बदलना आसान है।

QUESTION
47.
निम्नलिखित कथनों पर विचार करें:

कथन 1: हबल के कार्य ने बिग बैंग सिद्धांत के विकास में महत्वपूर्ण भूमिका निभाई।

कथन
2: बिग बैंग सिद्धांत ने सुझाव दिया कि ब्रह्मांड हमेश ा विस्तारित हो रहा है लेकिन पदार्थ का एक निरंतर औसत घनत्व बनाए रखता है।

उपरोक्त कथनों के संबंध में निम्नलिखित में से कौन सा सही है?

a) कथन-1 और कथन-2 दोनों सही हैं और कथन-2, कथन-1 की सही व्याख्या है।
b) कथन-1 और कथन-2 दोनों सही हैं और कथन-2, कथन-1 की सही व्याख्या नहीं है।
c) कथन-1 सही है लेकिन कथन-2 गलत है।
d) कथन-1 ग़लत है लेकिन कथन-2 सही है।
Correct Answer: C
Your Answer:
Explanation

Q. 47) Solution (c)

स्पष्टीकरण:

· एडविन हबल अपने अवलोकन कार्य के लिए प्रसिद्ध हैं जिसके कारण हबल का नियम प्रतिपादित हुआ, जिसमें कहा गया है कि
आकाशगंगाओं का वेग हमसे उनकी दूरी के समानुपाती होता है।

· दूर की आकाशगंगाओं से प्रकाश के लाल विस्थापन (रेड शिफ्ट) के साथ मिलकर इस अवलोकन ने एक विस्तारित ब्रह्मांड के लिए मजबूत
साक्ष्य प्रदान किए।

· हबल के काम ने बिग बैंग सिद्धांत के विकास में महत्वपूर्ण भूमिका निभाई, जो बताता है कि ब्रह्मांड बेहद गर्म और घने राज्य से शुरू हुआ
और तब से इसका विस्तार हो रहा है। (अतः कथन 1 सही है)

IASbaba
Score:
Web: http://ilp.iasbaba.com/
44.00 /
Email: ilp@iasbaba.com
Page 143 200
UPSC 2024 -
Exam Title : Environment,
Sc...
rupeshkr.
Email :
09ee80@gmail.com
Contact :

· ब्रह्माण्ड विज्ञान में स्थिर-अवस्था सिद्धांत, एक दृष्टिकोण है कि ब्रह्मांड हमेश ा विस्तारित हो रहा है लेकि न एक स्थिर औसत घनत्व
बनाए रखता है , जिसमें पदार्थ लगातार उसी दर पर नए तारों और आकाशगंगाओं का निर्माण कर रहा है जिससे पुराने तारे उनकी बढ़ती
दूरी के परिणामस्वरूप अप्राप्य हो जाते हैं । (इसलिए कथन 2 गलत है)

· एक स्थिर अवस्था वाले ब्रह्मांड का समय में कोई आरंभ या अंत नहीं होता है, और इसके भीतर किसी भी बिंदु से बड़े पैमाने पर दृश्य यानी
आकाशगंगाओं का औसत घनत्व और व्यवस्था - समान होती है। सभी संभावित युगों की आकाशगंगाएँ आपस में जुड़ी हुई हैं।

IMPORTANT POINTS/VALUE ADDITIONS:

· भारतीय वैज्ञानिकों ने हबल स्थिरांक का मान निर्धारित करने के लिए एक नई विधि प्रस्तावित की है।

· हबल का नियम कहता है कि आकाशगंगाएँ अपनी दूरी के अनुपातिक गति से पृथ्वी से दूर जा रही हैं, अर्थात आकाशगंगा पृथ्वी से जितनी
दूर होगी, उतनी ही तेजी से दूर चली जाएगी।

· हबल का नियम यह आधार बनाता है कि ब्रह्मांड का विस्तार हो रहा है।

· हबल स्थिरांक एक इकाई है जो बताती है कि ब्रह्मांड अंतरिक्ष में एक विशेष बिंदु से विभिन्न दूरी पर कितनी तेजी से फैल रहा है। इसे
(किमी/सेकेंड)/मेगापारसेक में मापा जाता है। मेगापारसेक बड़ी दूरी की इकाई है, जो 3.26 मिलियन प्रकाश वर्ष के बराबर है।

QUESTION
48.
निम्नलिखित में से कौन सा रेडियोधर्मी तत्व आमतौर पर परमाणु पतन ( nuclear fallout ) से जुड़ा होता है और परमाणु हथियारों के प
रीक्षण और दुर्घटनाओं में इसकी उपस्थिति के कारण स्वास्थ्य जोखिम पैदा करता है ?

a) ट्रिटियम
b) सीज़ियम-137
c) स्ट्रोंटियम-90
d) उपरोक्त सभी
Correct Answer: D
Your Answer:
Explanation

Q. 48) Solution (d)

स्पष्टीकरण:

· फुकु शिमा परमाणु दुर्घटना के बारह साल बाद, जापान बिजली संयंत्र के ठं डे पानी को समुद्र में छोड़ रहा है। जापानी अधिकारियों का कहना
है कि यह सुरक्षित है, लेकिन विशेषज्ञ बंटे हुए हैं।

· ट्रिटियम: ट्रिटियम हाइड्रोजन का एक रेडियोधर्मी आइसोटोप है। इसका उपयोग आमतौर पर परमाणु हथियारों में किया जाता है और इसे
परमाणु ऊर्जा उत्पादन के दौरान पर्यावरण में भी छोड़ा जा सकता है। ट्रिटियम निम्न -ऊर्जा बीटा विकिरण उत्सर्जित करता है और इसे पानी
के अणुओं में शामिल किया जा सकता है, अगर यह खाद्य श्रृंखला में प्रवेश करता है तो संभावित रूप से स्वास्थ्य संबंधी चिंताएँ पैदा हो सकती
हैं।

· सीज़ियम-137 (विकल्प बी): सीज़ियम-137 एक रेडियोधर्मी आइसोटोप है जो परमाणु विखंडन के दौरान उत्पन्न होता है। इस की अर्ध-
आयु अपेक्षाकृ त लं बी हो ती है, जिससे यह पर्यावरण में बना रहता है। सीज़ियम-137 मिट्टी और वनस्पति को दूषित कर सकता है, जिससे
सेवन करने पर मानव स्वास्थ्य को खतरा हो सकता है। परमाणु परीक्षण और दुर्घटनाओं से रेडियोधर्मी गिरावट में इसका महत्वपूर्ण योगदान
रहा है।

IASbaba
Score:
Web: http://ilp.iasbaba.com/
44.00 /
Email: ilp@iasbaba.com
Page 144 200
UPSC 2024 -
Exam Title : Environment,
Sc...
rupeshkr.
Email :
09ee80@gmail.com
Contact :

· स्ट्रोंटियम-90 (विकल्प सी): स्ट्रोंटियम-90 स्ट्रोंटियम का एक रेडियोधर्मी आइसोटोप है। यह परमाणु विखंडन के दौरान उत्पन्न होता है और
इस की अर्ध-आयु अपेक्षाकृ त लं बी हो ती है। स्ट्रोंटियम-90 रासायनिक रूप से कै ल्शियम के समान व्यवहार करता है और हड्डियों में जमा हो
सकता है, जिससे स्वास्थ्य जोखिम पैदा हो सकता है, खासकर बच्चों में हड्डियों के विकास के लिए। यह परमाणु पतन का भी एक घटक है।

IMPORTANT POINTS/VALUE ADDITIONS:

· पर्यावरण समूह ग्रीनपीस ने जापानी सरकार पर ट्रिटियम पर ध्यान कें द्रित करके पानी में विकिरण के स्तर से ध्यान भटकाने का आरोप
लगाया है - फ़िल्टर किए जाने के बाद भी अन्य रेडियोन्यूक्लाइड पानी में बने रहेंगे।

· ग्रीनपीस के अनुसार, दूषित पानी में कई रेडियोन्यूक्लाइड होते हैं, जिनके बारे में हम जानते हैं कि वे पर्यावरण और मानव स्वास्थ्य पर प्रभाव
डालते हैं - जिसमें स्ट्रोंटियम -90 भी शामिल है।

QUESTION
49.
' सुपरकं डक्टर्स/ अतिचालक ( Superconductors ) ' के बारे में निम्नलिखित कथनों पर विचार करें :

1. जब विभिन्न ठोस पदार्थों को एक निश्चित तापमान से नीचे ठं डा किया जाता है जिसे संक्रमण तापमान कहा जाता है तो उनमें विद्युत
प्रतिरोध पूरी तरह से गायब हो जाता है।

2. संक्रमण तापमान के नीचे सामग्री के आंतरिक भाग से चुंबकीय क्षेत्र का निष्कासन।

3. कोई भी ज्ञात सामग्री रोजमर्रा की परिस्थितियों में सुपरकं डक्टर्स / अतिचालक नहीं है।

उपरोक्त में से कितने कथन सही हैं?

a) के वल एक
b) के वल दो
c) सभी तीन
d) कोई नहीं
Correct Answer: C
Your Answer:
Explanation

Q. 49) Solution (c)

स्पष्टीकरण:

· अतिचालकता ( Superconductivity ) , विभिन्न ठोस पदार्थों में विद्युत प्रतिरोध का पूरी तरह से गायब हो जाना जब उन्हें एक विशिष्ट
तापमान से नीचे ठं डा किया जाता है। यह तापमान , जिसे संक्रमण तापमान कहा जाता है , विभिन्न सामग्रियों के लिए अलग - अलग होता है
लेकिन आम तौर पर 20 K (-253 °C) से नीचे होता है। ( अतः कथन 1 सही है )

· मीस्नर प्रभाव (Meissner effect) , एक ऐसी सामग्री के आंतरिक भाग से चुंबकीय क्षेत्र का निष्कासन जो सुपरकं डक्टर बनने की प्र
क्रिया में है , अर्थात , एक निश्चित तापमान से नीचे ठं डा होने पर विद्युत धाराओं के प्रवाह के प्रति अपना प्रतिरोध खो दे ता है , जिसे आमतौर
पर संक्रमण तापमान ( परम शून्य के करीब ) कहा जाता है। ( अतः कथन 2 सही है )

· 1911 में पहली बार खोजे गए सुपरकं डक्टर्स लगभग जादुई लग सकते हैं। हालाँकि, कोई भी ज्ञात सामग्री रोजमर्रा की परिस्थितियों में
अतिचालक नहीं है। (अतः कथन 3 सही है)

IMPORTANT POINTS/VALUE ADDITIONS:

· अधिकांश को अति-शीतित तापमान की आवश्यकता होती है, और उच्च तापमान पर कार्य करने वाले सुपरकं डक्टर्स की ओर हाल की
प्रगति के लिए क्रशिंग दाब की आवश्यकता होती है।

IASbaba
Score:
Web: http://ilp.iasbaba.com/
44.00 /
Email: ilp@iasbaba.com
Page 145 200
UPSC 2024 -
Exam Title : Environment,
Sc...
rupeshkr.
Email :
09ee80@gmail.com
Contact :

· एक सुपरकं डक्टर जो रोजमर्रा के तापमान और दबाव पर काम करता है , उसका उपयोग एमआरआई स्कै नर , नए इलेक्ट्रॉनिक उपकरणों
और उड़ने वाली ट्रेनों ( levitating trains ) में किया जा सकता है।

QUESTION
50.
मोशन कै प्चर तकनीक ( Mo-cap) और कं प्यूटर जनित इमेजरी ( CGI) के संदर्भ में निम्नलिखित कथनों पर विचार करें:

1. Mo-cap चेहरे के भाव और शरीर की गतिविधियों जैसी विशिष्ट गतिविधियों को ट्रैक करता है, ताकि स्क्रीन पर कं प्यूटर-जनरेटेड 3डी
कै रेक्टर को सुविधाएं प्रदान की जा सकें ।

2. CGI उन छवियों को बनाने और हेरफे र करने की प्रक्रिया को संदर्भित करता है जो भौतिक वातावरण में मौजूद नहीं हैं।

उपरोक्त में से कौन सा कथन सही है?

a) के वल 1
b) के वल 2
c) दोनों
d) कोई भी नहीं
Correct Answer: C
Your Answer:
Explanation

Q. 50) Solution (c)

स्पष्टीकरण:

· मोशन कै प्चर या mo-cap उस तकनीक का नाम है जो लोगों या वस्तुओं की गतिविधियों को रिकॉर्ड करती है। इस प्रकार कै प्चर किया
गया डेटा एक कं प्यूटर प्रोग्राम में स्थानांतरित किया जाएगा और बदले में इसे सीजीआई कै रेक्टर में अनुवादित किया जा सकता है।

· मो-कै प, चेहरे के भाव और शरीर की गतिविधियों जैसी विशिष्ट गतिविधियों को ट्रैक करता है, ताकि स्क्रीन पर कं प्यूटर-जनरेटेड 3डी
कै रेक्टर को सुविधाएं प्रदान की जा सकें । (अतः कथन 1 सही है)

· बुनियादी शब्दों में, कं प्यूटर जनित इमेजरी, या सीजीआई, फिल्म और टे लीविजन में चित्र या चरित्र बनाने के लिए कं प्यूटर के उपयोग को
संदर्भित करता है।

· इसे उन छवियों को बनाने और हेरफे र करने की प्रक्रिया के रूप में भी परिभाषित किया जा सकता है जो फिल्म या वीडियो पर कै प्चर
करने के लिए भौतिक वातावरण में मौजूद नहीं हैं। ये छवियां स्थिर या गतिशील हो सकती हैं और 2डी और 3डी दोनों फिल्मों में उपयोग की
जाती हैं। (अतः कथन 2 सही है)

IMPORTANT POINTS/VALUE ADDITIONS:

· सीजीआई का उपयोग विशिष्ट तत्वों, वस्तुओं या यहां तक कि संपूर्ण पृष्ठभूमि वातावरण बनाने के लिए अवधि/महाकाव्य या विज्ञान-फाई
शैलियों सहित फिल्मों की एक विस्तृत श्रृंखला में किया जाता है।

· यहां तक कि फिल्म के निर्माता भी जटिल विवरणों को शामिल करके दृश्यों या फिल्म की समग्र कोरियोग्राफी को बढ़ाने के लिए सीजीआई
तकनीक को अपनाने पर काम कर रहे हैं।

QUESTION
51.

IASbaba
Score:
Web: http://ilp.iasbaba.com/
44.00 /
Email: ilp@iasbaba.com
Page 146 200
UPSC 2024 -
Exam Title : Environment,
Sc...
rupeshkr.
Email :
09ee80@gmail.com
Contact :

“ ताकाकिया ( Takakia ) एक जीनस है जिसकी के वल दो प्रजातियाँ हैं। और ये दोनों के वल एक ही स्थान - तिब्बती पठार पर एक साथ
पाए जाते हैं। वैज्ञानिकों के एक अंतरराष्ट्रीय समूह ने पता लगाया है कि इसने ठं ढ , जीवन - घातक यूवी विकिरण और अन्य खतरनाक स्थि
तियों से बचने की क्षमता कै से विकसित की है।

यह निम्नलिखित में से किस समूह से संबंधित है?

a) शैवाल
b) काई ( Moss )
c) लिवरवॉर्ट
d) हॉर्नवॉर्ट
Correct Answer: B
Your Answer:
Explanation

Q. 51) Solution (b)

स्पष्टीकरण:

· ताकाकिया एक काई ( moss ) है , लिवरवॉर्ट , शैवाल या हॉर्नवॉर्ट नहीं। यह ब्रायोफाइटा प्रभाग से संबंधित है , जिसमें काई भी शामिल है
। जबकि ताकाकिया को एक आदिम मॉस माना जाता है और लिवरवॉर्ट्स के साथ कु छ विशेषताएं साझा करता है , इसे वर्गीकरण की दृष्टि से
मॉस समूह के भीतर वर्गीकृ त किया गया है। ( इसलिए विकल्प बी सही है )

· मॉसेस, लिवरवॉर्ट्स और हॉर्नवॉर्ट्स सभी गैर-संवहनी पौधों के अलग-अलग समूह हैं, प्रत्येक में अद्वितीय विशेषताएं और विकासवादी
इतिहास हैं।

IMPORTANT POINTS/VALUE ADDITIONS:

· पौधों को घटते तापमान और बढ़ते विकिरण के अनुकूल ढलने में लाखों साल लग गए लेकिन अब, जलवायु परिवर्तन के कारण इसका
निवास स्थान अब कु छ ही दशकों में बदल रहा है।

· ऐसा प्रतीत होता है कि ये कठोर प्रजातियाँ दूसरों की तुलना में तापमान वृद्धि का अच्छी तरह से सामना नहीं कर पाती हैं। अध्ययन अवधि
के दौरान उनकी आबादी काफी कम हो गई जबकि अन्य पौधों को वार्मिंग से लाभ हुआ। शोधकर्ताओं का मानना है कि यह प्रवृत्ति जारी
रहेगी।

· एक प्रजाति जिसने डायनासोरों को आते और जाते दे खा, वह मनुष्यों के आने से बच नहीं सकी ।

QUESTION
52.
निम्नलिखित में से कौन " ग्रीनवॉशिंग ( Greenwashing ) " का सबसे अच्छा वर्णन करता है ?

a) विनिर्माण में पर्यावरण-अनुकूल प्रथाओं को लागू करना


b) उत्पाद की पर्यावरण संवेदनशीलता का भ्रामक चित्रण
c) जलवायु परिवर्तन से निपटने के लिए हरित पहल की स्थापना करना
d) कॉर्पोरेट क्षेत्र में सतत नीतियों की वकालत करना
Correct Answer: B
Your Answer:
Explanation

Q. 52) Solution (b)

स्पष्टीकरण:

IASbaba
Score:
Web: http://ilp.iasbaba.com/
44.00 /
Email: ilp@iasbaba.com
Page 147 200
UPSC 2024 -
Exam Title : Environment,
Sc...
rupeshkr.
Email :
09ee80@gmail.com
Contact :

· ग्रीनवॉशिंग से तात्पर्य किसी कं पनी, उत्पाद या सेवा को पर्यावरण के अनुकूल या सतत के रूप में पेश करने की भ्रामक प्रथा से है, जबकि
वास्तव में ऐसा नहीं हुआ है।

· यह भ्रामक विपणन रणनीति यह धारणा दे ने के लिए डिज़ाइन की गई है कि कं पनी अक्सर विशिष्ट पर्यावरण-अनुकूल सुविधाओं या पहलों
को उजागर करके , पर्यावरण के प्रति जिम्मेदार होने के लिए महत्वपूर्ण प्रयास कर रही है।

· हालाँकि, कई मामलों में, ये दावे अतिरंजित हो सकते हैं या इनमें कोई सार नहीं हो सकता है, जिससे उपभोक्ताओं को यह विश्वास हो जाता
है कि वे पर्यावरण के प्रति जागरूक व्यवसायों का समर्थन कर रहे हैं, जबकि ऐसा नहीं हो सकता है।

· ग्रीनवॉशिंग स्थिरता की दिशा में वास्तविक प्रयासों को कमजोर कर सकती है और उपभोक्ताओं के लिए सूचित और जिम्मेदार विकल्प
चुनना चुनौतीपूर्ण बना सकती है।

IMPORTANT POINTS/VALUE ADDITIONS:

· एक प्रथा के रूप में ग्रीनवॉशिंग इतनी प्रचलित है कि जलवायु लक्ष्यों के पूरी तरह से कमजोर होने पर चिंताएं पैदा हो रही हैं, और इसे इतना
गंभीर माना गया कि संयुक्त राष्ट्र महासचिव ने इस पर ध्यान दे ने के लिए एक विशेषज्ञ समूह का गठन किया।

· हाल ही में, यूनाइटे ड किंगडम के विज्ञापन नियामक ने हवाई यात्रा के पर्यावरणीय प्रभाव के बारे में उपभोक्ताओं को कथित रूप से गुमराह
करने के लिए एयर फ्रांस, लुफ्थांसा और एतिहाद के विज्ञापनों पर प्रतिबंध लगा दिया।

QUESTION
53.
' ग्रीन क्रेडिट सिस्टम ( Green Credit System ) ' के संदर्भ में निम्नलिखित कथनों पर विचार करें :

1. बाज़ार-आधारित तंत्र के वल निजी क्षेत्र की कं पनियों द्वारा स्वैच्छिक पर्यावरणीय कार्यों को प्रोत्साहित करने के लिए डिज़ाइन किया गया
है।

2. अपनी प्रारंभिक अवस्था में यह दो प्रमुख गतिविधियों जल संरक्षण और वनीकरण पर कें द्रित है।

3. प्रशासक द्वारा जारी ग्रीन क्रेडिट प्रमाणपत्र स्टॉक एक्सचेंजों पर व्यापार योग्य होगा।

उपरोक्त में से कितने कथन सही हैं?

a) के वल एक
b) के वल दो
c) सभी तीन
d) कोई नहीं
Correct Answer: A
Your Answer:
Explanation

Q. 53) Solution (a)

स्पष्टीकरण:

· ग्रीन क्रेडिट प्रोग्राम (जीसीपी) एक अभिनव बाजार-आधारित तंत्र है जिसे व्यक्तियों, समुदायों, निजी क्षेत्र के उद्योगों और कं पनियों जैसे
विभिन्न हितधारकों द्वारा विभिन्न क्षेत्रों में स्वैच्छिक पर्यावरणीय कार्यों को प्रोत्साहित करने के लिए डिज़ाइन किया गया है । ( इसलिए
कथन 1 ग़लत है)

· अपने प्रारंभिक चरण में, जीसीपी दो प्रमुख गतिविधियों पर ध्यान कें द्रित करता है: जल संरक्षण और वनीकरण। ( इसलिए कथन 2 सही
है)

IASbaba
Score:
Web: http://ilp.iasbaba.com/
44.00 /
Email: ilp@iasbaba.com
Page 148 200
UPSC 2024 -
Exam Title : Environment,
Sc...
rupeshkr.
Email :
09ee80@gmail.com
Contact :

· एक बार सत्यापन पूरा हो जाने पर, प्रशासक एक ग्रीन क्रेडिट प्रमाणपत्र प्रदान करेगा जिसका व्यापार ग्रीन क्रेडिट प्लेटफॉर्म पर किया जा
सके गा, फिलहाल स्टॉक एक्सचेंजों पर नहीं। ( इसलिए कथन 3 गलत है)

IMPORTANT POINTS/VALUE ADDITIONS:

· जीसीपी का शासन ढांचा एक अंतर-मंत्रालयी संचालन समिति द्वारा समर्थित है और भारतीय वानिकी अनुसंधान और शिक्षा परिषद
(आईसीएफआरई) जीसीपी प्रशासक के रूप में कार्य करता है, जो कार्यक्रम कार्यान्वयन, प्रबंधन, निगरानी और संचालन के लिए जिम्मेदार
है।

· ग्रीन क्रेडिट दे ने के लिए प्रारूप पद्धति विकसित की गई है और हितधारक परामर्श के लिए इसे अधिसूचित किया जाएगा। ये पद्धतियाँ
प्रत्येक गतिविधि/प्रक्रिया के लिए मानक निर्धारित करती हैं, ताकि सभी क्षेत्रों में पर्यावरणीय प्रभाव और प्रतिस्थापना सुनिश्चित की जा सके ।

· एक उपयोगकर्ता-अनुकूल डिजिटल प्लेटफ़ॉर्म परियोजनाओं के पंजीकरण, उसके सत्यापन और ग्रीन क्रेडिट जारी करने की प्रक्रियाओं को
सुव्यवस्थित करेगा। विशेषज्ञों के साथ आईसीएफआरई द्वारा विकसित किया जा रहा ग्रीन क्रेडिट रजिस्ट्री और ट्रेडिंग प्लेटफॉर्म, पंजीकरण
और उसके बाद ग्रीन क्रेडिट की खरीद और बिक्री की सुविधा प्रदान करेगा।

· ग्रीन क्रेडिट प्राप्त करने के लिए, व्यक्तियों और संस्थाओं को कें द्र सरकार के समर्पित ऐप/वेबसाइट के माध्यम से अपनी गतिविधियों को
पंजीकृ त करना होगा। प्रशासक छोटी परियोजनाओं के लिए स्व-सत्यापन के साथ, एक निर्दिष्ट एजेंसी के माध्यम से गतिविधि का सत्यापन
करेगा।

QUESTION
54.
Q. 54 ) ' इकोमार्क योजना ( Ecomark Scheme ) ' के बारे में निम्नलिखित कथनों पर विचार करें :

1. यह घरेलू और उपभोक्ता उत्पादों के लिए मान्यता और लेबलिंग प्रदान करता है।

2. यह वि निर्माताओं को पर्यावरण के अनुकूल उत्पादन की ओर बढ़ने के लिए प्रेरित करेगा।

3. कें द्रीय प्रदूषण नियंत्रण बोर्ड भारतीय मानक ब्यूरो (बीआईएस) के साथ साझेदारी में इस योजना का संचालन करता है।

उपरोक्त में से कितने कथन सही हैं?

a) के वल एक
b) के वल दो
c) सभी तीन
d) कोई नहीं
Correct Answer: C
Your Answer:
Explanation

Q. 54) Solution (c)

स्पष्टीकरण:

· इकोमार्क योजना घरेलू और उपभोक्ता उत्पादों के लिए मान्यता और लेबलिंग प्रदान करती है जो भारतीय मानदं डों के अनुसार गुणवत्ता
मानकों को बनाए रखते हुए विशिष्ट पर्यावरणीय मानदं डों को पूरा करते हैं। (अतः कथन 1 सही है)

· इकोमार्क योजना के तहत मान्यता प्राप्त उत्पाद न्यूनतम पर्यावरणीय प्रभाव सुनिश्चित करते हुए विशिष्ट पर्यावरणीय मानदं डों का पालन
करेंगे।
यह पर्यावरणीय मुद्द ों के बारे में उपभोक्ताओं में जागरूकता पैदा करेगा और पर्यावरण के प्रति जागरूक विकल्पों को प्रोत्साहित करेगा।

IASbaba
Score:
Web: http://ilp.iasbaba.com/
44.00 /
Email: ilp@iasbaba.com
Page 149 200
UPSC 2024 -
Exam Title : Environment,
Sc...
rupeshkr.
Email :
09ee80@gmail.com
Contact :

· यह निर्माताओं को पर्यावरण के अनुकूल उत्पादन की ओर बढ़ने के लिए भी प्रेरित करेगा। यह योजना सटीक लेबलिंग सुनिश्चित करने और
उत्पादों के बारे में भ्रामक जानकारी को रोकने का प्रयास करती है। (अतः कथन 2 सही है)

· कें द्रीय प्रदूषण नियंत्रण बोर्ड भारतीय मानक ब्यूरो (बीआईएस) के साथ साझेदारी में इकोमार्क योजना का संचालन करता है, जो मानकों
और प्रमाणन के लिए राष्ट्रीय निकाय है। (अतः कथन 3 सही है)

IMPORTANT POINTS/VALUE ADDITIONS:

· यह सतत जीवन, पर्यावरण संरक्षण को बढ़ावा दे ने और व्यक्तिगत और सामूहिक पसंद के माध्यम से भारत में पर्यावरण-अनुकूल प्रथाओं को
मूर्त रूप दे ने में महत्वपूर्ण कदम है।

· यह वैश्विक सतत लक्ष्यों के अनुरूप है और पर्यावरण के संरक्षण और संरक्षण के लिए सरकार की प्रतिबद्धता को दर्शाता है।

QUESTION
55.
हाल ही में स्थानीय स्तर पर ' भीमथडी ( Bhimthadi ) ' घोड़े के नाम से जानी जाने वाली स्वदे श ी नस्ल को अब आधिकारिक तौर पर भा
रत की एक स्वतंत्र नस्ल के रूप में मान्यता दी गई है। इस संदर्भ में निम्नलिखित कथनों पर विचार करें :

1. यह भारत की स्वतंत्र नस्ल के रूप में मान्यता पाने वाली पहली स्वदे श ी घोड़ा नस्ल है।

2. यह भीमा नदी के किनारे दक्कन क्षेत्र में प्रमुख है।

3. बीकानेर में राष्ट्रीय अश्व अनुसंधान कें द्र एक नस्ल के लिए योग्य समरूप आबादी की पहचान और लक्षण वर्णन के लिए उत्तरदायी है।

उपरोक्त में से कितने कथन सही हैं?

a) के वल एक
b) के वल दो
c) सभी तीन
d) कोई नहीं
Correct Answer: A
Your Answer:
Explanation

Q. 55) Solution (a)

स्पष्टीकरण:

· राष्ट्रीय पशु आनुवंशि क संसाधन ब्यूरो ने पहले ही घोड़ों की सात स्वतंत्र भारतीय नस्लों को मान्यता दे दी है जो भूटिया, काठियावाड़ी ,
मणिपुरी, मारवाड़ी, स्पीति , ज़ांस्करी , कच्छी-सिंध हैं। और भीमथडी आठवां होगा । (इसलिए कथन 1 गलत है)

· दक्कनी घोड़े का नाम भीमा नदी के नाम पर रखा गया है , ' भीमथडी ' ' भीम ' को ' थडी ' से जोड़ती है , जिसका अर्थ नदी का किनारा
है। यह नस्ल मुख्य रूप से दक्कन के पठार पर स्थित है , जो पुणे , सतारा , सोलापुर , अहमदनगर , सांगली और कोल्हापुर जिलों के क्षेत्रों में
फैली हुई है। (अतः कथन 2 सही है)

· राष्ट्रीय पशु आनुवंशि क संसाधन ब्यूरो (एनबीएजीआर) ने दे श में 100 से अधिक नई नस्लों और समरूप आबादी को मान्यता दी है।
हालाँकि, अभी भी मूल AnGR की कई अनोखी आबादी हैं जिनकी पहचान करने की आवश्यकता है। जीरो नॉन-डिस्क्रिप्ट एएनजीआर
मिशन में दे श में लगभग 100 नस्लों की पहचान की परिकल्पना की गई है। (इसलिए कथन 3 गलत है)

IMPORTANT POINTS/VALUE ADDITIONS:

IASbaba
Score:
Web: http://ilp.iasbaba.com/
44.00 /
Email: ilp@iasbaba.com
Page 150 200
UPSC 2024 -
Exam Title : Environment,
Sc...
rupeshkr.
Email :
09ee80@gmail.com
Contact :

· कु ल 3,40,000 स्वदे श ी घोड़ों और लगभग 10,000 विदे श ी घोड़ों में से, विदे श ी घोड़े दौड़ या एनसीसी में भागीदारी के माध्यम से अधिक
ध्यान आकर्षित करते हैं। दुर्भाग्य से, हमारी स्वदे श ी नस्लों, जो विलुप्त होने के कगार पर हैं, की सुरक्षा के लिए कोई महत्वपूर्ण उपाय नहीं किए
जा रहे हैं।

· भीमथडी घोड़ों का ऐतिहासिक महत्व था क्योंकि 17 वीं और 18 वीं शताब्दी की मराठा सेना इन्हीं घोड़ों से बनी थी।

QUESTION
56.
हाल ही में खोजी गई जीनस बैटिलिप्स की प्रजाति का नाम डॉ . एपीजे अब्दुल कलाम के नाम पर " बैटिलिप्स कलामी " रखा गया है। यह प्र
जाति निम्नलिखित में से किस जीव से संबंधित है ?

a) स्प्रिंगटे ल ( Springtail )
b) निम्फ (Nymph)
c) गेरिडे ( Gerridae )
d) टार्डीग्रेड ( Tardigrade )
Correct Answer: D
Your Answer:
Explanation

Q. 56) Solution (d)

स्पष्टीकरण:

· कोचीन यूनिवर्सिटी ऑफ साइंस एंड टे क्नोलॉजी (कु सैट) के शोधकर्ताओं ने समुद्री टार्डिग्रेड की एक नई प्रजाति की पहचान की है, जिसका
नाम उन्होंने दिवंगत पूर्व राष्ट्रपति और वैज्ञानिक एपीजे अब्दुल कलाम के नाम पर रखा है। (इसलिए विकल्प d सही है)

· टार्डिग्रेड्स बेहद छोटे जीव हैं जिनका आकार माइक्रोमीटर में मापा जाता है । वे अपनी असाधारण लचीलापन और जीवित रहने की प्रवृत्ति
के लिए जाने जाते हैं। सूक्ष्मदर्शी का उपयोग करके अध्ययन करने पर , पानी में रहने वाले इन जीवों को असंभावित उपनाम ' जल भालू ( w
ater bears ) ' के नाम से भी जाना जाता है।

IMPORTANT POINTS/VALUE ADDITIONS:

· यह पृथ्वी के जीवमंडल के विभिन्न क्षेत्रों-पर्वत चोटियों, गहरे समुद्र, उष्णकटिबंधीय वर्षावनों और अंटार्क टिक में पाया गया है।

· टार्डीग्रेड ज्ञात सबसे लचीले जानवरों में से हैं, जिनकी व्यक्तिगत प्रजातियाँ अत्यधिक तापमान, अत्यधिक दबाव (उच्च और निम्न दोनों),
वायु की कमी, विकिरण, निर्जलीकरण और भुखमरी जैसी चरम स्थितियों में जीवित रहने में सक्षम हैं - जो अन्य ज्ञात प्रजातियों को जल्दी से
मार डालेगी।

QUESTION
57.
"आक्रामक विदे श ी प्रजातियों और उनके नियंत्रण पर मूल्यांकन रिपोर्ट'' के संदर्भ में निम्नलिखित कथनों पर विचार करें:

1. इसे आईपीसीसी-जलवायु परिवर्तन पर अंतर सरकारी पैनल द्वारा जारी किया गया है।

2. सभी विदे श ी प्रजातियाँ जैव विविधता पर नकारात्मक प्रभाव डालकर स्थापित और प्रसारित नहीं हैं।

3. रिपोर्ट के अनुसार विदे श ी पौधों, विदे श ी रोगाणुओं, विदे श ी कशेरुकियों में विदे श ी अकशेरुकी जीवों का उच्चतम प्रतिशत आक्रामक माना
जाता है।

4. कई आक्रामक विदे श ी प्रजातियों को जानबूझकर उनके कथित लाभों के लिए पेश किया गया है।

IASbaba
Score:
Web: http://ilp.iasbaba.com/
44.00 /
Email: ilp@iasbaba.com
Page 151 200
UPSC 2024 -
Exam Title : Environment,
Sc...
rupeshkr.
Email :
09ee80@gmail.com
Contact :

उपरोक्त में से कितने कथन सही हैं?

a) के वल एक
b) के वल दो
c) के वल तीन
d) सभी चार
Correct Answer: B
Your Answer:
Explanation

Q. 57) Solution (b)

स्पष्टीकरण:

· आक्रामक प्रजातियों पर अब तक किए गए सबसे व्यापक अध्ययन में, जैव विविधता और पारिस्थितिकी तंत्र सेवाओं पर अंतर सरकारी मंच
(आईपीबीईएस) ने अपने नए प्रकाशन - "आक्रामक विदे श ी प्रजातियों और उनके नियंत्रण पर मूल्यांकन रिपोर्ट'' में पाया है कि 37,000
विदे श ी प्रजातियाँ हैं, जिनमें पौधे और जीव भी शामिल हैं, जिन्हें कई मानवीय गतिविधियों द्वारा विश्व भर के क्षेत्रों और बायोम में लाया गया
है, जिनमें 3,500 से अधिक आक्रामक विदे श ी प्रजातियाँ शामिल हैं और आक्रामक विदे श ी प्रजातियों ने दर्ज किए गए 60% वैश्विक पौधों
और जीवों के विलुप्त होने में महत्वपूर्ण भूमिका निभाई है। (इसलिए कथन 1 गलत है)

· सभी विदे श ी प्रजातियाँ जैव विविधता, स्थानीय पारिस्थितिक तंत्र और प्रजातियों पर नकारात्मक प्रभाव डालते हुए स्थापित और फै लती नहीं
हैं, लेकिन एक महत्वपूर्ण अनुपात ऐसा करता है – अतः इसे आक्रामक विदे श ी प्रजातियों के रूप में जाना जाता है। (अतः
कथन 2 सही है)

· लगभग 6% विदे श ी पौधे; 22% विदे श ी अकशेरुकी; 14% विदे श ी कशेरुकी; और 11% विदे श ी रोगाणु आक्रामक माने जाते हैं, जो
प्रकृ ति और लोगों के लिए बड़ा खतरा पैदा करते हैं। (इसलिए कथन 3 गलत है)

· कई आक्रामक विदे श ी प्रजातियों को जानबूझकर उनके कथित लाभों के लिए, वानिकी, कृ षि, बागवानी, जलीय कृ षि, या पालतू जानवरों
के रूप में उनके नकारात्मक प्रभावों के विचार या ज्ञान के बिना पेश किया गया है। (अतः कथन 4 सही है)

IMPORTANT POINTS/VALUE ADDITIONS:

· आक्रामक विदे श ी प्रजातियाँ जैव विविधता हानि का एक प्रमुख चालक हैं और महत्वाकांक्षी कु नमिंग-मॉन्ट्रियल वैश्विक जैव विविधता
फ्रेमवर्क (KMGBF) के लक्ष्य 6 के तहत, विश्व को 2030 तक आक्रामक विदे श ी प्रजातियों के परिचय और स्थापना की दर को कम से
कम 50 प्रतिशत तक रोकना और कम करना है।

· आईपीबीईएस ने अपनी रिपोर्ट जारी की, 143 सदस्य दे श ों के प्रतिनिधियों ने रिपोर्ट को मंजूरी दे दी है।

· आईपीबीईएस एक स्वतंत्र अंतरसरकारी निकाय है जो जैव विविधता और पारिस्थितिकी तंत्र सेवाओं के लिए विज्ञान-नीति इंटरफे स को
मजबूत करने के लिए स्थापित किया गया है, जो आईपीसीसी के समान तरीके से काम करता है, जो संयुक्त राष्ट्र का जलवायु विज्ञान निकाय
है।

· यह अध्ययन , जो चार वर्षों की अवधि में हुआ है , 49 दे श ों के 86 प्रमुख विशेषज्ञों द्वारा किया गया है , जिसमें 13,000 से अधिक संदर्भ
शामिल हैं।

QUESTION
58.
निम्नलिखित युग्मों पर विचार करें:

प्रजाति का नाम IUCN स्थिति

1. हिमाल यी भेड़िया ( Himalayan Wolf ) निम्न चिंताजनक

IASbaba
Score:
Web: http://ilp.iasbaba.com/
44.00 /
Email: ilp@iasbaba.com
Page 152 200
UPSC 2024 -
Exam Title : Environment,
Sc...
rupeshkr.
Email :
09ee80@gmail.com
Contact :

2. ध्रुवीय भालू ( Polar bear ) लुप्तप्राय

3. लेसर फ्लोरिकन ( Lesser Florican ) लुप्तप्राय

उपरोक्त में से कितने सही हैं?

a) के वल एक
b) के वल दो
c) सभी तीन
d) कोई नहीं
Correct Answer: D
Your Answer:
Explanation

Q. 58) Solution (d)

स्पष्टीकरण:

· हिमालयन वुल्फ (कै निस ल्यूपस चांको), हिमालय में पाया जाने वाला एक प्रमुख ल्यूपिन शिकारी, जिसकी वर्गीकरण स्थिति एक पहेली थी,
का मूल्यांक न पहली बार अंतर्राष्ट्रीय प्रकृ ति संरक्षण संघ (आईयूसीएन) की रेड लिस्ट में किया गया है। इस जानवर को IUCN रेड
लिस्ट में ' सुभेद्य ( VU ) ' श्रेणी में रखा गया है । (इसलिए जोड़ी 1 ग़लत है)

· अपने पर्यावास स्थान - समुद्री बर्फ - के नुकसान के कारण अंतर्राष्ट्रीय प्रकृ ति संरक्षण संघ (आईयूसीएन) की लुप्तप्राय प्रजातियों की लाल
सूची में " सुभेद्य " के रूप में सूचीबद्ध किया गया है । (अतः जोड़ी 2 ग़लत है)

· लेसर फ्लोरिकन भारत में पाए जाने वाले बस्टर्ड परिवार के पक्षियों की चार प्रजातियों में से एक है। उन्हें अंतर्राष्ट्रीय प्रकृ ति संरक्षण संघ
(आईयूसीएन) द्वारा तैयार संकटग्रस्त प्रजातियों की लाल सूची में गंभीर रूप से लुप्तप्राय प्रजातियों के रूप में वर्गीकृ त किया गया है ।
(अतः जोड़ी 3 ग़लत है)

IMPORTANT POINTS/VALUE ADDITIONS:

· आईयूसीएन की रिपोर्ट में कहा गया है कि हिमालयी भेड़िये नेपाल और भारत की हिमालय श्रृंखला और तिब्बती पठार तक फैली एक उप-
जनसंख्या में थे।

· हाल ही में, अलास्का में बर्ड फ्लू के कारण ध्रुवीय भालू की मौत का पहला मामला दर्ज किया गया है, जिससे विशेषज्ञ पहले से ही तनावग्रस्त
मौजूदा प्रजातियों के बारे में चिंतित हैं।

· ध्रुवीय भालू की मृत्यु अत्यधिक रोगजनक एवियन इन्फ्लूएंजा (एचपीएआई) से हुई, जो घातक तनाव एच5एन1 से संक्रमित पक्षियों के शवों
को साफ करने से हुआ था। यदि संक्रमण फै लता है, तो यह आर्क टिक आबादी के लिए खतरनाक हो सकता है जो पहले से ही जलवायु
परिवर्तन के कारण समस्याओं का सामना कर रही है।

· लेसर फ्लोरिकन्स, बस्टर्ड परिवार के सबसे छोटे पक्षी, जो अपने सुंदर संभोग प्रदर्शन और मायावी मादाओं के लिए जाने जाते हैं, ये शक्ति
शाली उड़ने वाले पक्षी भी हैं जो गुजरात में अपने प्रजनन स्थलों से दक्षिणी भारत में अपने शीतकालीन प्रवास के मैदानों तक अपने वार्षिक
प्रवास के दौरान एक बार में खंभात की खाड़ी को पार करते हैं। गुजरात वन विभाग के एक अध्ययन में यह निष्कर्ष निकला है

QUESTION
59.
'स्टर्जन मछली ( Sturgeon fish ) ' के अवैध वन्यजीव व्यापार के बारे में निम्नलिखित कथनों पर विचार करें:

1. बुल्गारिया, रोमानिया, सर्बिया और यूक्रेन की मूल निवासी स्टर्जन मछली कै वियार ( caviar ) उत्पन्न करती है।

2. वोल्गा ( Volga ) बेलुगा, रूसी, स्टे लेट और स्टे रलेट स्टर्जन की कार्यात्मक आबादी वाला अंतिम नदी निकाय है।

IASbaba
Score:
Web: http://ilp.iasbaba.com/
44.00 /
Email: ilp@iasbaba.com
Page 153 200
UPSC 2024 -
Exam Title : Environment,
Sc...
rupeshkr.
Email :
09ee80@gmail.com
Contact :

3. वन्य जीवों और वनस्पतियों की लुप्तप्राय प्रजातियों में अंतर्राष्ट्रीय व्यापार पर कन्वेंश न (CITES) ने प्रजातियों को लुप्तप्राय के रूप में
सूचीबद्ध किया है।

4. IUCN ने अवैध व्यापार पर अंकुश लगाने के लिए सभी कै वियार उत्पादों के लिए एक अंतर्राष्ट्रीय लेबलिंग प्रणाली लागू की।

उपरोक्त में से कितने कथन सही हैं?

a) के वल एक
b) के वल दो
c) के वल तीन
d) सभी चार
Correct Answer: B
Your Answer: Unanswered
Explanation

Q. 59) Solution (b)

स्पष्टीकरण:

· शोधकर्ताओं ने पाया कि बुल्गारिया , रोमानिया , सर्बिया और यूक्रेन जैसे दे श ों की मूल मछली स्टर्जन , जो कै वियार पैदा करती है , अवैध
रूप से पकड़ी गई है। ( अतः कथन 1 सही है )

· अध्ययन के लेखकों के अनुसार, डेन्यूब बेलुगा (हुसो हुसो), रूसी (एसिपेंसर गुएलडेनस्टे डी ), स्टे लेट (एसिपेंसर स्टे लेटस ) और स्टे रलेट
(एसिपेंसर रूथेनस ) स्टर्जन की कार्यात्मक आबादी वाला अंतिम नदी निकाय है। (इसलिए कथन 2 गलत है)

· 1998 में वन्य जीवों और वनस्पतियों (CITES) की लुप्तप्राय प्रजातियों में अंतर्राष्ट्रीय व्यापार पर कन्वेंश न ने प्रजातियों को लुप्तप्राय के
रूप में सूचीबद्ध किया और डेन्यूब और काला सागर से स्टर्जन की मछली पकड़ने पर प्रतिबंध लगा दिया। (अतः कथन 3 सही है)

· 2000 में, CITES ने अवैध व्यापार पर अंकुश लगाने के लिए सभी कै वियार उत्पादों के लिए एक अंतर्राष्ट्रीय लेबलिंग प्रणाली भी लागू
की। (अतः कथन 4 ग़लत है)

IMPORTANT POINTS/VALUE ADDITIONS:

· हालाँकि, पर्यावास स्थान परिवर्तन के साथ-साथ मछली प्रजातियों के गहन दोहन ने उन्हें विलुप्त होने के कगार पर धके ल दिया है।

· कानूनी रूप से अंतरराष्ट्रीय स्तर पर पाए जाने योग्य कै वियार और मांस के वल पकड़े गए स्टर्जन से ही प्राप्त किया जा सकता है

· कै वियार और स्टर्जन व्यापार में तत्काल सुधार की आवश्यकता है ताकि यह सुनिश्चित किया जा सके कि स्टर्जन आबादी का भविष्य
सुनिश्चित हो सके ।

QUESTION
60.
निम्नलिखित में से किस दे श से भारत को 'प्रोजेक्ट चीता' के तहत भारत में पुनरुत्पादन के लिए चीता प्राप्त हुआ?

1. दक्षिण अफ्रीका

2. नामिबिया

3. ईरान

4. बोत्सवाना

उपरोक्त में से कितने सही हैं?

IASbaba
Score:
Web: http://ilp.iasbaba.com/
44.00 /
Email: ilp@iasbaba.com
Page 154 200
UPSC 2024 -
Exam Title : Environment,
Sc...
rupeshkr.
Email :
09ee80@gmail.com
Contact :

a) के वल एक
b) के वल दो
c) के वल तीन
d) सभी चार
Correct Answer: B
Your Answer:
Explanation

Q. 60) Solution (b)

स्पष्टीकरण:

· इस परियोजना को सितंबर, 2022 में दक्षिण अफ्रीका और नामीबिया से आने वाले 20 चीतों के साथ हरी झंडी दिखाई गई।

· भारत के कु नो राष्ट्रीय उद्यान, मध्य प्रदे श में चीता पुनरुत्पादन परियोजना है, जिसमें खराब स्वास्थ्य और प्रबंधन टीम की आलोचना के कारण
चीतों की मौत पर प्रतिक्रिया दे खी गई।

· ईरान ने अनुसंधान उद्दे श्यों के लिए भी नर और मादा चीता को भारत भेजने की अनुमति दे ने से इनकार कर दिया। ईरान एकमात्र दे श है
जहाँ एशियाई चीतों की आबादी है।

IMPORTANT POINTS/VALUE ADDITIONS:

· दक्षिण अफ्रीका एकमात्र ऐसा दे श हो सकता है जो भविष्य में चीता पुनरुत्पादन परियोजना के लिए भारत में चीते भेजेगा।

· भारत सावधानीपूर्वक चयन, स्वास्थ्य मानकों और भारतीय आवास में अनुकूलन क्षमता का उचित ध्यान रखते हुए दक्षिण अफ्रीका से 12 से
14 चीतों को लाने की योजना बना रहा है।

· एशियाई चीता गंभीर रूप से लुप्तप्राय है जबकि अफ़्रीकी चीता IUCN की लाल सूची में सुभेद्य है।

QUESTION
61.
निम्नलिखित जोड़ियों पर विचार करें:

नई प्रजाति अनूठी विशेषता

1. ओमॉर्गस खानदे श फोरेंसिक विज्ञान में भूमिका , मृत्यु के समय का पता लगाने में मदद करती है

2. बदीस लिमाकु मी रंग बदलने और परिवेश के साथ घुलने-मिलने की क्षमता

3. माइक्रिक्सलस कोटिगेहरेन्सिस भारत की सबसे अधिक संकटग्रस्त उभयचर प्रजाति है

उपरोक्त में से कितने जोड़े सही हैं?

a) के वल एक
b) के वल दो
c) सभी तीन
d) कोई नहीं
Correct Answer: C
Your Answer:
Explanation

Q. 61) Solution (c)

IASbaba
Score:
Web: http://ilp.iasbaba.com/
44.00 /
Email: ilp@iasbaba.com
Page 155 200
UPSC 2024 -
Exam Title : Environment,
Sc...
rupeshkr.
Email :
09ee80@gmail.com
Contact :

स्पष्टीकरण:

· एक नई भृंग प्रजाति (ओमोर्गस खानदे श ) की खोज भारत में की गई है जो फोरेंसिक विज्ञान के लिए महत्वपूर्ण है क्योंकि यह किसी जानवर
या इंसान की मृत्यु के समय का पता लगाने में मदद करता है। (अतः जोड़ी 1 सही है)

· वैज्ञानिकों ने हाल ही में नागालैंड की मिलक नदी से मछली की एक नई प्रजाति की खोज की है। नई खोजी गई प्रजाति बादिस लिमाकु मी है

· बदीस परिवार की मछली को रंग बदलने की क्षमता के कारण गिरगिट मछली के रूप में भी जाना जाता है। इससे उन्हें तनाव में होने पर
परिवेश के साथ घुलने-मिलने में मदद मिलती है। (अतः जोड़ी 2 सही है)

· नाचते हुए मेंढक (माइक्रिक्सलस) हाल ही में जारी ग्लोबल एम्फीबियन असेसमेंट के दूसरे संस्करण के आधार पर वाइल्डलाइफ ट्रस्ट ऑफ
इंडिया के अनुसार, पश्चिमी घाट के लिए स्थानिकमारी वाले कोटिगेहरेंसिस भारत की सबसे खतरनाक उभयचर प्रजाति हैं। (अतः जोड़ी 3
सही है)

IMPORTANT POINTS/VALUE ADDITIONS:

· ओमॉर्गस खानदे श नेक्रोफै गस है और इसलिए, इसे के राटिन बीटल भी कहा जाता है। किसी शरीर के विघटन के दौरान, प्रारंभिक अवस्था में
ब्लोफ़्लाइज़ सबसे पहले आती हैं। इस बीच, अंतिम क्रमिक चरण के राटिन फीडरों के आगमन के साथ है, इस प्रकार फोरेंसिक विज्ञान में
उनका महत्व है।

· हाल ही में गिरगिट मछली की प्रजातियों की खोज से संकेत मिलता है कि अन्य पूर्वोत्तर राज्यों की तुलना में नागालैंड में नदियों की खोज
कम की जाती है। खोजें अधिक अज्ञात मछली प्रजातियों की पहचान करने के प्रयासों को बढ़ाने की आवश्यकता पर प्रकाश डालती हैं।

· जलधाराओं के पास पाए जाने वाले नाचते हुए मेंढक संभोग का अनोखा प्रदर्शन करते हैं । नर अपने पिछले पैरों को एक-एक करके फैलाते
हैं और अपने जालदार पंजों को नृत्य की तरह तीव्र गति से हवा में लहराते हैं। यह साथियों को आकर्षित करने के साथ-साथ प्रतिस्पर्धा से बचने
के लिए है। इस कार्य को "फ़ु ट फ़्लैगिंग" कहा जाता है और इससे प्रजाति को उनका नाम मिलता है।

QUESTION
62.
हाल ही में जारी 'भारत के हाथी गलियारे, 2023' नामक रिपोर्ट के संबंध में निम्नलिखित कथनों पर विचार करें:

1. इसमें 2010 के बाद से हाथी गलियारों में 40 प्रतिशत की वृद्धि का उल्लेख किया गया है।

2. भारत में चिह्नित किए गए हाथी गलियारों की संख्या सबसे अधिक कर्नाटक में है, जो दे श के सभी सूचित हाथी गलियारों का 17% से
अधिक है।

3. दक्षिणी क्षेत्र में भारत में हाथियों की सबसे बड़ी आबादी रहती है।

उपरोक्त में से कितने कथन सही हैं?

a) के वल एक
b) के वल दो
c) सभी तीन
d) कोई नहीं
Correct Answer: B
Your Answer:
Explanation

Q. 62) Solution (b)

IASbaba
Score:
Web: http://ilp.iasbaba.com/
44.00 /
Email: ilp@iasbaba.com
Page 156 200
UPSC 2024 -
Exam Title : Environment,
Sc...
rupeshkr.
Email :
09ee80@gmail.com
Contact :

स्पष्टीकरण:

· भारत के चार हाथी- प्रभाव वाले क्षेत्रों में 15 हाथी रेंज वाले राज्यों से कु ल 150 हाथी गलियारों की सूचना मिली थी। भारत सरकार की
हाथी टास्क फोर्स ने 2010 के दौरान प्रकाशित गजह रिपोर्ट में 88 गलियारों को सूचीबद्ध किया। (इसलिए कथन 1 सही है)

· पश्चिम बंगाल में भारत में चिन्हित किए गए हाथी गलियारों की संख्या सबसे अधिक है, जो दे श के सभी सूचित हाथी गलियारों का 17% से
अधिक है। (इसलिए कथन 2 गलत है)

· चार हाथी- प्रभाव वाले क्षेत्रों में से, लगभग 35% हाथी गलियारे पूर्व-मध्य क्षेत्र में थे, इसके बाद 32% उत्तर-पूर्व क्षेत्र में थे।

· दक्षिणी क्षेत्र, जो भारत में हाथियों की सबसे बड़ी आबादी को आश्रय दे ता है, भारत में हाथी गलियारों का 21% हिस्सा है। (अतः कथन 3
सही है)

· उत्तरी क्षेत्र, जो चार क्षेत्रीय हाथियों की आबादी में सबसे छोटा है, में हाथी गलियारों की संख्या सबसे कम है, जो दे श में सभी रिपोर्ट किए
गए हाथी गलियारों का 12% है।

IMPORTANT POINTS/VALUE ADDITIONS:

· सीमा के विस्तार के कारण बिजली के झटके , कु ओं में गिरने और मानव - प्रधान कृ षि परिदृश्य में प्रवेश करने से हाथियों की मौत भी हो र
ही है।

· गलियारा भूमि का एक छोटा सा टु कड़ा माना जाता है जो हाथियों के निवास स्थान में आवाजाही के लिए कनेक्टिविटी प्रदान करता है,
मुख्यतः हाथी रिजर्व के परिदृश्य के भीतर।

· कृ षि आकस्मिकताओं के कारण परिदृश्य आंशि क रूप से खंडित हो जाते हैं, फिर गलियारे इन परिदृश्यों में फैले आवासों के बीच महत्वपूर्ण
कड़ी के रूप में काम करते हैं।

QUESTION
63.
निम्नलिखित युग्मों पर विचार करें:

संरक्षित क्षेत्र राज्य

1. कै मूर वन्यजीव अभयारण्य बिहार

2. यसलूर वन रेंज झारखंड

3. असोला भट्टी वन्यजीव अभयारण्य पंजाब

उपरोक्त में से कितने जोड़े सही हैं?

a) के वल एक
b) के वल दो
c) सभी तीन
d) कोई नहीं
Correct Answer: A
Your Answer:
Explanation

Q. 63) Solution (a)

स्पष्टीकरण:

IASbaba
Score:
Web: http://ilp.iasbaba.com/
44.00 /
Email: ilp@iasbaba.com
Page 157 200
UPSC 2024 -
Exam Title : Environment,
Sc...
rupeshkr.
Email :
09ee80@gmail.com
Contact :

· बिहार वन, पर्यावरण और जलवायु परिवर्तन विभाग के अधिकारियों के अनुसार, बिहार का बहुप्रतीक्षित दूसरा बाघ अभयारण्य 2023 के
अंत या 2024 की शुरुआत में बनने के लिए तैयार है।

· अधिकारी कै मूर वन्यजीव अभयारण्य को वाल्मिकी टाइगर रिजर्व (वीटीआर) के बाद राज्य का दूसरा बाघ रिजर्व घोषित करने के लिए
राष्ट्रीय बाघ रिजर्व संरक्षण प्राधिकरण (एनटीसीए) की मंजूरी प्राप्त करने की दिशा में काम कर रहे हैं। (इसलिए विकल्प 1 सही
है)

· हासन जिले के यसलूर वन रेंज में हाथी पकड़ने के अभियान के दौरान एक जंगली हाथी के साथ हुई मुठभेड़ में मौत हो गई । यह
ऑपरेश न वन विभाग के आदे श के अनुसार हाथियों की बड़े पैमाने पर रेडियो कॉलरिंग का हिस्सा था। (इसलिए विकल्प 2 गलत है)

· दिल्ली उच्च न्यायालय ने दिल्ली सरकार के वन विभाग को शहर के असोला भट्टी वन्यजीव अभयारण्य के अंदर प्रस्तावित कार्यक्रम 'वॉक
विद वाइल्डलाइफ' आयोजित करने से रोकते हुए कहा कि विभाग ने यांत्रिक तरीके से कार्यक्रम की अनुमति दी थी। इसमें कहा गया है कि
विभाग ने खतरे की आशंका का विश्लेषण नहीं किया क्योंकि घटना में किसी दुस्साहस की आशंका है।

· उच्च न्यायालय ने अपने आदे श में यह भी टिप्पणी की कि जबकि वन्यजीव अभयारण्य उनके प्राकृ तिक आवासों में वन्यजीवों के संरक्षण
और संरक्षण के लिए बनाए गए हैं, "हाल ही में, मनुष्य वन्यजीवों के आवास पर अतिक्रमण कर रहा है"। (इसलिए विकल्प 3 गलत है)

IMPORTANT POINTS/VALUE ADDITIONS:

· बाघों की आबादी में वृद्धि, जिसका कारण अवैध शिकार पर प्रतिबंध और साथ ही निवास स्थान का विखंडन और शिकार की कमी है, के
कारण मानव-पशु संघर्षों में उल्लेखनीय वृद्धि हुई है। महाराष्ट्र में बाघों के हमलों से बड़ी संख्या में मौतें हुई हैं।

· बढ़ते संघर्षों से निपटने के लिए कै मूर जैसे अतिरिक्त बाघ अभयारण्यों की स्थापना एक संभावित समाधान है। इसके अतिरिक्त, यसलूर वन
रेंज को वन्यजीव अभयारण्य के रूप में नामित करना, विशेष रूप से खंडित परिदृश्य के संदर्भ में, मनुष्यों और जानवरों के बीच लगातार
मुठभेड़ों को प्रबंधित करने और कम करने के लिए जरूरी है।

· अद्यतन 'वन और वन्यजीव क्षेत्रों में सतत इको-पर्यटन पर दिशानिर्देश' वैज्ञानिक योजना के आधार पर इको-पर्यटन को बढ़ावा दे ने का
आदे श दे ते हैं, जिसमें "लक्ष्य वन्यजीवों, निवास स्थान या भौगोलिक इकाई और उनके प्रबंधन की आवश्यकताओं के आकलन पर इको-पर्यटन
क्षेत्रों का सीमांकन" शामिल है। यह "पर्यटकों की यात्रा की संख्या और पैटर्न की तुलना में वन्यजीवों पर तनाव" की निगरानी भी अनिवार्य
करता है।

QUESTION
64.
जुगाली करने वाले जानवर मान वीय गतिविधि से महत्वपूर्ण मीथेन उत्सर्जन के लिए जिम्मेदार हैं। निम्नलिखित में से कौन सा जानवर जुगाली
करने वाली प्रजाति नहीं है?

a) गाय
b) भेड़
c) घोड़ा
d) बकरी
Correct Answer: C
Your Answer:
Explanation

Q. 64) Solution (c)

स्पष्टीकरण:

· जुगाली करने वाले जानवर, जैसे कि गाय, भेड़, बकरी, भैंस और अन्य, अपने खुरों और शाकाहारी चरने की आदतों की विशेषता रखते हैं,
जो जुगाली करने के अनूठे व्यवहार में संलग्न होते हैं।

IASbaba
Score:
Web: http://ilp.iasbaba.com/
44.00 /
Email: ilp@iasbaba.com
Page 158 200
UPSC 2024 -
Exam Title : Environment,
Sc...
rupeshkr.
Email :
09ee80@gmail.com
Contact :

· यह विशेष अनुकूलन इन खुर वाले स्तनधारियों को कोशिका भित्ति से आवश्यक पोषक तत्वों को निकालने, पौधों के सेलूलोज़ को प्रभावी
ढं ग से तोड़ने की अनुमति दे ता है।

· उनकी पाचन प्रक्रिया के एक प्रमुख पहलू में मीथेन उत्पादन शामिल है। जुगाली करने वालों का पेट चार भागों वाला होता है, रूमेन आंशि क
रूप से पचने वाले भोजन को संग्रहित करने और किण्वन प्रक्रिया को सुविधाजनक बनाने के लिए महत्वपूर्ण होता है।

· यह जटिल पाचन तंत्र पौधों की सामग्री को पोषक तत्वों में बदलने में महत्वपूर्ण भूमिका निभाता है, जो जुगाली करने वाली प्रजातियों की
उनकी पोषण संबंधी आवश्यकताओं के लिए सेलूलोज़ का उपयोग करने की उल्लेखनीय दक्षता को उजागर करता है।

· घोड़े जुगाली न करने वाले, साधारण पेट वाले शाकाहारी प्राणी होते हैं। वे पश्च आंत किण्वक हैं, जिसका अर्थ है कि बड़ी आंत रेश ेदार खाद्य
पदार्थों के किण्वन का मुख्य स्थल है। यह मवेश ी, बकरी, हिरण और भेड़ जैसे जुगाली करने वाले जानवरों से अलग है, जो रूमेन और
मल्टीकम्पार्टमेंट पेट के साथ अग्रगामी किण्वक होते हैं। (इसलिए विकल्प सी सही उत्तर है)

IMPORTANT POINTS/VALUE ADDITIONS:

· मीथेन, एक शक्तिशाली ग्रीनहाउस गैस है जो मुख्य रूप से जुगाली करने वाले पशुओं के डकारने से निकलती है।

· यह अनुमान लगाया गया है कि जुगाली करने वालों का पाचन तंत्र मानव गतिविधि से होने वाले सभी मीथेन उत्सर्जन के 27% के लिए
जिम्मेदार है।

QUESTION
65.
भारत सरकार द्वारा हाल ही में शुरू किए गए "मिष्टी ( MISHTI ) " कार्यक्रम का प्राथमिक उद्दे श्य क्या है?

a) सतत प्रथाओं के माध्यम से तटीय जिलों में शहरी बुनियादी ढांचे को बढ़ाना।
b) भारत के मैंग्रोव-समृद्ध क्षेत्रों में सतत कृ षि को बढ़ावा दे ना।
c) तटीय क्षेत्रों में जलवायु परिवर्तन के लिए एक अनुसंधान गठबंधन स्थापित करना।
d) तटीय जिलों में मैंग्रोव पुनर्वनीकरण और वनरोपण करना।
Correct Answer: D
Your Answer:
Explanation

Q. 65) Solution (d)

स्पष्टीकरण:

· भारत सरकार द्वारा हाल ही में भारत सरकार द्वारा मैंग्रोव इनिशिएटिव फॉर शोरलाइन हैबिटे ट्स एंड टैं जिबल इनकम (MISHTI) योजना
शुरू की गई थी, जिसका उद्दे श्य भारत के साथ-साथ अन्य दे श ों में पहले से मौजूद सर्वोत्तम प्रथाओं को अपनाकर इंडोनेशि या सहित भारत के
तटीय जिलों में मैंग्रोव पुनर्वनीकरण और वनीकरण करना है। (इसलिए विकल्प d सही है)

· इस कार्यक्रम की परिकल्पना तटीय राज्यों में मैंग्रोव से जुड़ी इकोटू रिज्म पहल और आजीविका सृजन को विकसित करने के लिए भी की
गई है।

· मिष्टी मैंग्रोव को बढ़ावा दे ने के लिए एक अंतरसरकारी गठबंधन 'मैंग्रोव एलायंस फॉर क्लाइमेट (एमएसी)' के प्रयासों में योगदान दे गी,
जिसका भारत (COP27) के दौरान सक्रिय सदस्य बन गया।

IMPORTANT POINTS/VALUE ADDITIONS:

· वर्तमान में, मैंग्रोव के अंतर्गत लगभग 5000 वर्ग किलोमीटर क्षेत्र है और मिष्टी कार्यक्रम के माध्यम से 9 राज्यों और 4 कें द्र शासित प्रदे श ों में
540 वर्ग किलोमीटर के अतिरिक्त क्षेत्र को कवर करने का प्रस्ताव है।

IASbaba
Score:
Web: http://ilp.iasbaba.com/
44.00 /
Email: ilp@iasbaba.com
Page 159 200
UPSC 2024 -
Exam Title : Environment,
Sc...
rupeshkr.
Email :
09ee80@gmail.com
Contact :

· इस योजना को पांच साल की अवधि के लिए लागू करने की योजना है। मिष्टी को CAMPA फं ड, MGNREGS और अन्य स्रोतों को
मिलाकर लागू किया जाना है।

QUESTION
66.
Q .66) पर्यावरण वैज्ञानिकों द्वारा प्रस्तावित "ग्रहीय सीमाओं ( planetary boundaries ) " की अवधारणा के संदर्भ में निम्नलिखित
पर विचार करें, जिसमें विनाशकारी पर्यावरणीय परिवर्तनों को रोकने के लिए मानवीय गतिविधियों की सीमा को परिभाषित करना शामिल
है:

1. महासाग रीय अम्लीकरण

2. भूमि उपयोग परिवर्तन

3. समतापमंडलीय ओजोन रिक्तीकरण

4. वैश्विक मीठे पानी का उपयोग

उपरोक्त में से कितनी 'ग्रहीय सीमाएँ' हैं?

a) के वल एक
b) के वल दो
c) के वल तीन
d) सभी चार
Correct Answer: D
Your Answer:
Explanation

Q. 66) Solution (d)

स्पष्टीकरण:

· ग्रहों की सीमाओं की रूपरेखा सबसे पहले 2009 में जोहान रॉकस्ट्रॉम और 28 अंतरराष्ट्रीय स्तर पर प्रसिद्ध वैज्ञानिकों के एक समूह द्वारा
प्रस्तावित की गई थी ताकि उन पर्यावरणीय सीमाओं को परिभाषित किया जा सके जिनके भीतर मानवता पृथ्वी की सतत ता और जैव
विविधता को बनाए रखने के लिए सुरक्षित रूप से काम कर सकती है।

· ये नौ ग्रहों की सीमाएँ हैं: जलवायु परिवर्तन, जीवमंडल अखंडता में परिवर्तन (जैव विविधता हानि और प्रजातियों का विलुप्त होना), समता
पमंडलीय ओजोन रिक्तीकरण, महासागर अम्लीकरण, जैव-भू-रासायनिक प्रवाह (फास्फोरस और नाइट्रोजन चक्र), भूमि-प्रणाली
परिवर्तन (उदाहरण के लिए वनोन्मूलन ), मीठे पानी का उपयोग (परिवर्तन) भूमि पर संपूर्ण जल चक्र के दौरान), वायुमंडलीय एयरोसोल
लोडिंग (वायुमंडल में सूक्ष्म कण जो जलवायु और जीवित जीवों को प्रभावित करते हैं), नई संस्थाओं का परिचय (माइक्रोप्लास्टिक्स,
अंतःस्रावी अवरोधक और कार्बनिक प्रदूषकों से युक्त)। (इसलिए विकल्प d सही उत्तर है)

IMPORTANT POINTS/VALUE ADDITIONS:

· साइंस एडवांसेज जर्नल में प्रकाशित एक नए अध्ययन के अनुसार , विश्व ने पृथ्वी की स्थिरता और लचीलापन बनाए रखने के लिए
आवश्यक नौ ग्रहीय सीमाओं में से छह का उल्लंघन किया है।

· इन सीमाओं का उल्लंघन किसी तात्कालिक तबाही का संकेत नहीं दे ता है बल्कि अपरिवर्तनीय पर्यावरणीय परिवर्तनों का खतरा पैदा करता
है।

· यह स्थिति पृथ्वी पर ऐसी स्थितियाँ पैदा कर सकती है जो अब हमारी वर्तमान जीवन शैली का समर्थन नहीं करेंगी।

IASbaba
Score:
Web: http://ilp.iasbaba.com/
44.00 /
Email: ilp@iasbaba.com
Page 160 200
UPSC 2024 -
Exam Title : Environment,
Sc...
rupeshkr.
Email :
09ee80@gmail.com
Contact :

QUESTION
67.
Q .67) हरित हाइड्रोजन का उत्पादन करने, उच्च दक्षता सुनिश्चित करने और पर्यावरणीय प्रभाव को कम करने के लिए इलेक्ट्रोलिसिस
प्रक्रिया में आमतौर पर कौन सी उन्नत तकनीक का उपयोग किया जाता है?

a) प्रोटॉन एक्सचेंज मेम्ब्रेन (पीईएम) इलेक्ट्रोलिसिस


b) भाप मीथेन सुधार ( Steam Methane Reforming )
c) कार्बन कै प्चर प्रौद्योगिकी के माध्यम से कोयला गैसीकरण।
d) हैबर-बॉश प्रक्रिया
Correct Answer: A
Your Answer:
Explanation

Q. 67) Solution (a)

स्पष्टीकरण:

· हाइड्रोजन उत्पादन के लिए ऊर्जा रूपांतरण प्रणाली के रूप में प्रोटॉन एक्सचेंज झिल्ली इलेक्ट्रोलिसिस ने काफी ध्यान आकर्षित किया है।
इसकी ऊर्जा दक्षता, कम पूंजी लागत, लचीलेपन, सुरक्षा और स्थायित्व के कारण इसे हरित हाइड्रोजन उत्पादन के लिए पसंदीदा विकल्प
माना जाता है । (इसलिए विकल्प ए सही है)

· ग्रे हाइड्रोजन: इसका उत्पादन कोयला या लिग्नाइट गैसीकरण (काला या भूरा) के माध्यम से, या प्राकृ तिक गैस या मीथेन के भाप मीथेन
सुधार (एसएमआर) नामक प्रक्रिया के माध्यम से किया जाता है। (इसलिए विकल्प बी गलत है)

· ब्लू हाइड्रोजन: यह कार्बन उत्सर्जन को कम करने के लिए कार्बन कै प्चर स्टोरेज (सीसीएस) या कार्बन कै प्चर उपयोग (सीसीयू)
प्रौद्योगिकियों के साथ संयुक्त प्राकृ तिक गैस या कोयला गैसीकरण के माध्यम से उत्पादित किया जाता है। (इसलिए विकल्प सी गलत है)

· हरित अमोनिया बनाने का एक तरीका पानी के इलेक्ट्रोलिसिस से हाइड्रोजन और हवा से अलग नाइट्रोजन का उपयोग करना है। फिर इन्हें
हैबर प्रक्रिया (जिसे हैबर-बॉश भी कहा जाता है) में डाला जाता है। (इसलिए विकल्प d गलत है)

IMPORTANT POINTS/VALUE ADDITIONS:

· भारत ने ग्रीन हाइड्रोजन और उसके डेरिवेटिव के उत्पादन, उपयोग और निर्यात के लिए भारत को वैश्विक कें द्र बनाने के लिए राष्ट्रीय ग्रीन
हाइड्रोजन मिशन शुरू किया। यह स्वच्छ ऊर्जा के माध्यम से आत्मनिर्भर बनने के भारत के लक्ष्य में योगदान दे गा और वैश्विक स्वच्छ ऊर्जा
संक्रमण के लिए प्रेरणा के रूप में काम करेगा।

· मिशन से अर्थव्यवस्था में उल्लेखनीय रूप से डीकार्बोनाइजेश न होगा, जीवाश्म ईंधन के आयात पर निर्भरता कम होगी और भारत हरित
हाइड्रोजन में प्रौद्योगिकी और बाजार का नेतृत्व संभालने में सक्षम होगा।

· हैबर प्रक्रिया में, अमोनिया, NH3 का उत्पादन करने के लिए हाइड्रोजन और नाइट्रोजन को उच्च तापमान और दाब पर एक साथ
प्रतिक्रिया की जाती है।

QUESTION
68.
'निरंतर जीवाश्म ईंधन ( unabated fossil fuels ) ' के संदर्भ में निम्नलिखित मानदं डों पर विचार करें:

1. यह बिना किसी हस्तक्षेप के उत्पादित और उपयोग किए जाने वाले जीवाश्म ईंधन को संदर्भित करता है जो संपूर्ण जीवन चक्र में उत्सर्जित
जीएचजी की मात्रा को काफी हद तक कम कर दे ता है।

2. जीवाश्म ईंधन का उपयोग जो बिजली संयंत्रों से 100% CO2 ग्रहण नहीं करता है।

3. जीवाश्म ईंधन का उपयोग जो ऊर्जा उत्पादन और आपूर्ति से 50-80% फ़्यूजिटिव मीथेन उत्सर्जन को कै प्चर नहीं करता है।

IASbaba
Score:
Web: http://ilp.iasbaba.com/
44.00 /
Email: ilp@iasbaba.com
Page 161 200
UPSC 2024 -
Exam Title : Environment,
Sc...
rupeshkr.
Email :
09ee80@gmail.com
Contact :

उपरोक्त में से कौन सा कथन सही है?

a) 1 और 2
b) 2 और 3
c) 1 और 3
d) उपरोक्त सभी
Correct Answer: C
Your Answer:
Explanation

Q. 68) Solution (c)

स्पष्टीकरण:

· संयुक्त राष्ट्र इंटरगवर्नमेंटल पैनल ऑन क्लाइमेट चेंज (आईपीसीसी) की एआर6 बेंचमार्क रिपोर्ट में कहा गया है कि निर्बाध जीवाश्म ईंधन वे
हैं जो "बिना किसी हस्तक्षेप के ग्रीनहाउस गैस उत्सर्जन को काफी हद तक कम करते हैं"। (इसलिए पहला कथन सही है )

· बिजली संयंत्रों से 90% या अधिक CO2 एकत्र करना। (इसलिए दूसरा कथन गलत है )

· या ऊर्जा आपूर्ति से 50-80% फ़्यूजिटिव मीथेन उत्सर्जन। (इसलिए तीसरा कथन सही है)

IMPORTANT POINTS/VALUE ADDITIONS:

· अंतर्राष्ट्रीय ऊर्जा एजेंसी (आईईए) ने अपनी रिपोर्ट, कार्बन कै प्चर, यूटिलाइजेश न एंड स्टोरेज में कहा कि बिजली और औद्योगिक संयंत्र जो
आधुनिक सीसीएस प्रौद्योगिकियों से लैस हैं, वे लगभग 90% CO2 ग्रहण करते हैं।

· हालाँकि, इंस्टीट्यूट फॉर एनर्जी इकोनॉमिक्स एंड फाइनेंशि यल एनालिसिस (IEEFA) - एक वैश्विक थिंक टैं क जो ऊर्जा बाजारों, रुझानों
और नीतियों से संबंधित मुद्द ों की जांच करता है - के 2022 के एक अध्ययन में पाया गया कि विश्व भर में 13 प्रमुख सीसीएस परियोजनाओं
में से अधिकांश का विश्लेषण किया गया है। जो ख़राब प्रदर्शन या पूरी तरह से विफल रहे हैं ।

· जर्मनी स्थित जलवायु विज्ञान और नीति संस्थान, क्लाइमेट एनालिटिक्स के एक अन्य विश्लेषण से पता चला है कि सीसीएस पर निर्भरता
2020 और 2050 के बीच वायुमंडल में 86 बिलियन टन अतिरिक्त ग्रीनहाउस गैसें जारी कर सकती है।

QUESTION
69.
'जीएम सरसों' के संदर्भ में निम्नलिखित कथनों पर विचार करें:

1. संकरण ( Hybridisation ) में दो आनुवंशि क रूप से भिन्न पौधों की किस्मों को क्रास करना शामिल है जो के वल अलग-अलग
प्रजातियों से होनी चाहिए।

2. ऐसे संकरण से प्राप्त पहली पीढ़ी (F1) की संतानों की पैदावार माता-पिता द्वारा व्यक्तिगत रूप से दी जा सकने वाली उपज से अधिक
होती है।

3. जीएम फसलें उन पौधों से प्राप्त होती हैं जिनके जीन को कृ त्रिम रूप से, आमतौर पर किसी अन्य जीव से आनुवंशि क सामग्री डालकर
संश ोधित किया जाता है।

निम्नलिखित में से कौन सा कथन सही है?

a) 1 और 2
b) 2 और 3
c) 1 और 3

IASbaba
Score:
Web: http://ilp.iasbaba.com/
44.00 /
Email: ilp@iasbaba.com
Page 162 200
UPSC 2024 -
Exam Title : Environment,
Sc...
rupeshkr.
Email :
09ee80@gmail.com
Contact :

d) उपरोक्त सभी
Correct Answer: B
Your Answer:
Explanation

Q. 69) Solution (b)

स्पष्टीकरण:

· संकरण में दो आनुवंशि क रूप से भिन्न पौधों की किस्मों को क्रास करना शामिल है जो एक ही प्रजाति से भी हो सकते हैं, जरूरी नहीं
कि अलग-अलग प्रजातियों की आवश्यकता हो। (इसलिए कथन 1 गलत है)

· ऐसे संकरण से प्राप्त पहली पीढ़ी (F1) की संतानों की पैदावार माता-पिता द्वारा व्यक्तिगत रूप से दी जा सकने वाली उपज से अधिक होती
है। (अतः कथन 2 सही है)

· जीएम फसलें उन पौधों से प्राप्त की जाती हैं जिनके जीन को कृ त्रिम रूप से संश ोधित किया जाता है, आमतौर पर किसी अन्य जीव से
आनुवंशि क सामग्री डालकर, इसे नए गुण प्रदान करने के लिए, जैसे कि बढ़ी हुई उपज, शाकनाशी के प्रति सहनशीलता, रोग या सूखे के प्रति
प्रतिरोध, या बेहतर पोषण मूल्य। (अतः कथन 3 सही है)

IMPORTANT POINTS/VALUE ADDITIONS:

· सरसों में संकरण आसान नहीं है, क्योंकि इसके फू लों में मादा (पिस्टिल) और नर (पुंकेसर) दोनों प्रजनन अंग होते हैं, जिससे पौधे बड़े पैमाने
पर स्व-परागण करते हैं।

· चूंकि एक पौधे के अंडों को दूसरे पौधे के परागकणों द्वारा निषेचित नहीं किया जा सकता है, इसलिए यह कपास, मक्का या टमाटर के
विपरीत संकर विकसित करने की गुंजाइश को सीमित कर दे ता है, जहां यह साधारण निर्बलीकरण या परागकोषों को भौतिक रूप से हटाने
के माध्यम से किया जा सकता है।

· आनुवंशि क संश ोधन (जीएम) द्वारा, दिल्ली विश्वविद्यालय के सेंटर फॉर जेनेटिक मैनिपुलेश न ऑफ क्रॉप प्लांट्स (सीजीएमसीपी) के
वैज्ञानिकों ने हाइब्रिड सरसों डीएमएच -11 विकसित किया है जिसमें बैसिलस एमाइलोलिकफे शियन्स नामक मिट्टी के जीवाणु से अलग किए
गए दो विदे श ी जीन (बार्नेज़ और बारस्टार) शामिल हैं ।

· DMH-11 एक स्वदे श ी रूप से विकसित ट्रांसजेनिक सरसों है। यह हर्बिसाइड टॉलरेंट (एचटी) सरसों का आनुवंशि क रूप से संश ोधित
संस्करण है।

· डीएमएच-11 भारतीय सरसों की किस्म 'वरुणा' और पूर्वी यूरोपीय 'अर्ली हीरा-2 ( Early Heera- 2) ' सरसों के मिश्रण का परिणाम
है।

QUESTION
70.
स्कॉटलैंड के ग्लासगो में COP26 शिखर सम्मेलन में, किस महत्वपूर्ण पहलू ने 'ग्लासगो संधि' को चिह्नित किया?

a) नए जलवायु लक्ष्यों का परिचय


b) समझौते में परमाणु ऊर्जा को शामिल करना
c) कोयले को चरणबद्ध तरीके से ख़त्म करना और जीवाश्म ईंधन को चरणबद्ध तरीके से ख़त्म करना
d) वैश्विक कार्बन कर प्रणाली की स्थापना
Correct Answer: C
Your Answer:
Explanation

Q. 70) Solution (c)

IASbaba
Score:
Web: http://ilp.iasbaba.com/
44.00 /
Email: ilp@iasbaba.com
Page 163 200
UPSC 2024 -
Exam Title : Environment,
Sc...
rupeshkr.
Email :
09ee80@gmail.com
Contact :

स्पष्टीकरण:

· स्कॉटलैंड के ग्लासगो में COP26 शिखर सम्मेलन में हुए समझौते में कोय ला उपयोग को धीरे-धीरे कम करने और जीवाश्म ईंधन को धीरे-
धीरे ख़त्म करने का आह्वान किया गया।

· यह पहली बार था कि संयुक्त राष्ट्र जलवायु समझौते में कोयले का स्पष्ट रूप से उल्लेख किया गया था। इस समझौते ने कार्बन बाजारों पर
गतिरोध के समाधान को भी चिह्नित किया। ( इसलिए विकल्प सी सही है)

IMPORTANT POINTS/VALUE ADDITIONS:

· सीओपी संयुक्त राष्ट्र (यूएन) द्वारा प्रतिवर्ष आयोजित एक अंतरराष्ट्रीय जलवायु बैठक है। COP कॉन्फ्रेंस ऑफ द पार्टीज़ का संक्षिप्त रूप है।
'पार्टियाँ' उन (अब) 198 दे श ों का संदर्भ है जो जलवायु परिवर्तन पर संयुक्त राष्ट्र फ्रेमवर्क कन्वेंश न (यूएनएफसीसीसी) नामक अंतर्राष्ट्रीय
संधि में शामिल हो गए हैं।

· संधि के 'पक्षों' ने "जलवायु प्रणाली में खतरनाक मानवजनित [मानव-जनित] हस्तक्षेप" को रोकने के लिए स्वैच्छिक कार्रवाई करने का
वादा किया है।

· कार्बन बाज़ार अनिवार्य रूप से व्यापारिक प्रणालियाँ हैं जिनमें कार्बन क्रेडिट बेचे और खरीदे जाते हैं। वे दे श ों या उद्योगों को अपने लक्ष्य से
अधिक ग्रीनहाउस गैस उत्सर्जन में कटौती के लिए कार्बन क्रेडिट अर्जित करने की अनुमति दे ते हैं।

· इन कार्बन क्रेडिट को पैसे के बदले में सबसे अधिक बोली लगाने वाले को बेचा जा सकता है। कार्बन क्रेडिट के खरीदार उत्सर्जन में कटौती
को अपना दिखा सकते हैं और अपने कटौती लक्ष्यों को पूरा करने के लिए उनका उपयोग कर सकते हैं।

· एक व्यापार योग्य कार्बन क्रेडिट एक टन कार्बन डाइऑक्साइड या कम की गई, पृथक की गई या टाली गई विभिन्न ग्रीनहाउस गैस की
समतुल्य मात्रा के बराबर है। एक बार क्रेडिट का उपयोग उत्सर्जन को कम करने, अलग करने या उससे बचने के लिए किया जाता है, तो यह
ऑफसेट बन जाता है और अब इसका व्यापार नहीं किया जा सकता है।

QUESTION
71.
'समुद्री पारिस्थितिकी के संकेतकों में पाए गए वैश्विक जलवायु-परिवर्तन रुझान' के संदर्भ में निम्नलिखित कथनों पर विचार करें:

1. इसमें कहा गया है कि सबसे अधिक प्रभावित क्षेत्रों में से एक भूमध्य रेखा के पास उष्णकटिबंधीय महासागर है, जहां पानी नीले से हरे रंग
में बदल रहा है।

2. इससे पता चलता है कि रंग परिवर्तन उन क्षेत्रों में अधिक है जहां महासागर अधिक स्तरीकृ त हो रहे हैं।

3. महासागर नीला और गहरा नीला दिखाई दे ता है क्योंकि नीले और बैंगनी रंग की तरंगदै र्ध्य बहुत कम होती है जो वापस परावर्तित हो जाती
है।

4. फाइटोप्लांकटन की मौजूदगी के कारण महासागर हरे दिखाई दे ते हैं।

उपरोक्त में से कौन सा कथन सही है?

a) 1, 2 और 3
b) 2, 3 और 4
c) 3 और 4
d) उपरोक्त सभी
Correct Answer: D
Your Answer:
Explanation

Q. 71) Solution (d)

IASbaba
Score:
Web: http://ilp.iasbaba.com/
44.00 /
Email: ilp@iasbaba.com
Page 164 200
UPSC 2024 -
Exam Title : Environment,
Sc...
rupeshkr.
Email :
09ee80@gmail.com
Contact :

स्पष्टीकरण:

· जुलाई की शुरुआत में नेचर जर्नल में प्रकाशित अध्ययन, 'समुद्री पारिस्थितिकी के संकेतकों में पाए गए वैश्विक जलवायु-परिवर्तन रुझान', से
:

· अध्ययन में कहा गया है कि सबसे अधिक प्रभावित क्षेत्रों में से एक भूमध्य रेखा के पास उष्णकटिबंधीय महासागर क्षेत्र है, जहां पानी नीले से
हरे रंग में बदल रहा है। (अतः कथन 1 सही है)

· निष्कर्षों से पता चलता है कि रंग में बदलाव उन क्षेत्रों में हो रहा है जहां महासागर अधिक स्तरीकृ त हो रहे हैं। (अतः कथन 2 सही है)

· विश्व भर के अधिकांश क्षेत्रों में, महासागर किसी कारण से नीले या गहरे नीले रंग के दिखाई दे ते हैं। नासा की एक रिपोर्ट के अनुसार, ऐसा
"प्रकाश के अवशोषण और प्रकीर्णन" के कारण होता है। (अतः कथन 3 सही है)

· विश्व के अन्य हिस्सों में महासागर हरे दिखाई दे ते हैं, जो पानी की ऊपरी सतह पर फाइटोप्लांकटन की मौजूदगी के कारण होता है। (अतः
कथन 4 सही है)

IMPORTANT POINTS/VALUE ADDITIONS:

· एक नए अध्ययन के अनुसार, पिछले दो दशकों में पृथ्वी के महासागरों का रंग काफी बदल गया है, संभवतः मानव-प्रेरित जलवायु परिवर्तन
के कारण। इसमें कहा गया है कि ग्रह पर कु ल भूमि क्षेत्र से अधिक, 56 प्रतिशत से अधिक महासागरों के रंग में बदलाव का अनुभव हुआ है।

· जब सूर्य का प्रकाश गहरे और साफ पानी पर पड़ता है, तो लंबी तरंग दै र्ध्य वाले रंग, जैसे लाल, पीला और हरा, पानी के अणुओं द्वारा
अवशोषित हो जाते हैं, लेकिन नीले और बैंगनी, जिनकी तरंग दै र्ध्य बहुत कम होती है, वापस परावर्तित हो जाते हैं।

· जहाँ प्रमुख नदियाँ अटलांटिक महासागर में विलीन होती हैं, वहाँ मृत पत्तियों और नदियों से निकलने वाले तलछट के कारण समुद्र का रंग
भूरा हो जाता है।

· दुनिया के अन्य हिस्सों में महासागर हरे दिखाई दे ते हैं, जो पानी की ऊपरी सतह पर फाइटोप्लांकटन की मौजूदगी के कारण होता है।

QUESTION
72.
'प्लास्टिक प्रदूषण' के बारे में निम्नलिखित कथनों पर विचार करें:

1. नए समुद्री सूक्ष्मजीव पर्यावास को "प्लास्टिस्फे यर ( plastisphere ) " नाम दिया गया है।

2. वार्षिक प्लास्टिक कचरे का के वल 30% ही पुनर्चक्रित किया जाता है।

3. सिगरेट के टु कड़े पर्यावरण में पाए जाने वाले प्लास्टिक कचरे का सबसे आम प्रकार हैं।

4. पीईटी (पॉलीइथाइलीन टे रेफ्थेलेट) विश्व का सबसे आम प्लास्टिक है और सबसे अधिक रिसाइकल करने योग्य प्लास्टिक भी है।

उपरोक्त में से कौन सा कथन सही है?

a) 1 और 2
b) 1, 3 और 4
c) 2, 3 और 4
d) उपरोक्त सभी
Correct Answer: B
Your Answer:
Explanation

Q. 72) Solution (b)

IASbaba
Score:
Web: http://ilp.iasbaba.com/
44.00 /
Email: ilp@iasbaba.com
Page 165 200
UPSC 2024 -
Exam Title : Environment,
Sc...
rupeshkr.
Email :
09ee80@gmail.com
Contact :

स्पष्टीकरण:

· माइक्रोप्लास्टिक सहित प्लास्टिक अब हमारे प्राकृ तिक वातावरण में सर्वव्यापी हैं। वे पृथ्वी के जीवाश्म रिकॉर्ड का हिस्सा बन रहे हैं और
हमारे वर्तमान भूवैज्ञानिक युग, एंथ्रोपोसीन के एक मार्क र बन रहे हैं। उन्होंने "प्लास्टिस्फे यर" नामक एक नए समुद्री सूक्ष्मजीव निवास को भी
अपना नाम दिया है। (अतः कथन 1 सही है)

· वार्षिक प्लास्टिक कचरे का के वल 9% ही पुनर्चक्रित किया जाता है। (इसलिए कथन 2 गलत है)

· सिगरेट के बट - जिनके फिल्टर में छोटे प्लास्टिक फाइबर होते हैं - पर्यावरण में पाए जाने वाले प्लास्टिक कचरे का सबसे आम प्रकार हैं। (
अतः कथन 3 सही है)

· पॉलीइथाइलीन टे रेफ्थेलेट, विश्व का सबसे आम प्लास्टिक और पीईटी सबसे अधिक रिसाइकिल होने वाला प्लास्टिक है और इसके उपोत्पाद
के लिए एक मजबूत बाजार है जिसका उपयोग पेय की बोतलें, खाद्य कं टे नर या कपड़ों के लिए फाइबर बनाने के लिए किया जाता है। (अतः
कथन 4 सही है)

महत्वपूर्ण बिंदु/मूल्यवर्धन :

· खाद्य रैपर, प्लास्टिक की बोतलें, प्लास्टिक की बोतल के ढक्कन, प्लास्टिक किराना बैग, प्लास्टिक स्ट्रॉ और स्टिरर सिगरेट बट् स के बाद
अगली सबसे आम वस्तुएं हैं। हम में से कई लोग हर दिन इन उत्पादों का उपयोग करते हैं।

QUESTION
73.
“राष्ट्रीय उद्यान में कियार, नाथ और किबार नालों के जलग्रहण क्षेत्र शामिल हैं, जो सभी दक्षिण-पश्चिम में मारवाह नदी (जिसे मारुसुदर नदी के
रूप में भी जाना जाता है) में गिरते हैं। भू-भाग आम तौर पर ऊबड़-खाबड़ और खड़ी स्थलाकृ ति है तथा उनके ऊपरी हिमनद भागों में खुलने
वाली ऊँ ची चोटियों से घिरी संकरी घाटियाँ हैं। इसमें लुभावनी अल्पाइन घास का मैदान शामिल है।

उपरोक्त विवरण निम्नलिखित में से किस राष्ट्रीय उद्यान को दर्शाता है?

a) किश्तवाड़ राष्ट्रीय उद्यान


b) दाचीगाम राष्ट्रीय उद्यान
c) हेमिस राष्ट्रीय उद्यान
d) सलीम अली राष्ट्रीय उद्यान
Correct Answer: A
Your Answer:
Explanation

Q. 73) Solution (a)

स्पष्टीकरण:

· हाल ही में वन्यजीव संरक्षण विभाग की शोध टीम ने कै मरा ट्रैप तस्वीरों के जरिए किश्तवाड़ हाई एल्टीट्यूड नेश नल पार्क में हिम तेंदुए
की मौजूदगी की पुष्टि की है । (इसलिए विकल्प ए सही है)

· कै मरा ट्रैप ने नांत नाला क्षेत्र में दो हिम तेंदुओं को कै द किया था। मारवाह और दच्छन क्षेत्र में कै मरा ट्रैप ने तीन हिम तेंदुओं की तस्वीरें कै द
की हैं।

IMPORTANT POINTS/VALUE ADDITIONS:

· हिम तेंदुए को उनकी घटती संख्या को दे खते हुए अंतर्राष्ट्रीय प्रकृ ति संरक्षण संघ की लाल सूची में सुभेद्य के रूप में सूचीबद्ध किया गया है।

· अनुमान है कि विश्व भर में 10,000 से भी कम परिपक्व हिम तेंदुए हैं।

IASbaba
Score:
Web: http://ilp.iasbaba.com/
44.00 /
Email: ilp@iasbaba.com
Page 166 200
UPSC 2024 -
Exam Title : Environment,
Sc...
rupeshkr.
Email :
09ee80@gmail.com
Contact :

· ये जानवर आमतौर पर गर्मियों के दौरान अल्पाइन घास के मैदानों और चट्टानी क्षेत्रों में 2,700 मीटर से 6,000 मीटर की ऊं चाई पर वृक्ष
रेखा के ऊपर रहते हैं। सर्दियों में, वे लगभग 1,200 मीटर से 2,000 मीटर की ऊं चाई तक नीचे आ जाते हैं।

QUESTION
74.
निम्नलिखित कथनों पर विचार करें:

1. बलराम अम्बाजी और जेसोर (बनासकांठा जिला), जंबुघोड़ा (पंचमहल जिला), रतनमहल (दाहोद जिला) एक राज्य में स्थित वन्यजीव
अभयारण्य हैं।

2. वे तेंदुए, धारीदार लकड़बग्घे, सियार, भेड़िये, जंगली बिल्लियाँ, नीले बैल, जंगली सूअर, भारतीय लोमड़ियों आदि के साथ-साथ स्लॉथ
भालू की प्रमुख प्रजातियों को सुरक्षित पर्यावास प्रदान करते हैं।

3. सीएजी रिपोर्ट में बड़े पैमाने पर उल्लंघन, खराब प्रबंधन और अतिक्रमण पर प्रकाश डाला गया है ।

निम्नलिखित में से कौन सा राज्य उपर्युक्त विवरण से सम्बंधित है?

a) गुजरात
b) मध्य प्रदे श
c) उत्तर प्रदे श
d) राजस्थान
Correct Answer: A
Your Answer:
Explanation

Q. 74) Solution (a)

स्पष्टीकरण:

· भारत के नियंत्रक महालेखा परीक्षक (सीएजी) ने बताया कि उल्लंघनों और अन्य विसंगतियों की एक श्रृंखला से गुजरात में वन्यजीवों को
खतरा हो रहा है, खासकर राज्य वन विभाग द्वारा प्रबंधित छह अभयारण्यों में।

· गुजरात में छह अभयारण्य - बलराम अम्बाजी और जेसोर ( बनासकांठा जिला); जम्बुघोडा ( पंचमहल जिला), रतनमहल (दाहोद जिला),
शूलपनेश्वर (नर्मदा जिला) और पूर्णा (तापी और डांग जिले) - सुस्त भालू, तेंदुए, धारीदार लकड़बग्घे, सियार, भेड़िये, जंगली बिल्लियाँ, नीले
बैल, जंगली सूअर, भारतीय लोमड़ी, भारतीय खरगोश, लंगूर, सरीसृप, पक्षी, भौंकने वाले हिरण , मृग और चीतल, अन्य को सुरक्षित पर्यावास
प्रदान करते हैं।

· रतनमहल अभयारण्य में स्लॉथ भालू रहते हैं, जो इस क्षेत्र की एक प्रमुख प्रजाति है।

· सीएजी ने 16 सितंबर, 2023 को राज्य विधानसभा में पेश अपनी रिपोर्ट में कहा कि धन के खराब व्यय के कारण कु प्रबंधन हुआ है,
जिससे जंगलों को अतिक्रमण, खनन और वनों की कटाई के लिए खुला छोड़ दिया गया है।

· 2016-17 में वन विभाग द्वारा आवंटित निधि का 1 प्रतिशत से भी कम खर्च किया गया। सीएजी ने बताया कि हालांकि राज्य सरकार ने
85,557.78 करोड़ रुपये आवंटित किए, लेकिन के वल 1,000 करोड़ रुपये या 0.78 प्रतिशत ही खर्च किए गए।

IMPORTANT POINTS/VALUE ADDITIONS:

· गुजरात भालू संरक्षण और कल्याण कार्य योजना ने भालू आबादी की निगरानी के लिए कोई प्रावधान विकसित नहीं किया। विभाग ने कोई
इकोटू रिज्म योजना नहीं बनाई ।

IASbaba
Score:
Web: http://ilp.iasbaba.com/
44.00 /
Email: ilp@iasbaba.com
Page 167 200
UPSC 2024 -
Exam Title : Environment,
Sc...
rupeshkr.
Email :
09ee80@gmail.com
Contact :

· राज्य ने अनुसूचित जनजाति और अन्य पारंपरिक वन निवासी (वन अधिकारों की मान्यता) अधिनियम, 2006 के कार्यान्वयन के बाद से
पिछले 14 वर्षों में वन्यजीव संरक्षण के लिए समर्पित राष्ट्रीय उद्यानों और अभयारण्यों में किसी भी महत्वपूर्ण वन्यजीव आवास क्षेत्र की
घोषणा नहीं की है।

QUESTION
75.
“यह एक शहर के भीतर स्थित कु छ और सबसे छोटे राष्ट्रीय उद्यानों में से एक है। दक्षिणी उष्णकटिबंधीय शुष्क सदाबहार वनों के बचे हुए
कु छ अवशेषों में से एक घास के मैदानों और झाड़ियों के साथ मोज़ेक का एक पैटर्न बनाता है। अच्छी तरह से विकसित बरगद के पेड़
आमतौर पर वुडलैंड्स पर प्रभा वी होते हैं। काले हिरण, चित्तीदार हिरण, सियार, पाम सिवेट विभिन्न आकार के खुले घास के मैदानों के लिए
अच्छी तरह से अनुकूलित हैं।

निम्नलिखित में से कौन सा राष्ट्रीय उद्यान उपरोक्त विवरण से सम्बंधित है?

a) संजय गांधी राष्ट्रीय उद्यान


b) गिंडी राष्ट्रीय उद्यान
c) माउंट हैरियट राष्ट्रीय उद्यान
d) बन्नेरघट्टा राष्ट्रीय उद्यान
Correct Answer: B
Your Answer:
Explanation

Q. 75) Solution (b)

स्पष्टीकरण:

· संजय गांधी राष्ट्रीय उद्यान मुंबई के उत्तरी भाग में स्थित यह राष्ट्रीय उद्यान अपनी जैव विविधता और प्राचीन कन्हेरी गुफाओं के लिए जाना
जाता है। यहां का सबसे प्रमुख आवास-प्रकार वन आवास है। अके ले फू ल वाले पौधों की, अनुमानतः 1300 प्रजातियाँ मौजूद हैं। यहां के
अधिकांश जंगल दक्षिणी मिश्रित-पर्णपाती जंगल के अनुरूप हैं, जिनमें टे क्टोना, अल्बिज़िया, टर्मिनलिया, होलारेना , फ़िरमियाना ,
डालबर्गिया आदि का प्रभुत्व है। जीव-जंतुओं में चित्तीदार हिरण, सांभर, बार्किंग हिरण, ब्लैक- नेप्ड खरगोश और तेंदुए का प्रभुत्व है। (इस
लिए विकल्प ए ग़लत है)

· गिंडी राष्ट्रीय उद्यान चेन्नई शहर में स्थित है। यह भारत के सबसे छोटे राष्ट्रीय उद्यानों में से एक है। यह वनस्पतियों और जीवों की विभिन्न
प्रजातियों का घर है। यह किसी शहर के भीतर स्थित कु छ राष्ट्रीय उद्यानों में से एक है। दक्षिणी उष्णकटिबंधीय शुष्क सदाबहार वनों के बचे
हुए कु छ अवशेषों में से एक राष्ट्रीय उद्यान में घास के मैदानों और झाड़ियों के साथ मोज़ेक का एक पैटर्न बनाता है, लेकिन अच्छी तरह से
विकसित बरगद के पेड़ आमतौर पर वुडलैंड्स पर हावी होते हैं। ब्लैकबक्स और स्पॉटे ड हिरण विभिन्न आकार के खुले घास के मैदानों के लिए
अच्छी तरह से अनुकूलित हैं। (इसलिए विकल्प बी सही है)

· माउंट हैरियट नेश नल पार्क दक्षिण अंडमान द्वीप के उत्तरी भाग पर स्थित है, जो अंडमान और निकोबार द्वीप समूह की राजधानी पोर्ट
ब्लेयर से सड़क मार्ग से लगभग 15 किलोमीटर दूर है। राष्ट्रीय उद्यान अपनी विविध वनस्पतियों और जीवों के लिए जाना जाता है। वन क्षेत्र
विभिन्न प्रकार की पौधों की प्रजातियों का घर है, जिनमें ऑर्किड, फ़र्न और अन्य उष्णकटिबंधीय वनस्पति शामिल हैं। पार्क में विभिन्न पक्षी
प्रजातियों का भी निवास है, जो इसे पक्षी दे खने के लिए एक गंतव्य बनाता है। (इसलिए विकल्प सी गलत है)

· बन्नेरघट्टा राष्ट्रीय उद्यान भारत का एक राष्ट्रीय उद्यान है, जो कर्नाटक के बैंगलोर के पास स्थित है। पार्क में सदाबहार प्राथमिक वन हैं, और
चिरियातापु में वन का प्रकार मिश्रित पर्णपाती है, जो प्राथमिक और द्वितीयक वनों का संयोजन है। तीन प्रकार के वनों को उष्णकटिबंधीय
सदाबहार, पहाड़ी शीर्ष उष्णकटिबंधीय सदाबहार और तटीय के रूप में वर्गीकृ त किया गया है। कु ल मिलाकर 134 पौधों और पेड़ों की
प्रजातियाँ बताई गई हैं, जिनमें 74 दे श ी और 51 प्रचलित प्रजातियाँ शामिल हैं। (इसलिए विकल्प d गलत है)

IMPORTANT POINTS/VALUE ADDITIONS:

· एन्नोर से बचाए गए छह तेल से लथपथ स्पॉट-बिल्ड पेलिकन गिंडी नेश नल पार्क में पुनर्वास के अधीन हैं और जंगल में वापस छोड़ने से पहले
उन्हें टै ग किया जाएगा।

IASbaba
Score:
Web: http://ilp.iasbaba.com/
44.00 /
Email: ilp@iasbaba.com
Page 168 200
UPSC 2024 -
Exam Title : Environment,
Sc...
rupeshkr.
Email :
09ee80@gmail.com
Contact :

· कोसस्थलैयार में तेल रिसाव के बाद, कई पक्षियों पर प्रतिकू ल प्रभाव पड़ा। भारतीय वन्यजीव ट्रस्ट, बेसेंट मेमोरियल एनिमल डिस्पेंसरी और
वन विभाग के विशेषज्ञों की एक वन्यजीव टीम रिसाव के प्रभाव का आकलन कर रही है और पाया है कि लगभग दो दर्जन पेलिकन और
पेंटेड स्टॉर्क पर भारी प्रभाव पड़ा है।

QUESTION
76.
निम्नलिखित में से कौन सी पौधे की प्रजाति आक्रामक ( invasive ) है?

1. युकलिप्टु स

2. प्रोसोपिस जूलीफ्लोरा

3. कै सिया फिस्टु ला

4. लैंटाना कै मारा

5. कोनोकार्पस

नीचे दिए गए विकल्पों में से सही कोड चुनें।

a) 2, 3 और 4
b) 2, 3, 4 और 5
c) 1, 2, 4 और 5
d) उपरोक्त सभी
Correct Answer: C
Your Answer:
Explanation

Q. 76) Solution (c)

स्पष्टीकरण:

· पश्चिमी घाट में, जहां अतीत में घास के मैदानों और शोला जंगलों को परिवर्तित करके नीलगिरी और मवेशि यों के विशाल वृक्षारोपण किए
गए थे, नीलगिरि तहर का मूल निवास स्थान तबाह हो गया है। आजकल, विदे श ी प्रजातियों के व्यापक वृक्षारोपण के कारण खाद्य पौधों की
अनुपलब्धता के कारण भारतीय बाइसन तमिलनाडु के कोडाइकनाल शहर में अक्सर आते हैं। ( इसलिए 1 आक्रामक प्रजाति है)

· कोनोकार्पस प्रजातियों के प्रबंधन पर चिंताओं के कारण हाल ही में गुजरात और, पिछले साल, तेलंगाना ने उनके उपयोग पर प्रतिबंध लगा
दिया है। (अतः 5 आक्रामक प्रजाति है)

· अगले वर्ष दे श भर में वनीकरण और भूनिर्माण परियोजनाओं में प्रजातियों को बढ़ाने और इसके लाखों पौधों का उपयोग करने से
हतोत्साहित होने की संभावना है। कोनोकार्पस के साथ-साथ ल्यूकेना और मैंगियम भी आक्रामक पौधों की प्रजातियाँ हैं।

· प्रोसोपिस जूलीफ्लोरा और लैंटाना कै मारा भी सार्वजनिक भूमि और कॉमन्स पर हावी हैं। (इसलिए 2 और 4 आक्रामक प्रजाति है )

· कै सिया फिस्टु ला फै बेसी परिवार का एक फू ल वाला पौधा है। यह प्रजाति भारतीय उपमहाद्वीप और दक्षिण पूर्व एशिया के निकटवर्ती
क्षेत्रों की मूल निवासी है। ( इसलिए 3 आक्रामक प्रजाति नहीं है)

IMPORTANT POINTS/VALUE ADDITIONS:

· वनस्पतियों की कई विदे श ी प्रजातियाँ, जो ज्यादातर अंग्रेजों द्वारा लाई गईं, मुख्य रूप से पश्चिमी घाट में काफी हद तक बढ़ गई हैं।

IASbaba
Score:
Web: http://ilp.iasbaba.com/
44.00 /
Email: ilp@iasbaba.com
Page 169 200
UPSC 2024 -
Exam Title : Environment,
Sc...
rupeshkr.
Email :
09ee80@gmail.com
Contact :

· लैंटाना कै मारा से घिरे नीलगिरी और पुलनी पहाड़ियों की ऊपरी ढलानों पर नीलगिरी, बबूल (बबूल) और पाइनस के विशाल वृक्षारोपण दे खे
जा सकते हैं।

· प्रोसोपिस जूलीफ्लोरा , पार्थेनियम हिस्टे रोफोरस और यूपेटोरियम ओडोरेटम को निचली ढलानों पर दे खा जा सकता है। ऑस्ट्रेलिया से लाई
गई अधिकांश प्रजातियाँ जैसे यूकेलिप्टस, वॉटल (बबूल) अत्यधिक आक्रामक हो गई हैं।

· कै सिया फ़िस्टु ला भारत में के रल राज्य का आधिकारिक राज्य फू ल है। यह एक लोकप्रिय सजावटी पौधा भी है और इसका उपयोग हर्बल
चिकित्सा में भी किया जाता है। दिल्ली के हेरिटे ज एरिया में कै सिया फिस्टु ला का पौधारोपण है। आम और सार्वजनिक क्षेत्र में सजावटी उद्दे श्यों
के लिए स्वदे श ी प्रजातियों का उपयोग करने की आवश्यकता है।

QUESTION
77.
निम्नलिखित कथनों पर विचार करें:

1. इकोक्लाइन ( Ecocline ) एक पारिस्थितिकी तंत्र से दूसरे पारिस्थितिकी तंत्र में क्रमिक लेकिन निरंतर परिवर्तन का क्षेत्र है।

2. इकोसाइड ( Ecocide ) रासायनिक कीटनाशकों और शाकनाशियों का पारिस्थितिक विकल्प है।

3. इकोटोन ( Ecotone ) दो बायोम के बीच जंक्शन या संक्रमण क्षेत्र का एक क्षेत्र है।

उपरोक्त में से कितने कथन सही हैं?

a) के वल एक
b) के वल दो
c) सभी तीन
d) कोई नहीं
Correct Answer: B
Your Answer:
Explanation

Q. 77) Solution (b)

स्पष्टीकरण:

· इकोक्लाइन एक पारिस्थितिकी तंत्र से दूसरे पारिस्थितिकी तंत्र में क्रमिक लेकिन निरंतर परिवर्तन का एक क्षेत्र है जब प्रजातियों की संरचना
के संदर्भ में दोनों के बीच कोई स्पष्ट सीमा नहीं होती है। (अतः कथन 1 सही है)

· पारिस्थितिकी-हत्या या 'किसी के पर्यावास को समाप्त करना ' , बंदरगाह विस्तार परियोजनाओं, वनों की कटाई, अवैध रेत खनन, नदियों
को प्रदूषित करना और अनुपचारित सीवेज छोड़ना आदि जैसे कृ त्यों को संदर्भित करता है, जो सुभेद्य प्राकृ तिक पारिस्थितिकी तंत्र और
स्थानीय आजीविका को नष्ट करते हैं। ( इसलिए कथन 2 गलत है)

· इकोटोन दो बायोम (विविध पारिस्थितिक तंत्र) के बीच जंक्शन या संक्रमण क्षेत्र का एक क्षेत्र है। (इसलिए कथन 3 सही है)

· इकोटोन वह क्षेत्र है जहां दो समुदाय मिलते हैं और एकीकृ त होते हैं। उदाहरण के लिए, मैंग्रोव वन समुद्री और स्थलीय पारिस्थितिकी तंत्र के
बीच एक इकोटोन का प्रतिनिधित्व करते हैं। अन्य उदाहरण घास के मैदान (जंगल और रेगिस्तान के बीच), मुहाना (मीठे पानी और खारे पानी
के बीच) और नदी के किनारे या दलदली भूमि (सूखे और गीले के बीच) हैं।

महत्वपूर्ण बिंदु/मूल्यवर्धन :

· इकोक्लाइन पर्यावरणीय ढाल (ऊं चाई, तापमान (थर्मोक्लाइन), लवणता (हेलोक्लाइन), गहराई इत्यादि जैसे अजैविक कारकों में क्रमिक
परिवर्तन) में होता है।

IASbaba
Score:
Web: http://ilp.iasbaba.com/
44.00 /
Email: ilp@iasbaba.com
Page 170 200
UPSC 2024 -
Exam Title : Environment,
Sc...
rupeshkr.
Email :
09ee80@gmail.com
Contact :

· इकोसाइड का अर्थ गैरकानूनी या अनियंत्रित कार्य जो इस ज्ञान के साथ किए जाते हैं कि उन कृ त्यों के कारण पर्यावरण को गंभीर और
व्यापक या दीर्घकालिक क्षति होने की पर्याप्त संभावना है।

· इकोटोन में निकटवर्ती पारिस्थितिक तंत्रों की मध्यवर्ती स्थितियाँ होती हैं। इसलिए यह तनाव का क्षेत्र है । आमतौर पर, जैसे-जैसे हम समुदाय
या पारिस्थितिकी तंत्र से दूर जाते हैं, बाहर जाने वाले समुदाय की प्रजातियों की संख्या और जनसंख्या घनत्व कम हो जाता है। एक अच्छी
तरह से विकसित इकोटोन में कु छ ऐसे जीव होते हैं जो आसपास के समुदायों से पूरी तरह से अलग होते हैं।

QUESTION
78.
निम्नलिखित राष्ट्रीय उद्यानों को उत्तर से दक्षिण क्रम में व्यवस्थित करें:

1. जिम कॉर्बेट नेश नल पार्क

2. दुधवा राष्ट्रीय उद्यान

3. गोविंद पशु विहार राष्ट्रीय उद्यान

4. राजाजी राष्ट्रीय उद्यान

नीचे से सही कोड चुनें.

a) 1, 2, 3, 4
b) 2, 3. 4. 1
c) 3, 4, 1, 2
d) 4, 1, 2, 3
Correct Answer: C
Your Answer:
Explanation

Q. 78) Solution (c)

स्पष्टीकरण:

· तीन राष्ट्रीय उद्यान उत्तराखंड के हैं और के वल एक उत्तर प्रदे श का है। यूपी के उस राष्ट्रीय उद्यान को जानकर सही उत्तर मिलता है।

· चूंकि दुधवा राष्ट्रीय उद्यान उत्तर प्रदे श में स्थित है और अन्य तीन इसके उत्तर में हैं, विकल्प सी सही है।

IMPORTANT POINTS/VALUE ADDITIONS:

· गोविंद पशु विहार राष्ट्रीय उद्यान उत्तराखंड के उत्तरकाशी जिले में स्थित है। पश्चिमी हिमालय में बसा यह स्थान अपनी विविध वनस्पतियों
और जीवों के लिए जाना जाता है। इसमें अल्पाइन घास के मैदान, ग्लेशि यर और उच्च ऊं चाई वाले परिदृश्य शामिल हैं, जो हिम तेंदुए,
हिमालयी तहर और कई पक्षी प्रजातियों जैसी प्रजातियों को आवास प्रदान करते हैं।

· राजाजी राष्ट्रीय उद्यान उत्तराखंड में शिवालिक पर्वतमाला में फै ला हुआ है और इसका नाम भारत के अंतिम गवर्नर-जनरल सी.
राजगोपालाचारी के नाम पर रखा गया है। यह एक राष्ट्रीय उद्यान और बाघ अभयारण्य है, जो हाथियों, बाघों, तेंदुओं और हिरणों की कई
प्रजातियों सहित अपनी समृद्ध जैव विविधता के लिए जाना जाता है। पार्क की विशेषता विविध परिदृश्य हैं, जिनमें नदी घाटियाँ और घने
जंगल शामिल हैं।

· जिम कॉर्बेट राष्ट्रीय उद्यान उत्तराखंड के नैनीताल जिले में स्थित है। यह भारत का सबसे पुराना राष्ट्रीय उद्यान है, जिसे 1936 में स्थापित
किया गया था, जो बंगाल के बाघों, हाथियों और विभिन्न पक्षी प्रजातियों सहित अपने विविध वन्य जीवन के लिए प्रसिद्ध है। यह हिमालय की
तलहटी में फै ला हुआ है, जिसमें नदी बेल्ट, घास के मैदान और घने जंगल शामिल हैं।

IASbaba
Score:
Web: http://ilp.iasbaba.com/
44.00 /
Email: ilp@iasbaba.com
Page 171 200
UPSC 2024 -
Exam Title : Environment,
Sc...
rupeshkr.
Email :
09ee80@gmail.com
Contact :

· दुधवा राष्ट्रीय उद्यान भारत के उत्तर प्रदे श के लखीमपुर खीरी जिले में भारत-नेपाल सीमा पर स्थित है। यह एक संरक्षित क्षेत्र है जो अपनी
दलदली हिरण आबादी के लिए जाना जाता है। दुधवा राष्ट्रीय उद्यान में घास के मैदान, घने जंगल और मनमोहक तराई पारिस्थितिकी तंत्र है।
यह पार्क बाघों, तेंदुओं और विभिन्न प्रकार की पक्षी प्रजातियों का घर है।

QUESTION
79.
IPCC की AR6 सिंथेसिस रिपोर्ट के बारे में निम्नलिखित कथनों पर विचार करें:

1. अत्यधिक संवेदनशील क्षेत्रों में 3 अरब से अधिक लोगों की जलवायु संबंधी घटनाओं से मरने की संभावना निम्न संवेदनशील क्षेत्रों की
तुलना में 15 गुना अधिक है।

2. मौजूदा अनुकूलन अंतराल सीमित संसाधनों, कम जलवायु साक्षरता और राजनीतिक प्रतिबद्धता की कमी के कारण बढ़ते रहेंगे।

3. यह जलवायु-लचीले विकास, जीवाश्म ईंधन के उपयोग को कम करने, जलवायु निवेश और कम कार्बन वाली जीवन शैली की
आवश्यकता पर जोर दे ता है।

उपरोक्त में से कौन सा कथन सटीक है?

a) 1 और 2
b) 2 और 3
c) 1 और 3
d) उपरोक्त सभी
Correct Answer: D
Your Answer:
Explanation

Q. 79) Solution (d)

स्पष्टीकरण:

· सुभेद्य समुदाय, जो जलवायु परिवर्तन के लिए ऐतिहासिक रूप से कम ज़िम्मेदार हैं, असंगत रूप से प्रभावित होते हैं। इसमें कहा गया है कि
अत्यधिक संवेदनशील क्षेत्रों में तीन अरब से अधिक लोगों की जलवायु संबंधी घटनाओं से मरने की संभावना 15 गुना अधिक है। (अतः
कथन 1 सही है।)

· ग्रीनहाउस गैस उत्सर्जन को कम करने में कु छ प्रगति के बावजूद, मौजूदा अनुकूलन अंतराल हैं। इन अंतरालों को सीमित संसाधनों, कम
जलवायु साक्षरता और राजनीतिक प्रतिबद्धता की कमी जैसे कारकों के लिए जिम्मेदार ठहराया जाता है। (अतः कथन 2 सही है।)

· इसमें जलवायु-लचीले विकास, जीवाश्म ईंधन के उपयोग को कम करने, जलवायु निवेश के लिए वित्त बढ़ाने और निम्न कार्बन वाली जीवन
शैली को सुविधाजनक बनाने की आवश्यकता पर जोर दिया गया है। (अतः कथन 3 सही है।)

IMPORTANT POINTS/VALUE ADDITIONS:

· विश्व के प्रमुख जलवायु वैज्ञानिकों के संयुक्त राष्ट्र समर्थित निकाय, इंटरगवर्नमेंटल पैनल ऑन क्लाइमेट चेंज (आईपीसीसी) ने चेतावनी दी है
कि विश्व 2030 के दशक तक 1.5 डिग्री सेल्सियस ग्लोबल वार्मिंग सीमा को तोड़ने की राह पर है।

· जैसा कि पेरिस समझौते में सहमति व्यक्त की गई थी, 1.5 डिग्री सेल्सियस की सीमा को पार करने से विभिन्न गंभीर परिणाम हो सकते हैं,
जैसे अप्रत्याशित वैश्विक जल चक्र, विनाशकारी बाढ़, चरम समुद्र स्तर की घटनाएं और अधिक तीव्र उष्णकटिबंधीय चक्रवात।

QUESTION
80.

IASbaba
Score:
Web: http://ilp.iasbaba.com/
44.00 /
Email: ilp@iasbaba.com
Page 172 200
UPSC 2024 -
Exam Title : Environment,
Sc...
rupeshkr.
Email :
09ee80@gmail.com
Contact :

इको ब्रिज ( Eco bridges ) और इको डक्ट ( Eco ducts ) हाल ही में खबरों में थे, इनके संबंध में निम्नलिखित
कथनों पर विचार करें:

1. इसका उद्दे श्य वन्यजीव कनेक्टिविटी को बढ़ाना और भू- परिदृश्य को सन्निहित रूप दे ना है।

2. उत्तराखंड में सरीसृपों और छोटे जानवरों के लिए इको ब्रिज है।

3. पेंच टाइगर रिजर्व में बाघ, तेंदुए ने इको ब्रिज का इस्तेमाल किया ।

उपरोक्त में से कौन सा कथन सही है?

a) 1 और 2
b) 2 और 3
c) 1 और 3
d) उपरोक्त सभी
Correct Answer: D
Your Answer:
Explanation

Q. 80) Solution (d)

स्पष्टीकरण:

· इको-डक्ट् स या इको-ब्रिज का उद्दे श्य वन्यजीव कनेक्टिविटी को बढ़ाना है जो राजमार्गों या लॉगिंग के कारण बाधित हो सकती है। (अतः
कथन 1 सही है)

· आम तौर पर इन ब्रिजों / पुलों को परिदृश्य के साथ एक सन्निहित रूप दे ने के लिए क्षेत्र से पौधे लगाए जाते हैं।

· रामनगर वन प्रभाग ने हाल ही में सरीसृपों और छोटे स्तनधारियों के लिए अपना पहला इको- ब्रिज बनाया है। (अतः कथन 2 सही है)

· भारतीय वन्यजीव संस्थान (डब्ल्यूआईआई) के 2020 के एक अध्ययन में कहा गया है कि अगले पांच से छह वर्षों में भारत में निर्माण के
लिए लगभग 50,000 किलोमीटर सड़क परियोजनाओं की पहचान की गई है, जबकि कई राजमार्गों को चार लेन में अपग्रेड किया जा रहा
है।

· राष्ट्रीय बाघ संरक्षण प्राधिकरण, नई दिल्ली ने तीन प्रमुख स्थलों की पहचान की थी जो पशु गलियारों को काट रहे थे, जिनमें असम में
काजीरंगा -कार्बी आंगलोंग परिदृश्य के माध्यम से राष्ट्रीय राजमार्ग 37 और कर्नाटक में नागरहोल टाइगर रिजर्व के माध्यम से राज्य राजमार्ग
33 शामिल थे।

· कै मरों ने इन अंडरपासों का उपयोग करने वाली लगभग 18 प्रजातियों को कै द किया, जिनमें बाघ, तेंदुआ और गोल्डन सियार शामिल थे।
(अतः कथन 3 सही है)

IMPORTANT POINTS/VALUE ADDITIONS:

· पेंच में 750 मीटर लंबा पुल है जो संभवतः विश्व का सबसे बड़ा अंडरपास है। अधिकांश प्रजातियाँ इसका उपयोग करती हैं। 750 मीटर के
इस पुल को पार करने में स्लॉथ भालू को दो साल लगे, भेड़िया और पैंगोलिन को एक साल से भी कम समय लगा, जबकि चित्तीदार हिरण
और जंगली बिल्ली को मुश्किल से एक महीना लगा।

· भारत में पशु संरक्षण के लिए सबसे बड़े अंडरपासों में से एक - 1.4 किमी - मध्य प्रदे श -महाराष्ट्र सीमा पर बनाया जा रहा है।

QUESTION
81.
'होप स्पॉट नेटवर्क ( Hope Spot Network ) ' के बारे में निम्नलिखित कथनों पर विचार करें:

IASbaba
Score:
Web: http://ilp.iasbaba.com/
44.00 /
Email: ilp@iasbaba.com
Page 173 200
UPSC 2024 -
Exam Title : Environment,
Sc...
rupeshkr.
Email :
09ee80@gmail.com
Contact :

1. यह कं जर्वेशन इंटरनेश नल और IUCN की एक संयुक्त पहल है।

2. एक होप स्पॉट उच्च प्रजाति समृद्धि वाला क्षेत्र है जिसमें संवहनी पौधों की कम से कम 1500 प्रजातियाँ और स्थलीय क्षेत्र में उच्च स्तर
की प्रजाति स्थानिकता है।

3. होप स्पॉट के वल समुद्री संरक्षित क्षेत्र (एमपीए) हो सकते हैं।

उपरोक्त में से कितने कथन सही हैं?

a) के वल एक
b) के वल दो
c) सभी तीन
d) कोई नहीं
Correct Answer: D
Your Answer:
Explanation

Q. 81) Solution (d)

स्पष्टीकरण:

· होप स्पॉट नेटवर्क मिशन ब्लू और IUCN की एक संयुक्त पहल है। (इसलिए कथन 1 गलत है)

· होप स्पॉट महासागर का एक क्षेत्र है जिसे अपने वन्य जीवन और महत्वपूर्ण पानी के नीचे के आवासों के कारण विशेष सुरक्षा की
आवश्यकता है। उन्हें जैव विविधता, कार्बन सिंक और महत्वपूर्ण आवास में उनके योगदान के लिए चुना गया है। (इसलिए कथन 2 गलत
है)

· वे समुद्री संरक्षित क्षेत्र (एमपीए - आईयूसीएन द्वारा परिभाषित) हो सकते हैं जिन पर ध्यान दे ने की आवश्यकता है या कोई नई साइट हो
सकती है । (इसलिए कथन 3 गलत है)

IMPORTANT POINTS/VALUE ADDITIONS:

· होप स्पॉट हमें भविष्य के लिए योजना बनाने और वर्तमान समुद्री संरक्षित क्षेत्रों (एमपीए) से परे दे खने की अनुमति दे ते हैं, जो भूमि पर
राष्ट्रीय उद्यानों की तरह हैं जहां मछली पकड़ने और गहरे समुद्र में खनन जैसे दोहन कारी उपयोग प्रतिबंधित हैं।

· होप स्पॉट अक्सर ऐसे क्षेत्र होते हैं जिन्हें नई सुरक्षा की आवश्यकता होती है, लेकिन वे मौजूदा एमपीए भी हो सकते हैं जहां अधिक कार्रवाई
की आवश्यकता होती है। वे बड़े हो सकते हैं, वे छोटे हो सकते हैं, लेकिन वे सभी निम्न कारणों से आशा प्रदान करते हैं: प्रजातियों की एक
विशेष बहुतायत या विविधता, असामान्य या प्रतिनिधि प्रजातियाँ, आवास या पारिस्थितिक तंत्र, दुर्लभ, संकटग्रस्त या स्थानिक प्रजातियों की
विशेष आबादी, संभावित क्षमता वाली साइट नकारात्मक मानवीय प्रभावों से विपरीत क्षति, प्रमुख प्रवास गलियारे या प्रजनन स्थल जैसी
प्राकृ तिक प्रक्रियाओं की उपस्थिति, महत्वपूर्ण ऐतिहासिक, सांस्कृ तिक या आध्यात्मिक मूल्य, समुदाय के लिए विशेष आर्थिक महत्व ।

· विचार यह है कि कोई भी व्यक्ति अपनी विशेष साइट को नामांकित कर सकता है—एक ऐसी साइट जो आशा दे ती है। सामूहिक रूप से ये
सभी होप स्पॉट समुद्र संरक्षण के लिए सामुदायिक समर्थन की एक वैश्विक लहर पैदा करेंगे जिसे नेता और नीति निर्माता नजरअंदाज नहीं कर
सकते। अधिक होप स्पॉट बनाने और हमारे मौजूदा होप स्पॉट का समर्थन करने से हम 2030 तक 30% महासागरों की रक्षा के अपने लक्ष्य
को प्राप्त करने के करीब पहुंच सकते हैं।

QUESTION
82.
'फ्लोरिअनोपोलिस घोषणा ( Florianopolis Declaration ) ' निम्नलिखित में से किससे संबंधित है?

a) एक आवश्यक आर्थिक गतिविधि के रूप में वाणिज्यिक व्हेलिंग को बढ़ावा दे ना

IASbaba
Score:
Web: http://ilp.iasbaba.com/
44.00 /
Email: ilp@iasbaba.com
Page 174 200
UPSC 2024 -
Exam Title : Environment,
Sc...
rupeshkr.
Email :
09ee80@gmail.com
Contact :

b) विश्व की व्हेल आबादी की अनिश्चितकालीन सुरक्षा की अनुमति दे ना


c) व्हेल समर्थक राष्ट्रों को अपनी गतिविधियाँ बढ़ाने के लिए प्रोत्साहित करना
d) वाणिज्यिक व्हेलिंग गतिविधियों पर बाध्यकारी नियम प्रदान करना
Correct Answer: B
Your Answer:
Explanation

Q. 82) Solution (b)

स्पष्टीकरण:

· 2018 की फ्लोरिअनोपोलिस घोषणा इस बात की पुष्टि करती है कि वाणिज्यिक व्हेलिंग पर रोक, जो 1986 से प्रभावी है, ने कु छ व्हेल
आबादी की वसूली में योगदान दिया है, और व्हेल आबादी के लिए कई जैसे , पानी के भीतर शोर, जहाज़ हमले, समुद्री मलबा और जलवायु
परिवर्तन, मौजूदा और उभरते खतरों जैसे उलझाव, बायकै च के संचयी प्रभावों के बारे में जागरूक किया है। (
इसलिए विकल्प बी सही है)

· यह इस बात से सहमत है कि 21वीं सदी में अंतर्राष्ट्रीय व्हेलिंग आयोग की भूमिका में अन्य बातों के साथ-साथ व्हेल आबादी को उनके पूर्व-
औद्योगिक स्तरों पर वापस लाने को सुनिश्चित करने की जिम्मेदारी भी शामिल है, और इस संदर्भ में वाणिज्यिक व्हेलिंग पर रोक बनाए रखने
के महत्व की पुष्टि करता है।

IMPORTANT POINTS/VALUE ADDITIONS:

· समकालीन गैर-घातक सिटासियन ( non-lethal cetacean ) अनुसंधान विधियों की प्रचुरता के अस्तित्व को स्वीकार करता है और
इसलिए इस बात से सहमत है कि घातक अनुसंधान विधियों का उपयोग अनावश्यक है; यह सुनिश्चित करने का प्रयास किया गया है कि
स्वदे श ी समुदायों के लाभ के लिए आदिवासी निर्वाह व्हेलिंग को शिकारियों की सुरक्षा और सीतासियों के कल्याण को ध्यान में रखते हुए
आयोग के प्रबंधन और संरक्षण उद्दे श्यों को पूरा करना चाहिए।

· इस घोषणा को अपनाने के बाद जापान ने IWC छोड़ दिया और वाणिज्यिक व्हेलिंग फिर से शुरू कर दी।

QUESTION
83.
भारतीय वन अधिनियम 1927 के संदर्भ में निम्नलिखित कथनों पर विचार करें:

1. संरक्षित वनों में अनुमति न होने तक सभी गतिविधियाँ प्रतिबंधित हैं

2. सभी गतिविधियों की अनुमति है जब तक कि यह आरक्षित वनों में निषिद्ध न हो

3. राज्य सरकार किसी भी ग्राम समुदाय को आरक्षित वन के रूप में गठित किसी भी भूमि पर सरकारी अधिकार सौंप सकती है।

उपरोक्त में से कौन सा कथन गलत है?

a) 1 और 2
b) 2 और 3
c) 1 और 3
d) उपरोक्त सभी
Correct Answer: A
Your Answer:
Explanation

Q. 83) Solution (a)

स्पष्टीकरण:

IASbaba
Score:
Web: http://ilp.iasbaba.com/
44.00 /
Email: ilp@iasbaba.com
Page 175 200
UPSC 2024 -
Exam Title : Environment,
Sc...
rupeshkr.
Email :
09ee80@gmail.com
Contact :

· राज्य सरकार किसी भी वन भूमि या बंजर भूमि को आरक्षित वन के रूप में गठित कर सकती है , जो सरकार की संपत्ति है , इन वनों से
उपज भी बेच सकती है।

· आधिकारिक अधिसूचना जारी करने से पहले राज्य सरकार को उस क्षेत्र से संबंधित व्यक्तियों की जांच करने और उन्हें निपटान प्रदान करने
के लिए एक वन निपटान अधिकारी (सिविल न्यायालयों की शक्ति है) नियुक्त करने की आवश्यकता है।

· आरक्षित वनों में अनुमति न होने तक सभी गतिविधियाँ प्रतिबंधित हैं। (इसलिए कथन 1 ग़लत है)

· आरक्षित वन में शामिल नहीं की गई किसी भी भूमि को संरक्षित वन माना जाता है। संरक्षित वन 2 प्रकार के होते हैं: सीमांकित संरक्षित वन
और असीमांकित संरक्षित वन

· संरक्षित वनों में, सभी गतिविधियों की अनुमति है जब तक कि यह निषिद्ध न हो। (इसलिए कथन 2 गलत है)

· राज्य सरकार किसी भी ग्राम समुदाय को आरक्षित वन के रूप में गठित किसी भी भूमि पर सरकारी अधिकार (या उस पर) सौंप सकती है।
(अतः कथन 3 सही है)

· राज्य सरकार ग्राम वन के प्रबंधन को विनियमित करने के लिए नियम बना सकती है, जिसमें ऐसी शर्तें निर्धारित की जा सकती हैं जिनके
तहत समुदाय को लकड़ी या अन्य वन उपज या चारागाह प्रदान किया जा सकता है।

IMPORTANT POINTS/VALUE ADDITIONS:

· 1927 का भारतीय वन अधिनियम, 1878 के अपने पूर्ववर्ती से काफी हद तक प्रभावित, लकड़ी से संबंधित शुल्कों को रेखांकित करते
हुए, वन संरक्षण और प्रबंधन के लिए एक कानूनी ढांचे के रूप में कार्य करता है।

· यह क्षेत्रों को आरक्षित, संरक्षित या ग्राम वन के रूप में नामित करने की प्रक्रियाओं को चित्रित करता है, जिनमें से प्रत्येक में सुरक्षा की
अलग-अलग डिग्री होती है।

· 2017 के संश ोधन ने बांस को पेड़ की परिभाषा से हटा दिया, जिससे गैर-वन क्षेत्रों में अनुमति के बिना इसके आर्थिक उपयोग की अनुमति
मिल गई।

QUESTION
84.
'एज प्रजाति ( EDGE Species ) ' के बारे में निम्नलिखित कथनों पर विचार करें:

1. EDGE प्रजाति की वंश ावली ( tree of life ) पर बहुत कम या कोई करीबी रिश्तेदार नहीं होते हैं।

2. EDGE प्रजातियां आमतौर पर अपने दिखने, रहने और व्यवहार करने के तरीके के साथ-साथ अपनी आनुवंशि क संरचना में भी बेहद
अलग होती हैं।

उपरोक्त में से कौन सा कथन सही है?

a) के वल 1
b) के वल 2
c) दोनों
d) कोई भी नहीं
Correct Answer: C
Your Answer:
Explanation

Q. 84) Solution (c)

स्पष्टीकरण:

IASbaba
Score:
Web: http://ilp.iasbaba.com/
44.00 /
Email: ilp@iasbaba.com
Page 176 200
UPSC 2024 -
Exam Title : Environment,
Sc...
rupeshkr.
Email :
09ee80@gmail.com
Contact :

· EDGE प्रजाति की वंश ावली ( tree of life ) पर बहुत कम या कोई करीबी रिश्तेदार नहीं होते हैं। (अतः कथन 1 सही है)

· EDGE प्रजातियाँ आमतौर पर अपने दिखने, रहने और व्यवहार के साथ-साथ अपनी आनुवंशि क संरचना में बेहद अलग होती हैं। (अतः
कथन 2 सही है)

महत्वपूर्ण बिन्दु/मूल्यवर्धन

· EDGE of Existence कार्यक्रम एक वैश्विक संरक्षण पहल है जो विशेष रूप से संकटग्रस्त प्रजातियों पर ध्यान कें द्रित करती है जो
अद्वितीय विकासवादी इतिहास की एक महत्वपूर्ण मात्रा का प्रतिनिधित्व करती हैं।

· यदि वे गायब हो गए, तो ग्रह पर उनके जैसा कु छ भी नहीं बचेगा।

QUESTION
85.
'डार्क स्काई पार्क ( Dark Sky Park ) ' के संदर्भ में निम्नलिखित कथनों पर विचार करें:

1. पेंच टाइगर रिजर्व भारत का पहला डार्क स्काई पार्क बन गया।

2. यह मुख्य रूप से रात के आकाश को प्रकाश प्रदूषण से बचाने की आवश्यकता से प्रेरित है।

3. इसे जिला योजना समुदाय (डीपीसी) द्वारा वित्त पोषित किया जाता है, जो गहन तारा-दर्शन के लिए क्षेत्र निर्दिष्ट करता है।

उपरोक्त में से कौन सा कथन सही है?

a) 1 और 2
b) 2 और 3
c) 1 और 3
d) उपरोक्त सभी
Correct Answer: D
Your Answer:
Explanation

Q. 85) Solution (d)

स्पष्टीकरण:

· पेंच टाइगर रिजर्व (पीटीआर) ने भारत का पहला और एशिया में पांचवां डार्क स्काई पार्क बनने का गौरव प्राप्त किया है। (अतः कथन 1
सही है)

· पेंच
को पहले डार्क स्काई पार्क के रूप में नामित करने का निर्णय रात के आकाश को प्रकाश प्रदूषण से बचाने की आवश्यकता से प्रेरित है। (अ
तः कथन 2 सही है)

· भारत के पहले और एशिया के आठवें डार्क स्काई पार्क के रूप में, पेंच टाइगर रिज़र्व एक अछू ते रात के आकाश को संरक्षित करने की
अपनी प्रतिबद्धता को रेखांकित करता है।

· जिला योजना समुदाय (डीपीसी) द्वारा वित्त पोषित, हाल ही में उद्घाटन की गई रात्रि आकाश वेधशाला, बाघोली के पास के क्षेत्र को गहन
तारों को दे खने के लिए नामित करती है। (अतः कथन 3 सही है)

· डार्क -स्काई प्रिजर्व एक ऐसा क्षेत्र है, जो आमतौर पर एक पार्क या वेधशाला के आसपास होता है, जो कृ त्रिम प्रकाश प्रदूषण को सीमित
करता है।

IASbaba
Score:
Web: http://ilp.iasbaba.com/
44.00 /
Email: ilp@iasbaba.com
Page 177 200
UPSC 2024 -
Exam Title : Environment,
Sc...
rupeshkr.
Email :
09ee80@gmail.com
Contact :

IMPORTANT POINTS/VALUE ADDITIONS:

· डार्क -स्काई आंदोलन का प्राथमिक उद्दे श्य आम तौर पर खगोल विज्ञान का समर्थन करना है।

· पहल के हिस्से के रूप में, प्रकाश प्रदूषण को कम करने के लिए बफर क्षेत्र के वाघोली , सिलारी , पिपरिया और खापा गांवों में 100 से
अधिक स्ट्रीट और सामुदायिक लाइटों को जमीन की ओर लगी लाइटों से बदल दिया गया है।

· भारत का पहला डार्क -स्काई संरक्षण भारतीय खगोलीय वेधशाला (आईएओ) है, जो हानले में स्थित एक उच्च ऊं चाई वाला खगोल विज्ञान
स्टे श न है और भारतीय खगोल भौतिकी संस्थान द्वारा संचालित है।

QUESTION
86.
बेसल कन्वेंश न में 'बेसल प्रतिबंध संश ोधन' के बारे में निम्नलिखित कथनों पर विचार करें:

1. इसे कानून बनने के लिए कन्वेंश न के 3/4 पक्षों द्वारा अनुसमर्थन की आवश्यकता थी और क्रोएशिया के अनुसमर्थन के बाद लागू हुआ ।

2. संश ोधन पुनर्चक्रण को छोड़कर विकसित दे श ों की सूची से विकासशील दे श ों में इलेक्ट्रॉनिक कचरे सहित खतरनाक कचरे के निर्यात पर
प्रतिबंध लगाता है।

3. भारत ने बेसल प्रतिबंध संश ोधन को मंजूरी दे दी है।

उपरोक्त में से कितने कथन सही हैं?

a) के वल एक
b) के वल दो
c) सभी तीन
d) कोई नहीं
Correct Answer: A
Your Answer:
Explanation

Q. 86) Solution (a)

स्पष्टीकरण:

· बेसल प्रतिबंध संश ोधन को कानून बनने के लिए कन्वेंश न के 3/4 पक्षों द्वारा अनुसमर्थन की आवश्यकता थी। क्रोएशिया द्वारा इसकी पुष्टि
के बाद सितंबर 2019 में यह अंततः एक अंतरराष्ट्रीय कानून बन गया। (अतः कथन 1 सही है)

· संश ोधन विकसित (ज्यादातर ओईसीडी) दे श ों की सूची से विकासशील दे श ों में खतरनाक कचरे (इलेक्ट्रॉनिक कचरे सहित) के निर्यात पर
प्रतिबंध लगाता है। बेसल प्रतिबंध रीसाइक्लिंग सहित किसी भी कारण से निर्यात पर लागू होता है। (इसलिए कथन 2 गलत है)

· यह कन्वेंश न में एक नया अनुच् छे द बन जाएगा और 90 दिनों के बाद 97 दे श ों में लागू हो जाएगा। हालाँकि, अमेरिका, कनाडा, जापान,
ऑस्ट्रेलिया, - न्यूजीलैंड, दक्षिण कोरिया, रूस, भारत, ब्राजील और मैक्सिको जैसे दे श ों ने अभी तक प्रतिबंध की पुष्टि नहीं की है। (इसलिए
कथन 3 गलत है)

IMPORTANT POINTS/VALUE ADDITIONS:

· कई लोगों का मानना था कि पुनर्चक्रण के लिए निर्यात सहित खतरनाक कचरे के शिपमेंट पर पूर्ण प्रतिबंध की आवश्यकता है। इसके
परिणामस्वरूप 1995 में बेसल कन्वेंश न में एक संश ोधन को अपनाया गया जिसे बेसल प्रतिबंध संश ोधन कहा गया।

· बेसल प्रतिबंध संश ोधन को वैश्विक पर्यावरण न्याय के लिए एक ऐतिहासिक समझौते के रूप में दे खा जाता है।

IASbaba
Score:
Web: http://ilp.iasbaba.com/
44.00 /
Email: ilp@iasbaba.com
Page 178 200
UPSC 2024 -
Exam Title : Environment,
Sc...
rupeshkr.
Email :
09ee80@gmail.com
Contact :

· अमेरिका प्रति व्यक्ति सबसे अधिक अपशिष्ट पैदा करता है लेकिन उसने प्रतिबंध संश ोधन का सक्रिय रूप से विरोध किया है।

· 2021 के बजट भाषण में, कें द्रीय वित्त मंत्री ने 2024 तक जहाज रीसाइक्लिंग क्षमता को दोगुना करने और यूरोप और जापान से भारत में
अधिक जहाजों को आकर्षित करने की बात कही।

QUESTION
87.
निम्नलिखित में से कौन सी समिति पश्चिमी घाट के 'इको सेंसिटिव ज़ोन (ESZ)' की सुरक्षा से संबंधित है?

1. माधव गाडगिल समिति

2. के . कस्तूरीरंगन समिति

3. शैलेश नायक समिति

सही कोड चुनें.

a) 1 और 2
b) 2 और 3
c) 1 और 3
d) उपरोक्त सभी
Correct Answer: A
Your Answer:
Explanation

Q. 87) Solution (a)

स्पष्टीकरण:

· पश्चिमी घाटों की सुरक्षा और संरक्षण के लिए, MoEFCC ने प्रोफेसर माधव गाडगिल की अध्यक्षता में पश्चिमी घाट पारिस्थितिकी विशेषज्ञ
पैनल (WGEEP) नामक दो समितियों का गठन किया था। (इसलिए 1 सही है)

· डब्ल्यूजीईईपी की सिफारिशों की समीक्षा के लिए डॉ. के . कस्तूरीरंगन की अध्यक्षता में उच्च स्तरीय कार्य समूह (एचएलडब्ल्यूजी) का गठन
किया गया । (इसलिए दूसरा सही है)

· HLWG की रिपोर्ट के आधार पर, MoEFCC ने एक मसौदा अधिसूचना जारी की है, जिसमें जैविक और सांस्कृ तिक रूप से विविध
क्षेत्रों को पर्यावरण-संवेदनशील क्षेत्र घोषित किया गया है, जिसके लिए सुरक्षा और संरक्षण की आवश्यकता है।

· पश्चिमी घाट पर्यावरण-संवेदनशील क्षेत्र के मसौदे को 6 जुलाई, 2022 को पांचवीं बार फिर से अधिसूचित किया गया था और अधिसूचना
को अंतिम रूप दे ने के लिए राज्य सरकारों के साथ परामर्श में सहायता के लिए विशेष रूप से एक समिति भी गठित की गई है। ड्राफ्ट
नोटिफिके शन की वैधता 30 जून 2024 तक है ।

· शैलेश नायक समिति पश्चिमी घाट से संबंधित नहीं थी और तटीय विनियमन क्षेत्रों की समीक्षा के लिए नियुक्त की गई थी। (इसलिए तीसरा
गलत है)

महत्वपूर्ण बिंदु/मूल्यवर्धन :

· पर्यावरण, वन और जलवायु परिवर्तन मंत्रालय (MoEFCC) पश्चिमी घाट राज्यों सहित विभिन्न राज्यों और कें द्र शासित प्रदे श ों में वन,
वन्यजीव और पर्यावरण के संरक्षण के लिए कई योजनाएं लागू कर रहा है।

IASbaba
Score:
Web: http://ilp.iasbaba.com/
44.00 /
Email: ilp@iasbaba.com
Page 179 200
UPSC 2024 -
Exam Title : Environment,
Sc...
rupeshkr.
Email :
09ee80@gmail.com
Contact :

· प्रतिपूरक वनीकरण प्रबंधन और योजना प्राधिकरण के तहत धनराशि का उपयोग पश्चिमी घाट में वन और वन्यजीवों के संरक्षण के लिए भी
किया जाता है।

QUESTION
88.
'डार्क किण्वन ( Dark Fermentation ) ' शब्द का क्या अर्थ है?

a) यह एक ऐसी प्रक्रिया है जिसका उद्दे श्य वायुमंडलीय CO2 स्तर को कम करना है।
b) यह अपशिष्ट जल से हाइड्रोजन ईंधन उत्पन्न करने की एक तकनीक है।
c) यह परमाणु कचरे के निपटान की एक विधि है।
d) यह जैविक कचरे से मीथेन उत्पादन की एक प्रक्रिया है।
Correct Answer: B
Your Answer:
Explanation

Q. 88) Solution (b)

स्पष्टीकरण:

· डार्क किण्वन ( Dark fermentation ) कार्बनिक सब्सट्रेट का जैव-हाइड्रोजन में किण्वक रूपांतरण है।

· यह बैक्टीरिया के विभिन्न समूहों द्वारा प्रकट एक जटिल प्रक्रिया है, जिसमें अवायवीय रूपांतरण के समान तीन चरणों का उपयोग करके
जैव रासायनिक प्रतिक्रियाओं की एक श्रृंखला शामिल है।

· डार्क किण्वन प्रक्रिया में अपशिष्ट जल का उपयोग जैव-हाइड्रोजन उत्पादन के लिए संभावित सब्सट्रेट के रूप में किया जाता है।

IMPORTANT POINTS/VALUE ADDITIONS:

· डार्क किण्वन प्रकाश और ऑक्सीजन की अनुपस्थिति में बाध्य अवायवीय और ऐच्छिक अवायवीय द्वारा किया जाता है।

· डार्क किण्वन में, बैक्टीरिया सब्सट्रेट पर कार्य करते हैं और हाइड्रोजन उत्पन्न करते हैं। डार्क किण्वन के लिए सब्सट्रेट लिग्नोसेल्यूलोसिक
बायोमास, कार्बोहाइड्रेट सामग्री जैसे उद्योग से अपशिष्ट जल, चीनी युक्त फसल अवशेष और नगरपालिका ठोस अपशिष्ट है।

· पहले चरण में, बायोमास का पूर्व-उपचार डार्क किण्वन की दक्षता को बहुत प्रभावित करता है। दक्षता को प्रभावित करने वाले अन्य
पैरामीटर शामिल सूक्ष्मजीव और सब्सट्रेट की चीनी सामग्री हैं।

QUESTION
89.
प्रवाल / मूंगे ( Corals ) के बारे में निम्नलिखित कथनों पर विचार करें:

1. प्रवाल अलैंगिक या लैंगिक रूप से प्रजनन कर सकते हैं।

2. काले प्रवाल मांसाहारी होते हैं।

3. भित्ति निर्माणकारी / रीफ बिल्डिंग प्रवाल महासागरों के उथले और गहरे पानी दोनों में पाए जा सकते हैं।

उपरोक्त में से कितने कथन सही हैं?

a) के वल एक
b) के वल दो
c) सभी तीन

IASbaba
Score:
Web: http://ilp.iasbaba.com/
44.00 /
Email: ilp@iasbaba.com
Page 180 200
UPSC 2024 -
Exam Title : Environment,
Sc...
rupeshkr.
Email :
09ee80@gmail.com
Contact :

d) कोई नहीं
Correct Answer: B
Your Answer:
Explanation

Q. 89) Solution (b)

स्पष्टीकरण:

· प्रवालों / मूंगों की कई प्रजनन रणनीतियाँ होती हैं - वे नर या मादा या दोनों हो सकते हैं, और अलैंगिक या लैंगिक रूप से प्रजनन कर सकते
हैं।

· कॉलोनी के आकार को बढ़ाने के लिए अलैंगिक प्रजनन महत्वपूर्ण है, और यौन प्रजनन से आनुवंशि क विविधता बढ़ती है और नई
कॉलोनियाँ शुरू होती हैं जो माता-पिता से दूर हो सकती हैं। (इसलिए कथन 1 सही है)

· काले मूंगे मांसाहारी होते हैं। काले मूंगे उथले पानी में और 26,000 फीट (8,000 मीटर) से अधिक की गहराई तक उगते हुए पाए जा
सकते हैं, और कु छ व्यक्तिगत मूंगे 4,000 से अधिक वर्षों तक जीवित रह सकते हैं। (अतः कथन 2 सही है)

· काले मूंगे फिल्टर फीडर होते हैं और छोटे ज़ोप्लांकटन को खाते हैं जो गहरे पानी में प्रचुर मात्रा में होते हैं। जबकि रंगीन उथले पानी के मूंगे
ऊर्जा के लिए सूर्य और प्रकाश संश्लेषण पर निर्भर हैं।

· मूंगे विश्व भर के महासागरों में उथले और गहरे पानी दोनों में पाए जाते हैं, लेकिन चट्टान बनाने वाले मूंगे के वल उथले उष्णकटिबंधीय और
उपोष्णकटिबंधीय पानी में पाए जाते हैं। (इसलिए कथन 2 गलत है)

· ऐसा इसलिए है क्योंकि उनके ऊतकों में पाए जाने वाले शैवाल को प्रकाश संश्लेषण के लिए प्रकाश की आवश्यकता होती है और वे
70-85°F (22-29°C) के बीच पानी का तापमान पसंद करते हैं।

IMPORTANT POINTS/VALUE ADDITIONS:

· हाल ही में वैज्ञानिकों ने एक रिमोट-नियंत्रित पनडु ब्बी का उपयोग करके ऑस्ट्रेलिया के तट पर ग्रेट बैरियर रीफ और कोरल सागर में सतह से
2,500 फीट (760 मीटर) नीचे रहने वाले काले मूंगों की पांच नई प्रजातियों की खोज की।

· काले मूंगे या एंटीपैथेरियन औपनिवेशि क जीव हैं जो समुद्री एनीमोन और पथरीले मूंगों से संबंधित हैं। इनका नाम उनके कठोर, काले या भूरे
कं काल के रंग के कारण रखा गया है। काले मूंगों की 150 से अधिक प्रजातियों का वर्णन किया गया है। वर्तमान में हवाई से काले मूंगों की
कम से कम 14 प्रजातियाँ ज्ञात हैं।

· काले मूंगे सभी महासागरों में पाए जाते हैं, लेकिन उष्णकटिबंधीय और उपोष्णकटिबंधीय समुद्रों के गहरे पानी के आवासों में सबसे आम हैं।

QUESTION
90.
'क्रिस्टलीय सिलिकॉन प्रौद्योगिकी ( Crystalline Silicon technology ) ' के बारे में निम्नलिखित कथनों पर विचार करें:

1. यह मुख्य रूप से ऊर्जा रूपांतरण के लिए कार्बनिक पदार्थों का उपयोग करता है।

2. इसकी दक्षता के कारण यह बड़े पैमाने पर अनुप्रयोगों तक ही सीमित है।

3. इसका उपयोग विशेष रूप से पवन ऊर्जा प्रणालियों में किया जाता है।

उपरोक्त में से कितने कथन सही हैं?

a) के वल एक
b) के वल दो
c) सभी तीन

IASbaba
Score:
Web: http://ilp.iasbaba.com/
44.00 /
Email: ilp@iasbaba.com
Page 181 200
UPSC 2024 -
Exam Title : Environment,
Sc...
rupeshkr.
Email :
09ee80@gmail.com
Contact :

d) कोई नहीं
Correct Answer: D
Your Answer:
Explanation

Q. 90) Solution (d)

स्पष्टीकरण:

· क्रिस्टलीय सिलिकॉन तकनीक मुख्य रूप से क्रिस्टलीय अर्धचालक सामग्रियों का उपयोग करती है, न कि कार्बनिक पॉलिमर का। (इसलिए
कथन 1 गलत है)

· यह छोटे पैमाने के अनुप्रयोगों तक सीमित नहीं है; इसका व्यापक रूप से छोटे पैमाने और बड़े पैमाने की सौर ऊर्जा प्रणालियों दोनों में
उपयोग किया जाता है। (इसलिए कथन 2 गलत है)

· क्रिस्टलीय सिलिकॉन सौर सेल क्रिस्टलीय संरचना वाले अर्धचालक पदार्थों से बने होते हैं। दो मुख्य प्रकार हैं: मोनोक्रिस्टलाइन सिलिकॉन
और पॉलीक्रिस्टलाइन सिलिकॉन।

· इसका उपयोग फोटोवोल्टिक (पीवी) प्रणालियों और कें द्रित सौर ऊर्जा प्रणालियों दोनों में किया जाता है। (इसलिए कथन 3 गलत है)

IMPORTANT POINTS/VALUE ADDITIONS:

· अंतर्राष्ट्रीय सौर गठबंधन की- विश्व सौर प्रौद्योगिकी रिपोर्ट 2023 सौर फोटोवोल्टिक्स (सौर पीवी) पर कें द्रित है, जो क्रिस्टलीय सिलिकॉन
प्रौद्योगिकी में उल्लेखनीय प्रगति पर प्रकाश डालती है।

· सौर पीवी ने असाधारण वृद्धि का अनुभव किया है, 2050 तक कु ल नवीकरणीय ऊर्जा हिस्सेदारी का 56.4% हासिल करने की उम्मीद
है।

· 98% हिस्सेदारी के साथ क्रिस्टलीय सिलिकॉन प्रौद्योगिकी बाजार पर हावी है; विशेष रूप से, मोनोक्रिस्टलाइन और ऑर्गेनिक पीवी और
पेरोव्स्काइट पीवी जैसी उभरती प्रौद्योगिकियां भविष्य के लिए आशाजनक हैं।

QUESTION
91.
'समुद्री शैवाल ( Seaweeds ) ' के बारे में निम्नलिखित कथनों पर विचार करें:

1. समुद्री शैवाल विशिष्ट एंजियोस्पर्म (समुद्री फू ल वाले पौधे) हैं।

2. समुद्री शैवाल खारे पानी में जीवन के लिए अनुकूलित उच्च पौधों का एकमात्र समूह है

3. समुद्री शैवाल का उपयोग गण्डमाला ( goiter ) के उपचार, आंतों और पेट संबंधी विकारों के लिए दवा के रूप में किया जाता है।

4. अगर-अगर ( agar-agar ) और एल्गिनेट्स, आयोडीन जैसे उत्पाद जो व्यावसायिक मूल्य के हैं, समुद्री शैवाल से निकाले जाते हैं।

उपरोक्त में से कितने कथन सही हैं?

a) के वल एक
b) के वल दो
c) के वल तीन
d) सभी चार
Correct Answer: B
Your Answer: Unanswered
Explanation

IASbaba
Score:
Web: http://ilp.iasbaba.com/
44.00 /
Email: ilp@iasbaba.com
Page 182 200
UPSC 2024 -
Exam Title : Environment,
Sc...
rupeshkr.
Email :
09ee80@gmail.com
Contact :

Q. 91) Solution (b)

स्पष्टीकरण:

· समुद्री घास विशेष एंजियोस्पर्म (समुद्री फू ल वाले पौधे) हैं जो दिखने में घास के समान होते हैं। वे फू ल पैदा करते हैं, उनमें पट्टा जैसी या
अंडाकार पत्तियां और एक जड़ प्रणाली होती है, जबकि समुद्री शैवाल (थैलॉइड पौधे) मैक्रोस्कोपिक शैवाल होते हैं, जिसका अर्थ है कि उनमें
जड़ों, तनों और पत्तियों जैसे वास्तविक ऊतकों का कोई भेदभाव नहीं होता है। (इसलिए कथन 1 गलत है)

· समुद्री घासें रेतीले या कीचड़युक्त तल वाले उथले तटीय जल में उगती हैं और उन्हें अपेक्षाकृ त शांत क्षेत्रों की आवश्यकता होती है।

· समुद्री घास खारे पानी में जीवन के लिए अनुकूलित उच्च पौधों का एकमात्र समूह है। (इसलिए कथन 2 गलत है)

· समुद्री शैवाल मनुष्यों के लिए भोजन, जानवरों के लिए चारा और पौधों के लिए उर्वरक के रूप में महत्वपूर्ण हैं। समुद्री शैवाल का उपयोग
गण्डमाला के उपचार, आंतों और पेट संबंधी विकारों के लिए दवा के रूप में किया जाता है । (इसलिए कथन 3 सही है) अगर-अगर और
एल्गिनेट्स, आयोडीन जैसे उत्पाद जो वाणिज्यिक मूल्य के हैं, समुद्री शैवाल से निकाले जाते हैं। (अतः कथन 4 सही है)

IMPORTANT POINTS/VALUE ADDITIONS:

· भारत में प्रमुख समुद्री घास के मैदान तमिलनाडु के दक्षिण पूर्वी तट और कु छ लक्षद्वीप द्वीपों के लैगून में पाए जाते हैं। अंडमान और
निकोबार द्वीप समूह के आसपास भी घास की कु छ क्यारियाँ हैं । तमिलनाडु तट और लक्षद्वीप द्वीपों पर समुद्री घासों की प्रचुर वृद्धि मुख्य रूप
से पानी की उच्च लवणता और रेतीले सब्सट्रेटम के कारण है।

· समुद्री शैवाल, बड़े और दृश्यमान समुद्री पौधे समुद्र के अंतर्ज्वारीय और उथले उपज्वारीय क्षेत्रों में चट्टानों, मूंगों और अन्य जलमग्न परतों से
जुड़े पाए जाते हैं।

QUESTION
92.
निम्नलिखित में से कौन 'फाइटोप्लांकटन / पादपप्लवक ' हैं?

1. साइनोबैक्टीरीया

2. सिलिका-संलग्न डायटम

3. डाइनोफ्लैगलेट्स

4. हरित शैवाल

5. चाक-लेपित कोकोलिथोफ़ोर्स ( Chalk-coated coccolithophores )

सही कोड चुनें.

a) 1, 2, 3 और 4
b) 1, 3, 4 और 5
c) 1, 3 और 4
d) 1, 2, 3, 4 और 5
Correct Answer: D
Your Answer:
Explanation

Q. 92) Solution (d)

स्पष्टीकरण:

IASbaba
Score:
Web: http://ilp.iasbaba.com/
44.00 /
Email: ilp@iasbaba.com
Page 183 200
UPSC 2024 -
Exam Title : Environment,
Sc...
rupeshkr.
Email :
09ee80@gmail.com
Contact :

· 'प्लैंकटन' शब्द का तात्पर्य जीवों के उस समूह से है जो नदियों, झीलों और महासागरों के सतही जल में तैरते हैं।

· ग्रीक शब्द फाइटो (पौधा) और प्लैंकटन (भटकने या बहने के लिए बने) से व्युत्पन्न, फाइटोप्लांकटन सूक्ष्म पौधे जीव हैं जो नमकीन और
ताजा दोनों जलीय वातावरण में रहते हैं।

· कु छ फाइटोप्लांकटन बैक्टीरिया हैं, कु छ प्रोटिस्ट हैं, और अधिकांश एकल-कोशिका वाले पौधे हैं। सामान्य प्रकारों में सायनोबैक्टीरिया,
सिलिका-एन्के स्ड डायटम, डाइनोफ्लैगलेट्स, हरा शैवाल और चाक-लेपित कोकोलिथोफोरस शामिल हैं। (इसलिए विकल्प d सही है)

IMPORTANT POINTS/VALUE ADDITIONS:

· भूमि पौधों की तरह, फाइटोप्लांकटन में सूर्य के प्रकाश को ग्रहण करने के लिए क्लोरोफिल होता है, और वे इसे रासायनिक ऊर्जा में बदलने
के लिए प्रकाश संश्लेषण का उपयोग करते हैं। वे कार्बन डाइऑक्साइड का उपभोग करते हैं, और ऑक्सीजन छोड़ते हैं।

· सभी फाइटोप्लांकटन प्रकाश संश्लेषण करते हैं, लेकिन कु छ अन्य जीवों को खाकर अतिरिक्त ऊर्जा प्राप्त करते हैं। ये सूक्ष्म शैवाल ध्रुवीय
क्षेत्रों सहित सभी समुद्रों और महासागरों के प्रकाशयुक्त क्षेत्रों में मौजूद हैं।

· उनका कु ल बायोमास भूमि पर मौजूद कु ल पौधों की तुलना में कई गुना अधिक है और वे जलीय वातावरण में "चारागाह भूमि" के रूप में
काम करते हैं।

· उनके आकार के आधार पर, फाइटोप्लांकटन को इस प्रकार वर्गीकृ त किया जा सकता है: 1. मैक्रोप्लांकटन (1 मिमी से अधिक), 2.
माइक्रोप्लांकटन (1 मिमी से कम, 0.06 मिमी के बीच ), 3. नैनोप्लांकटन (5 और 60 माइक्रोमीटर के बीच ) और 4. अल्ट्राप्लांकटन (5
माइक्रोमीटर से कम )।

QUESTION
93.
'कृ षि सतत ता का समग्र सूचकांक (CIAS)' के बारे में निम्नलिखित कथनों पर विचार करें:

1. नीति आयोग ने कृ षि सतत ता का समग्र सूचकांक (सीआईएएस) तैयार किया है।

2. इससे पता चलता है कि भारतीय कृ षि पद्धतियाँ मध्यम रूप से सतत हैं।

3. वर्तमान समय में सबसे अधिक सतत कृ षि मिज़ोरम में की जाती है।

उपरोक्त में से कौन सा कथन सही है?

a) 1 और 2
b) 2 और 3
c) 1 और 3
d) उपरोक्त सभी
Correct Answer: B
Your Answer:
Explanation

Q. 93) Solution (b)

स्पष्टीकरण:

· भारतीय कृ षि अनुसंधान परिषद ने कृ षि स्थिरता का एक समग्र सूचकांक (सीआईएएस) तैयार किया है। (इसलिए कथन 1 गलत है)

· सूचकांक दर्शाता है कि भारतीय कृ षि पद्धतियाँ मध्यम रूप से सतत हैं। (अतः कथन 2 सही है)

IASbaba
Score:
Web: http://ilp.iasbaba.com/
44.00 /
Email: ilp@iasbaba.com
Page 184 200
UPSC 2024 -
Exam Title : Environment,
Sc...
rupeshkr.
Email :
09ee80@gmail.com
Contact :

· वर्तमान में, सबसे अधिक सतत कृ षि मिजोरम, के रल, आंध्र प्रदे श , मध्य प्रदे श और पश्चिम बंगाल में की जाती है, जबकि शुष्क राजस्थान में
कृ षि सबसे कम सतत है। (अतः कथन 3 सही है)

महत्वपूर्ण बिंदु/मूल्यवर्धन :

· सूचकांक के अनुसार, बेहतर प्रदर्शन करने वाले राज्यों में बड़े पैमाने पर फसल विविधीकरण, कृ षि बुनियादी ढांचे में सुधार, कृ षि ऋण और
सतत इनपुट उपयोग का अनुभव हुआ है।

· कृ षि स्थिरता के विभिन्न पैटर्न को मापने के लिए सीआईएएस के पास 51 संकेतक हैं। उन्हें मृदा स्वास्थ्य, जल संसाधन, जैव विविधता और
पारिस्थितिकी और सामाजिक-आर्थिक वर्गों के अंतर्गत वर्गीकृ त किया गया है।

· गंगा के मैदानी इलाकों में स्थित राज्य-उत्तर प्रदे श , पंजाब, बिहार और हरियाणा-साथ ही चावल-प्रधान झारखंड और असम भी अधिक
जोखिम में हैं। ये राज्य भारत के अधिकांश मुख्य खाद्य पदार्थों का उत्पादन करते हैं, जिससे सतत ता को खतरा होने पर खाद्य सुरक्षा खतरे में
पड़ जाती है।

QUESTION
94.
भारत में सूखे के बारे में निम्नलिखित कथनों पर विचार करें:

1. भारत के वल चार प्रकार के सूखे को चिह्नित करता है और उनकी निगरानी करता है: जो मौसम संबंधी, जल विज्ञान संबंधी, कृ षि संबंधी
और पारिस्थितिक संबंधी हैं ।

2. पारिस्थितिक सूखा पानी की उपलब्धता में प्रासंगिक कमी है जो पारिस्थितिकी तंत्र को भेद्यता की सीमा से परे ले जाता है, पारिस्थितिकी
तंत्र सेवाओं को प्रभावित करता है।

उपरोक्त में से कौन सा कथन सही है?

a) के वल 1
b) के वल 2
c) दोनों
d) कोई भी नहीं
Correct Answer: B
Your Answer:
Explanation

Q. 94) Solution (b)

स्पष्टीकरण:

· भारत तीन प्रकार के सूखे की निगरानी करता है - मौसम विज्ञान, जल विज्ञान और कृ षि। (इसलिए कथन 1 गलत है)

· पारिस्थितिक सूखा इसे "पानी की उपलब्धता में एक प्रासंगिक कमी के रूप में परिभाषित करता है जो पारिस्थितिकी तंत्र को भेद्य ता की
सीमा से परे ले जाता है, पारिस्थितिकी तंत्र सेवाओं को प्रभावित करता है और प्राकृ तिक और/या मानव प्रणालियों में फीडबैक को ट्रिगर
करता है।" (अतः कथन 2 सही है)

· जो चीज़ पारिस्थितिक सूखे को अलग करती है, वह सूखे के संदर्भ में मनुष्यों और पारिस्थितिक तंत्रों के बीच अंतर्संबंध पर जोर दे ना है।

IMPORTANT POINTS/VALUE ADDITIONS:

· भारत में पिछले पांच दशकों में हर तीन साल में कम से कम एक बार सूखे की सूचना मिली है। संयुक्त राष्ट्र कन्वेंश न ऑन कॉम्बैटिंग
डेजर्टिफिके शन (यूएनसीसीडी) की 2022 की रिपोर्ट के अनुसार, सूखे के प्रभाव के कारण इसे अपने सकल घरेलू उत्पाद का दो से पांच

IASbaba
Score:
Web: http://ilp.iasbaba.com/
44.00 /
Email: ilp@iasbaba.com
Page 185 200
UPSC 2024 -
Exam Title : Environment,
Sc...
rupeshkr.
Email :
09ee80@gmail.com
Contact :

प्रतिशत का नुकसान हुआ है, जिसमें एक वैश्विक सूखा सूचकांक भी शामिल है, जिसमें भारत की सूखे की संवेदनशीलता की तुलना उप
सहारा अफ्रीका से की गई है।

· एक निश्चित अवधि के लिए दीर्घकालिक औसत की तुलना में वर्षा में कमी होती है । भारत मौसम विज्ञान विभाग निर्दिष्ट करता है कि मौसम
संबंधी सूखा वह स्थिति है जब क्षेत्र में प्राप्त मौसमी वर्षा उसके दीर्घकालिक औसत मूल्य के 75% से कम होती है।

· यदि वर्षा की कमी 26-50% के बीच है तो इसे "मध्यम सूखा" और जब कमी सामान्य मूल्य के 50% से अधिक हो जाती है तो इसे "गंभीर
सूखा" के रूप में वर्गीकृ त किया जाता है।

· वर्षा को सूखे का सबसे बड़ा कारण माना जाता है, लेकिन जब सूखा घोषित करने की बात आती है तो यह एकमात्र कारक नहीं होता है।

· हाइड्रोलॉजिकल सूखे को कम सतह और उप-सतह जल आपूर्ति, जैसे कि नदियों, जलाशयों और भूजल स्रोतों द्वारा परिभाषित किया जाता
है।

· भारत के जल प्राधिकरण, जल शक्ति मंत्रालय का कहना है कि वे सूखे की स्थिति का एक महत्वपूर्ण संकेतक हैं और "औसत (या औसत से
ऊपर) वर्षा के समय भी उत्पन्न हो सकते हैं, जब पानी के बढ़ते उपयोग से भंडार कम हो जाता है।"

· कृ षि सूखा तब होता है जब मौसम संबंधी सूखा चार सप्ताह तक बना रहता है, जिससे मिट्टी की नमी और वनस्पति में कमी आती है।

· संकट के संकेत, जैसे मवेशि यों की बिक्री, चारे की अनुपलब्धता, पलायन, सीमित पेयजल आपूर्ति, श्रम रोजगार की उच्च मांग और वस्तुओं
की कीमत इस बात के संकेत हैं कि सूखे का प्रभाव फै ल गया है और आजीविका प्रभावित हुई है।

· राज्यों को सूखा घोषित करने के लिए, उन्हें 2016 के सूखा मैनुअल द्वारा निर्धारित कु छ मानदं डों के लिए अर्हता प्राप्त करनी होगी

QUESTION
95.
भारत में भालू ( Bears ) के बारे में निम्नलिखित कथनों पर विचार करें:

1. स्लॉथ भालू प्रजाति विश्व भर में पाई जाने वाली सबसे छोटी भालू प्रजाति है।

2. हिमालयी भूरा भालू अपनी वैश्विक स्थिर आबादी के आधार पर निम्न चिंताजनक है।

3. हाल ही में भारत में पहली बार तिब्बती भूरा भालू दे खा गया है।

4. उत्तर-पूर्वी क्षेत्र में पाया जाने वाला सन बियर ( Sun Bear ), 'मिरमेकोफै गी ( myrmecophagy ) ' के लिए अनुकूलित है।

उपरोक्त में से कितने कथन सही हैं?

a) के वल एक
b) के वल दो
c) के वल तीन
d) सभी चार
Correct Answer: B
Your Answer: Unanswered
Explanation

Q. 95) Solution (b)

स्पष्टीकरण:

· सूर्य भालू / सन बियर (हेलार्क्टोस मलायनस) ज्यादातर दक्षिण-पूर्व एशिया में वितरित है। भारत में यह उत्तर-पूर्वी क्षेत्र में होता है,
हालाँकि यह आम नहीं है। यह विश्व भर में पाई जाने वाली आठ भालू प्रजातियों में से सबसे छोटी है। सन बियर को पालतू जानवरों के

IASbaba
Score:
Web: http://ilp.iasbaba.com/
44.00 /
Email: ilp@iasbaba.com
Page 186 200
UPSC 2024 -
Exam Title : Environment,
Sc...
rupeshkr.
Email :
09ee80@gmail.com
Contact :

व्यापार और विभिन्न उत्पाद उपलब्ध कराने दोनों से खतरा है। IUCN ने सन बियर को सुभेद्य के रूप में सूचीबद्ध किया है। (इसलिए कथन
1 गलत है)

· भूरा भालू (उर्सस आर्क टोस) अपने भारी शरीर और भूरे बालों के कारण हिमालयी काले भालू से अलग है। IUCN ने विश्व स्तर पर स्थिर
जनसंख्या प्रवृत्ति के आधार पर इस प्रजाति को निम्न चिंताजनक प्रजातियों में से एक के रूप में सूचीबद्ध किया है । हालाँकि, एशिया
(और भारत) में इसकी अधिकांश रेंज में इसकी आबादी या इन बिखरे हुए इलाकों की कनेक्टिविटी के बारे में बहुत कम जानकारी है। ऐसा
अनुमान लगाया जाता है कि यह भूटान में पहले ही विलुप्त हो चुका है। (अतः कथन 2 सही है)

· भारत ने दुर्लभ तिब्बती भूरे भालू का अपना पहला पुष्ट रिकॉर्ड दर्ज किया है। सिक्किम वन विभाग और डब्ल्यूडब्ल्यूएफ-इंडिया द्वारा लगाए
गए कै मरा ट्रैप में भालू को उत्तरी सिक्किम के ऊं चाई वाले इलाकों में दे खा गया था। (अतः कथन 3 सही है)

· स्लॉथ भालू (मेलर्सस उर्सिनस) भारतीय उपमहाद्वीप के लिए स्थानिक है और भारत, नेपाल, भूटान और श्रीलंका में पाया जाता है।

· यह विशेष प्रजाति रुडयार्ड किपलिंग की जंगलबुक में बहुत पसंदीदा चरित्र बालू ( Baloo ) के लिए प्रेरणा है।

· स्लॉथ भालू भालू की प्रजातियों में अद्वितीय है, क्योंकि वह 'मिरमेकोफैगी' के लिए अनुकूलित होता है, यानी चींटियों और दीमकों को खाने
के लिए, जो इसके आहार का एक बड़ा हिस्सा है। IUCN ने स्लॉथ बियर को सुभेद्य के रूप में सूचीबद्ध किया है। (अतः कथन 4 ग़लत
है)

महत्वपूर्ण बिंदु/मूल्यवर्धन :

· तिब्बती भूरे भालू की उप-प्रजातियाँ भारत में भालू की मौजूदा चार प्रजातियों एशियाई काले (हिमालयी काले के साथ), स्लॉथ, सन और
हिमालयी भूरे भालू में जोड़ी गईं।

· सभी भारतीय भालू प्रजातियों को सीआईटीईएस में परिशिष्ट I और वन्यजीव (संरक्षण) अधिनियम, 1972 की अनुसूची I के तहत
सूचीबद्ध किया गया है। यह प्रजातियों को शिकार और व्यापार से पूर्ण सुरक्षा प्रदान करता है।

QUESTION
96.
Q .96) 'ग्लोबल ग्रीन ग्रोथ इंस्टीट्यूट (GGGI)' के संदर्भ में निम्नलिखित कथनों पर विचार करें:

1. ग्लोबल ग्रीन ग्रोथ इंस्टीट्यूट (GGGI) एक संधि-आधारित अंतरराष्ट्रीय, अंतर-सरकारी संगठन है।

2. इसका मुख्यालय सियोल, दक्षिण कोरिया में है।

3. यह अपने हरित विकास सूचकांक के साथ हरित विकास को बढ़ावा दे ने के लिए समर्पित है।

4. ग्लोबल ग्रीन ग्रोथ इंस्टीट्यूट और एनर्जी एफिशिएंसी सर्विसेज लिमिटे ड (ईईएसएल) एशिया लो कार्बन बिल्डिंग ट्रांजिशन (एएलसीबीटी)
परियोजना को लागू करने के लिए सहयोग करेंगे।

उपरोक्त में से कितने कथन सही हैं?

a) के वल एक
b) के वल दो
c) के वल तीन
d) सभी चार
Correct Answer: D
Your Answer:
Explanation

Q. 96) Solution (d)

स्पष्टीकरण:

IASbaba
Score:
Web: http://ilp.iasbaba.com/
44.00 /
Email: ilp@iasbaba.com
Page 187 200
UPSC 2024 -
Exam Title : Environment,
Sc...
rupeshkr.
Email :
09ee80@gmail.com
Contact :

· ग्लोबल ग्रीन ग्रोथ इंस्टीट्यूट (जीजीजीआई) एक संधि-आधारित अंतरराष्ट्रीय, अंतर-सरकारी संगठन है जो विकासशील दे श ों और उभरती
अर्थव्यवस्थाओं में मजबूत, समावेश ी और सतत आर्थिक विकास को समर्थन और बढ़ावा दे ने के लिए समर्पित है। (अतः कथन 1 सही है)

· यह सियोल, दक्षिण कोरिया में स्थित है। (अतः कथन 2 सही है)

· जीजीजीआई का लक्ष्य हरित विकास के मॉडल की ओर वैश्विक परिवर्तन करना है, हालांकि इसे दे श स्तर पर अलग किया जाएगा, इसके
मूल में ऐसी रणनीतियां होंगी जो एक साथ गरीबी में कमी, सामाजिक समावेश न, पर्यावरणीय स्थिरता और आर्थिक विकास हासिल करेंगी।

· हरित परिवर्तन की दिशा में विश्व की प्रगति सुनिश्चित करने के लिए व्यापक माप और ट्रैकिंग प्रणालियों की आवश्यकता है। इस रिपोर्ट के
साथ, जीजीजीआई ग्रीन ग्रोथ इंडेक्स के माध्यम से विश्व भर के दे श ों के हरित विकास प्रदर्शन को मापने और ट्रैक करने के लिए एक नया मंच
प्रस्तुत कर रहा है। (अतः कथन 3 सही है)

· एनर्जी एफिशिएंसी सर्विसेज लिमिटे ड (ईईएसएल) ने एक अंतरराष्ट्रीय अंतरसरकारी संगठन ग्लोबल ग्रीन ग्रोथ इंस्टीट्यूट (जीजीजीआई) के
सहयोग से भारत में एशिया लो कार्बन बिल्डिंग ट्रांजिशन (एएलसीबीटी) प्रोजेक्ट शुरू करने की घोषणा की। (अतः कथन 4 सही है)

IMPORTANT POINTS/VALUE ADDITIONS:

· ALCBT- नोडल मंत्रालय के रूप में आवास और शहरी मामलों के मंत्रालय ( MoHUA) के मार्गदर्शन में , इस रणनीतिक पहल का उद्दे श्य
दे श में चरम बिजली की मांग में अभूतपूर्व वृद्धि का मुकाबला करना है , जो मुख्य रूप से शीतलन समाधानों की बढ़ती आवश्यकता से प्रेरित
है।

· ग्लोबल ग्रीन ग्रोथ इंस्टीट्यूट (जीजीजीआई) के सहयोग से एशिया लो कार्बन बिल्डिंग ट्रांजिशन (एएलसीबीटी) परियोजना बढ़ती बिजली की
मांग की चुनौतियों का समाधान करने और कार्बन उत्सर्जन में कमी में महत्वपूर्ण योगदान दे ने के लिए तैयार है।

· मुख्य फोकस में 60 भारतीय इमारतों में एयर कं डीशनिंग सिस्टम को रेट्रोफिटिंग करना शामिल है, जो ऊर्जा दक्षता के प्रति हमारी
प्रतिबद्धता को प्रदर्शित करता है।

QUESTION
97.
समुद्र के कानून पर संयुक्त राष्ट्र कन्वेंश न (UNCLOS) से संबंधित निम्नलिखित कथनों पर विचार करें:

1. UNCLOS विशेष रूप से समुद्री क्षेत्रों और स्वतंत्र दे श ों के विशेष आर्थिक क्षेत्रों के प्रबंधन को नियंत्रित करता है।

2. अंतर्राष्ट्रीय महासागर तल UNCLOS के अधीन नहीं है जब तक कि किसी दे श ने सम्मेलन पर हस्ताक्षर या पुष्टि नहीं की हो।

3. UNCLOS के अनुसार "मानव जाति की साझी विरासत" सिद्धांत समुद्र तल और उसके खनिज संसाधनों पर लागू होता है।

उपरोक्त में से कितने कथन सही हैं?

a) के वल एक
b) के वल दो
c) सभी तीन
d) कोई नहीं
Correct Answer: A
Your Answer:
Explanation

Q. 97) Solution (a)

स्पष्टीकरण:

IASbaba
Score:
Web: http://ilp.iasbaba.com/
44.00 /
Email: ilp@iasbaba.com
Page 188 200
UPSC 2024 -
Exam Title : Environment,
Sc...
rupeshkr.
Email :
09ee80@gmail.com
Contact :

· दे श अपने स्वयं के समुद्री क्षेत्र और विशिष्ट आर्थिक क्षेत्रों का प्रबंधन करते हैं, जबकि उच्च समुद्र और अंतर्राष्ट्रीय महासागर तल समुद्र के
कानून पर संयुक्त राष्ट्र कन्वेंश न द्वारा शासित होते हैं। (इसलिए, कथन 1 गलत है)

· इसे राष्ट्रों पर लागू माना जाता है, भले ही उन्होंने इस पर हस्ताक्षर किए हों या इसकी पुष्टि की हो या नहीं। (इसलिए, कथन 2 गलत है)

· संधि के तहत, समुद्र तल और उसके खनिज संसाधनों को "मानव जाति की साझी विरासत" माना जाता है, जिसे इस तरह से प्रबंधित किया
जाना चाहिए कि आर्थिक लाभों को साझा करने, समुद्री वैज्ञानिक अनुसंधान के लिए समर्थन और समुद्री पर्यावरण की रक्षा के माध्यम से
मानवता के हितों की रक्षा की जा सके । (अतः, कथन 3 सही है)

IMPORTANT POINTS/VALUE ADDITIONS:

· अंतर्राष्ट्रीय सीबेड अथॉरिटी, संयुक्त राष्ट्र की संस्था जो विश्व के महासागर तल को नियंत्रित करती है, उन वार्ताओं को फिर से शुरू करने की
तैयारी कर रही है जो खनन के लिए अंतर्राष्ट्रीय सीबेड को खोल सकती हैं, जिसमें हरित ऊर्जा संक्रमण के लिए महत्वपूर्ण सामग्री भी शामिल
है।

· वर्षों से चल रही वार्ता एक महत्वपूर्ण बिंदु पर पहुंच रही है जहां प्राधिकरण को जल्द ही खनन परमिट आवेदन स्वीकार करना शुरू करना
होगा, जिससे कम शोध वाले समुद्री पारिस्थितिक तंत्र और गहरे समुद्र के आवासों पर संभावित प्रभावों पर चिंताएं बढ़ जाएंगी।

· गहरे समुद्र के दोहन में रुचि रखने वाली खनन कं पनियाँ अन्वेषण लाइसेंस प्राप्त करने में मदद करने के लिए दे श ों के साथ साझेदारी कर रही
हैं।

QUESTION
98.
“वैज्ञानिकों ने स्तनपायी की एक लंबे समय से लुप्त प्रजाति को फिर से खोजा है, जिसमें हेजहोग की रीढ़, एक चींटीखोर की थूथन और एक
छछू ं दर के पैर शामिल हैं, यह इसे आखिरी बार दर्ज किए जाने के 60 साल से अधिक समय बाद संभव हुआ । एटनबरो की लंबी चोंच वाली
इकिडना का नाम ब्रिटिश प्रकृ तिवादी डेविड एटनबरो के नाम पर रखा गया है।''

निम्नलिखित में से किस दे श में इकिडना की उपरोक्त वर्णित प्रजाति की खोज की गई?

a) ऑस्ट्रेलिया
b) पापुआ न्यू गिनी
c) भारत
d) इंडोनेशि या
Correct Answer: D
Your Answer:
Explanation

Q. 98) Solution (d)

स्पष्टीकरण:

· यह इंडोनेशि याई न्यू गिनी के साइक्लोप्स पर्वत में पाया जाता है।

· इसके अन्य स्तनधारियों से इतना अलग दिखने का कारण यह है कि यह मोनोट्रेम्स का सदस्य है - एक अंडे दे ने वाला समूह जो लगभग 200
मिलियन वर्ष पहले शेष स्तनपायी जीवन वृक्ष से अलग हो गया था।

· इस प्रजाति को पहले के वल एक बार 1961 में एक डच वनस्पतिशास्त्री द्वारा वैज्ञानिक रूप से दर्ज किया गया था। एक अलग इकिडना
प्रजाति पूरे ऑस्ट्रेलिया और तराई न्यू गिनी में पाई जाती है।

महत्वपूर्ण बिंदु/मूल्यवर्धन :

IASbaba
Score:
Web: http://ilp.iasbaba.com/
44.00 /
Email: ilp@iasbaba.com
Page 189 200
UPSC 2024 -
Exam Title : Environment,
Sc...
rupeshkr.
Email :
09ee80@gmail.com
Contact :

· इकिडना का नाम आधी नर , आधे सांप वाले ग्रीक पौराणिक प्राणी के साथ साझा किया गया है , और टीम द्वारा उन्हें शर्मीले , रात्रिचर बि
ल में रहने वालों के रूप में वर्णित किया गया है , जिन्हें ढूं ढना बेहद मुश्किल है।

QUESTION
99.
Q. 99 ) सुंदरबन बाघों ( Sundarban Tigers ) के बारे में निम्नलिखित कथनों पर विचार करें :

1. यह पैंथेरा टाइग्रिस टाइग्रिस की विभिन्न उपप्रजातियां हैं।

2. सुंदरबन के बाघ अन्य बाघों की तुलना में आदमखोर बाघों ( man eater tigers ) के लक्षण अधिक प्रदर्शित करते हैं।

3. उन्होंने अर्ध-जलीय जीवन शैली अपना ली है और पानी में शिकार करने में सक्षम हैं।

उपरोक्त में से कौन सा कथन गलत है?

a) 1 और 2
b) 2 और 3
c) 1 और 3
d) उपरोक्त सभी
Correct Answer: A
Your Answer:
Explanation

Q. 99) Solution (a)

स्पष्टीकरण:

· बाघों की 5 उप-प्रजातियाँ (वर्तमान जीवित जनसंख्या) की पहचान की गई है और वे इस प्रकार हैं: भारतीय या रॉयल बंग ाल टाइगर (पैंथे
रा टाइग्रिस टाइग्रिस ), इंडोचाइनीज़ टाइगर (पेंथेरा टाइग्रिस कॉर्बेटी ), साइबेरियन (अमूर) बाघ (पैंथेरा टाइग्रिस अल्ताइका ), साउथ
चाइना टाइगर (पैंथेरा टाइग्रिस अमोयेन्सिस ), सुमात्रा टाइगर (पेंथेरा टाइग्रिस सुमात्रा) ( इसलिए कथन 1 ग़लत है)

· उत्परिवर्तन दर को ध्यान में रखते हुए, जिसके कारण आनुवंशि क परिवर्तन हुआ, आम तौर पर एक जानवर जो 1 मिलियन वर्षों की अवधि
के लिए अलग-थलग था, उसे अलग-अलग प्रजातियों के रूप में वर्गीकृ त किया गया था और जो 20,000 और 50,000 वर्षों के बीच
आनुवंशि क रूप से अलग-थलग था, वह एक अलग उप-प्रजाति था।

· सुंदरबन टाइगर के मामले में , यह दूसरों के साथ सन्निहित क्षेत्र का हिस्सा था और शायद लगभग 500 से 1000 साल पहले अलग हो गया
था।

· वन विभाग द्वारा प्रदान किए गए एक अनुमान में दावा किया गया है कि 1985 और 2010 के बीच बाघों द्वारा 410 लोगों पर हमला
किया गया, के वल 95 लोग जीवित बचे। यह स्थापित नहीं है कि सुंदरबन के बाघ स्पष्ट रूप से अधिक आदमखोर हैं। आँकड़े इसे मिथक
बनाते हैं। (इसलिए कथन 2 गलत है)

· सुंदरबन टाइगर कु छ विशिष्ट रूपात्मक अनुकूलन प्रदर्शित करता है जो इसे सुंदरबन द्वीपों के मैंग्रोव निवास स्थान के लिए विशेष रूप से
अनुकूल बनाता है और पानी में शिकार करने में सक्षम बनाता है । (अतः कथन 1 सही है)

IMPORTANT POINTS/VALUE ADDITIONS:

· बाघ (पैंथेरा टाइग्रिस ) एक लुप्तप्राय बड़ी बिल्ली है जो सदियों से भारतीय संस्कृ ति और इतिहास से जुड़ी हुई है। यह निर्विवाद रूप से
जीवित सबसे राजसी जानवरों में से एक है। दुनिया में ऐसा कोई जंगली जानवर नहीं है जो एक ही सांस में अत्यधिक सम्मान, भय, अत्यधिक
जिज्ञासा या बस प्रकृ ति के जादू की भावना पैदा करता हो।

IASbaba
Score:
Web: http://ilp.iasbaba.com/
44.00 /
Email: ilp@iasbaba.com
Page 190 200
UPSC 2024 -
Exam Title : Environment,
Sc...
rupeshkr.
Email :
09ee80@gmail.com
Contact :

· टाइगर - जिसकी जनसांख्यिकीय स्थिति इसकी संपूर्ण वितरण सीमा में अनिश्चित है, 13 एशियाई दे श ों में मौजूद है, जिसमें 70% आबादी
भारतीय उपमहाद्वीप में मौजूद है। अपने बड़े शरीर के आकार और मांसाहारी आहार के कारण, बाघ स्वाभाविक रूप से कम जनसंख्या घनत्व
में पाए जाते हैं।

QUESTION
100.
निम्नलिखित में से कौन "पिसिवोर्स रैप्टर्स ( Piscivores raptors ) " का सबसे अच्छा वर्णन करता है?

a) शिकारी पक्षी जो मुख्य रूप से छोटे स्तनधारियों को खाते हैं।


b) शिकारी पक्षी जो मछली का शिकार करने और खाने में माहिर होते हैं।
c) रैप्टर जो विशेष रूप से सरीसृपों को अपने मुख्य भोजन स्रोत के रूप में लक्षित करते हैं।
d) कीड़ों और अरचिन्डों ( arachnids ) को प्राथमिकता दे ने वाले शिकारी पक्षी
Correct Answer: B
Your Answer:
Explanation

Q. 100) Solution (b)

स्पष्टीकरण:

· पिसिवोर्स ऐसे जीव हैं जो मुख्य रूप से मछली खाते हैं। रैप्टर्स के संदर्भ में, जो शिका री पक्षी हैं, पिसिवोर्स वे हैं जो मछली का शिकार करने
और खाने में माहिर हैं। (इसलिए विकल्प बी सही है)

· इन रैप्टर्स में अक्सर तेज पंजे और गहरी दृष्टि जैसे अनुकूलन होते हैं, जो उन्हें जल निकायों से मछली पकड़ने में सक्षम बनाते हैं। मछली खाने
वाले शिकारी पक्षियों के उदाहरणों में ऑस्प्रे और कु छ ईगल शामिल हैं।

IMPORTANT POINTS/VALUE ADDITIONS:

· ग्रे-हेडेड फिश ईगल्स अपने शिकार को पानी से निकालने में माहिर होते हैं। उनकी प्रजाति का नाम, इचिथिएटस , ग्रीक के 'मछली' के लिए
इचथस और 'ईगल' के लिए एटोस से लिया गया है।

· मछली खाने वालों में आम तौर पर जीवित शिकार से निपटने की क्षमता होती है जो उनके शरीर के आकार की तुलना में बड़े होते हैं, और
इसलिए उनके अंतराल बड़े होते हैं।

· कु छ अपने शिकार को पूरा निगल लेते हैं, और अन्य शिकार को फाड़कर छोटे -छोटे टु कड़ों में काट दे ते हैं। हालाँकि, तकनीकें डीमर्सल
मछली समूह के भीतर प्रजातियों के बीच व्यापक रूप से भिन्न होती हैं।

· पिसिवोर्स में आम तौर पर जबड़े पर तथा मुंह और ग्रसनी के अंदर शंक्वाकार पीछे की ओर इशारा करने वाले दांतों की कई पंक्तियाँ होती
हैं। ये मुख्य रूप से शिकार को भागने से रोकते हैं और निगलने में सहायता करते हैं।

IASbaba
Score:
Web: http://ilp.iasbaba.com/
44.00 /
Email: ilp@iasbaba.com
Page 191 200

You might also like